Pharm FINAL part 3

Ace your homework & exams now with Quizwiz!

13. Over secretion of the adrenocortical hormones leads to: A. Cushing's syndrome B. Addison's disease

A

C. stop cough reflex- non productive

When giving an antitussive, the nurse remembers that they are used primarily for? A. relieve nasal congestion B. thin secretions C. stop cough reflex- non productive D. suppress productive and non productive coughs

A nursing student asks a nurse to clarify the differences between the mechanisms of spontaneous mutation and conjugation in acquired resistance of microbes. What will the nurse say? a. Conjugation results in a gradual increase in resistance. b. Conjugation results in random changes in the microbe's DNA. c. Spontaneous mutation leads to resistance to only one antimicrobial agent. d. Spontaneous mutation can transfer DNA from one organism to another.

c. Spontaneous mutation leads to resistance to only one antimicrobial agent. Spontaneous mutation generally confers resistance to only one drug. Conjugation can occur quickly; spontaneous mutation is gradual. Spontaneous mutation is random; conjugation is not. Conjugation can occur with the transfer of DNA from one organism to another.

1. A patient will be receiving mitoxantrone (Novantrone), 12 mg/m2 every 3 weeks, as part of treatment for prostate cancer. Each dose is mixed into a 50-mL bag of D5W and needs to infuse over 15 minutes. The infusion pump delivers the dose at milliliters per hour. Identify the nurse will set the pump to infuse at what rate. _______

ANS: 200 ml/hr DIF: COGNITIVE LEVEL: Applying (Application) REF: N/A TOP: NURSING PROCESS: Implementation

patient newly diagnosed with tuberculosis asks the nurse why oral medications must be given in the clinic. The nurse will tell the patient that medications are given in the clinic so that: a. clinic staff can observe adherence to drug regimens. b. nurses can monitor for drug toxicities. c. providers can adjust doses as needed. d. the staff can ensure that the U.S. Food and Drug Administration (FDA) regulations are met.

ANS: A Adherence to drug regimens is a serious problem in the treatment of TB. Directly observed therapy (DOT) combined with intermittent dosing helps ensure adherence and increases the chance of success. Patients report drug side effects to providers; it is not necessary to give drugs in the clinic to monitor this. Doses are adjusted based on response to treatment and not on DOT. The FDA regulations do not require DOT.

13. A prescriber orders verapamil [Covera-HS] for a patient who is taking digoxin [Lanoxin] and warfarin. The nurse will expect the prescriber to ____ the dose of ____. a. lower; digoxin b. increase; digoxin c. lower; warfarin d. increase; warfarin

ANS: A Calcium channel blockers, such as verapamil, can increase levels of digoxin, so patients taking these drugs may need to have their digoxin dose reduced. Increasing the dose of digoxin can result in digoxin toxicity. Verapamil does not affect warfarin levels.

6. A patient with a tracheostomy has difficulty removing excessive, thick mucus from the respiratory tract. The nurse expects that which drug will be ordered to aid in the removal of mucus? a. Guaifenesin (Humibid) b. Benzonatate (Tessalon Perles) c. Diphenhydramine (Benadryl) d. Dextromethorphan (Robitussin DM)

ANS: A Expectorants such as guaifenesin work to loosen and thin sputum and the bronchial secretions, thereby indirectly diminishing the tendency to cough. The other drugs listed do not have this effect. DIF: COGNITIVE LEVEL: Understanding (Comprehension) REF: p. 573 TOP: NURSING PROCESS: Planning

8. When administering mineral oil, the nurse recognizes that it can interfere with the absorption of which substance? a. Fat-soluble vitamins b. Water-soluble vitamins c. Minerals d. Electrolytes

ANS: A Mineral oil can decrease the absorption of fat-soluble vitamins (A, D, E, and K). The other options are incorrect. DIF: COGNITIVE LEVEL: Applying (Application) REF: p. 820 TOP: NURSING PROCESS: Planning

6. The nurse is monitoring a patient who is receiving muromonab-CD3 (Orthoclone OKT3) after an organ transplant. Which effect is possible with muromonab-CD3 therapy? a. Chest pain b. Hypotension c. Confusion d. Dysuria

ANS: A Muromonab-CD3 may cause chest pain, fever, chills, tremor, gastrointestinal disturbances (nausea, vomiting, diarrhea), and other effects as noted in Table 48-2. The other options are incorrect. DIF: COGNITIVE LEVEL: Applying (Application) REF: p. 773 TOP: NURSING PROCESS: Evaluation

D. heart palpitations

A patient is taking a decongestant to help reduce symptoms of a cold. The nurse will instruct the patient to observe for which possible symptoms, which may indicate an adverse effect of this drug? A. increased cough B. dry mouth C. slower heart rate D. heart palpitations

ANS: D New concerns have arisen over the potential for long-term users of proton pump inhibitors (PPIs) to develop osteoporosis. This is thought to be due to the inhibition of stomach acid, and it is speculated that PPIs speed up bone mineral loss. The other options are incorrect.

The nurse is teaching a patient who will be taking a proton pump inhibitor as long-term therapy about potential adverse effects. Which statement is correct? a.Proton pump inhibitors can cause diarrhea. b.These drugs can cause nausea and anorexia. c.Proton pump inhibitors cause drowsiness. d.Long-term use of these drugs may contribute to osteoporosis.

Textbook Critical Thinking and Prioritization Questions 1. A patient with a history of decreased renal function tells the nurse, "I have finally found an antacid that gives me great relief!" The nurse checks the antacid's content and finds that the antacid is a combination of aluminum hydroxide and magnesium hydroxide. What is the nurse's priority action at this time? Explain your answer.

The nurse's priority action is to educate the patient about which antacids are safe for patients with decreased renal function. Both calcium-and magnesium-based antacids are more likely to accumulate to toxic levels in patients with renal disease. Therefore, if antacids are used in renal patients, preparations that are only aluminum-based should be chosen because they are generally more easily excreted than other categories of antacid. Sodium-based antacids may cause fluid retention and metabolic changes, so they also must be avoided by patients with renal problems.

A patient with histoplasmosis is being treated with itraconazole [Sporanox]. The nurse will teach this patient to report which symptoms? a. Gynecomastia and decreased libido b. Headache and rash c. Nausea, vomiting, and anorexia d. Visual disturbances

c. Nausea, vomiting, and anorexia

NCLEX EXAMINATION REVIEW QUESTIONS 1. A 30-year-old patient is taking simethicone for excessive flatus associated with diverticulitis. During a patient teaching session, the nurse explains the mechanism of action of simethicone by saying: a. "It neutralize gastric pH, thereby preventing gas." b. "It buffers the effects of pepsin on the gastric wall." c. "It decreases gasgtric acid secretion and thereby minimizes flatus." d. "It causes mucus-coated gas bubbles to break into smaller ones."

d "It causes mucus-coated gas bubbles to break into smaller ones."

Which drug would the nurse question if it were prescribed to treat nitroglycerin-induced tachycardia? A) Propranolol B) Verapamil C) Diltiazem D) Atropine

d Beta blockers (e.g., propranolol) and calcium channel blockers (e.g., verapamil and diltiazem) are used to treat the reflex tachycardia that may result from nitroglycerin. Atropine is not indicated because it is a vagolytic used to increase heart rate.

Factors to be considered before initiation of drug therapy with thyroid drug

desire ratio T3 to T4, the cost, and the desired duration of effect

Mechanism of Action: Antithyroid Drugs: Methimazole and Propylthiouracil

inhibit the incorporation of iodine molecules into the amino acid tyrosine, a process required to make the precursors of T3 and T4- impede formation of thyroid hormone **propylthiouracil has added effect of inhibiting conversion of T4 to T3 in peripheral circulation **neither can inactivate existing thyroid hormone -effects primarily limited to thyroid gland and overall effect is to decrease thyroid hormone level

5. When counseling a male patient about the possible adverse effects of antihypertensive drugs, the nurse will discuss which potential problem? a. Impotence b. Bradycardia c. Increased libido d. Weight gain

ANS: A Sexual dysfunction is a common complication of antihypertensive medications and may be manifested in men as decreased libido or impotence. The other options are incorrect. DIF: COGNITIVE LEVEL: Understanding (Comprehension) REF: p. 358 TOP: NURSING PROCESS: Assessment

3. Which drugs are useful for preventing preterm labor? (Select all that apply.) a. Antibiotics b. Hydroxyprogesterone c. Indomethacin d. Magnesium sulfate e. Terbutaline

ANS: A, B Two drug interventions may help to prevent preterm labor: antibiotics and hydroxyprogesterone. Indomethacin, magnesium sulfate, and terbutaline are all used to stop preterm labor after it starts. PTS: 1 DIF: Cognitive Level: Knowledge REF: p. 780

12. A patient with type 2 diabetes mellitus has been found to have trace proteinuria. The prescriber writes an order for an angiotensin-converting enzyme (ACE) inhibitor. What is the main reason for prescribing this class of drug for this patient? a. Cardioprotective effects b. Renal protective effects c. Reduces blood pressure d. Promotes fluid output

ANS: B ACE inhibitors have been shown to have a protective effect on the kidneys because they reduce glomerular filtration pressure. This is one reason that they are among the cardiovascular drugs of choice for diabetic patients. The other drugs do not have this effect. DIF: COGNITIVE LEVEL: Applying (Application) REF: p. 349 TOP: NURSING PROCESS: Assessment

6. A patient has prescriptions for two inhalers. One inhaler is a bronchodilator, and the other is a corticosteroid. Which instruction regarding these inhalers will the nurse give to the patient? a. "Take the corticosteroid inhaler first." b. "Take the bronchodilator inhaler first." c. "Take these two drugs at least 2 hours apart." d. "It does not matter which inhaler you use first."

ANS: B An inhaled bronchodilator is used before the inhaled corticosteroid to provide bronchodilation before administration of the anti-inflammatory drug. DIF: COGNITIVE LEVEL: Applying (Application) REF: p. 592 TOP: NURSING PROCESS: Implementation

8. When reviewing the laboratory values of a patient who is taking antithyroid drugs, the nurse will monitor for which adverse effect? a. Decreased glucose levels b. Decreased white blood cell count c. Increased red blood cell count d. Increased platelet count

ANS: B Antithyroid drugs may cause bone marrow suppression, resulting in agranulocytosis, leukopenia, thrombocytopenia, and other problems. The other options are incorrect. DIF: COGNITIVE LEVEL: Applying (Application) REF: p. 495 TOP: NURSING PROCESS: Evaluation

9. A patient is being discharged on anticoagulant therapy. The nurse will include in the patient-education conversation that it is important to avoid herbal products that contain which substance? a. Valerian b. Ginkgo c. Soy d. Saw palmetto

ANS: B Capsicum pepper, feverfew, garlic, ginger, ginkgo, St. John's wort, and ginseng are some herbals that have potential interactions with anticoagulants, especially with warfarin. DIF: COGNITIVE LEVEL: Understanding (Comprehension) REF: p. 417 TOP: NURSING PROCESS: Assessment

3. Which nursing diagnosis is appropriate for a patient receiving antidysrhythmics? a. Risk for infection b. Deficient knowledge c. Deficient fluid volume d. Urinary retention

ANS: B Deficient knowledge related to lack of experience with medication therapy is a potential nursing diagnosis for a patient receiving antidysrhythmics. The other options are incorrect. DIF: COGNITIVE LEVEL: Applying (Application) REF: p. 406 TOP: NURSING PROCESS: Nursing Diagnosis

7. A patient has been advised to add a nasal spray (an adrenergic decongestant) to treat a cold. The nurse will include which instruction? a. "You won't see effects for at least 1 week." b. "Limit use of this spray to 3 to 5 days." c. "Continue the spray until nasal stuffiness has resolved." d. "Avoid use of this spray if a fever develops."

ANS: B Frequent, long-term, or excessive use of adrenergic nasal decongestants may lead to rebound congestion if used beyond the recommended time. The other instructions are incorrect. DIF: COGNITIVE LEVEL: Applying (Application) REF: p. 571 TOP: NURSING PROCESS: Implementation

7. When monitoring a patient who is on immunosuppressant therapy with azathioprine (Imuran), the nurse will monitor which laboratory results? a. Serum potassium levels b. White blood cell (leukocyte) count c. Red blood cell count d. Serum albumin levels

ANS: B Leukopenia is a potential adverse effect of azathioprine therapy, so white blood cells need to be monitored. The other options are incorrect. DIF: COGNITIVE LEVEL: Applying (Application) REF: p. 773 TOP: NURSING PROCESS: Evaluation

10. A patient in the neurologic intensive care unit is being treated for cerebral edema. Which class of diuretic is used to reduce intracranial pressure? a. Loop diuretics b. Osmotic diuretics c. Thiazide diuretics d. Vasodilators

ANS: B Mannitol, an osmotic diuretic, is commonly used to reduce intracranial pressure and cerebral edema resulting from head trauma. DIF: COGNITIVE LEVEL: Understanding (Comprehension) REF: p. 456 TOP: NURSING PROCESS: Planning

5. A patient with late-stage HIV infection also has Pneumocystis jirovecii pneumonia. The nurse anticipates treatment with which medication for this pneumonia? a. Ivermectin (Stromectol) b. Atovaquone (Mepron) c. Praziquantel (Biltricide) d. Metronidazole (Flagyl)

ANS: B Pentamidine and atovaquone are used for the treatment of pneumonia caused by P. jirovecii. The other options are not used for this pneumonia. DIF: COGNITIVE LEVEL: Understanding (Comprehension) REF: p. 684 TOP: NURSING PROCESS: Planning

5. A nurse is reviewing the phenomenon of reflex tachycardia with a group of nursing students. Which statement by a student indicates understanding of this phenomenon? a. "Baroreceptors in the aortic arch stimulate the heart to beat faster." b. "Reflex tachycardia can negate the desired effects of vasodilators." c. "Reflex tachycardia is more likely to occur when beta blockers are given." d. "Venous dilation must occur for reflex tachycardia to occur."

ANS: B Reflex tachycardia, which is a compensatory mechanism in response to decreased blood pressure, can negate the desired effect of a vasodilator by eventually increasing blood pressure. Baroreceptors relay information to the vasomotor center of the medulla; the medulla sends impulses to the heart. Beta blockers are given to counter reflex tachycardia. Reflex tachycardia can be produced by dilation of both arterioles and veins.

10. While assessing a patient who is taking a beta blocker for angina, the nurse knows to monitor for which adverse effect? a. Nervousness b. Hypertension c. Bradycardia d. Dry cough

ANS: C Adverse effects of beta blockers include bradycardia, hypotension, dizziness, lethargy, impotence, and several other effects, but not dry cough or nervousness. DIF: COGNITIVE LEVEL: Understanding (Comprehension) REF: p. 367 TOP: NURSING PROCESS: Evaluation

12. A patient is receiving a dose of edrophonium (Tensilon). The nurse recognizes that this drug is given to determine the diagnosis of which disease? a. Parkinson's disease b. Multiple sclerosis c. Myasthenia gravis d. Alzheimer's disease

ANS: C Edrophonium, another indirect-acting cholinergic drug, is commonly used to diagnose myasthenia gravis. Edrophonium is not used in diagnosing Parkinson's disease, multiple sclerosis, or Alzheimer's disease. DIF: COGNITIVE LEVEL: Understanding (Comprehension) REF: p. 325 TOP: NURSING PROCESS: Planning

4. When monitoring patients on antitubercular drug therapy, the nurse knows that which drug may cause a decrease in visual acuity? a. Rifampin (Rifadin) b. Isoniazid (INH) c. Ethambutol (Myambutol) d. Streptomycin

ANS: C Ethambutol may cause a decrease in visual acuity or even blindness resulting from retrobulbar neuritis. The other options are incorrect. DIF: COGNITIVE LEVEL: Understanding (Comprehension) REF: p. 664 TOP: NURSING PROCESS: Assessment

9. When considering the various types of contraceptive drugs, the nurse is aware that which type most closely duplicates the normal hormonal levels of the female menstrual cycle? a. Monophasic b. Biphasic c. Triphasic d. Short acting

ANS: C The triphasic drugs most closely duplicate the normal hormonal levels of the female menstrual cycle. The other options are incorrect. DIF: COGNITIVE LEVEL: Understanding (Comprehension) REF: p. 540 TOP: NURSING PROCESS: Assessment

Which medications are most likely to cause postural hypotension? (Select all that apply.) a. Minoxidil b. Diltiazem [Cardizem] c. Prazosin [Minipress] d. Captopril [Capoten] e. Losartan [Cozaar]

ANS: C, D, E Postural, or orthostatic, hypotension is defined as a fall in blood pressure related to a change in position. Minoxidil and diltiazem dilate arterioles; therefore, they are not likely to cause postural hypotension. Prazosin, captopril, and losartan all reportedly can cause orthostatic hypotension.

4. A patient who is HIV- positive has been receiving medication therapy that includes zidovudine (Retrovir). However, the prescriber has decided to stop the zidovudine because of its dose-limiting adverse effect. Which of these conditions is the dose-limiting adverse effect of zidovudine therapy? a. Retinitis b. Renal toxicity c. Hepatotoxicity d. Bone marrow suppression

ANS: D Bone marrow suppression is often the reason that a patient with HIV infection has to be switched to another anti-HIV drug such as didanosine. The two drugs can be taken together, cutting back on the dosages of both and thus decreasing the likelihood of toxicity. The other options are incorrect. DIF: COGNITIVE LEVEL: Applying (Application) REF: p. 654 TOP: NURSING PROCESS: Implementation

14. A nursing student asks the nurse why multi-drug therapy is often used to treat hypertension. Which statement by the student indicates a need for further teaching? a. "Multi-drug therapy often means that drugs may be given in lower doses." b. "Some agents are used to offset adverse effects of other agents." c. "Treatment of hypertension via different mechanisms increases success." d. "Two or more drugs will lower blood pressure more quickly."

ANS: D Multi-drug therapy does not lower blood pressure more quickly. Using more than one drug often means that doses can be decreased. Some agents can offset adverse effects of other agents. Treatment via different mechanisms increases the likelihood of success.

7. When educating a patient recently placed on inhaled corticosteroids, the nurse will discuss which potential adverse effects? a. Fatigue and depression b. Anxiety and palpitations c. Headache and rapid heart rate d. Oral candidiasis and dry mouth

ANS: D Oral candidiasis and dry mouth are two possible adverse effects of inhaled corticosteroids. The other responses are incorrect. DIF: COGNITIVE LEVEL: Understanding (Comprehension) REF: p. 592 TOP: NURSING PROCESS: Planning

Osmotic diuretics: Mannitol

--Intravenous infusion only --May crystallize when exposed to low temperatures --Use of a filter is required

26. Glucocorticoids are administered topically for: A. Control of steroid-responsive bronchospastic states B. Rhinitis and to prevent the recurrence of polyps after surgical removal C. Inflammations of the eye, ear, and skin D. Shock, status asthmaticus and spinal cord injury

C

C. Urinary retention

Prior to administering an antihistamine to a patient, it is most important for the nurse to assess the patient for a history of which condition? A. Chronic urticaria B. Motion sickness C. Urinary retention D. Insomnia

3. A patient is on vitamin D supplemental therapy. The nurse will monitor for which signs of toxicity during this therapy? a. Tinnitus b. Anorexia c. Diarrhea d. Hypotension

ANS: B The toxic effects of vitamin D are those associated with hypertension, such as weakness, fatigue, headache, anorexia, dry mouth, metallic taste, nausea, vomiting, abdominal cramps, ataxia, and bone pain. If not recognized and treated, these symptoms can progress to impairment of renal function and osteoporosis. The other options listed are not signs of vitamin D toxicity. DIF: COGNITIVE LEVEL: Applying (Application) REF: p. 845 TOP: NURSING PROCESS: Assessment

Thiazide diuretics indications

-HTN (one of the most prescribed group of drugs for this) -edematous states -idiopathic hypercalciruia -diabetes insipidus -HF due to diastolic dysfunction -adjunct drugs in tx of edema r/t HF< hepatic cirrhosis, or corticosteroid or estrogen therapy

Osmotic diuretics drug effects

-Increases glomerular filtration rate and renal plasma flow; helps to prevent kidney damage during ARF -Reduces intracranial pressure or cerebral edema associated with head trauma -Reduces excessive intraocular pressure

Potassium-sparing diuretics drug effects

-Prevent potassium from being pumped into the tubule, thus preventing its secretion -Competitively block aldosterone receptors and inhibit their action -Promote the excretion of sodium and water

Potassium-sparing diuretics adverse effects

-dizziness, headache -cramps, N/V/D -urinary frequency, weakness, hyperkalemia -spironolactone: gynecomastia, amenorrhea, irregular menses, postmenopausal bleeding

Thiazide diuretics adverse effects

-dizziness, headache, blurred vision -anorexia, N/V/D -impotence -jaundice, leukopenia -urticaria, photosensitivity -hypokalemia, hyperglycemia, hyperuricemia, hypochloremic alkalosis

Loop diuretics adverse effects

-dizziness, headache, tinnitus, blurred vision -N/V/D -SJS (with torsemide) -agranulocytosis, neutropenia, thrombocytopenia -hypokalemia, hyperglycemia, hyperuricemia

Interactions: Thyroid drugs

-enhance activity of oral anticoagulants -taking concurrently with digitalis glycosides may decrease serum digitalis glycoside levels -cholestyramine binds to thyroid hormone in GI tract-may reduce absorption of both drugs -diabetic patients may require increased dosages of hypoglycemic drugs -used with epinephrine in patients with coronary disease may induce coronary insufficiency -insulin: decreased efficacy of insulin resulting in increased blood glucose levels -estrogen: reduced thyroid drug activity -digoxin: decreased digoxin effectiveness -phenytoin and fosphenytoin: reduced levothyroxine effectiveness -phenobarbital: reduced levothyroxine effectiveness

Thiazide diuretics MOA

-inhibit tubular reabsorption of sodium, chloride, and potassium ions -action primarily in the distal convoluted tubule -results in water, sodium, and chloride excretion -potassium is also excreted to a lesser extent -dilate the arterioles by direct relaxation

Thiazide diuretics drug effects

-lowered peripheral vascular resistance -depletion of sodium , water, and potassium

Potassium-sparing diuretics indications

-spironolactone and triamterene: Hyperaldosteronism, Hypertension, Reversing potassium loss caused by potassium-losing drugs, Certain cases of HF -amiloride: -Treatment of HF

Carbonic anhydrase inhibitors MOA

-the enzyme carbonic anhydrase helps to make H+ ions available for exchange with sodium and water in the proximal tubules -CAIs block the action of carbonic anhydrase, preventing the exchange of H+ ions with sodium and water -inhibition of carbonic anhydrase reduces H+ ion concentration in renal tubules -there is an increased exertion of bicarb, sodium, water, and K+ -reabsorption of water is decreased, and urine volume is increased

Thiazide diuretics

-thiazides should not be used if creatinine clearance is less than 30-50 mL/min (normal is 125 mL/min) -metolazone remains effective to a creatinine clearance of 10 mL/min

Thyroid Drugs: Levothyroxine (Levoxyl, Levothroid, Synthroid, others), T4

100% T4 (thyroxine) makes its effects most predicable, half-life long enough that it only needs to be admin once a day, switching between brands during treatment can destabilize the course of treatment, thyroid function test need to be monitored when switching products

NCLEX EXAMINATION REVIEW QUESTIONS 7. The order reads: "Give cimetidine (Tagamet) 300 mg in 100 mL normal saline IVPB tid and at bedtime. Infuse over 30 minutes." The infusion pump can only be programmed to deliver over 60 minutes (mL per hour). The nurse will set the pump to deliver how many mL/hour for each IVPB dose?

200 mL/hr

21. MOA for adrenal drugs in general: A. Exert effect by modifying enzyme activity B. Inhibit inflammatory and immune responses

A

A. loratadine (claritin)

A patient asks the nurse about the newer antihistamines. He wants one that doesn't cause drowsiness. Which of these is appropriate? A. loratadine (claritin) B. diphenhydramine (bendadryl) C. dimenhydrinate ( drammamine) D. meclizine (antivert)

11. The adrenal medulla secretes the following catecholamines: A. Epinephrine B. Norepinephrine C. Glucocorticoids D. Mineralocorticoids (primarily aldosterone)

A, B

The nurse is providing care to a patient following a non-accidental traumatic brain injury. The patient has developed diabetes insipidus due to the injury. What medication is most often used in the management of diabetes insipidus? A. desmopressin (DDAVP) B. corticotrophin (Acthar) C. octreotide (Sandostatin) D. somatropin (Humatrope)

A. desmopressin (DDAVP) Vasopressin (Pitressin) and desmopressin (DDAVP) are used to prevent or control polydipsia (excessive thirst), polyuria, and dehydration in patients with diabetes insipidus caused by a deficiency of endogenous antidiuretic hormone.

A woman has been taking a progestin-only oral contraceptive and will begin using a vaginal ring. The nurse will teach the patient to insert the ring: a. the day the last pill is taken and use backup contraception for 7 days. b. 1 week before taking the last pill. c. 1 to 5 days after taking the last pill and use backup contraception for 2 days. d. within 7 days after taking the last pill.

ANS: A When patients who have been using a progestin-only OC begin using a vaginal ring, they should be instructed to insert the ring on the last day of the pill and use backup contraception for 7 days.

2. When a patient is receiving cisplatin (Platinol-AQ) chemotherapy, the nurse will monitor for which adverse effects? (Select all that apply.) a. Tinnitus b. Heart failure c. Hearing loss d. Elevated blood urea nitrogen and creatinine levels e. Numbness or tingling in the extremities f. Elevated glucose and ketone levels

ANS: A, C, D, E Cisplatin can cause nephrotoxicity, ototoxicity, and peripheral neuropathy. Nephrotoxicity is manifested by rising blood urea nitrogen and creatinine levels; ototoxicity is manifested by tinnitus, hearing loss, and dizziness; peripheral neuropathy is manifested by numbness or tingling of the extremities. DIF: COGNITIVE LEVEL: Applying (Application) REF: p. 736 TOP: NURSING PROCESS: Evaluation

9. Abatacept (Orencia) is prescribed for a patient with severe rheumatoid arthritis. The nurse checks the patient's medical history, knowing that this medication would need to be used cautiously if which condition is present? a. Coronary artery disease b. Chronic obstructive pulmonary disease c. Diabetes mellitus d. Hypertension

ANS: B Abatacept must be used cautiously in patients with recurrent infections or chronic obstructive pulmonary disease. The other options are incorrect. DIF: COGNITIVE LEVEL: Applying (Application) REF: p. 762 TOP: NURSING PROCESS: Assessment

7. The nurse is creating a plan of care for a patient with a new diagnosis of hypertension. Which is a potential nursing diagnosis for the patient taking antihypertensive medications? a. Diarrhea b. Sexual dysfunction c. Urge urinary incontinence d. Impaired memory

ANS: B Sexual dysfunction is a potential nursing diagnosis related to possible adverse effects of antihypertensive drug therapy. The other nursing diagnoses are not appropriate. DIF: COGNITIVE LEVEL: Applying (Application) REF: p. 355 TOP: NURSING PROCESS: Nursing Diagnosis

4. The nurse is counseling a patient about calcium supplements. Which dietary information is appropriate during this teaching session? a. "Take oral calcium supplements with meals." b. "There are no drug interactions with calcium products." c. "Avoid foods that are high in calcium, such as beef, egg yolks, and liver." d. "Be sure to eat foods high in calcium, such as dairy products and salmon."

ANS: D Foods high in calcium include dairy products, fortified cereals, calcium-fortified orange juice, sardines, and salmon. Patients can be encouraged to add dietary sources of calcium to their diets. Oral-dosage forms of calcium need to be given 1 to 3 hours after meals. Calcium salts will bind with tetracycline and quinolone antibiotics and result in an insoluble complex. DIF: COGNITIVE LEVEL: Analyzing (Analysis) REF: p. 843 |pp. 857-858 TOP: NURSING PROCESS: Implementation

Which needle length and gauge should the nurse choose to administer subcutaneous heparin? A. 1/2 inch; 20 gauge B. 5/8 inch; 25 gauge C. 1.5 inch; 18 gauge D. 1 inch; 26 gauge

B. 5/8 inch; 25 gauge Heparin should be administered subcutaneously into the fatty layer of the abdomen with a 1/2- to 5/8-inch needle, 25 or 26 gauge. The only appropriate option for injection in the list shown is the 5/8-inch, 25-gauge needle.

28. Contraindictions for adrenal drugs: A. Drug allergies B. Serious infections, including septicemia, systemic fungal infections, and varicella C. All of the above

C

A 63-year-old male patient has been receiving a heparin infusion for 2 days for treatment of pulmonary embolism. Which symptom most clearly relates to an adverse effect of heparin? A. Heart rate of 60 beats per minute B. Blood pressure of 160/88 mm Hg C. Discolored urine D. Wheezing

C. Discolored urine The primary and most serious adverse effect of heparin is bleeding. Bleeding can occur from any site and may be manifested in various ways, including reduced blood pressure, increased heart rate, bruises, petechiae, hematomas, red or black stools, cloudy or discolored urine, pelvic pain, headache, and lumbar pain.

Which is a priority nursing diagnosis for a patient receiving desmopressin (DDAVP)? A. Risk for injury B. Acute pain C. Excess fluid volume D. Deficient knowledge regarding medication

C. Excess fluid volume Desmopressin is a form of antidiuretic hormone, which increases sodium and water retention, leading to an alteration in fluid volume. Although the other nursing diagnoses may be appropriate, they are not a priority using Maslow's hierarchy of needs.

35. Adverse effects for adrenal drugs of this system include: - Muscle weakness, loss of muscle mass, osteoporosis A. Cardiovascular B. CNS C. Endocrine D. GI E. Integumentary F. Musculoskeletal G. Ocular H. Other

F

A. albuterol (Ventolin)

Which medication will the nurse teach a patient with asthma to use when experiencing an acute asthma attack? A. albuterol (Ventolin) B. salmeterol (Serevent) C. theophylline (Theo-Dur) D. montelukast (Singulair)

C. St. John's wort

Which over-the-counter/herbal product, when taken with theophylline, can decrease theophylline's serum drug levels? A. Caffeine B. Diphenhydramine C. St. John's wort D. Echinacea

C. "Wait 1 to 2 minutes before you take a second puff of the same drug."

Which statement would be included when teaching a patient about the proper use of metered-dose inhalers? A. "After you inhale the medication once, repeat until you obtain relief." B. "Make sure that you puff out air repeatedly after you inhale the medication." C. "Wait 1 to 2 minutes before you take a second puff of the same drug." D. "Hold the inhaler in your mouth, take a deep breath, and then compress the inhaler."

A patient who has erectile dysfunction asks about medications to treat this disorder, but tells the nurse he doesn't want to have to plan sexual activity several hours in advance. Which medication will the nurse expect the provider to order for this patient? a. Avanafil [Stendra] b. Sildenafil [Viagra] c. Tadalafil [Cialis] d. Vardenafil [Levitra]

a. Avanafil [Stendra] Avanafil is used to treat ED and, unlike the other agents, has a shorter onset of action, with effects occurring in 15 minutes after taking the drug and may be taken 30 minutes prior to intercourse. The other agents must be taken 2 hours prior to intercourse.

A patient with Cushing's syndrome has undergone surgery and radiation treatment. The nurse will expect to teach the patient about which medication? a. Cosyntropin b. Dexamethasone c. Fludrocortisone acetate [Florinef] d. Ketoconazole [Nizoral]

d. Ketoconazole [Nizoral] The role of drugs in the treatment of Cushing's syndrome is limited; drugs are used only as adjuncts to surgery and radiation therapy. The most effective agent is ketoconazole, which suppresses steroid synthesis. Cosyntropin is used to diagnose adrenal insufficiency. Dexamethasone is a glucocorticoid used for replacement therapy and to diagnose Cushing's syndrome. Fludrocortisone is used for chronic mineralocorticoid replacement.

1. A patient will be receiving testosterone cypionate (Depo-Testosterone) 400 mg intramuscularly every 4 weeks. The medication is available in a 200-mg/mL strength. Identify how many milliliters will the nurse draw up for each injection. _______

ANS: 2 ml DIF: COGNITIVE LEVEL: Applying (Application) REF: N/A TOP: NURSING PROCESS: Implementation

2. A patient will be receiving oral theophylline (Theo-Dur), 600 mg/day, in three divided doses. Identify how many milligrams will the patient receive per dose. _______

ANS: 200 mg DIF: COGNITIVE LEVEL: Applying (Application) REF: N/A TOP: NURSING PROCESS: Implementation

A prescriber has ordered pilocarpine [Pilocar]. A nurse understands that the drug stimulates muscarinic receptors and would expect the drug to have which action? a.Reduction of excessive secretions in a postoperative patient b.Lowering of intraocular pressure in patients with glaucoma c.Inhibition of muscular activity in the bladder d.Prevention of hypertensive crisis

ANS: B Pilocarpine is a muscarinic agonist used mainly for topical therapy of glaucoma to reduce intraocular pressure. Pilocarpine is not indicated for the treatment of excessive secretions and mucus; in fact, pilocarpine is used to treat dry mouth. Pilocarpine does not inhibit muscular activity in the bladder. Pilocarpine is not used to prevent hypertensive crisis.

2. Which drug classes are considered first-line treatment for heart failure? (Select all that apply.) a. Angiotensin-converting enzyme (ACE) inhibitors b. Angiotensin II receptor blockers (ARBs) c. Digoxin (cardiac glycoside) d. Beta blockers e. Nesiritide (Natrecor), the B-type natriuretic peptide

ANS: A, B, D ACE inhibitors, ARBs, and beta blockers are now considered the first-line treatments for heart failure. Digoxin is used when the first-line treatments are not successful; nesiritide is considered a last-resort treatment. DIF: COGNITIVE LEVEL: Applying (Application) REF: p. 378 TOP: NURSING PROCESS: Planning

14. The nurse knows to administer acarbose (Precose), an alpha-glucosidase inhibitor, at which time? a. 30 minutes before breakfast b. With the first bite of each main meal c. 30 minutes after breakfast d. Once daily at bedtime

ANS: B When an alpha-glucosidase inhibitor is taken with the first bite of a meal, excessive postprandial blood glucose elevation (a glucose spike) can be reduced or prevented. DIF: COGNITIVE LEVEL: Applying (Application) REF: p. 510 TOP: NURSING PROCESS: Implementation

3. A nurse is caring for a patient who will begin taking hydralazine to treat hypertension. Which statement by the patient indicates understanding of the nurse's teaching about this drug? a. "I will need to ask for assistance when getting up out of a chair." b. "I will also take a beta blocker medication with this drug to prevent rapid heart rate." c. "I may develop joint pain, but this side effect will decrease over time." d. "This drug may cause excessive hair growth on my face, arms, and back."

ANS: B Hydralazine can cause severe reflex tachycardia; a beta blocker is usually given to counter this effect. Postural hypotension is minimal with hydralazine. Patients should be taught to report joint pain, which indicates an SLE syndrome and requires discontinuation of the drug. Minoxidil can cause excessive hair growth, not hydralazine.

24. Glucocorticoids are administered by inhalation for: A. Control of steroid-responsive bronchospastic states B. Rhinitis and to prevent the recurrence of polyps after surgical removal C. Inflammations of the eye, ear, and skin D. Shock, status asthmaticus and spinal cord injury

A

ANS: A Calcium antacids are not used as frequently as other antacids because their use may lead to the development of kidney stones; they also cause increased gastric acid production. The other options are incorrect.

A patient is asking advice about which over-the-counter antacid is considered the most safe to use for heartburn. The nurse explains that calcium antacids are not used as frequently as other antacids because a.their use may result in kidney stones. b.they cause decreased gastric acid production. c.they cause severe diarrhea. d.their use may result in fluid retention and edema.

ANS: C Neuromuscular blocking agents relax skeletal muscle to produce flaccid paralysis; the first muscles to be affected are the levator muscle of the eyelids and the muscles of mastication. These agents do not affect the CNS. Because the drug has just taken effect, a toxic dose is not yet a concern. Mechanical ventilation is necessary when the drug affects the intercostal muscles and the diaphragm, interfering with respiration.

A patient receives a neuromuscular blocking agent before a procedure. The patient's eyes close. The nurse knows this is a sign that the patient: a.has fallen asleep. b.has received a toxic dose of the medication. c.is beginning to feel the drug's effects. d.may need mechanical ventilation

1. A patient has been admitted for an exacerbation of chronic obstructive pulmonary disease and will be receiving methylprednisolone (Solu-Medrol) 30 mg intravenously every 6 hours. The medication is available in 40-mg/mL vials. Identify how many milliliters will the nurse draw up for this dose. _______

ANS: 0.75 mL DIF: COGNITIVE LEVEL: Applying (Application) REF: N/A TOP: NURSING PROCESS: Implementation

A patient is experiencing severe diarrhea, flushing and life threatening hypotension associated with carcinoid crisis. The nurse will prepare to administer which drug? A. octreotide (Sandostatin) B. vasopressin (Pitressin) C. somatropin (Humatrope) D. cosyntropin (Cortrosyn)

A. Octreotide (Sandostatin)

The nurse knows that which statement is accurate for enoxaparin (Lovenox)? A. It equally reduces the activity of thrombin and factor Xa. B. It has selective inhibition of factor Xa and no effect on thrombin. C. It reduces the activity of factor Xa more than the activity of thrombin. D. It has a lower bioavailability and shorter half-life than unfractionated heparin.

C. It reduces the activity of factor Xa more than the activity of thrombin. Enoxaparin acts primarily on factor Xa and also but to a lesser degree on thrombin. Unfractionated heparin equally reduces the action of thrombin and factor Xa. Fondaparinux (Arixtra) causes selective inhibition of factor Xa. Low-molecular-weight (LMW) heparins, such as enoxaparin, have greater bioavailability and a longer half-life than those of unfractionated heparin.

When administering a loop diuretic to a patient, it is most important for the nurse to determine if the patient is also taking which drug? A.lithium (Eskalith) B.acetaminophen (Tylenol) C.penicillin D.theophylline

Correct answer: A Rationale: Use of loop diuretics with lithium can increase the risk of lithium toxicity. Drug interactions with loop diuretic therapy can occur with concurrent use of nonsteroidal antiinflammatory drugs (NSAIDs), and vancomycin can cause increased neuro- and ototoxicity when used with loop diuretics. There is no associated risk of drug interaction when taking acetaminophen, penicillin, or theophylline with loop diuretics.

While preparing an infusion of mannitol (Osmitrol), the nurse notices small crystals in the IV tubing. The most appropriate action by the nurse is to A.administer the infusion slowly. B.discard the solution and obtain another bag of medication. C.obtain a filter, and then infuse the solution. D.return the fluid to the IV bag to dissolve the crystals.

Correct answer: B Rationale: Even though a filter should always be used with this medication infusion, a solution with crystals present in the bag or tubing should never be infused. The nurse should first discard the solution and then restart the infusion with a new bag, ensuring that there is a filter on the IV line.

23. Indications for adrenal drugs: A. Adrenocortical deficiency B. Collagen diseases C. Dermatologic diseases D. GI diseases E. Exacerbations of chronic respiratory illnesses, such as asthma and COPD F. Organ transplant (decrease immune response) G. Palliative management of leukemias and lymphomas H. Spinal cord injury I. All of the above

I

1st- C. albuterol 2nd- B. ipratropium 3rd- A. beclamethasone

The physician orders ipratropium bromide (Atrovent), albuterol (Proventil), and beclomethasone (Beclovent) inhalers for your patient. Put the drugs in the correct order in which they will be administered, using the choices A through C listed below. A. beclomethasone (Beclovent) B. ipratropium bromide (Atrovent) C. albuterol (Proventil)

A. Coronary artery disease

The nurse is providing care to a patient prescribed a nonselective adrenergic agonist bronchodilator. Which medical diagnosis on this patient's chart would alert the nurse to question this order? A. Coronary artery disease B. Chronic obstructive pulmonary disease (COPD) C. Hyperkalemia D. Premature labor

Textbook Critical Thinking and Prioritization Questions 2. A patient tells the nurse, "I like taking antacids because they coat my stomach and protect my ulcer." What is the nurse's priority when answering the patient's question?

The priority is to explain to the patient how antacids work and how they are to be used for short-term relief only. Antacids do not coat the stomach. They elevate the gastric pH so that an acidic environment, which could cause more problems for patients with ulcers, does not exist. However, long-term use of antacids may mask symptoms of serious problems, such as gastric ulcers. In addition, overuse of antacids may cause acid rebound. Patients with chronic heartburn problems need to be checked thoroughly for the presence of more serious conditions.

NCLEX EXAMINATION REVIEW QUESTIONS 5. A patient who is taking oral tetracycline complains of heartburn and requests an antacid. Which action by the nurse correct? a. Give the tetracycline, but delay the antacid for 1 to 2 hours. b. Give the antacid, but delay the tetracycline for at least 4 hours. c. Administer both medications together. d. Explain that the antacids cannot be given while the patient is taking the tetracycline.

a Give the tetracycline, but delay the antacids for 1 to 2 hours.

A. Liver enzymes

Which laboratory value would the nurse assess before administering zafirlukast (Accolate) to a patient? A. Liver enzymes B. Cardiac enzymes C. Renal function tests D. Complete blood count

Preparation for the NCLEX® Examination Questions 1. The nurse will teach clients that antacids are effective in the treatment of hyperacidity based on which mechanism of action? a. Neutralizes gastric acid b. Decreases gastric pH c. Decreases stomach motility d. Decreases duodenal pH

a Neutralizing gastric acid Antacids work by neutralizing gastric acid, which would cause an increase in pH. They do not affect gastric motility.

A 14-year-old male patient has not begun puberty. His parents tell the nurse that their son does not want to go to school, because he gets teased. The nurse learns that the boy's father did not begin puberty until age 16 years. Laboratory tests on this child do not reveal true hypogonadism. What will the nurse tell these parents when they ask what can be done for their son? a. "A limited course of androgen therapy may be prescribed, but it is not necessary." b. "He will eventually begin puberty, so this is nothing to worry about." c. "He will probably have to receive injections of androgens for 3 to 4 years." d. "The risk of accelerated growth plate closure is too great to warrant androgen therapy."

a. "A limited course of androgen therapy may be prescribed, but it is not necessary."

The nurse is teaching a pharmacology refresher course to a group of nurses. A student asks what host factors affect the choice of agents in antimicrobial therapy. The nurse will tell the students that such host factors include what? (Select all that apply.) a. Age b. Gender c. Immune system status d. Infection site e. Weight

a. Age c. Immune system status d. Infection site Host factors, such as the patient's age, immune status, and the site of the infection, affect the choice of antimicrobial therapy used to treat an infection. Gender and weight do not affect the choice of antimicrobial but may affect the dose and route of administration.

A patient is admitted with nausea, vomiting, diarrhea, and abdominal pain. The patient appears emaciated and complains of feeling weak. The nurse notes a heart rate of 98 beats per minute and a blood pressure of 88/54 mm Hg. The nurse reviews the chart and notes an increased serum potassium level and a decreased serum sodium level. The nurse expects the provider to order which medication initially? a. Cosyntropin [Cortrosyn] b. Dexamethasone c. Fludrocortisone [Florinef] d. Hydrocortisone

a. Cosyntropin [Cortrosyn] Cosyntropin is used to diagnose adrenal insufficiency, which this patient shows signs of having. Dexamethasone is used to diagnose Cushing's syndrome. Fludrocortisone, a mineralocorticoid, is used with hydrocortisone to treat primary adrenal insufficiency; both of these drugs will be administered after the diagnosis has been confirmed.

A patient is receiving tobramycin three times daily. A tobramycin peak level is 4.5 and the trough is 1.2. What will the nurse do? a. Give the next dose as ordered. b. Hold the next dose and notify the provider. c. Monitor the patient for signs of nephrotoxicity. d. Tell the patient to report tinnitus.

a. Give the next dose as ordered. These levels are within normal limits, so the next dose may be given safely. It is not necessary to withhold the next dose. These levels do not indicate any increased risk of nephrotoxicity or ototoxicity.

NCLEX EXAMINATION REVIEW QUESTIONS 4. A patient with a history of renal problems is asking for advice about which antacid he should use. The nurse will make which recommendation? a. "Patients with renal problems cannot use antacids." b. "Aluminum-based antacids are the best choice for you." c. "Calcium-based antacids are the best choice for you." d. "Magnesium-based antacids are the best choice for you."

b "Aluminum-based antacids are the best choice for you."

An adult male patient will begin androgen therapy for testicular failure. Which statement by the patient indicates understanding of the treatment regimen? a. "I will need to have x-rays of my hands and feet every 6 months." b. "My libido may improve while I am taking this medication." c. "Taking this drug may lead to the development of prostate cancer." d. "This will restore fertility, so I can have a child."

b. "My libido may improve while I am taking this medication."

A patient has a viral sinus infection, and the provider tells the patient that antibiotics will not be prescribed. The patient wants to take an antibiotic and asks the nurse what possible harm could occur by taking an antibiotic. Which response by the nurse is correct? a. "Antibiotics are mutagenic and can produce changes that cause resistance." b. "Even normal flora can develop resistance and transfer this to pathogens." c. "Host cells become resistant to antibiotics with repeated use." d. "Patients who overuse antibiotics are more likely to have nosocomial infections."

b. "Even normal flora can develop resistance and transfer this to pathogens." Antibiotics make conditions favorable for the overgrowth of microbes with acquired resistance. Normal flora, present at all times, can develop resistance and can transfer this resistance to pathogens if they occur. Even when pathogens are not present, antibiotic use can promote resistance in the future. Antibiotics are not mutagenic. Host cells are not affected. Antibiotic use does not increase the risk of nosocomial infection in a particular patient but does increase resistance in resident organisms in a particular hospital.

A patient is receiving an intraperitoneal aminoglycoside during surgery. To reverse a serious side effect of this drug, the nurse may expect to administer which agent? a. Amphotericin B b. Calcium gluconate c. Neuromuscular blocker d. Vancomycin

b. Calcium gluconate Aminoglycosides can inhibit neuromuscular transmission, especially during intraperitoneal or intrapleural instillation, and this risk is increased when neuromuscular blocking agents and general anesthetics are given. Calcium can reverse neuromuscular blockade. Amphotericin B, additional neuromuscular blockers, and vancomycin are not indicated.

According to studies of estrogen/progestin therapy (EPT), what are its known benefits? (Select all that apply.) a. Cardiovascular protection in older patients b. Decreased osteoporosis risk c. Glycemic control d. Improved wound healing e. Prevention of colorectal cancer

b. Decreased osteoporosis risk c. Glycemic control d. Improved wound healing EPT can lower osteoporosis risk, improve glycemic control, and aid in wound healing. Preparations with estrogen alone can provide cardiovascular protection or help prevent colorectal cancer, but not preparations containing progestin.

A nurse caring for a patient notes that the patient has a temperature of 104°F and a heart rate of 110 beats per minute. The patient's skin is warm and moist, and the patient complains that the room is too warm. The patient appears nervous and has protuberant eyes. The nurse will contact the provider to discuss: a. cretinism. b. Graves' disease. c. myxedema. d. Plummer's disease.

b. Graves' disease. The signs and symptoms in this patient are consistent with hyperthyroidism, and because the patient's eyes are protuberant, they also are consistent with Graves' disease. Cretinism is hypothyroidism in children. Myxedema is severe hypothyroidism. Plummer's disease is a hyperthyroidism condition without exophthalmos.

When monitoring the laboratory values of a patient who is taking antithyroid drugs, the nurse knows to watch for a. increased platelet counts b. decreased WBC counts c. decreased BUN level d. increased blood glucose levels

b. decreased WBC counts

A patient who has erectile dysfunction asks a nurse whether sildenafil [Viagra] would be a good medication for him to take. Which aspect of this patient's history would be of most concern? a. Benign prostatic hypertrophy b. Mild hypertension c. Occasional use of nitroglycerin d. Taking finasteride

c. Occasional use of nitroglycerin Patients taking nitroglycerin should not take sildenafil. Having BPH is not a contraindication. Mild hypertension requires caution but is not a contraindication. Patients with BPH taking finasteride may take sildenafil.

Osmotic diuretics adverse effects

convulsions, thrombophlebitis, pulmonary congestion, headaches, chest pain, tachycardia, blurred vision, chills, fever

The pharmacy has called a patient to notify her that the current brand of thyroid replacement hormone is on back order. The patient calls the clinic to ask what to do. Which is the best response by the nurse? a. Go ahead and take the other brand that the pharmacy has available for now b. You can stop the medication until your current brand is available c. You can split the thyroid pills that you have left so that they will last longer d. Let me ask your physician what needs to be done; we will need to watch how you do if you switch brands

d. Let me ask your physician what needs to be done; we will need to watch how you do if you switch brands

Contraindications: Thyroid drugs

known drug allergy, recent myocardial infarction, adrenal insufficiency, and hyperthyroidism

Contraindications: Antithyroid Drugs

known drug allergy, use in pregnancy is controversial-according to FDA propythiouracil to be used during first trimester only, then methimazole is used for remainder f pregnancy *case reports of scalp abnormalities when methimazole is used, both drugs classified as pregnancy D drugs

Carbonic anhydrase inhibitors adverse effects

metabolic acidosis, anorexia, hematuria, photosensitivity, melena, hypokalemia, drowsiness, paresthesias, urticaria, glycosuria in diabetic pts

11. A pregnant woman is experiencing hypertension. The nurse knows that which drug is commonly used for a pregnant patient who is experiencing hypertension? a. Mannitol (Osmitrol) b. Enalapril (Vasotec) c. Hydrochlorothiazide (HydroDIURIL) d. Methyldopa (Aldomet)

ANS: D Methyldopa is used in the treatment of hypertension during pregnancy. The other options are incorrect. DIF: COGNITIVE LEVEL: Understanding (Comprehension) REF: p. 347 TOP: NURSING PROCESS: Planning

1. A patient is to receive filgrastim (Neupogen) 5 mcg/kg/day. The patient weighs 198 pounds. Identify how many micrograms of medication will this patient receive each day. _______

ANS: 450 mcg Convert pounds to kilograms: Calculate mcg/day for this patient: DIF: COGNITIVE LEVEL: Applying (Application) REF: N/A TOP: NURSING PROCESS: Implementation

4. When assessing for potential adverse effects of fludrocortisone (Florinef), the nurse monitors for signs and symptoms of which condition? A. Hypokalemia B. Hypovolemia C. Hyponatremia D. Hypercalcemia

A

1. A patient will be receiving metoprolol (Lopressor) 5 mg IV push for angina. The medication is available in a strength of 1 mg/mL. Identify how much medication will the nurse draw up for each dose. _______

ANS: 5 mL DIF: COGNITIVE LEVEL: Applying (Application) REF: N/A TOP: NURSING PROCESS: Implementation

A patient who has drug-sensitive tuberculosis has completed 2 months of the standard four-drug therapy and asks the nurse how long he will have to take medication. Which response by the nurse is correct? a. "As long as you remain symptomatic, you will not have to take more medication." b. "The four-drug regimen will continue for 3 more months." c. "You will have to take maintenance drugs indefinitely." d. "You will need to take only two drugs for the next 4 months."

ANS: D Patients with drug-sensitive tuberculosis take four drugs for 2 months during the induction phase, followed by two drugs for 4 months in the continuation phase. Drug therapy does not stop after the induction phase, even for asymptomatic patients. Although drug therapy is prolonged, it is not indefinite.

A nurse is performing a physical assessment on a patient with tuberculosis who takes rifampin [Rifadin]. What would be an expected finding? a. Crystalluria b. Myopathy c. Peripheral neuropathy d. Red-orange-tinged urine

ANS: D Urine tinged red-orange is a normal finding associated with rifampin's adverse effects. Peripheral neuropathy, myopathy, and crystalluria are not manifestations of adverse effects of rifampin.

13. The nurse is caring for a 15-year-old patient who has undergone a liver transplant. Which provider order will the nurse question? a. Cyclosporine [Sandimmune] and ketoconazole [Nizoral] b. Everolimus [Zortress] 1 mg twice daily c. Prednisone 60 mg daily d. Tacrolimus [Prograf] 50 mcg/kg twice daily the day after surgery

B Everolimus is not approved in patients younger than 18 years of age. Ketoconazole can decrease the metabolism of cyclosporine and is often given concurrently to allow lower dosing and lower cost. Prednisone and tacrolimus are commonly used, and these doses are correct.

14. Under secretion of the adrenocortical hormones leads to: A. Cushing's syndrome B. Addison's disease

B

ANS: C Medications are not to be taken, unless prescribed, within 1 to 2 hours of taking an antacid because of their impact on the absorption of many medications in the stomach.

At 0900, the nurse is about to give morning medications, and the patient has asked for a dose of antacid for severe heartburn. Which schedule for the antacid and medications is correct? a.Give both the antacid and medications at 0900. b.Give the antacid at 0900, and then the medications at 0930. c.Give the medications at 0900, and then the antacid at 1000. d.Give the medications at 0900, and then the antacid at 0915.

10. Which conditions is aminoglutethimide (Cytadren) used to treat? (Select all that apply.) A. Thyroid cancer B. Adrenal cancer C. Testicular cancer D. Cushing's syndrome E. Metastatic breast cancer

B, D, E

A nurse is reviewing a patient's medications before administration. Which drug-to-drug interactions will most concern the nurse in a patient with a history of heart failure and a potassium level of 5.5 mEq/L? a. Furosemide [Lasix] and enalapril [Vasotec] b. Amlodipine [Norvasc] and spironolactone [Aldactone] c. Captopril [Capoten] and spironolactone [Aldactone] d. Metoprolol [Lopressor] and furosemide [Lasix]

C ACE inhibitors increase the risk of hyperkalemia, and combining this drug with a potassium-sparing diuretic creates a significant risk of hyperkalemia. Furosemide and enalapril, an ACE inhibitor, would not be contraindicated in this patient. Amlodipine and spironolactone would not cause hyperkalemia. The combination of metoprolol, a beta blocker, and furosemide would not be contraindicated in this patient.

2. A patient with a liver transplant has been receiving cyclosporine [Sandimmune] for 6 months. The nurse reviews this patient's laboratory results and notes a sharp increase in the blood urea nitrogen (BUN) and serum creatinine. Vital signs are normal, and the patient reports no discomfort. What does the nurse suspect? a. Hepatotoxicity b. Infection c. Organ rejection d. Nephrotoxicity

D An elevation of BUN and serum creatinine is an indication of nephrotoxicity, which occurs in 75% of patients taking cyclosporine. Hepatotoxicity would cause elevations in liver enzymes, not the BUN and creatinine. Infection would be associated with fever. Organ rejection of a renal transplant would cause elevation in the BUN and creatinine but also would cause tenderness at the graft site and fever.

A patient with a creatinine clearance of 20 mL/min is admitted to the medical surgical unit. The patient is in need of rapid diuresis. Which class of diuretic does the nurse anticipate administering? A.Potassium-sparing B.Thiazide C.Osmotic D.Loop

Correct answer: D Rationale: The loop diuretics provide rapid diuresis because of their rapid onset of action. Loop diuretics are effective for patients with reduced kidney function (creatinine clearance below 25 mL/min).

16. Mineralocorticoids are: A. Topical B. Inhaled C. Nasal D. Systemic E. All of the above

D

A nurse teaches a health education class to male adolescents. Which of these topics should be a priority for the nurse to include? Adoption of healthy lifetime hygienic practices Effects of performance-enhancing androgens Pituitary gland regulation of sexual development Self-esteem issues related to delayed onset of puberty

Effects of performance-enhancing androgens

20. Anti-adrenals, adrenal steroid inhibitors, include: A. Beclomethasone (several formulations) B. Fluticasone propionate C. Hydrocortisone (several formulations) D. Cortisone E. Methylprednisolone F. Prednisone G. Fludrocortisone acetate H. Aminoglutethimide

H

A patient has a systemic candidal infection, and the provider has ordered oral fluconazole [Diflucan] 400 mg on day 1 and 200 mg once daily thereafter. What will the nurse do? a. Administer the drug as ordered. b. Contact the provider to discuss cutting the dosing in half. c. Contact the provider to discuss giving 400 mg on all days. d. Contact the provider to discuss giving the drug intravenously.

a. Administer the drug as ordered.

A clinic nurse is assessing an adolescent male patient who has been receiving androgen therapy for hypogonadism via a transdermal patch. The patient's last clinic visit was 4 weeks earlier. Which part of the interval history is of most concern to the nurse? a. Five-pound weight gain b. Increased growth of pubic hair c. Rash at the site of the patch d. Presence of acne

a. Five-pound weight gain

A nurse monitors for which adverse cardiovascular effects in a male patient taking testosterone [Androderm]? (Select all that apply.) Postural hypotension Atrial fibrillation Pedal edema and weight gain Prolonged QT interval Decrease in high-density lipoprotein (HDL) levels

Pedal edema and weight gain Decrease in high-density lipoprotein (HDL) levels

Androgens are prescribed for a patient with anemia. The nurse is aware that the action of the androgen in this patient is what? Iron replacement Promoting synthesis of erythropoietin Prevention of blood loss Increase in bone formation

Promoting synthesis of erythropoietin

A. Drowsiness and dizziness

The nurse is discussing use of antitussive medications with a patient. What common side effect does the nurse include in the patient teaching? A. Drowsiness and dizziness B. Diarrhea and abdominal cramping C. Tremors and anxiety D. Bradycardia and increased lacrimation

C. a patient who has had recent abdominal surgery

The nurse knows that an antitussive cough medication would be the best choice for which patient? A. a patient with a productive cough B. a patient with chronic paranasal sinusitis C. a patient who has had recent abdominal surgery D. a patient who has influenza

B. Therapeutic

The nurse receives lab values for a patient with a theophylline level of 14 mcg/mL. How does the nurse interpret this theophylline level? A. Subtherapeutic B. Therapeutic C. Toxic D. Life-threatening

A nurse is teaching a patient about the use of sildenafil [Viagra] for erectile dysfunction. Which statement by the patient indicates understanding of the teaching? a. "If my erection lasts longer than 4 hours, I should contact my provider." b. "I should not use nitroglycerin within 12 hours of using sildenafil." c. "I should take this drug about 15 minutes before sexual activity." d. "This drug may cause me to have an erection when I don't want one."

a. "If my erection lasts longer than 4 hours, I should contact my provider." Priapism can occur and can cause tissue damage to the penis, so an erection lasting longer than 4 hours should be reported. Patients who use nitroglycerin should not use sildenafil within 24 hours of taking this drug. Patients should be advised to take sildenafil 30 minutes to 4 hours before anticipated sexual activity. Sildenafil does not cause an erection without sexual stimuli.

D. "This inhaler is not to be used alone to treat an acute asthma attack."

What statement by the patient, who is using the anticholinergic inhaler ipratropium bromide (Atrovent), indicates to the nurse that teaching has been successful? A. "I will increase my intake of caffeine." B. "I may gain weight as a result of taking this medication." C. "Nausea and vomiting are common adverse effects of this medication, so I will always take it with meals." D. "This inhaler is not to be used alone to treat an acute asthma attack."

C. tachycardia

When a patient is taking parenteral xanthine derivatives such as aminophylline, the nurse should monitor for which adverse effect? A. decreased respirations B. hypotension C. tachycardia D. hypoglycemia

The nurse recognizes that all patients with stable angina should take which drug? A) Aspirin B) Warfarin C) Ibuprofen D) Acetaminophen

a All patients with stable angina are encouraged to take 75 to 162 mg of aspirin daily to decrease platelet aggregation, which can lead to coronary occlusion.

NCLEX EXAMINATION REVIEW QUESTIONS 2. When evaluating the medication list of a patient who will be starting therapy with an H2 receptor antagonist, the nurse is aware that which drug may interact with it? a. codeine b. pencillin c. Phenytoin d. acetaminophen

c Phenytoin

A nursing student asks a nurse to explain the differences between amphotericin B [Abelcet] and the azoles group of antifungal agents. Which statement by the nurse is correct? a. "Amphotericin B can be given orally or intravenously." b. "Amphotericin B increases the levels of many other drugs." c. "Azoles have lower toxicity than amphotericin B." d. "Only the azoles are broad-spectrum antifungal agents."

c. "Azoles have lower toxicity than amphotericin B."

A nurse is teaching a male adult patient about the use of testosterone gel. Which statement by the patient indicates an understanding of the teaching? a. "I should apply this to my forearms and neck after showering." b. "I should keep treated areas exposed to the air so that they can dry." c. "I should not let my child touch the gel to prevent behavioral problems." d. "I should not swim or bathe for 3 to 4 hours after applying the gel."

c. "I should not let my child touch the gel to prevent behavioral problems."

To help with the insomnia associated with thyroid hormone replacement therapy, the nurse will teach the patient to a. take half the dose at lunchtime and the other half 2 hours later b. use a sedative to assist with falling asleep c. take the dose upon awakening in the morning d. reduce the dosage as needed if sleep is impaired

c. take the dose upon awakening in the morning

ANS: D Pancuronium is a competitive neuromuscular blocker and is used to paralyze skeletal muscles by neuromuscular blockade. The first muscles affected are those of the eyelids, so patients given this medication cannot open their eyes. Inability to move the extremities is an expected effect. Because these medications do not have central nervous system (CNS) effects, the patient is conscious and awake; therefore, caregivers should continue to talk to the patient while providing care. Atracurium, not pancuronium, can cause hypotension secondary to histamine release. High potassium levels would reduce paralysis. Patients with a history of MG would have increased paralysis. Nothing indicates that this patient is experiencing extreme effects from this medication.

A nurse is caring for an intubated patient who is receiving pancuronium for neuromuscular blockade. The patient's eyes are closed, and the patient is not moving any extremities. The heart rate is 76 beats per minute, and the blood pressure is 110/70 mm Hg. The nurse caring for this patient will do what? a.Request an order for an antihistamine to prevent a further drop in blood pressure. b.Request an order for serum electrolytes to evaluate for hyperkalemia. c.Review the patient's chart for a history of myasthenia gravis (MG). d.Talk to the patient while giving care and explain all procedures

1. A patient with a Pseudomonas species urinary infection will be receiving amikacin (Amikin) 15 mg/kg once daily via intravenous infusion. The patient weighs 198 pounds, and the medication is available in an injection solution strength of 250 mg/mL. Identify how many milliliters of medication will be drawn up for this injection. (record answer using one decimal place) _______

ANS: DIF: COGNITIVE LEVEL: Applying (Application) REF: N/A TOP: NURSING PROCESS: Implementation

1. The nurse is giving morning medications. The Medication Administration Record has an order for levothyroxine, 75 mcg PO. The drug-dispensing cabinet contains levothyroxine tablets in milligram strengths instead of micrograms. Calculate the milligram equivalent dose of 75 mcg. _______

ANS: DIF: COGNITIVE LEVEL: Applying (Application) REF: N/A TOP: NURSING PROCESS: Implementation

1. The order reads, "Give 1500 mL of normal saline over 12 hours. The tubing drop factor is 15 gtt/mL." The nurse will set the gravity drip infusion at how many drops per minute (gtt/min). _______

ANS: DIF: COGNITIVE LEVEL: Applying (Application) REF: N/A TOP: NURSING PROCESS: Implementation

Loop diuretics drug effects

--Potent diuresis and subsequent loss of fluid --Decreased fluid volume causes a reduction in: -Blood pressure -Pulmonary vascular resistance -Systemic vascular resistance -Central venous pressure -Left ventricular end-diastolic pressure --Potassium and sodium depletion

Loop diuretics indications

-Edema associated with HF or hepatic or renal disease -To control hypertension -To increase renal excretion of calcium in patients with hypercalcemia -In cases of HF resulting from diastolic dysfunction

Osmotic diuretics MOA

-works mostly in proximal tubule -non absorbable, producing an osmotic effect -pull water into the renal tubules from surrounding tissues -inhibit tubular resorption of water and solutes, thus producing rapid diuresis

The order reads: "Give ocreotide (Sandostatin) 50 mcg subcut twice a day." The medication is available in an injectable form of 0.05 mg/mL. How many milliliters will the nurse draw up for the ordered dose?

1 mL

30. Adverse effects for adrenal drugs of this system include: - Heart failure, cardiac edema, hypertension—all caused by electrolyte imbalances (hypokalemia, hypernatremia) A. Cardiovascular B. CNS C. Endocrine D. GI E. Integumentary F. Musculoskeletal G. Ocular H. Other

A

7. To prevent the development of oral candidiasis when using corticosteroid inhalers, which instruction is MOST important for the nurse to teach the patient? A. Rinse the mouth after each use. B. Minimize use of an inhaler to every other day. C. Swish and swallow with Mycostatin after each use. D. Report any gingival irritation to the health care provider.

A

ANS: C Excessive use of sodium bicarbonate may lead to systemic alkalosis. The other options are incorrect.

A 75-year-old woman comes into the clinic with complaints of muscle twitching, nausea, and headache. She tells the nurse that she has been taking sodium bicarbonate 5 or 6 times a day for the past 3 weeks. The nurse will assess for which potential problem that may occur with overuse of sodium bicarbonate? a.Constipation b.Metabolic acidosis c.Metabolic alkalosis d.Excessive gastric mucus

1. A patient will be taking fluconazole (Diflucan) 100 mg/day PO for 2 weeks. The patient is unable to swallow tablets, so an oral suspension that contains 10 mg/mL is available. Identify how many milliliters will the nurse administer with each dose. _______

ANS: 10 mL DIF: COGNITIVE LEVEL: Applying (Application) REF: N/A TOP: NURSING PROCESS: Implementation

A patient comes to the clinic and receives valacyclovir [Valtrex] for a herpes-zoster virus. The nurse instructs the patient to take the medication: a.without regard to meals. b.without any dairy products. c.each morning. d.on an empty stomach.

ANS: A The patient may take the medication without regard to meals. The patient does not need to avoid dairy products, take the pill only in the morning, or take it on an empty stomach.

9. The nurse is monitoring a patient who has severe bone marrow suppression following antineoplastic drug therapy. Which is considered a principal early sign of infection? a. Fever b. Diaphoresis c. Tachycardia d. Elevated white blood cell count

ANS: A Fever and/or chills may be the first sign of an oncoming infection. Elevated white blood cell count will not occur because of the bone marrow suppression. The other options are incorrect. DIF: COGNITIVE LEVEL: Applying (Application) REF: p. 729 TOP: NURSING PROCESS: Assessment

A patient calls a family planning clinic and tells the nurse that her vaginal ring, which has been in place for 2 weeks, came out sometime during the night while she was sleeping. The nurse will instruct her to: a. clean the ring with warm water, reinsert it, and use condoms for 7 days. b. discard the ring and insert a new ring after 1 week has passed. c. discard the ring and insert a new one to begin a new cycle. d. wash the ring in hot, soapy water and reinsert it.

ANS: A If a ring is expelled before 3 weeks have passed, it can be reinserted after being washed in warm water. If the ring has been out longer than 3 hours, backup contraception should be used for 7 days. Because this ring was expelled while the patient was sleeping, using backup contraception is prudent. Discarding the ring is not indicated if it can be cleaned. The ring should not be washed in hot water.

5. A patient is recovering from a myocardial infarction but does not have symptoms of heart failure. The nurse will expect to teach this patient about: a. ACE inhibitors and beta blockers. b. biventricular pacemakers. c. dietary supplements and exercise. d. diuretics and digoxin.

ANS: A This patient is classified as having Stage B heart failure with no current symptoms but with structural heart disease strongly associated with the development of heart failure. Treatment at this stage includes an ACE inhibitor and a beta blocker to help prevent the progression of symptoms. Biventricular pacemakers are used for patients in Stage C heart failure and have more advanced structural disease and symptoms. Dietary supplements and exercise have not been proven to prevent structural heart disease. Diuretics and digoxin are used for patients with Stage C heart failure.

Which are adverse effects of quinine [Qualaquin]? (Select all that apply.) a. Cinchonism b. Hemolysis in patients with G6PD deficiency c. Increased ventricular rate d. Nightmares e. Prolonged QT interval

ANS: A,B,C Quinine can cause cinchonism, hemolysis in susceptible patients, and an increased ventricular rate. Mefloquine can cause nightmares and a prolonged QT interval.

A nurse is teaching a group of nursing students about influenza prevention. Which statement by a student indicates understanding of the teaching? a."I may develop a mild case of influenza if I receive the vaccine by injection." b."I should receive the vaccine every year in October or November." c."If I have a cold I should postpone getting the vaccine." d."The antiviral medications are as effective as the flu vaccine for preventing the flu."

ANS: B Influenza vaccine should be given every year in October or November. The vaccine will not cause influenza. Minor illnesses, such as a cold, are not a contraindication for receiving the vaccine. Antiviral medications are not as effective as the flu vaccine in preventing influenza.

1. A patient must be treated immediately for acute organ transplant rejection. The nurse anticipates that muromonab-CD3 (Orthoclone OKT3) will be ordered. What is the priority assessment before beginning drug therapy with muromonab-CD3? a. Serum potassium level b. Fluid volume status c. Electrocardiogram d. Blood glucose level

ANS: B Assess fluid volume status because muromonab-CD3 is contraindicated in the presence of fluid overload. The other options are incorrect. DIF: COGNITIVE LEVEL: Applying (Application) REF: p. 777 TOP: NURSING PROCESS: Assessment

What are the effects of prolactin secretion in males? (Select all that apply.) a.Breast development b.Decreased libido c.Delayed puberty d.Galactorrhea e.Infertility

ANS: B, C, D Prolactin can cause decreased libido, delayed puberty, and galactorrhea in males, but it does not cause breast development or infertility.

2. A patient with a history of angina will be started on ranolazine (Ranexa). The nurse is reviewing the patient's history and will note potential contraindications to this drug therapy if which condition is present? (Select all that apply.) a. Type 2 diabetes mellitus b. Prolonged QT interval on the electrocardiogram c. Heart failure d. Closed-angle glaucoma e. Decreased liver function

ANS: B, E Ranolazine is contraindicated in patients with pre-existing QT prolongation or hepatic impairment. The other options are not contraindications. DIF: COGNITIVE LEVEL: Applying (Application) REF: p. 370 TOP: NURSING PROCESS: Assessment

A patient receives topical atropine to facilitate an eye examination. The nurse will tell the patient to remain in a darkened room or to wear sunglasses for several hours until the effects of the medication wear off. This teaching is based on the nurse's knowledge that muscarinic antagonists cause: a.elevation of intraocular pressure. b.miosis and ciliary muscle contraction. c.paralysis of the iris sphincter. d.relaxation of ciliary muscles.

ANS: C By blocking muscarinic receptors in the eye, atropine causes paralysis of the iris sphincter, which prevents constriction of the pupil; consequently, the eye cannot adapt to bright light. This also causes an elevation in intraocular pressure, which increases the risk of glaucoma. However, it is not an indication for wearing darkened glasses. Muscarinic agonists cause miosis; atropine causes mydriasis. The effect of relaxing ciliary muscles focuses the eye for far vision, causing blurred vision.

4. A nurse checks a patient's vital signs in the hospital and notes a blood pressure of 146/98 mm Hg. What will the nurse do? a. Instruct the patient to consume a low-sodium diet. b. Prepare the patient for an electrocardiogram and blood tests. c. Recheck the patient's blood pressure in the other arm. d. Request an order for a thiazide diuretic.

ANS: C Diagnosis of hypertension should be based on several blood pressure readings, not just one. High readings should be confirmed in the contralateral arm. Low-sodium diets are indicated for patients with confirmed hypertension. An electrocardiogram and blood tests are indicated for patients with confirmed hypertension to rule out primary causes. Thiazide diuretics are first-line drugs for confirmed hypertension.

15. A patient who has type 2 diabetes will begin taking glipizide [Glucotrol]. Which statement by the patient is concerning to the nurse? a. "I will begin by taking this once daily with breakfast." b. "It is safe to drink grapefruit juice while taking this drug." c. "I may continue to have a glass of wine with dinner." d. "I will need to check my blood sugar once daily or more."

ANS: C Glipizide is a sulfonylurea antidiabetic agent and can cause a disulfiram-like reaction when combined with alcohol. Patients should be taught to avoid alcohol while taking this medication. The initial dosing is once daily with breakfast. There is no drug interaction with grapefruit juice. Patients will need to monitor their blood glucose.

3. The nurse is reviewing the use of anticholinergic drugs. Anticholinergic drugs block the effects of which nervous system? a. Central nervous system b. Somatic nervous system c. Sympathetic nervous system d. Parasympathetic nervous system

ANS: D Anticholinergic drugs block or inhibit the actions of acetylcholine in the parasympathetic nervous system. The other options are incorrect. DIF: COGNITIVE LEVEL: Remembering (Knowledge) REF: p. 331 TOP: NURSING PROCESS: General

7. The nurse is reviewing discharge teaching for a patient who will be taking digoxin (Lanoxin) therapy. The nurse will teach the patient to avoid which foods when taking the digoxin? a. Leafy green vegetables b. Dairy products c. Grapefruit juice d. Bran muffins

ANS: D Bran, in large amounts, may decrease the absorption of oral digitalis drugs. The other foods do not affect digoxin levels. DIF: COGNITIVE LEVEL: Understanding (Comprehension) REF: p. 385 TOP: NURSING PROCESS: Implementation

2. Which cephalosporin may be used to treat meningitis? a. Cefaclor b. Cefazolin c. Cefoxitin d. Cefotaxime

ANS: D Cefotaxime has increased ability to reach the cerebrospinal fluid (CSF) and to treat meningitis. Cefaclor, cefazolin, and cefoxitin do not reach effective concentrations in the CSF.

2. A patient has a prescription for oxybutynin (Ditropan), an anticholinergic drug. When reviewing the patient's medical history, which condition, if present, would be considered a contraindication to therapy with this drug? a. Diarrhea b. Hypertension c. Neurogenic bladder d. Uncontrolled angle-closure glaucoma

ANS: D Contraindications include drug allergy, urinary or gastric retention, and uncontrolled angle-closure glaucoma. Neurogenic bladder is an indication for oxybutynin. The other options are incorrect. DIF: COGNITIVE LEVEL: Applying (Application) REF: p. 334 TOP: NURSING PROCESS: Assessment

3. A patient has had an overdose of an intravenous cholinergic drug. The nurse expects to administer which drug as an antidote? a. Atenolol (Tenormin) b. Bethanechol (Urecholine) c. Dobutamine d. Atropine sulfate

ANS: D Prompt administration of atropine sulfate can reverse a toxic dose of cholinergic drugs. The other drugs listed are not antidotes to cholinergic toxicity. DIF: COGNITIVE LEVEL: Understanding (Comprehension) REF: p. 324 TOP: NURSING PROCESS: Assessment

11. A nurse prepares to administer a scheduled dose of digoxin. The nurse finds a new laboratory report showing a plasma digoxin level of 0.7 ng/mL. What action should the nurse take? a. Withhold the drug for an hour and reassess the level. b. Withhold the drug and notify the prescriber immediately. c. Administer Digibind to counteract the toxicity. d. Check the patient's apical pulse, and if it is within a safe range, administer the digoxin.

ANS: D The optimum plasma digoxin range is 0.5 to 0.8 ng/mL. The patient's pulse should be checked before administration, as always, and the digoxin should be administered. The digoxin does not have to be withheld, nor does the prescriber need to be notified. If the digoxin level is demonstrating a trend of increasing, the issue should be discussed in rounds. Digibind is not indicated, because the digoxin level is therapeutic.

9. What action is often recommended to help reduce tolerance to transdermal nitroglycerin therapy? a. Omit a dose once a week. b. Leave the patch on for 2 days at a time. c. Cut the patch in half for 1 week until the tolerance subsides. d. Remove the patch at bedtime, and then apply a new one in the morning.

ANS: D To prevent tolerance, remove the transdermal patch at night for 8 hours, and apply a new patch in the morning. Transdermal patches must never be cut or left on for 2 days, and doses must not be omitted. DIF: COGNITIVE LEVEL: Applying (Application) REF: p. 365 TOP: NURSING PROCESS: Implementation

Textbook Case Studies A 50-year-old attorney has self-treated for heartburn for years by drinking large amounts of antacids. She finally made an appointment with her family practice physician, who referred her to a gastroenterologist. Her family practice physician instructed her to stop taking the antacids. In a few weeks, the attorney had an endoscopy, and it was discovered that she had gastroesophgeal reflux disease (GERD) and gastritis secondary to stress-induced hyper-acidity. The gastoenterologist has prescribed the proton pump inhibitor (PPI) omeprazole (Prilosec) 20 mg once a day. 4. What patient teaching is important regarding the PPI?

Administer omeprazole before meals, and take the capsule whole and not crushed, opened, or chewed. It needs to be taken on an empty stomach, 30 to 60 minutes before breakfast, with a full glass of water.

A. expected adverse effect of this medication

After a nebulizer treatment with the beta agonist, albuterol, the patient complains of feeling a little shaky with slight hand tremors. The patient's heart rate is 98 BPM increased from the pretreatment rate of 88BPM. The nurse knows that this reaction is an? A. expected adverse effect of this medication B. allergic reaction to this medication C. indication that he has received an overdose of the medication D. idiosyncratic reaction to this medication

B. rinse out the mouth with water

After the patient takes a dose of an inhaled corticosteroid, such as fluticasone (flovent), what is the most important action the patient needs to do next? A. hold the breath for 60 seconds B. rinse out the mouth with water C. follow the corticosteroid with a bronchodilator inhaler, if ordered D. repeat the dose in 15 minutes if the patient feels short of breath

ANS: A Sometimes H2 receptor antagonists such as cimetidine may cause adverse effects related to the central nervous system in the elderly, including confusion and disorientation. The nurse needs to be alert for mental status changes when giving these drugs, especially if the changes are new to the patient.

An elderly patient had gastric surgery due to a gastrointestinal bleed 3 days ago, and he has been stable since the surgery. This evening, his daughter tells the nurse, "He seems to be more confused this afternoon. He's never been like this. What could be the problem?" The nurse reviews the patient's medication record and suspects that which drug could be the cause of the patient's confusion? a.cimetidine (Tagamet) b.pantoprazole (Protonix) c.clarithromycin (Biaxin) d.sucralfate (Carafate)

8. The nurse knows that which immunosuppressants are among the most effective? a. Azathioprine [Imuran] and everolimus [Zortress] b. Cyclosporine [Sandimmune] and tacrolimus [Prograf] c. Methotrexate [Rheumatrex] and muromonab-CD3 [Orthoclone OKT3] d. Sirolimus [Rapamune] and methylprednisolone

B Cyclosporine and tacrolimus are the most effective immunosuppressants available.

15. Glucocorticoids are: A. Topical B. Inhaled C. Nasal D. Systemic E. All of the above

E

D. improvement should be seen within a week of use

For patients taking a leukotriene receptor antagonist, the nurse should include which information in the patient teaching? A. if a dose is missed, the patient may take a double dose to maintain blood levels B. the patient should gargle or rinse the mouth after using the inhaler C. the medication should be taken at the first sign of bronchospasm D. improvement should be seen within a week of use

19. Mineralocorticoids include: A. Beclomethasone (several formulations) B. Fluticasone propionate C. Hydrocortisone (several formulations) D. Cortisone E. Methylprednisolone F. Prednisone G. Fludrocortisone acetate H. Aminoglutethimide

G

36. Adverse effects for adrenal drugs of this system include: - Increased intraocular pressure, glaucoma A. Cardiovascular B. CNS C. Endocrine D. GI E. Integumentary F. Musculoskeletal G. Ocular H. Other

G

B. albuterol (Proventil)

One of the attendees expresses concern regarding her granddaughter's asthma. The attendee tells the nurse that she is afraid that she will not know which of her granddaughter's medications to give first in case of an asthma attack. Which medication should the nurse inform the attendee to administer first for an acute asthma attack? A. ipratropium (Atrovent) B. albuterol (Proventil) C. budesonide (Pulmicort Turbuhaler) D. montelukast (Singulair)

A. Step 1

One of the attendees tells the nurse that he has asthma and is being treated with a short-acting inhaled beta2 agonist. The nurse identifies this treatment as which step of the stepwise therapy for the treatment of asthma? A. Step 1 B. Step 2 C. Step 3 D. Step 4

B. the medications only treat the symptoms.

The common cold is treated with empiric therapy, which means: A. the medications cure the cold. B. the medications only treat the symptoms. c. herbal medications are useful to eliminate symptoms. D. it is prevented with careful use of medications.

B. drink extra fluid, unless contraindicated, to aid in expectoration of sputum

The nurse is administering an expectorant and will provide which teaching? A. avoid fluids for 30-45 min after the dose B. drink extra fluid, unless contraindicated, to aid in expectoration of sputum C. avoid driving or operating heavy machinery while taking this med D. expect secretions to become thicker

A. Antihistamines B. Nasal decongestants C. Antitussives D. Expectorants

The nurse is discussing management of symptoms of an upper respiratory tract infection. Which classes of medications are often used in treating the symptoms of upper respiratory tract infections? (Select all that apply.) A. Antihistamines B. Nasal decongestants C. Antitussives D. Expectorants E. Antibiotics

Interactions: Antithyroid Drugs

additive leukopenic effects when taken in conjunction with other bone marrow depressants and increase in activity of oral anticoagulants

A nurse is caring for a patient who has a superficial fungal infection and notes that the provider has ordered 200 mg of ketoconazole [Nizoral] 200 mg PO once daily. Which action by the nurse is correct? a. Administer the drug as ordered. b. Contact the provider to discuss twice daily dosing. c. Discuss a 400-mg daily drug regimen with the provider. d. Request an order for a different antifungal medication.

d. Request an order for a different antifungal medication.

Which of the following is not an adverse effect of itraconazole? a. Cardiosuppression b. Hepatotoxicity c. Nausea, vomiting, diarrhea d. Visual disturbances

d. Visual disturbances

Adverse Effects: Thyroid drugs

usually due to overdose; most significant is cardiac dysrhythmia with the risk of life-threatening or fatal irregularities, tachycardia, palpitations, angina, hypertension, insomnia, tremors, anxiety, menstrual irregularities, weight loss, sweating, heart intolerance, fever

Mechanism of Action: Thyroid Drugs

work in same manner as endogenous thyroid hormones, affecting many body systems; cellular level-induce the metabolic rate like the rate of protein, carbohydrate, and lipid metabolism, increase oxygen consumption, body temperature, blood volume, and overall cellular growth and differentiation. stimulate cardiovascular system by increasing number of myocardial beta-adrenergic receptors - increases sensitivity of heart to catecholamines and increases cardiac output. also increase renal blood flow and glomerular filtration rate that results in diuretic effect

Osmotic diuretics indications

-Treatment of patients in the early, oliguric phase of acute renal failure (ARF) -To promote excretion of toxic substances -To reduce intracranial pressure -Treatment of cerebral edema -NOT INDICATED FOR PERIPHERAL EDEMA

Which application instruction is the priority for a nurse to provide to a patient starting to use AndroGel testosterone gel? "Apply the gel to clean, dry skin of the shoulders or upper arms." "Squeeze the entire packet into your palms and then rub the gel into your skin." "Wait about 5 or 6 hours before showering or swimming." "Wash your hands after applying the gel to prevent transfer to others."

"Wash your hands after applying the gel to prevent transfer to others."

D. Longer duration of action

A nurse is providing education to a patient taking two different medications. The nurse identifies which characteristic as the advantage of salmeterol (Serevent) over other beta2 agonists such as albuterol (Proventil)? A. Shorter onset of action B. Better side effect profile C. Quicker peak action D. Longer duration of action

B. sinus tachyardia

A patient has been receiving aminophylline (xanthine derivative) infusion for 24 hours. The nurse will assess for which adverse effect when assessing the patient during the infusion? A. CNS depression B. sinus tachyardia C. increased appetite D. temporary urinary retention

The laboratory calls the nursing unit to report a drop in the platelet count to 90,000/mm3 for a patient receiving heparin for the treatment of postoperative deep vein thrombosis. Which action by the nurse is the most appropriate? A. Call the healthcare provider to discuss the reduction or withdrawal of heparin. B. Call the healthcare provider to discuss increasing the heparin dose to achieve a therapeutic level. C. Obtain vitamin K and prepare to administer it by intramuscular (IM) injection. D. Observe the patient and monitor the activated partial thromboplastin time (aPTT) as indicated.

A. Call the healthcare provider to discuss the reduction or withdrawal of heparin. Heparin-induced thrombocytopenia (HIT) is a potential immune-mediated adverse effect of heparin infusions that can prove fatal. HIT is suspected when the platelet counts fall significantly. A platelet count below 100,000/mm3 would warrant discontinuation of the heparin.

What would the nurse assess when monitoring for the therapeutic effectiveness of vasopressin? A. Fluid balance B. Patient's pain scale C. Serum albumin levels D. Adrenocorticotropic hormone (ACTH) levels

A. Fluid balance Vasopressin causes decreased water excretion in the renal tubule, thus decreasing urine output. It is used to treat diabetes insipidus, which presents with polyuria and dehydration.

When teaching a patient regarding desmopressin (DDAVP), the nurse will inform the patient to monitor for which potential side effects? (Select all that apply.) A. Headache B. Weight gain C. Nasal irritation D. Hyperglycemia E. Hypotension

A. Headache Desmopressin works to decrease urine output; thus the patient would retain fluid and gain weight. Headache may also occur as a sequela of fluid retention. Because it is administered intranasally, it can be irritating; thus nostrils should be rotated. Desmopressin does not affect serum glucose levels.

1. A patient will be receiving monthly injections of cyanocobalamin (Nascobal). The dose is 100 mcg/month IM. The medication is available in a strength of 1000 mcg/mL. Identify how many milliliters will the nurse draw up into the syringe. (record answer using one decimal place) _______

ANS: 0.1 mL DIF: COGNITIVE LEVEL: Applying (Application) REF: N/A TOP: NURSING PROCESS: Implementation

4. The nurse assesses a newly diagnosed patient for short-term complications of diabetes. What does this assessment include? a. Evaluation for hyperglycemia, hypoglycemia, and ketoacidosis b. Cranial nerve testing for peripheral neuropathy c. Pedal pulse palpation for arterial insufficiency d. Auscultation of the carotids for bruits associated with atherosclerosis

ANS: A High blood sugar, low blood sugar, and ketoacidosis are short-term complications of diabetes. Microvascular and macrovascular complications, such as peripheral neuropathy, are long-term complications of diabetes. Arterial insufficiency and atherosclerosis also are long-term complications of diabetes.

8. An older adult patient with congestive heart failure develops crackles in both lungs and pitting edema of all extremities. The physician orders hydrochlorothiazide [HydroDIURIL]. Before administering this medication, the nurse reviews the patient's chart. Which laboratory value causes the nurse the most concern? a. Elevated creatinine clearance b. Elevated serum potassium level c. Normal blood glucose level d. Low levels of low-density lipoprotein (LDL) cholesterol

ANS: A Hydrochlorothiazide should not be given to patients with severe renal impairment; therefore, an elevated creatinine clearance would cause the most concern. Thiazide diuretics are potassium-wasting drugs and thus may actually improve the patient's potassium level. Thiazides may elevate the serum glucose level in diabetic patients. Thiazides increase LDL cholesterol; however, this patient's levels are low, so this is not a risk.

8. In assessing a patient before administration of a cardiac glycoside, the nurse knows that which lab result can increase the toxicity of the drug? a. Potassium level 2.8 mEq/L b. Potassium level 4.9 mEq/L c. Sodium level 140 mEq/L d. Calcium level 10 mg/dL

ANS: A Hypokalemia increases the chance of digitalis toxicity. The other levels listed are incorrect. DIF: COGNITIVE LEVEL: Analyzing (Analysis) REF: p. 384 TOP: NURSING PROCESS: Assessment

A patient who is being treated for HIV infection has a 5-mm area of induration after a routine TST. The patient's chest radiograph is normal, and there are no other physical findings. The nurse will expect this patient to begin treatment with which drugs? a. Isoniazid and rifabutin b. Isoniazid and rifampin c. Isoniazid and rifapentine d. Isoniazid and pyrazinamide

ANS: A Rifabutin is used off-label as an alternative to rifampin to treat TB in patients with HIV, because it has less impact on the metabolism of protease inhibitors. The effects of rifapentine on protease inhibitors is similar to those of rifampin. Pyrazinamide is not indicated.

10. The potassium-sparing diuretic spironolactone [Aldactone] prolongs survival and improves heart failure symptoms by which mechanism? a. Blocking aldosterone receptors b. Increasing diuresis c. Reducing venous pressure d. Reducing afterload

ANS: A Spironolactone prolongs survival in patients with HF primarily by blocking receptors for aldosterone. Spironolactone cause only minimal diuresis. It does not reduce afterload, and it does not reduce venous pressure enough to prolong survival, because it causes only minimal diuresis.

2. A patient will be starting therapy with a corticosteroid. The nurse reviews the patient's orders and notes that an interaction may occur if the corticosteroid is taken with which of these drug classes? a. Nonsteroidal anti-inflammatory drugs b. Antibiotics c. Opioid analgesics d. Antidepressants

ANS: A The use of corticosteroids with aspirin, other NSAIDs, and other ulcerogenic drugs produces additive gastrointestinal effects and an increased chance for the development of gastric ulcers. The other options are incorrect. DIF: COGNITIVE LEVEL: Understanding (Comprehension) REF: p. 526 TOP: NURSING PROCESS: Assessment

While performing an admission assessment on a patient, the nurse learns that the patient is taking furosemide [Lasix], digoxin, and spironolactone [Aldactone]. A diet history reveals the use of salt substitutes. The patient is confused and dyspneic and complains of hand and foot tingling. Which is an appropriate nursing action for this patient? a. Contact the provider to request orders for an electrocardiogram and serum electrolyte levels. b. Evaluate the patient's urine output and request an order for intravenous potassium. c. Hold the next dose of furosemide and request an order for intravenous magnesium sulfate. d. Request an order for intravenous insulin to help this patient regulate extracellular potassium.

ANS: A This patient is taking a potassium-sparing diuretic and is ingesting dietary potassium in salt substitutes. The patient shows signs of hyperkalemia: confusion, shortness of breath, and tingling of the hands and feet. The cardiac effects can be fatal, especially if a patient also is taking digoxin. The most appropriate first action would be to obtain an ECG and serum electrolyte levels to evaluate the extent of the patient's hyperkalemia. The patient does not need increased potassium. Furosemide is a potassium-wasting diuretic and would be helpful. Magnesium sulfate is not indicated. Until the patient's status has been assessed further, intravenous insulin is not indicated.

Which antibiotics may be administered topically? (Select all that apply.) a. Bacitracin b. Daptomycin c. Ofloxacin d. Polymyxin B e. Rifampin

ANS: A, D Bacitracin and Polymyxin B are both topical antibiotics. Daptomycin, ofloxacin, and rifampin are not formulated to be administered topically.

A nursing student asks a nurse why tuberculosis resistance is so high. Which reasons are correct? (Select all that apply.) a. Adherence to drug regimens is erratic. b. Drug doses are often too low. c. Many hosts are immunocompromised. d. Many regimens contain too few drugs. e. Some TB drugs are bacteriostatic.

ANS: A,B,D There are many reasons for the emergence of drug-resistant organisms in the treatment of TB. They include erratic patient adherence, drug doses that are too low, and drug regimens with too few drugs. The immunocompetence of the host does not affect drug resistance, nor does the fact that some agents are bacteriostatic.

A patient has a positive test for influenza type A and tells the nurse that symptoms began 5 days before being tested. The prescriber has ordered oseltamivir [Tamiflu]. The nurse will tell the patient that oseltamivir: a.may decrease symptom duration by 2 or 3 days. b.may not be effective because of the delay in starting treatment. c.may reduce the severity but not the duration of symptoms. d.will alleviate symptoms within 24 hours of the start of therapy.

ANS: B Oseltamivir is most effective when begun within 2 days after symptom onset. When started within 12 hours of symptom onset, it may decrease duration of symptoms by 2 to 3 days. The drug reduces both symptom severity and symptom duration when used in a timely fashion. It does not rapidly alleviate symptoms.

2. A child with seasonal rhinitis has used budesonide [Rhinocort Aqua] for several years. The parents are concerned that the child's rate of growth has slowed. What will the nurse do? a. Reassure the parents that this is an expected side effect. b. Suggest that the parents discuss using fluticasone [Flonase] with the provider. c. Tell the parents to administer the drug only when symptoms are severe. d. Tell the parents that antihistamines work as well as intranasal glucocorticoids.

ANS: B A worrisome systemic effect of intranasal glucocorticoids is suppression of linear growth in children. Although rare, it can occur; however, it is less likely with fluticasone and mometasone, so these two preparations are better options for children. Reassuring parents that this is an expected side effect is incorrect. Intranasal glucocorticoids should be given daily and not as needed. Antihistamines are not as effective as glucocorticoids, because antihistamines work only against one mediator of allergic inflammation.

10. A patient who has been taking antihypertensive drugs for a few months complains of having a persistent dry cough. The nurse knows that this cough is an adverse effect of which class of antihypertensive drugs? a. Beta blockers b. Angiotensin-converting enzyme (ACE) inhibitors c. Angiotensin II receptor blockers (ARBs) d. Calcium channel blockers

ANS: B ACE inhibitors cause a characteristic dry, nonproductive cough that reverses when therapy is stopped. The other drug classes do not cause this cough. DIF: COGNITIVE LEVEL: Understanding (Comprehension) REF: p. 349 TOP: NURSING PROCESS: Assessment

9. The nurse is reviewing the mechanism of action of antidiarrheal drugs. Which type of antidiarrheal medication works by decreasing the intestinal muscle tone and peristalsis of the intestines? a. Adsorbents such as Pepto-Bismol b. Anticholinergics such as belladonna alkaloids c. Probiotics such as Lactinex d. Lubricants such as mineral oil

ANS: B Anticholinergic drugs work to slow peristalsis by reducing the rhythmic contractions and the smooth muscle tone of the gastrointestinal tract. The other options are incorrect. DIF: COGNITIVE LEVEL: Understanding (Comprehension) REF: p. 815 TOP: NURSING PROCESS: Planning

7. A patient is in the urgent care center after experiencing a black widow spider bite. The nurse prepares to give which product to treat this injury? a. Live vaccine b. Antivenins or antisera c. Tetanus immune globulin d. Active immunizing drug

ANS: B Antivenins, also known as antisera, are used to prevent or minimize the effects of poisoning by poisonous snakes and spiders. They provide the person who has been bitten with the substance needed to overcome the effects of the venom. DIF: COGNITIVE LEVEL: Understanding (Comprehension) REF: p. 786 TOP: NURSING PROCESS: Implementation

15. A nursing student asks a nurse how digoxin causes dysrhythmias. The nurse correctly states that digoxin: a. reduces automaticity in the AV node. b. increases automaticity in the Purkinje fibers. c. increases automaticity in the SA node. d. speeds up AV conduction.

ANS: B Digoxin increases automaticity in the Purkinje fibers, which contributes to dysrhythmias caused by digoxin. Decreased automaticity in the AV node is a desired effect of digoxin. Digoxin does not increase automaticity in the SA node. It does not increase AV node conduction.

4. A patient who has experienced preterm births with her previous two pregnancies will begin receiving hydroxyprogesterone caproate [Makena] to prevent preterm delivery with her current pregnancy. Which statement by the patient indicates a need for further teaching about this drug? a. "I should report any calf pain immediately." b. "I will eventually learn to administer the injections of this drug myself." c. "This drug may cause fluid retention and glucose intolerance." d. "This medication must be given once a week until 37 weeks' gestation."

ANS: B Hydroxyprogesterone caproate is given IM and must be given by a healthcare provider and not by the patient. Although serious side effects are rare, thrombosis and thromboembolism may occur, so the patient should report signs of thrombosis immediately. Fluid retention and glucose intolerance may occur. The medication is given weekly from 16 to 21 weeks' gestation up to 37 weeks or delivery of the baby. PTS: 1 DIF: Cognitive Level: Application REF: pp. 780-781

A patient is about to begin treatment for latent tuberculosis. The patient is an alcoholic, has difficulty complying with drug regimens, and has mild liver damage. What will the nurse tell this patient? a. "You must stop drinking before adequate treatment can begin." b. "You must take isoniazid with close monitoring of hepatic function." c. "You must take rifampin daily for 4 months." d. "You will begin a regimen of isoniazid and rifampin."

ANS: B Isoniazid has an increased incidence of hepatotoxicity, especially when given with alcohol. Patients who consume alcohol or who have liver damage should receive isoniazid with caution and should have close monitoring of liver function. It is unrealistic to ask the alcoholic to stop drinking to undergo treatment. Rifampin is toxic to the liver, especially in alcoholics. Giving both drugs would only increase the risk of hepatotoxicity.

A nurse is caring for a postoperative patient who has a nasogastric tube with continuous suction. The nurse notes that the patient has shallow respirations and suspects that this patient has developed: a.metabolic acidosis. b.metabolic alkalosis. c.respiratory acidosis. d.respiratory alkalosis.

ANS: B Metabolic alkalosis occurs with excessive loss of acid, such as gastric acid, or an excessive increase in alkalinizing salts. The body compensates for metabolic alkalosis by hypoventilating in an attempt to retain CO2. Metabolic acidosis is usually caused by ingestion of acids or excessive loss of bicarbonate and is compensated by hyperventilation. Respiratory acidosis results from hypoventilation. Respiratory alkalosis is the result of hyperventilation.

10. A patient is taking nystatin (Mycostatin) oral lozenges to treat an oral candidiasis infection resulting from inhaled corticosteroid therapy for asthma. Which instruction by the nurse is appropriate? a. "Chew the lozenges until they are completely dissolved." b. "Let the lozenge dissolve slowly and completely in your mouth without chewing it." c. "Rinse your mouth with water before taking the inhaler." d. "Rinse your mouth with mouthwash after taking the inhaler."

ANS: B Nystatin may be given orally in the form of lozenges, or troches, which need to be slowly and completely dissolved in the mouth for optimal effects; tablets are not to be chewed or swallowed whole. The other options are incorrect. Patients taking an inhaled corticosteroid must rinse their mouths with water thoroughly after taking the inhaler. DIF: COGNITIVE LEVEL: Applying (Application) REF: p. 676 TOP: NURSING PROCESS: Planning

7. A patient is receiving heparin therapy as part of the treatment for a pulmonary embolism. The nurse monitors the results of which laboratory test to check the drug's effectiveness? a. Bleeding times b. Activated partial thromboplastin time (aPTT) c. Prothrombin time/international normalized ratio (PT/INR) d. Vitamin K levels

ANS: B Ongoing aPTT values are used to monitor heparin therapy. PT/INR is used to monitor warfarin therapy. The other two options are not used to monitor anticoagulant therapy. DIF: COGNITIVE LEVEL: Understanding (Comprehension) REF: p. 432 TOP: NURSING PROCESS: Evaluation

4. During a teaching session for a patient on antithyroid drugs, the nurse will discuss which dietary instructions? a. Using iodized salt when cooking b. Avoiding foods containing iodine c. Restricting fluid intake to 2500 mL/day d. Increasing intake of sodium- and potassium-containing foods

ANS: B Patients on antithyroid therapy need to avoid iodine-containing foods. These foods may interfere with the effectiveness of the antithyroid drug. The other options are incorrect. DIF: COGNITIVE LEVEL: Applying (Application) REF: p. 496 TOP: NURSING PROCESS: Implementation

10. A patient receiving a cephalosporin develops a secondary intestinal infection caused by Clostridium difficile. What is an appropriate treatment for this patient? a. Adding an antibiotic, such as vancomycin [Vancocin], to the patient's regimen b. Discontinuing the cephalosporin and beginning metronidazole [Flagyl] c. Discontinuing all antibiotics and providing fluid replacement d. Increasing the dose of the cephalosporin and providing isolation measures

ANS: B Patients who develop C. difficile infection (CDI) as a result of taking cephalosporins or other antibiotics need to stop taking the antibiotic in question and begin taking either metronidazole or vancomycin. Adding one of these antibiotics without withdrawing the cephalosporin is not indicated. CDI must be treated with an appropriate antibiotic, so stopping all antibiotics is incorrect. Increasing the cephalosporin dose would only aggravate the CDI.

A patient has developed muscarinic antagonist toxicity from ingestion of an unknown chemical. The nurse should prepare to administer which medication? a.Atropine [Sal-Tropine] IV b.Physostigmine [Antilirium] c.An acetylcholinesterase activator d.Pseudoephedrine [Ephedrine]

ANS: B Physostigmine is indicated for muscarinic antagonist toxicity. Atropine is a drying agent and would only complicate the drying action that arises from the muscarinic antagonist. An acetylcholinesterase activator would only contribute to dryness that arises from the muscarinic antagonist. Ephedrine is not indicated for muscarinic antagonist toxicity.

5. When monitoring a patient who is taking a systemically administered glucocorticoid, the nurse will monitor for signs of which condition? a. Dehydration b. Hypokalemia c. Hyponatremia d. Hypoglycemia

ANS: B Systemic glucocorticoid drugs may cause potassium depletion, hyperglycemia, and hypernatremia. The other options are incorrect. DIF: COGNITIVE LEVEL: Understanding (Comprehension) REF: p. 526 TOP: NURSING PROCESS: Evaluation

Two nurses are discussing the major differences between physostigmine [Antilirium] and neostigmine [Prostigmin]. One nurse correctly makes which statement about physostigmine [Antilirium]? a."It is not effective for treating poisoning by muscarinic blocking drugs." b."It can readily cross the blood-brain barrier." c."It does not cause any side effects." d."It can be given by all routes."

ANS: B The basic pharmacology of physostigmine is nearly identical to that of neostigmine, except that physostigmine readily crosses membranes, including the blood-brain barrier; neostigmine does not. Physostigmine and neostigmine are both effective for treating poisoning by muscarinic blocking drugs. Neither physostigmine nor neostigmine is devoid of side effects. Physostigmine can be given only intramuscularly (IM) or intravenously (IV); neostigmine is given orally (PO), IM, IV, and subcutaneously (subQ).

A patient who takes imipramine has a moderate infection with P. vivax that is chloroquine resistant. Which medication will the nurse expect the provider to order? a. Artemether/lumefantrine [Coartem] b. Atovaquone/proguanil [Malarone] c. Artesunate d. Quinine

ANS: B The combination of atovaquone and proguanil is highly effective for the prophylaxis and treatment of chloroquine-resistant malaria. Artemether/lumefantrine inhibits CYP2D6 and can raise the levels of drugs that are substrates for this enzyme, including imipramine. Artesunate is an investigational drug used for severe malaria. Quinine has many toxic side effects.

6. A nurse is working in an immunization clinic. A new colleague asks, "When is the first dose of the diphtheria, tetanus, and acellular pertussis (DtaP, Daptacel) given?" The nurse knows that this series is started at what age? a. At birth b. 6 weeks c. 3 months d. 1 year

ANS: B The first dose of the series of three injections is given at 6 weeks of age. The other options are incorrect. DIF: COGNITIVE LEVEL: Remembering (Knowledge) REF: p. 788 TOP: NURSING PROCESS: Assessment

10. After starting treatment for type 2 diabetes mellitus 6 months earlier, a patient is in the office for a follow-up examination. The nurse will monitor which laboratory test to evaluate the patient's adherence to the antidiabetic therapy over the past few months? a. Hemoglobin levels b. Hemoglobin A1C level c. Fingerstick fasting blood glucose level d. Serum insulin levels

ANS: B The hemoglobin A1C level reflects the patient's adherence to the therapy regimen for several months previously, thus evaluating how well the patient has been doing with diet and drug therapy. The other options are incorrect. DIF: COGNITIVE LEVEL: Applying (Application) REF: p. 517 TOP: NURSING PROCESS: Evaluation

A nurse is preparing to administer a dose of growth hormone and reconstitutes the medication. After adding the diluent, the nurse notices that the preparation is cloudy. What will the nurse do? a.Administer the drug as ordered. b.Discard the drug and prepare another dose. c.Notify the prescriber. d.Shake the drug to dissipate the particles.

ANS: B The medication should not be injected if the preparation is cloudy or contains particulate matter. The drug should be discarded and another dose drawn up and administered. There is no need to notify the prescriber. The preparation should never be shaken

5. A patient who does not consume alcohol or nicotine products reports a strong family history of hypertension and cardiovascular disease. The patient has a blood pressure of 126/82 and a normal weight and body mass index for height and age. The nurse will expect to teach this patient about: a. ACE inhibitors and calcium channel blocker medications. b. the DASH diet, sodium restriction, and exercise. c. increased calcium and potassium supplements. d. thiazide diuretics and lifestyle changes.

ANS: B This patient has prehypertension without other risk factors. Lifestyle changes are indicated at this point. If blood pressure rises to hypertension levels, other measures, including drug therapy, will be initiated. Calcium and potassium supplements are not indicated.

7. A nurse is caring for a patient in the intensive care unit who is receiving intravenous lidocaine. The patient is drowsy and confused and reports numbness of the fingers and toes. Which standing order will the nurse initiate at this time? a. Administer diazepam. b. Reduce the rate of infusion. c. Discontinue the infusion. d. Prepare for mechanical ventilation.

ANS: B This patient is showing signs that are common with high therapeutic levels of lidocaine. Because lidocaine is rapidly degraded, slowing the rate of infusion can help remove excess drug from the circulation. Seizures are possible with toxic doses; diazepam should be used to control seizures. It is not necessary to discontinue the infusion, because this patient is showing signs common to high therapeutic doses. Respiratory arrest is possible with toxic doses; mechanical ventilation may be needed.

3. A patient who has been anticoagulated with warfarin (Coumadin) has been admitted for gastrointestinal bleeding. The history and physical examination indicates that the patient may have taken too much warfarin. The nurse anticipates that the patient will receive which antidote? a. Vitamin E b. Vitamin K c. Protamine sulfate d. Potassium chloride

ANS: B Vitamin K is given to reverse the anticoagulation effects of warfarin toxicity. Protamine sulfate is the antidote for heparin overdose. The other options are incorrect. DIF: COGNITIVE LEVEL: Understanding (Comprehension) REF: p. 416 TOP: NURSING PROCESS: Implementation

1. The nurse is reviewing therapy with glucocorticoid drugs. Which conditions are indications for glucocorticoid drugs? (Select all that apply.) a. Glaucoma b. Cerebral edema c. Chronic obstructive pulmonary disease and asthma d. Organ transplantation e. Varicella f. Septicemia

ANS: B, C, D Cerebral edema, chronic obstructive pulmonary disease, asthma, and organ transplantation are indications for glucocorticoid therapy. Glaucoma, varicella, and septicemia are all contraindications to glucocorticoid therapy. DIF: COGNITIVE LEVEL: Applying (Application) REF: p. 525 TOP: NURSING PROCESS: Planning

1. During therapy with hematopoietic drugs, the nurse will monitor the patient for which adverse effects? (Select all that apply.) a. Hypotension b. Edema c. Diarrhea d. Black, tarry stools e. Nausea and vomiting f. Headache

ANS: B, C, E, F Potential adverse effects of hematopoietic drugs include edema, anorexia, nausea, vomiting, diarrhea, dyspnea, fever, and headache. See Table 47-1 for a complete list. The other options are not adverse effects of these drugs. DIF: COGNITIVE LEVEL: Applying (Application) REF: p. 752 TOP: NURSING PROCESS: Evaluation

A nursing student asks a nurse why pegylated interferon alfa is used instead of regular interferon for a patient with hepatitis C. The nurse will tell the student that pegylated interferon: a.decreases the need for additional medications. b.has fewer adverse effects than interferon. c.is administered less frequently than interferon. d.may be given orally to increase ease of use.

ANS: C Pegylated interferon alfa preparations are preferred because of their convenience and superior efficacy. These preparations may be given once weekly instead of three or more times per week like the regular interferon. Using pegylated interferons does not decrease the need for additional medications. Pegylated interferons have similar adverse effects. Pegylated interferons are not given orally.

6. The nurse is reviewing drug therapy for hypertension. According to the JNC-8 guidelines, antihypertensive drug therapy for a newly diagnosed hypertensive African-American patient would most likely include which drug or drug classes? a. Vasodilators alone b. ACE inhibitors alone c. Calcium channel blockers with thiazide diuretics d. Beta blockers with thiazide diuretics

ANS: C According to the JNC-8 guidelines, calcium channel blockers and diuretics are recommended as first-line therapy for management of hypertension in African-American patients. The other drugs are not recommended as first-line drugs for this group. DIF: COGNITIVE LEVEL: Applying (Application) REF: p. 355 TOP: NURSING PROCESS: Implementation

6. The nurse will monitor for which adverse effect when administering an anticholinergic drug? a. Excessive urination b. Diaphoresis c. Dry mouth d. Pupillary constriction

ANS: C Anticholinergic drugs commonly cause the adverse effects of dry mouth, blurred vision, constipation, and urinary retention. They also cause mydriasis (pupillary dilation). DIF: COGNITIVE LEVEL: Understanding (Comprehension) REF: p. 333 TOP: NURSING PROCESS: Assessment

5. A patient is receiving a third session of chemotherapy with daunorubicin (Cerubidine). The nurse will assess the patient for which signs of a potential severe toxic effect of this drug? a. Tinnitus and hearing loss b. Numbness and tingling in the fingers c. A weight gain of 2 pounds or more in 24 hours d. Decreased blood urea nitrogen and creatinine levels

ANS: C Cardiac toxicity may occur, so frequent checking of heart and breath sounds is necessary and daily weights need to be recorded (with reporting of an increase of 2 pounds or more in 24 hours or 5 pounds or more in 1 week). DIF: COGNITIVE LEVEL: Applying (Application) REF: p. 743 TOP: NURSING PROCESS: Assessment

16. A patient is taking linaclotide (Linzess) to treat irritable bowel syndrome (IBS). The nurse will monitor this patient for which adverse effect? a. Chest pain b. Chronic constipation c. Abdominal pain d. Elevated blood glucose levels

ANS: C Common adverse effects of linaclotide (Linzess) are diarrhea, abdominal pain, and flatulence. Elevated blood glucose levels, chest pain, and chronic constipation are not adverse effects of linaclotide. DIF: COGNITIVE LEVEL: Applying (Application) REF: p. 823 TOP: NURSING PROCESS: Assessment

1. A patient who has a helminthic infection has a prescription for pyrantel (Antiminth). Which is one of the common adverse effects that the patient may experience while on this therapy? a. Vertigo b. Seizures c. Diarrhea d. Insomnia

ANS: C Diarrhea and abdominal pain are some of the possible gastrointestinal effects of pyrantel. See Table 43-11 for other adverse effects. The other options are incorrect. DIF: COGNITIVE LEVEL: Understanding (Comprehension) REF: p. 688 TOP: NURSING PROCESS: Implementation

7. A patient is receiving an infusion of fresh frozen plasma. Based on this order, the nurse interprets that this patient has which condition? a. Hypovolemic shock b. Anemia c. Coagulation disorder d. Previous transfusion reaction

ANS: C Fresh frozen plasma is used as an adjunct to massive blood transfusion in the treatment of patients with underlying coagulation disorders. The other options are not indications for fresh frozen plasma. DIF: COGNITIVE LEVEL: Understanding (Comprehension) REF: p. 470 TOP: NURSING PROCESS: Assessment

12. A patient is in the intensive care unit and receiving an infusion of milrinone (Primacor) for severe heart failure. The prescriber has written an order for an intravenous dose of furosemide (Lasix). How will the nurse give this drug? a. Infuse the drug into the same intravenous line as the milrinone. b. Stop the milrinone, flush the line, and then administer the furosemide. c. Administer the furosemide in a separate intravenous line. d. Notify the prescriber that the furosemide cannot be given at this time.

ANS: C Furosemide must not be injected into an intravenous line with milrinone because it will precipitate immediately. The infusion must not be stopped because of the patient's condition. A separate line will be needed. The other options are incorrect. DIF: COGNITIVE LEVEL: Applying (Application) REF: p. 381 TOP: NURSING PROCESS: Planning

10. A patient is receiving hydroxychloroquine therapy but tells the nurse that she has never traveled out of her city. The nurse knows that a possible reason for this drug therapy is which condition? a. Lyme disease b. Toxoplasmosis c. Systemic lupus erythematosus d. Intestinal tapeworms

ANS: C Hydroxychloroquine, which is used for malaria, also possesses anti-inflammatory actions and has been used to treat rheumatoid arthritis and systemic lupus erythematosus. The other options are incorrect. DIF: COGNITIVE LEVEL: Understanding (Comprehension) REF: p. 681 TOP: NURSING PROCESS: Assessment

17. The nurse educator is providing patient education about the Cardiac Arrhythmia Suppression Trial (CAST). The nurse correctly explains that the trial demonstrated what effect from the pharmacologic suppression of dysrhythmias? a. It reduced mortality by 50% but increased morbidity. b. It significantly reduced the risk of a second myocardial infarction (MI). c. It doubled the risk of a second MI. d. It should be used in all patients who have had an MI, regardless of rhythm.

ANS: C In the CAST, class IC dysrhythmic drugs were used to prevent dysrhythmias after MI. These drugs were found to actually double the rate of mortality. The antidysrhythmic drugs did not reduce mortality or the risk of a second MI. They should not be used for any MI patients with associated dysrhythmias unless the dysrhythmias are life threatening.

2. A patient is admitted with lower abdominal pain and nausea. The nurse performing the initial assessment notes that the patient's abdomen is distended and firm, and hypoactive bowel sounds are present. The patient has not had a stool for 3 days. The nurse will contact the provider, who will: a. order a bulk-forming laxative. b. order extra fluids and fiber. c. perform diagnostic tests. d. prescribe a cathartic laxative.

ANS: C Laxatives are contraindicated for patients with abdominal pain, nausea, cramps, or other symptoms of abdominal disease or an acute surgical abdomen. Laxatives should not be used in patients with obstruction or impaction. This patient shows signs of abdominal obstruction, and laxatives could cause a bowel perforation secondary to increased peristalsis. A bulk-forming laxative is contraindicated. Patients with acute abdomens should be kept NPO pending diagnosis. A cathartic laxative is contraindicated.

1. The nurse is giving oral mineral oil as an ordered laxative dose. The nurse will take measures to prevent which potential problem that may occur with mineral oil? a. Fecal impaction b. Electrolyte imbalances c. Lipid pneumonia d. Esophageal blockage

ANS: C Lipid pneumonia may occur if the oral mineral oil is accidentally aspirated into the respiratory tract. The other options are incorrect. DIF: COGNITIVE LEVEL: Applying (Application) REF: p. 820 TOP: NURSING PROCESS: Implementation

5. When reviewing the medication profile of a patient with a new order for desmopressin (DDAVP), the nurse notes that a drug interaction will occur if which drug is taken with desmopressin? a. Aspirin b. Digoxin c. Lithium d. Penicillin

ANS: C Lithium may cause a decreased therapeutic effect of desmopressin. The other options are incorrect. DIF: COGNITIVE LEVEL: Understanding (Comprehension) REF: p. 486 TOP: NURSING PROCESS: Assessment

13. The nurse is reviewing the orders for a patient and notes a new order for an angiotensin-converting enzyme (ACE) inhibitor. The nurse checks the current medication orders, knowing that this drug class may have a serious interaction with what other drug class? a. Calcium channel blockers b. Diuretics c. Nonsteroidal anti-inflammatory drugs d. Nitrates

ANS: C Nonsteroidal anti-inflammatory drugs (NSAIDs) such as ibuprofen can reduce the antihypertensive effect of ACE inhibitors. In addition, the use of NSAIDs and ACE inhibitors may also predispose patients to the development of acute renal failure. DIF: COGNITIVE LEVEL: Applying (Application) REF: p. 349 TOP: NURSING PROCESS: Assessment

A patient has been taking isoniazid [Nydrazid] for 4 months for latent tuberculosis. The patient reports bilateral tingling and numbness of the hands and feet, as well as feeling clumsy. The nurse expects the provider to: testbanks_and_xanax a. discontinue the isoniazid. b. lower the isoniazid dose and add rifampin. c. order pyridoxine 100 mg per day. d. recheck the tuberculin skin test to see whether it worsens.

ANS: C Patients sometimes develop peripheral neuropathy, characterized by paresthesias, clumsiness, and muscle aches. If these occur, they may be reversed by administering pyridoxine (vitamin B6). It is not necessary to discontinue the isoniazid. Lowering the isoniazid dose and adding rifampin is not indicated. Rechecking the TST is not indicated.

1. When reviewing the health history of a patient, the nurse will note that a potential contraindication to potassium supplements exists if the patient has which problem? a. Burns b. Diarrhea c. Renal disease d. Cardiac tachydysrhythmias

ANS: C Potassium supplements are contraindicated in the presence of renal disease; the other conditions listed may be treated with potassium supplements. DIF: COGNITIVE LEVEL: Applying (Application) REF: p. 474 TOP: NURSING PROCESS: Assessment

7. The nurse is reviewing the medication administration record of a patient who is taking isoniazid (INH). Which drug or drug class has a significant drug interaction with isoniazid? a. Pyridoxine (vitamin B6) b. Penicillins c. Phenytoin (Dilantin) d. Benzodiazepines

ANS: C Taking INH with phenytoin will cause decreased metabolism of the phenytoin, leading to increased drug effects. Pyridoxine is often given with isoniazid to prevent peripheral neuropathy. The other options are incorrect. DIF: COGNITIVE LEVEL: Understanding (Comprehension) REF: p. 664 TOP: NURSING PROCESS: Implementation

4. When monitoring a patient's response to oral antidiabetic drugs, the nurse knows that which laboratory result would indicate a therapeutic response? a. Random blood glucose level 180 mg/dL b. Blood glucose level of 50 mg/dL after meals c. Fasting blood glucose level between 92 mg/dL d. Evening blood glucose level below 80 mg/dL

ANS: C The American Diabetes Association recommends a fasting blood glucose level of between 80 and 130 mg/dL for diabetic patients. The other options are incorrect. DIF: COGNITIVE LEVEL: Applying (Application) REF: p. 504 TOP: NURSING PROCESS: Evaluation

9. A patient is started on a diuretic for antihypertensive therapy. The nurse expects that a drug in which class is likely to be used initially? a. Loop diuretics b. Osmotic diuretics c. Thiazide diuretics d. Potassium-sparing diuretics

ANS: C The Eighth Report of the Joint National Committee on Prevention, Detection, Evaluation, and Treatment of High Blood Pressure (JNC-8) guidelines reaffirmed the role of thiazide diuretics as one of the first-line treatment for hypertension. The other drug classes are not considered first-line treatments. DIF: COGNITIVE LEVEL: Applying (Application) REF: p. 452 TOP: NURSING PROCESS: Implementation

A patient has been treated for an acute attack of P. vivax malaria. For relapse prevention, the nurse expects the provider to order which medication? a. Chloroquine [Aralen Phosphate] b. Doxycycline [Vibramax] c. Primaquine phosphate d. Quinine [Qualaquin]

ANS: C The agent of choice for preventing relapse of P. vivax malaria is primaquine, which is highly active against the hepatic forms of P. vivax. Chloroquine is the drug of choice for treating mild to moderate acute attacks of either type of malaria. Doxycycline is used for chloroquine- resistant malaria. Quinine has little effect on hepatic forms of malaria, so it is not useful for relapse prevention.

5. When reviewing the mechanisms of action of diuretics, the nurse knows that which statement is true about loop diuretics? a. They work by inhibiting aldosterone. b. They are very potent, having a diuretic effect that lasts at least 6 hours. c. They have a rapid onset of action and cause rapid diuresis. d. They are not effective when the creatinine clearance decreases below 25 mL/min.

ANS: C The loop diuretics have a rapid onset of action; therefore, they are useful when rapid onset is desired. Their effect lasts for about 2 hours, and a distinct advantage they have over thiazide diuretics is that their diuretic action continues even when creatinine clearance decreases below 25 mL/min. DIF: COGNITIVE LEVEL: Understanding (Comprehension) REF: p. 454 TOP: NURSING PROCESS: Assessment

16. A patient with heart failure who takes a thiazide diuretic and digoxin [Lanoxin] is admitted for shortness of breath. The patient's heart rate is 66 beats per minute, and the blood pressure is 130/88 mm Hg. The serum potassium level is 3.8 mEq/L, and the digoxin level is 0.8 ng/mL. The nurse admitting this patient understands that the patient: a. has digoxin toxicity. b. is showing signs of renal failure. c. is experiencing worsening of the disease. d. needs a potassium-sparing diuretic.

ANS: C This patient has a normal serum potassium level, and the digoxin level is normal. The patient is showing signs of pulmonary edema, which indicates progression of heart failure. The digoxin level is within normal limits, and the heart rate is above 60 beats per minute, so digoxin toxicity is not likely. There is no sign of renal failure. A potassium-sparing diuretic is not indicated.

6. A patient who has type 2 diabetes is scheduled for an oral endoscopy and has been NPO (nothing by mouth) since midnight. What is the best action by the nurse regarding the administration of her oral antidiabetic drugs? a. Administer half the original dose. b. Withhold all medications as ordered. c. Contact the prescriber for further orders. d. Give the medication with a sip of water.

ANS: C When the diabetic patient is NPO, the prescriber needs to be contacted for further orders regarding the administration of the oral antidiabetic drugs. The other options are incorrect. DIF: COGNITIVE LEVEL: Applying (Application) REF: p. 518 TOP: NURSING PROCESS: Implementation

Which are expected effects of cardiac glycosides? (Select all that apply.) a. Decreased cardiac output b. Decreased force of contraction c. Decreased heart rate d. Modulation of neurohormonal systems e. Positive inotropic effects

ANS: C, D, E Digoxin slows the heart rate, modulates the activity of neurohormonal systems, and increases the force of contraction. It does not decrease cardiac output or reduce the force of contraction.

After starting an antiviral protease inhibitor, a patient with HIV telephones the nurse, complaining, "I'm so hungry and thirsty all the time! I'm urinating 10 or 12 times a day." The nurse recognizes these findings to be consistent with: a.pancreatic infiltration by HIV. b.allergic reaction. c.nonadherence to the antiviral regimen. d.hyperglycemia.

ANS: D Protease inhibitors have been associated with hyperglycemia, new-onset diabetes, abrupt exacerbation of existing diabetes, and diabetic ketoacidosis. These symptoms are not consistent with pancreatic infiltration or an allergic reaction. No evidence indicates that the patient is noncompliant.

A nurse reads a tuberculin skin test on a patient and notes a 6- to 7-mm area of induration. The patient is a young adult who has recently immigrated from a country with a high prevalence of tuberculosis. The patient has no other risk factors. The nurse will expect the provider to: a. begin treating this patient with a two-drug regimen of isoniazid and rifampin. b. order a chest radiograph and a sputum culture to assess for active tuberculosis. c. order a nucleic acid amplification test of the patient's sputum. d. reassure the patient that this is not considered a positive test result

ANS: D testbanks_and_xanax This patient has a moderate risk of tuberculosis; the area of induration would have to be 10 mm or greater to be considered a positive skin test result. Without other clinical signs, there is no indication to treat this patient or to perform diagnostic testing, so a chest x-ray or sputum cultures are not recommended.

4. When administering heparin subcutaneously, the nurse will follow which procedure? a. Aspirating the syringe before injecting the medication b. Massaging the site after injection c. Applying heat to the injection site d. Using a - to -inch 25- to 28-gauge needle

ANS: D A - to -inch 25- to 28-gauge needle is the correct needle to use for a subcutaneous heparin injection. The other options would encourage hematoma formation at the injection site. DIF: COGNITIVE LEVEL: Applying (Application) REF: p. 429 TOP: NURSING PROCESS: Implementation

9. A 63-year-old male patient is scheduled for a physical examination, and he tells the nurse that he wants to start taking a vitamin formula that includes saw palmetto for prostate health. Which is the nurse's best response? a. "I've heard many good things about saw palmetto." b. "It's not a good idea to start herbal therapy at your age." c. "There are very few adverse effects with saw palmetto therapy." d. "The doctor will need to draw some blood and do a digital rectal exam first."

ANS: D A prostatic-specific antigen test and digital rectal examination needs to be performed before initiation of treatment with saw palmetto for benign prostatic hyperplasia. Adverse effects may include gastrointestinal upset, headache, back pain, and dysuria. DIF: COGNITIVE LEVEL: Applying (Application) REF: p. 560 TOP: NURSING PROCESS: Planning

9. The nurse working in a preoperative admitting unit administers an anticholinergic medication to a patient before surgery. What is the purpose of this drug in the preoperative setting? a. Control the heart rate b. Relax the patient c. Reduce urinary frequency d. Reduce oral and gastrointestinal secretions

ANS: D Anticholinergic drugs are given preoperatively to control oral and gastrointestinal secretions during surgery. The other options are incorrect. DIF: COGNITIVE LEVEL: Applying (Application) REF: p. 333 TOP: NURSING PROCESS: Planning

9. A patient is experiencing the exoerythrocytic phase of malaria. The nurse expects which drug to be used for this patient? a. Quinine b. Chloroquine (Aralen) c. Mefloquine (Lariam) d. Primaquine

ANS: D Primaquine is one of the few antimalarial drugs that can destroy the malarial parasites while they are in their exoerythrocytic phase. The other drugs are effective during the erythrocytic, or blood, phase. DIF: COGNITIVE LEVEL: Understanding (Comprehension) REF: p. 683 TOP: NURSING PROCESS: Assessment

2. The nurse will monitor a patient for signs and symptoms of hyperkalemia if the patient is taking which of these diuretics? a. Hydrochlorothiazide (HydroDIURIL) b. Furosemide (Lasix) c. Acetazolamide (Diamox) d. Spironolactone (Aldactone)

ANS: D Spironolactone (Aldactone) is a potassium-sparing diuretic, and patients taking this drug must be monitored for signs of hyperkalemia. The other drugs do not cause hyperkalemia but instead cause hypokalemia. DIF: COGNITIVE LEVEL: Understanding (Comprehension) REF: p. 457 TOP: NURSING PROCESS: Evaluation

9. A patient with primary hypertension is prescribed drug therapy for the first time. The patient asks how long drug therapy will be needed. Which answer by the nurse is the correct response? a. "This therapy will take about 3 months." b. "This therapy will take about a year." c. "This therapy will go on until your symptoms disappear." d. "Therapy for high blood pressure is usually lifelong."

ANS: D There is no cure for the disease, and treatment will be lifelong. The other answers are not appropriate. DIF: COGNITIVE LEVEL: Understanding (Comprehension) REF: p. 355 TOP: NURSING PROCESS: Implementation

A prescriber orders ramipril [Altace] for an obese patient with type 2 diabetes mellitus who has developed hypertension. The nurse provides teaching before dismissing the patient home. Which statement by the patient indicates understanding of the teaching? a. "I am less likely to develop diabetic nephropathy when taking this medication." b. "I should check my blood sugar more often, because hyperglycemia is a side effect of this drug." c. "Taking this medication helps reduce my risk of stroke and heart attack." d. "This medication will probably prevent the development of diabetic retinopathy."

C Ramipril [Altace] is approved for reducing the risk of stroke and myocardial infarction (MI) in patients at high risk for a major cardiovascular event because they have hypertension in conjunction with a history of stroke or MI or because they have diabetes. ACE inhibitors cannot be used for primary prevention of diabetic nephropathy, but they can delay the onset of overt nephropathy in patients who already have less advanced nephropathy. ACE inhibitors do not affect serum electrolytes or glucose. One ACE inhibitor, enalapril, can reduce the risk of diabetic retinopathy in some patients with type 1 diabetes mellitus.

12. The adrenal cortex secretes the following corticosteroids: A. Epinephrine B. Norepinephrine C. Glucocorticoids D. Mineralocorticoids (primarily aldosterone)

C, D

27. Glucocorticoids are administered parenteral for: A. Control of steroid-responsive bronchospastic states B. Rhinitis and to prevent the recurrence of polyps after surgical removal C. Inflammations of the eye, ear, and skin D. Shock, status asthmaticus and spinal cord injury

D

A patient who is taking nitrofurantoin calls the nurse to report several side effects. Which side effect of this drug causes the most concern and would require discontinuation of the medication? a. Anorexia, nausea, and vomiting b. Brown-colored urine c. Drowsiness d. Tingling of the fingers

D Tingling of the fingers can indicate peripheral neuropathy, which can be an irreversible side effect of nitrofurantoin. The other side effects are not serious and can be reversed.

Preferred thyroid drug

Levothyroxine because its hormonal content is standardized and its effect is predictable

C. Atrial fibrillation

The nurse would question an order for pseudoephedrine in a patient with a history of which condition? A. Pneumonia B. Osteoporosis C. Atrial fibrillation D. Peptic ulcer disease

D. They decrease inflammation.

What is the role of corticosteroids in the treatment of acute respiratory disorders? A. They stimulate the immune system. B. They directly dilate the bronchi. C. They increase gas exchange in the alveoli. D. They decrease inflammation.

B. montelukast (singulair)

Which drug acts by blocking leukotrines, thus reducing inflammation in the lungs? A. cromolyn (Intal) B. montelukast (singulair) C. theophylline (elixophyllin) D. albuterol (proventil)

The nurse is preparing to administer amphotericin B intravenously. The nurse will expect to pretreat the patient with which medications? a. Acetaminophen [Tylenol], diphenhydramine [Benadryl], and meperidine [Demerol] b. Aspirin, diphenhydramine [Benadryl], and meperidine [Demerol] c. Ibuprofen [Motrin], diphenhydramine [Benadryl], and meperidine [Demerol] d. Morphine sulphate [Morphine] and acetaminophen ]Tylenol]

a. Acetaminophen [Tylenol], diphenhydramine [Benadryl], and meperidine [Demerol]

Antithyroid Drugs: Propylthiouracil (PTU)

about 2 weeks of therapy may be necessary before symptoms improve, available only in oral form as 50-mg tablet, Methimazole only alternative drug and is rarely used

Preparation for the NCLEX® Examination Questions 9. PPIs have the ability to almost totally inhibit gastric acid secretion. Because of this possibility, the use of the medication can lead to what condition? a. Gastric ulcer formation b. Gastroesoophageal reflux disease (GERD) c. Achlohydria d. Diverticulosis

c Achlohydria Becasue PPIs stop the final step of acid secretion, they can block up to 90% of acid secretion, leading to achlorhydria (without acid).

A 14-year-old male patient who plays football is admitted to the hospital. The nurse notes that the patient has short stature for his age according to a standard growth chart. The patient is muscular, has a deep voice, and needs to shave. The nurse notifies the provider of these findings. Which test will the nurse expect the provider to order? a. Coagulation studies b. Complete blood count (CBC) with differential c. Liver function tests and serum cholesterol d. Serum glucose and hemoglobin A1c

c. Liver function tests and serum cholesterol

A nurse is reviewing the culture results of a patient receiving an aminoglycoside. The report reveals an anaerobic organism as the cause of infection. What will the nurse do? a. Contact the provider to discuss an increased risk of aminoglycoside toxicity. b. Continue giving the aminoglycoside as ordered. c. Request an order for a different class of antibiotic. d. Suggest adding a penicillin to the patient's drug regimen.

c. Request an order for a different class of antibiotic Aminoglycosides are not effective against anaerobic microbes, so another class of antibiotics is indicated. There is no associated increase in aminoglycoside toxicity with anaerobic infection. The aminoglycoside will not be effective, so continuing to administer this drug is not indicated. Adding another antibiotic is not useful, because the aminoglycoside is not necessary.

The nurse correlates the action of calcium channel blockers with what? A) Increased urine output B) Increased cardiac afterload C) Decreased renal blood flow D) Decreased peripheral resistance

d Calcium channel blockers promote arteriolar dilation, which leads to decreased peripheral resistance. Additionally, they decrease oxygen demand by decreasing heart rate and cardiac contractility.

In reviewing cardiac physiology, the nurse recognizes that the most important determinant of cardiac oxygen demand is what? A) Blood volume B) Myocardial contractility C) Coronary artery perfusion D) Intramyocardial wall tension

d Intramyocardial wall tension is the most significant determinant of coronary oxygen demand. Increased cardiac load stretches the cardiac muscle and increases oxygen demand. Additionally, oxygen demand can be increased as a result of the increased pressure against which the heart must pump in the event of increased cardiac afterload.

The nurse should question which medication if it is ordered for a patient with vasospastic angina? A) Nitroglycerin B) Diltiazem C) Verapamil D) Atenolol

d Vasospastic angina is caused by coronary artery spasm leading to decreased blood flow to the myocardium. Treatment is directed to increasing oxygen delivery, and beta blockers work primarily to decrease oxygen demand. Beta blockers (such as atenolol) are not effective in vasospastic angina.

A provider has ordered oral voriconazole [Vfend] for a patient who has a systemic fungal infection. The nurse obtains a medication history and learns that the patient takes phenobarbital for seizures. The nurse will contact the provider to discuss which possibility? a. Administering intravenous voriconazole b. Reducing the dose of phenobarbital c. Reducing the dose of voriconazole d. Using a different antifungal agent

d. Using a different antifungal agent

A patient with osteopenia asks a nurse about the benefits of hormone therapy in preventing osteoporosis. Which statement by the nurse is correct? a. "Estrogen can help reverse bone loss." b. "Hormone therapy increases bone resorption." c. "Hormone therapy does not decrease fracture risk." d. "When hormone therapy is discontinued, bone mass is quickly lost."

d. "When hormone therapy is discontinued, bone mass is quickly lost." Benefits of HT for patients with osteopenia are not permanent; bone loss resumes when HT is discontinued. HT does not reverse bone loss that has already occurred. HT reduces bone resorption. HT can decrease fracture risk by a small amount.

A nurse is obtaining a history from a patient who will receive tadalafil [Cialis] for erectile dysfunction. The patient reports that he expects to have sexual activity three or four times a week. What should the nurse tell the patient? a. Adverse effects of tadalafil resolve more quickly than those of sildenafil. b. There is an increased risk of priapism with frequent sexual activity. c. He should take tadalafil 30 to 60 minutes before sexual activity. d. He should talk to his provider about daily dosing of tadalafil.

d. He should talk to his provider about daily dosing of tadalafil. Tadalafil is approved for daily dosing and can be given daily for men who anticipate sexual activity twice a week or more. Tadalafil has longer effects—up to 36 hours—than sildenafil, and adverse effects take longer to resolve. There is no increased risk of priapism associated with an increase in sexual activity. Because tadalafil has longer effects, dosing and sexual activity do not have to be closely timed.

The nurse has been caring for a patient who has been taking antibiotics for 3 weeks. Upon assessing the patient, the nurse notices the individual has developed oral thrush. What describes the etiology of the thrush? a. Antibiotic resistance b. Community-acquired infection c. Nosocomial infection d. Superinfection

d. Superinfection Oral thrush is a manifestation of a superinfection. The development of thrush is not a symptom of antibiotic resistance. Oral thrush typically is not a community-acquired infection. The development of thrush is not a nosocomial infection.

Indications: Thyroid Drugs

replace what the thyroid gland itself cannot produce to achieve normal thyroid hormone levels (euthyroid condition), also used for diagnoses of suspected hyperthyroidism (TSH-suppression test) and prevent or treat various types of goiters, used for replacement hormonal therapy in patients whose thyroid gland was removed or destroyed during radiation for cancer or hyperthyroidism

ANS: A Stress-related mucosal damage is an important issue for critically ill patients. Stress ulcer prophylaxis (or therapy to prevent severe gastrointestinal [GI] damage) is undertaken in almost every critically ill patient in an intensive care unit and for many patients on general medical surgical units. Procedures performed commonly in critically ill patients, such as passing nasogastric tubes, placing patients on ventilators, and others, predispose patients to bleeding of the GI tract. Guidelines suggest that all such patients receive either a histamine receptor-blocking drug or a proton pump inhibitor. The other options are incorrect.

A patient in the intensive care unit has a nasogastric tube and is also receiving a proton pump inhibitor (PPI). The nurse recognizes that the purpose of the PPI is which effect? a.Prevent stress ulcers b.Reduce bacteria levels in the stomach c.Reduce gastric gas formation (flatulence) d.Promote gastric motility

B. BPH D. narrow-angle glaucoma E. asthma

A patient is asking about taking an antihistamine for springtime allergies. The nurse assesses for contraindications to antihistamine therapy. Which of these conditions if present, would be contraindicated? select all that apply.... A. type 2 DM B. BPH C. hyperthyroidism D. narrow-angle glaucoma E. asthma

ANS: D Simethicone alters the elasticity of mucus-coated bubbles, causing them to break, and is an over-the-counter antiflatulent. The other options are incorrect.

A patient is complaining of excessive and painful gas. The nurse checks the patient's medication orders and prepares to administer which drug for this problem? a.famotidine (Pepcid) b.aluminum hydroxide and magnesium hydroxide (Maalox or Mylanta) c.calcium carbonate (Tums) d.simethicone (Mylicon)

ANS: B Aluminum-based antacids have a constipating effect as well as an acid-neutralizing capacity. The other options are incorrect.

A patient is receiving an aluminum-containing antacid. The nurse will inform the patient to watch for which possible adverse effect? a.Diarrhea b.Constipation c.Nausea d.Abdominal cramping

A. allergy to soy lecithin B. allergy to peanuts

A patient is to receive a new prescription for an ipratropium (atrovent) inhaler. The nurse will assess for which potential contraindications? select all that apply... A. allergy to soy lecithin B. allergy to peanuts C. allergy to iodine products D. hypertension E. seizure disorders

ANS: B Smoking may impair the absorption of H2 antagonists. The other factors are correct interventions for this medication.

A patient who has been taking cimetidine (Tagamet) for hyperacidity calls the clinic to say that the medication has not been effective. The nurse reviews his history and notes that which factor may be influencing the effectiveness of this drug? a.He takes the cimetidine with meals. b.He smokes two packs of cigarettes a day. c.He drinks a glass of water with each dose. d.He takes an antacid 3 hours after the cimetidine dose.

B. The nurse should ask the patient to come to the office for an evaluation of his respiratory status.

A patient with chronic bronchitis calls the office for a refill of his albuterol inhaler. He just had the prescription filled 2 weeks ago, but he says it is empty. When asked, he tells the nurse, "I use it whenever I need it, but now when I use it I feel so sick. I've been needing to use it more often." What is the most appropriate action by the nurse? A. The nurse should confirm the pharmacy location for the needed refill. B. The nurse should ask the patient to come to the office for an evaluation of his respiratory status. C. The nurse should tell the patient not to use this drug too often. D. The nurse should consult the prescriber for a different inhaler prescription.

ANS: B Cisatracurium is used for muscle relaxation during intubation. It is eliminated by spontaneous degradation, not by hepatic metabolism or renal excretion. Atropine is not used for muscle relaxation; it is an anticholinergic drug used to control secretions during procedures. Rocuronium is eliminated by hepatic metabolism. Vecuronium is excreted in bile and can cause prolonged paralysis in patients with liver dysfunction.

A patient with hepatitis requires endotracheal intubation. Which agent does the nurse expect to be administered to facilitate this procedure? a.Atropine b.Cisatracurium [Nimbex] c.Rocuronium [Zemuron] d.Vecuronium [Norcuron]

The nurse is discussing upper and lower UTIs. Which of these conditions can be treated at home? (Select all that apply.) a. Acute cystitis b. Acute urethritis c. Recurrent UTI d. Severe pyelonephritis e. Acute bacterial prostatitis

A, B, C Acute cystitis, urethritis, and recurrent UTIs can be treated on an outpatient basis. Severe pyelonephritis and acute bacterial prostatitis require hospitalization and IV antibiotics.

18. Glucocorticoids include: A. Beclomethasone (several formulations) B. Fluticasone propionate C. Hydrocortisone (several formulations) D. Cortisone E. Methylprednisolone F. Prednisone G. Fludrocortisone acetate H. Aminoglutethimide

A, B, C, D, E, F

The nurse is caring for a group of patients taking warfarin (Coumadin). Which patient or patients is/are at moderate to high risk for harm as a result of warfarin therapy? (Select all that apply.) A. A 79-year-old man with variant genes that code for VKORC1 and CYP2CP B. A 50-year-old woman with a current INR of 2.2 treated for deep vein thrombosis C. A 26-year-old pregnant woman with new onset of symptoms of a pulmonary embolus D. A 65-year-old man on day 4 after hip replacement with a new order for warfarin E. A young adult with a prosthetic heart valve, for whom an interacting drug is being deleted from the regimen

A, C, E Patients with genetic alterations in VKORC1 and CYP2CP are at increased risk of warfarin-induced bleeding. An INR of 2.2 indicates a therapeutic warfarin level. Warfarin can cause fetal hemorrhage and is listed as Pregnancy Risk Category X. Warfarin could be prescribed for the prevention of deep vein thrombosis after hip replacement surgery. The day of prescription would not likely be a day to expect a dangerous adverse effect from warfarin, because its half-life is 1.5 to 2 days. Warfarin interacts with many other agents. The greatest risk for harm is when an interacting drug is being added to or deleted from the regimen.

Which instruction should be included in the teaching for a patient for whom fluticasone propionate [Flovent] MDI has been ordered? A. "Gargle after using your inhaler." B. "Take the medication immediately at the onset of an attack." C. "Take your albuterol first, followed by the Flovent 5 minutes later." D. "Make sure you monitor your fingerstick blood glucose level each morning."

A. "Gargle after using your inhaler." Patients should be taught to gargle after using inhaled glucocorticoids, such as fluticasone propionate, to minimize dysphonia and oropharyngeal candidiasis. The medication is not used to abort an acute attack. Short-acting bronchodilators, such as albuterol, should be administered 5 minutes before inhaled glucocorticoids to enhance delivery to the bronchial tree. Oral glucocorticoids pose a risk of hyperglycemia.

A patient with nonvalvular atrial fibrillation is to be discharged on dabigatran etexilate (Pradaxa). Which statement should the nurse include in the discharge teaching? A. "The medication must be stored in the manufacturer-supplied bottle." B. "Once a new bottle is opened, the capsules maintain efficacy for 90 days." C. "If you have difficulty swallowing the capsule, you can open it and mix it with food." D. "You will need to learn how to give yourself a subcutaneous injection in your abdomen."

A. "The medication must be stored in the manufacturer-supplied bottle." Dabigatran is unstable, especially when exposed to moisture, and should be stored in the manufacturer-supplied bottle, which has a desiccant cap. Current labeling of the pill bottle indicates that once the bottle is opened, the pills should be used within 30 days. However, recent evidence indicates that they maintain efficacy for 60 days, provided they have been stored in the original container. Capsules should be swallowed intact, because chewing, crushing, or opening enhances absorption by 75% and increases the risk of bleeding. The medication is administered orally, not subcutaneously.

A patient with antidiuretic hormone deficiency is receiving desmopressin (DDAVP). The nurse will teach this patient to: a.avoid grapefruit juice. b.increase sodium intake. c.monitor blood pressure. d.reduce fluid intake.

ANS : D

1. The medication order for a 4-year-old child reads, "Give digoxin elixir, 15 mcg/kg, PO now." Convert the micrograms to milligrams. _______

ANS: 0.015 mg DIF: COGNITIVE LEVEL: Applying (Application) REF: N/A TOP: NURSING PROCESS: Implementation

1. A patient will be on a tacrolimus (Prograf) infusion after receiving a liver transplant. The order reads, "Give 0.03 mg/kg/day as a continuous IV infusion." The patient weighs 159 pounds, and the medication injection solution is available in a 5-mg/mL strength. Identify how many milliliters will the nurse draw up for this infusion. (record answer using two decimal places) _______

ANS: 0.43 mL DIF: COGNITIVE LEVEL: Applying (Application) REF: N/A TOP: NURSING PROCESS: Implementation

1. A patient is to receive glycopyrrolate (Robinul) 4 mcg/kg IM 30 minutes before a procedure. The patient weighs 110 pounds; the medication is available in a strength of 0.2 mg/mL. Identify how many milliliters of medication will the nurse draw up into the syringe. _______

ANS: 1 mL To calculate mcg/kg, multiply The patient will receive 200 mcg, or 0.2 mg. The medication is available in a strength of 0.2 mg/mL; therefore, the patient will receive 1 mL. DIF: COGNITIVE LEVEL: Applying (Application) REF: N/A TOP: NURSING PROCESS: Implementation

1. The nurse is preparing to administer a bolus dose of verapamil (Calan) as follows: "Give 5-mg bolus of verapamil, IV push, over 2 minutes. May repeat in 30 minutes if needed." The medication is available in a 2.5-mg/mL strength solution. Identify how many milliliters will the nurse draw into the syringe for this dose. _______

ANS: 2 mL DIF: COGNITIVE LEVEL: Applying (Application) REF: N/A TOP: NURSING PROCESS: Implementation

2. The order for a child reads, "Give furosemide (Lasix) 2 mg/kg IV STAT." The child weighs 33 pounds. Identify how many milligrams will the child receive for this dose. _______

ANS: 30 mg First, convert 33 pounds to kilograms: Next, calculate mg/kg: DIF: COGNITIVE LEVEL: Applying (Application) REF: N/A TOP: NURSING PROCESS: Implementation

1. A patient is to receive enalapril (Vasotec) 5 mg IV every 6 hours. Each dose is given over 5 minutes. The medication is available in an injectable form, 1.25 mg/mL. Identify how many milliliters of medication will the nurse draw up for each dose. _______

ANS: 4 mL DIF: COGNITIVE LEVEL: Applying (Application) REF: N/A TOP: NURSING PROCESS: Implementation

1. A patient is receiving memantine (Namenda) 10 mg PO daily. The patient is unable to swallow pills, so an oral solution that contains 2 mg/mL is ordered. Identify how much will the patient receive per dose. _______

ANS: 5 mL DIF: COGNITIVE LEVEL: Applying (Application) REF: N/A TOP: NURSING PROCESS: Implementation

1. A patient has received an overdose of enoxaparin (Lovenox). The order was for 30 mg, but the patient received 90 mg. The nurse notes that the patient is showing signs of bleeding (oozing blood from the intravenous sites, increased bruising) and notifies the physician, who prescribes protamine sulfate to cover the excess amount of enoxaparin that the patient received. Calculate how much protamine sulfate the patient will receive. _______

ANS: 60 mg Protamine sulfate is used to reverse the effects of low-molecular-weight heparins (LMWHs). A 1-mg dose of protamine is administered for each milligram of the LMWH. This patient received 60 mg of enoxaparin more than the ordered dose of 30 mg; therefore, 60 mg of protamine sulfate will be used as an antidote. DIF: COGNITIVE LEVEL: Applying (Application) REF: p. 416 TOP: NURSING PROCESS: Implementation

2. A patient will be receiving aldesleukin [IL-2] (Proleukin), 600,000 IU/kg every 8 hours for 14 doses. The patient weighs 220 pounds. Identify how many IU of medication will this patient receive per dose. _______

ANS: 60 million (60,000,000) IU Convert pounds to kilograms: Calculate IU/kg for this patient: DIF: COGNITIVE LEVEL: Applying (Application) REF: N/A TOP: NURSING PROCESS: Implementation

1. A patient is to receive a daily dose of fludarabine (Fludara), 25 mg/m2/day for 5 consecutive days. Each dose is diluted in a 125-mL bag of normal saline and is to infuse over 30 minutes. The nurse will set the infusion pump to what rate in milliliters per hour? _______

ANS: 250 ml/hr DIF: COGNITIVE LEVEL: Applying (Application) REF: N/A TOP: NURSING PROCESS: Implementation

1. A 10-year-old child will be receiving docusate sodium (Colace), 120 mg/day PO, divided into 3 doses. Identify how many milligrams will the child receive per dose. _______

ANS: 40 mg DIF: COGNITIVE LEVEL: Applying (Application) REF: N/A TOP: NURSING PROCESS: Implementation

2. A patient will be receiving diphenhydramine (Benadryl) via a PEG tube, 25 mg, every 8 hours for an allergic rash. The medication is available as a 12.5 mg/5 mL syrup. Identify how many milliliters will the nurse administer with each dose. _______

ANS: 45.5 mg DIF: COGNITIVE LEVEL: Applying (Application) REF: N/A TOP: NURSING PROCESS: Implementation

1. A health care worker will be receiving hepatitis B immunoglobulin (BayHep B), 0.06 mg/kg IM now and repeated in 30 days as part of hepatitis B prophylaxis after a needle stick accident. The patient weighs 264 pounds. Identify how many milligrams will the patient receive for each dose. (record answer using one decimal place) _______

ANS: 7.2 mg DIF: COGNITIVE LEVEL: Applying (Application) REF: N/A TOP: NURSING PROCESS: Implementation

A nurse is preparing to administer medications to a patient recently started on delavirdine [Rescriptor]. Which concurrent prescription should the nurse question before administration? a.Alprazolam [Xanax] b.Diphenhydramine [Benadryl] c.Morphine d.Penicillin

ANS: A To prevent toxicity from excessive drug levels, patients should not take alprazolam while taking delavirdine. Diphenhydramine, morphine, and penicillin are not contraindicated for patients taking delavirdine.

8. A patient with a urinary tract infection is given a prescription for TMP/SMZ. When reviewing the drug with the patient, the nurse learns that the patient has type 1 diabetes mellitus and consumes alcohol heavily. What will the nurse do? a. Contact the provider to request a different antibiotic for this patient. b. Obtain frequent blood glucose determinations while giving TMP/SMZ. c. Suggest that the patient take a potassium supplement while taking TMP/SMZ. d. Tell the patient to avoid excessive fluid intake while taking TMP/SMZ.

ANS: A Alcoholics are likely to be folate deficient and have an increased risk of megaloblastic anemia when taking TMP/SMZ, so withholding this drug in this population is recommended. TMP/SMZ shares hypersensitivity reactions with oral sulfonylurea-type hypoglycemics that are used with type 2 diabetes mellitus, so it is not necessary to assess the blood glucose level more often. TMP/SMZ can cause hyperkalemia, so potassium supplements are contraindicated. Patients taking TMP/SMZ should consume more fluids to maintain renal blood flow and prevent renal damage.

10. A nurse is providing teaching to a patient who is admitted to the hospital for initiation of treatment with amiodarone [Cordarone] for atrial fibrillation that has been refractory to other medications. Which statement by the patient indicates a need for further teaching? a. "I may have itching, malaise, and jaundice, but these symptoms will subside." b. "I need to use sunblock to help keep my skin from turning bluish gray." c. "I should not drink grapefruit juice while taking this medication." d. "I should report shortness of breath and cough and stop taking the drug immediately."

ANS: A Amiodarone has many toxic effects. Liver toxicity is rare but serious and should be reported and the drug discontinued. Dermatologic toxicity can occur, and sunblock helps protect the skin, which, with prolonged exposure to the sun, can turn bluish gray. Drinking grapefruit juice can increase amiodarone levels. Pulmonary toxicity is the greatest concern, and patients with pulmonary symptoms should report these to the provider.

4. A patient newly diagnosed with heart failure is admitted to the hospital. The nurse notes a pulse of 90 beats per minute. The nurse will observe this patient closely for: a. decreased urine output. b. increased blood pressure. c. jugular vein distension. d. shortness of breath.

ANS: A As the heart rate increases, ventricular filling decreases, and cardiac output and renal perfusion decrease. Tachycardia does not elevate blood pressure. Jugular vein distension and shortness of breath occur with fluid volume overload.

4. A nurse is teaching a group of nursing students about antidysrhythmic medications. Which statement by a student indicates understanding of the teaching? a. "Antidysrhythmic drugs can cause new dysrhythmias or worsen existing ones." b. "Adverse effects of these drugs are mainly noncardiac in nature." c. "For most antidysrhythmic drugs, there is evidence of reduced mortality." d. "Use of these drugs may be necessary even if the benefits are unknown."

ANS: A Because antidysrhythmic drugs have prodysrhythmic actions, they can exacerbate existing dysrhythmias or generate new ones. Most adverse effects are cardiac related. There is evidence of increased mortality with many of these drugs. Use of these drugs should be limited to situations in which there is a clear benefit and only if that benefit outweighs any risks.

16. A 5-year-old patient seen in an outpatient clinic is noted to have hypertension on three separate visits. Ambulatory blood pressure monitoring confirms that the child has hypertension. As an initial intervention with the child's parents, the nurse will expect to: a. perform a detailed health history on the child. b. provide teaching about antihypertensive medications. c. reassure the parents that their child may outgrow this condition. d. teach the parents about lifestyle changes and a special diet.

ANS: A Because the incidence of secondary hypertension is much higher in children than adults, it is important to obtain an accurate health history to help uncover primary causes. Once the type of hypertension is established, the teaching interventions may be useful. Hypertension must be treated, and it is incorrect to reassure parents that their child may just outgrow the condition.

1. The nurse is about to administer a stat dose of intravenous atropine sulfate to a patient who is experiencing a symptomatic cardiac dysrhythmia. During administration of this drug, the nurse will monitor the patient closely for which adverse effect? a. Tachycardia b. Bradycardia c. Ectopic beats d. Cardiac standstill

ANS: A Cardiovascular effects of cholinergic blockers include increased heart rate and dysrhythmias. One indication for use is the treatment of sinus bradycardia accompanied by hemodynamic compromise. The other options are incorrect. DIF: COGNITIVE LEVEL: Applying (Application) REF: p. 333 TOP: NURSING PROCESS: Evaluation

3. A nurse is teaching a patient who will begin taking verapamil [Calan] for hypertension about the drug's side effects. Which statement by the patient indicates understanding of the teaching? a. "I may become constipated, so I should increase fluids and fiber." b. "I may experience a rapid heart rate as a result of taking this drug." c. "I may have swelling of my hands and feet, but this will subside." d. "I may need to increase my digoxin dose while taking this drug."

ANS: A Constipation is common with verapamil and can be minimized by increasing dietary fiber and fluids. Verapamil lowers the heart rate. Peripheral edema may occur secondary to vasodilation, and patients should notify their prescriber if this occurs, because the prescriber may use diuretics to treat the condition. Verapamil and digoxin have similar cardiac effects; also, verapamil may increase plasma levels of digoxin by as much as 60%, so digoxin doses may need to be reduced.

8. A patient will be starting vitamin D supplements. The nurse reviews his medical record for contraindications, including which condition? a. Renal disease b. Cardiac disease c. Hypophosphatemia d. There are no contraindications to vitamin D supplements.

ANS: A Contraindications to vitamin D products include known allergy to the product, hypercalcemia, renal dysfunction, and hyperphosphatemia. DIF: COGNITIVE LEVEL: Applying (Application) REF: p. 845 TOP: NURSING PROCESS: Assessment

The nurse is monitoring a patient who is receiving an inhalation treatment with pentamidine [NebuPent]. The patient begins to cough and states that he is having trouble breathing. What is the best action by the nurse? a. Administer a PRN dose of a bronchodilator. b. Discontinue the inhalation treatment. c. Nothing, because these are expected adverse effects of pentamidine. d. Reduce the rate of the inhalation treatment.

ANS: A Cough and bronchospasm are the most common adverse reactions associated with inhaled pentamidine. These reactions can be controlled with an inhaled bronchodilator. Reducing the rate of the inhalation treatment or discontinuing the treatment is not an appropriate action. Doing nothing is not sufficient when the patient is having trouble breathing.

A patient is hospitalized with head trauma after a motor vehicle accident. The nurse caring for the patient notes a marked increase in the output of pale, dilute urine. The nurse suspects which condition? a.Diabetes insipidus b.Diabetes mellitus c.Syndrome of inappropriate antidiuretic hormone secretion (SIADH) d.Water intoxication

ANS: A Deficiency of antidiuretic hormone (ADH) produces hypothalamic diabetes insipidus, in which large volumes of dilute urine are produced. Head trauma can cause the hypothalamus to stop producing ADH. Diabetes mellitus is an endocrine disorder of the pancreas that causes the production of large volumes of nondilute urine. SIADH is a condition in which too much ADH is produced, causing oliguria. Water intoxication occurs with SIADH.

10. A patient is in the emergency department with a new onset of rapid-rate atrial fibrillation, and the nurse is preparing a continuous infusion. Which drug is most appropriate for this dysrhythmia? a. Diltiazem (Cardizem) b. Atenolol (Tenormin) c. Lidocaine d. Adenosine (Adenocard)

ANS: A Diltiazem (Cardizem) is indicated for the temporary control of a rapid ventricular response in a patient with atrial fibrillation or flutter and paroxysmal supraventricular tachycardia. It is given by continuous infusion after a loading dose given by IV bolus. The other options are incorrect. DIF: COGNITIVE LEVEL: Applying (Application) REF: p. 405 TOP: NURSING PROCESS: Planning

13. A patient who has a viral upper respiratory infection reports having a runny nose and a cough that prevents sleep and asks the nurse to recommend an over-the-counter medication. Which medication will the nurse recommend? a. Diphenhydramine [Benadryl] b. Fexofenadine/pseudoephedrine [Allegra-D] c. Guaifenesin [Mucinex] d. Phenylephrine drops

ANS: A Diphenhydramine is effective in suppressing cough and also has sedative effects when used in doses to suppress cough. Fexofenadine/pseudoephedrine is a combination antihistamine/decongestant and will not help with cough. Guaifenesin helps make coughs more productive but will not suppress cough or help with sleep. Phenylephrine drops have decongestant properties.

6. A patient with volume overload begins taking a thiazide diuretic. The nurse will tell the patient to expect which outcome when taking this drug? a. Improved exercise tolerance b. Increased cardiac output c. Prevention of cardiac remodeling d. Prolonged survival

ANS: A Diuretics help reduce fluid volume overload, which, by reducing pulmonary edema, can improve exercise tolerance. Diuretics do not improve cardiac output. ACE inhibitors are used to prevent cardiac remodeling and to improve long-term survival.

A patient with acromegaly asks the nurse about treatments for this ondition. What will the nurse tell the patient? a.Drugs are generally used after surgical and radiation therapies have been tried. b.Drug therapy is easy to administer. c.Drug therapy is inexpensive. d.Drug therapy is generally short term.

ANS: A Drugs for acromegaly are generally reserved for patients who do not respond to other therapies or for whom these therapies are not viable options. Drug therapy requires daily subQ injections and is expensive. Drug therapy is indefinite, not short term.

9. A patient is to undergo orthopedic surgery, and the prescriber will order a cephalosporin to be given preoperatively as prophylaxis against infection. The nurse expects the provider to order which cephalosporin? a. First-generation cephalosporin b. Second-generation cephalosporin c. Third-generation cephalosporin d. Fourth-generation cephalosporin

ANS: A First-generation cephalosporins are widely used for prophylaxis against infection in surgical patients, because they are as effective, less expensive, and have a narrower antimicrobial spectrum than second-, third-, and fourth-generation cephalosporins.

15. A patient is receiving lactulose (Enulose) three times a day. The nurse knows that the patient is not constipated and is receiving this drug for which reason? a. High ammonia levels due to liver failure b. Prevention of constipation c. Chronic renal failure d. Chronic diarrhea

ANS: A Lactulose (Enulose) produces a laxative effect but also works to reduce blood ammonia levels by converting ammonia to ammonium. Ammonium is a water-soluble cation that is trapped in the intestines and cannot be reabsorbed into the systemic circulation. This effect has proved helpful in reducing elevated serum ammonia levels in patients with severe liver disease. The other options are incorrect. DIF: COGNITIVE LEVEL: Applying (Application) REF: p. 822 TOP: NURSING PROCESS: Planning

8. Which condition would cause the nurse to withhold a PRN order for magnesium hydroxide? a. Chronic renal failure b. Cirrhosis c. Hemorrhoids d. Prostatitis

ANS: A Magnesium can accumulate to toxic levels in patients with renal dysfunction. The nurse should withhold the medication. Magnesium hydroxide is not contraindicated for patients with hemorrhoids, prostatitis, or cirrhosis.

A child who ingested a handful of aspirin tablets from a medicine cabinet at home is brought to the emergency department. The nurse caring for the child notes a respiratory rate of 48 breaths per minute. The nurse understands that this child's respiratory rate is the result of the body's attempt to compensate for: a. metabolic acidosis. b. metabolic alkalosis. c. respiratory acidosis. d. respiratory alkalosis.

ANS: A Metabolic acidosis can result from the ingestion of aspirin. The body responds by hyperventilating to reduce CO2, which represents volatile carbonic acid, and raise pH. This child has a rapid respiratory rate in response to metabolic acidosis. In patients with metabolic alkalosis, the body responds with hypoventilation in an effort to increase the CO2 level. Patients with respiratory acidosis usually have retention of CO2 secondary to hypoventilation, and compensation is the result of retention of bicarbonate by the kidneys, which is a slow process. Respiratory alkalosis is caused by hyperventilation; treatment involves having the patient rebreathe CO2 or administering sedatives.

10. A patient with type 1 diabetes reports mixing NPH and regular insulin to allow for one injection. What should the nurse tell the patient? a. This is an acceptable practice. b. These two forms of insulin are not compatible and cannot be mixed. c. Mixing these two forms of insulin may increase the overall potency of the products. d. NPH insulin should only be mixed with insulin glargine.

ANS: A NPH insulin is the only insulin suitable for mixing with short-acting insulins, such as insulin aspart [NovoLog]. These insulins are compatible and are mixed frequently for management of diabetics. The overall potency of each insulin is not increased by mixing them. Insulin glargine cannot be mixed with any other insulin for administration.

6. A patient who is in her first trimester of pregnancy asks the nurse to recommend nonpharmaceutical therapies for morning sickness. What will the nurse suggest? a. Avoiding fatty and spicy foods b. Consuming extra clear fluids c. Eating three meals daily d. Taking foods later in the day

ANS: A Nausea and vomiting of pregnancy (NVP) can be treated with nondrug measures, including avoiding fatty and spicy foods. Consuming extra fluids does not help with nausea and vomiting (N/V) but may be needed to prevent dehydration. Patients should be advised to eat small portions of food throughout the day rather than three complete meals. "Morning sickness" may actually occur all day, so delaying intake is not recommended.

A patient who is a missionary has returned from Africa to the United States and has completed treatment for an acute attack of P. vivax malaria. The provider has determined that clinical cure has been achieved. The nurse will expect to teach this patient about: a. beginning medications for a radical cure. b. beginning suppressive therapy. c. reporting visual disturbances and headaches. d. using insect repellents and mosquito netting.

ANS: A Once a clinical cure has been achieved with treatment of an acute attack, radical cure is begun if the patient is out of a malaria-endemic region and not likely to become reinfected. Because this patient has returned to the United States, radical cure therapy may begin. Suppressive therapy is used to prevent infection in individuals traveling to areas where malaria is endemic. Insect repellants and mosquito netting are nondrug measures to prevent infection and are not necessary for this patient.

5. A patient has received two doses of dinoprostone [Prepidil] to initiate labor. It has been 6 hours since the last dose. The nurse assesses the patient and notes that the cervix is ripe and dilated to 4 cm, but contractions are diminishing in intensity and frequency. What will the nurse expect to do? a. Administer oxytocin. b. Monitor urine output. c. Prepare for a cesarean section. d. Watch closely for fetal distress.

ANS: A Once the cervix is ripe and labor has occurred, labor should progress, with contractions increasing in duration and frequency. This woman is showing signs of stalled labor and, because the cervix is ripe, can be induced with oxytocin. Large doses of oxytocin can cause water intoxication, which is not a risk at this point in this patient. There is no indication for a cesarean section at this point. The risk of fetal distress is not increased at this point. PTS: 1 DIF: Cognitive Level: Application REF: p. 783

1. During an intravenous (IV) infusion of amphotericin B, a patient develops tingling and numbness in his toes and fingers. What will the nurse do first? a. Discontinue the infusion immediately. b. Reduce the infusion rate gradually until the adverse effects subside. c. Administer the medication by rapid IV infusion to reduce these effects. d. Nothing; these are expected side effects of this medication.

ANS: A Once the intravenous infusion of amphotericin B has begun, vital signs must be monitored frequently to assess for adverse reactions such as cardiac dysrhythmias, visual disturbances, paresthesias (numbness or tingling of the hands or feet), respiratory difficulty, pain, fever, chills, and nausea. If these adverse effects or a severe reaction occur, the infusion must be discontinued (while the patient is closely monitored) and the prescriber contacted. The other options are incorrect. DIF: COGNITIVE LEVEL: Applying (Application) REF: p. 676 TOP: NURSING PROCESS: Implementation

11. When administering digoxin immune Fab (Digibind) to a patient with severe digoxin toxicity, the nurse knows that each vial can bind with how much digoxin? a. 0.5 mg b. 5 mg c. 5.5 mg d. 15 mg

ANS: A One vial of digoxin immune Fab binds 0.5 mg of digoxin. The other options are incorrect. DIF: COGNITIVE LEVEL: Remembering (Knowledge) REF: p. 383 TOP: NURSING PROCESS: Assessment

A patient is about to begin therapy with ethambutol. The nurse knows that, before initiating treatment with this drug, it is important to obtain which test(s)? a. Color vision and visual acuity b. Complete blood cell (CBC) count c. Hearing testing and a tympanogram d. Hepatic function tests

ANS: A Optic neuritis is a dose-related adverse effect of ethambutol. Patients receiving this drug should have color vision and visual acuity testing before therapy starts and periodically thereafter. A CBC, hearing evaluations, and hepatic function testing are not recommended.

A patient who has human immunodeficiency virus (HIV) infection has developed pneumocystis pneumonia (PCP) and will receive parenteral pentamidine [Pentam 300]. The nurse caring for this patient will: a. administer the drug with the patient supine and monitor the blood pressure closely. b. inform the patient to report coughing and shortness of breath. c. teach the patient that darkening of the urine may occur with this drug. d. tell the patient to report yellow discoloration of the sclerae.

ANS: A Parenteral pentamidine can cause sudden and severe hypotension in 1% of patients, so patients should receive the drug while lying down, and nurses should monitor the blood pressure while giving the drug. Coughing and shortness of breath occur with the inhaled form of the drug. Darkening of the urine is a side effect of metronidazole. Yellow discoloration of the sclerae occurs with nitazoxanide.

A nurse is teaching a patient who is preparing to travel for a month-long missionary trip to Africa. The provider has ordered chloroquine [Aralen Phosphate] as suppressive therapy. Which statement by the patient indicates a need for further teaching? a. "I should continue taking the drug for 2 weeks after returning home." b. "I should take 500 mg of this drug once each week." c. "I should take the first dose 1 week before leaving for Africa." d. "I should take this drug with meals."

ANS: A Patients taking chloroquine as suppressive therapy should take the medication for 4 weeks, not 2 weeks, after returning from an area where infection is likely. The patient is correct to state that 500 mg of the drug should be taken weekly, that the first dose should be taken 1 week before travel, and that the drug should be taken with meals.

5. A 21-year-old male athlete admits to using androgenic steroids. The nurse tells him that which of these is a possible adverse effect of these drugs? a. Liver damage b. Renal failure c. Heart failure d. Stevens-Johnson syndrome

ANS: A Peliosis of the liver, the formation of blood-filled cavities, is a potential effect of androgenic anabolic steroid therapy and may be life threatening. Other serious hepatic effects are hepatic neoplasms (liver cancer), cholestatic hepatitis, jaundice, and abnormal liver function. The other options are incorrect. DIF: COGNITIVE LEVEL: Understanding (Comprehension) REF: p. 557 TOP: NURSING PROCESS: Implementation

1. A patient with elevated lipid levels has a new prescription for nicotinic acid (niacin). The nurse informs the patient that which adverse effects may occur with this medication? a. Pruritus, cutaneous flushing b. Tinnitus, urine with a burnt odor c. Myalgia, fatigue d. Blurred vision, headaches

ANS: A Possible adverse effects of nicotinic acid include pruritus, cutaneous flushing, and gastrointestinal distress. Tinnitus, urine with a burnt odor, and headaches are possible adverse effects of bile acid sequestrants. Headaches are also possible adverse effects of HMG-CoA reductase inhibitors, as are myalgia and fatigue. DIF: COGNITIVE LEVEL: Understanding (Comprehension) REF: p. 444 TOP: NURSING PROCESS: Implementation

2. A patient is taking a vasodilator that relaxes smooth muscles in veins. To help minimize drug side effects, the nurse caring for this patient will: a. caution the patient not to get up abruptly. b. encourage the patient to increase fluid intake. c. tell the patient to report shortness of breath. d. warn the patient about the possibility of bradycardia.

ANS: A Postural hypotension occurs when moving from a supine or seated position to an upright position. It is caused by relaxation of smooth muscle in veins, which allows blood to pool in veins and decreases venous return of blood to the heart. Patients taking such drugs should avoid abrupt transitions to prevent falls. Prolonged use of vasodilators can lead to expansion of blood volume and fluid overload, so increasing fluid intake is not appropriate. Shortness of breath is a symptom associated with heart failure. Tachycardia can occur when the blood pressure drops as a result of the baroreceptor reflex.

9. A prescriber has ordered propranolol [Inderal] for a patient with recurrent ventricular tachycardia. The nurse preparing to administer this drug will be concerned about what in the patient's history? a. Asthma b. Exercise-induced tachyarrhythmias c. Hypertension d. Paroxysmal atrial tachycardia associated with emotion

ANS: A Propranolol is contraindicated in patients with asthma, because it is a nonselective beta-adrenergic antagonist and can cause bronchoconstriction and exacerbate asthma. It is used to treat tachyarrhythmias and paroxysmal atrial tachycardia evoked by emotion, so it is not contraindicated for patients with these conditions. It lowers blood pressure, so it would be helpful in patients with hypertension.

9. Which action is most appropriate regarding the nurse's administration of a rapid-acting insulin to a hospitalized patient? a. Give it within 15 minutes of mealtime. b. Give it after the meal has been completed. c. Administer it once daily at the time of the midday meal. d. Administer it with a snack before bedtime.

ANS: A Rapid-acting insulins, such as insulin lispro and insulin aspart, are able to mimic closely the body's natural rapid insulin output after eating a meal; for this reason, both insulins are usually administered within 15 minutes of the patient's mealtime. The other options are incorrect. DIF: COGNITIVE LEVEL: Applying (Application) REF: p. 505 TOP: NURSING PROCESS: Planning

A patient with high-risk factors for tuberculosis will begin therapy for latent TB with isoniazid and rifampin. The nurse learns that this patient takes oral contraceptives. The nurse will counsel this patient to discuss ____ with her provider. a. another birth control method b. reducing the rifampin dose c. reducing the isoniazid dose d. increasing the oral contraceptive dose

ANS: A Rifampin induces cytochrome P450 enzymes and can accelerate the metabolism of many drugs, including oral contraceptive pills (OCPs). Women taking OCPs should consider a nonhormonal form of birth control. Reducing the dose of rifampin or isoniazid is not indicated. Increasing the OCP dose is not recommended.

1. One patient has cancer of the bone; another has cancer in the connective tissues of the thigh muscles; a third patient has cancer in the vascular tissues. Which of these is the correct term for these tumors? a. Sarcoma b. Leukemia c. Carcinoma d. Lymphoma

ANS: A Sarcomas are malignant tumors that arise from connective tissues. These tissues can be found in bone, cartilage, muscle, blood, lymphatic, and vascular tissues. The other options are incorrect. DIF: COGNITIVE LEVEL: Understanding (Comprehension) REF: p. 711 TOP: NURSING PROCESS: Assessment

2. A patient is taking digoxin (Lanoxin) and a loop diuretic daily. When the nurse enters the room with the morning medications, the patient states, "I am seeing a funny yellow color around the lights." What is the nurse's next action? a. Assess the patient for symptoms of digoxin toxicity. b. Withhold the next dose of the diuretic. c. Administer the digoxin and diuretic together as ordered. d. Document this finding, and reassess in 1 hour.

ANS: A Seeing colors around lights is one potential indication of developing digoxin toxicity. If a patient complains of this, the nurse needs to assess for other signs and symptoms of digoxin toxicity including bradycardia, headache, dizziness, confusion, nausea, and blurred vision, and then notify the prescriber. Administering the drug or withholding the diuretic are incorrect options. DIF: COGNITIVE LEVEL: Analyzing (Analysis) REF: p. 382 TOP: NURSING PROCESS: Assessment

2. A patient is about to undergo a kidney transplant. She will be given an immunosuppressant drug before, during, and after surgery to minimize organ rejection. During the preoperative teaching session, which information will the nurse include about the medication therapy? a. Several days before the surgery, the medication will be administered orally. b. The oral doses need to be taken 1 hour before meals to maximize absorption. c. Mix the oral liquid with juice in a disposable Styrofoam cup just before administration. d. Intramuscular injections of the medication will be needed for several days preceding surgery.

ANS: A Several days before transplant surgery, immunosuppressant drugs need to be taken by the oral route, if possible, to avoid intramuscular injections and the risk for infection caused by the injections. Avoid Styrofoam containers because the medication may adhere to the side of the container. These medications are taken with food to minimize gastrointestinal upset. DIF: COGNITIVE LEVEL: Applying (Application) REF: p. 778 TOP: NURSING PROCESS: Implementation

8. A patient's blood pressure elevates to 270/150 mm Hg, and a hypertensive emergency is obvious. He is transferred to the intensive care unit and started on a sodium nitroprusside (Nipride) drip to be titrated per his response. With this medication, the nurse knows that the maximum dose of this drug should be infused for how long? a. 10 minutes b. 30 minutes c. 1 hour d. 24 hours

ANS: A Sodium nitroprusside is a potent vasodilator and may lead to extreme decreases in the patient's blood pressure. For this reason, it is never infused at the maximum dose for more than 10 minutes. If this drug does not control a patient's blood pressure after 10 minutes, it will most likely be ordered to be discontinued. The other times listed are incorrect. DIF: COGNITIVE LEVEL: Understanding (Comprehension) REF: p. 357 TOP: NURSING PROCESS: Planning

8. A patient with Stage C heart failure (HF) who has been taking an ACE inhibitor, a beta blocker, and a diuretic begins to have increased dyspnea, weight gain, and decreased urine output. The provider orders spironolactone [Aldactone]. The nurse will instruct the patient to: a. avoid potassium supplements. b. monitor for a decreased heart rate. c. take extra fluids. d. use a salt substitute instead of salt.

ANS: A Spironolactone is added to therapy for patients with worsening symptoms of HF. Because spironolactone is a potassium-sparing diuretic, patients should not take supplemental potassium. Patients taking digoxin need to monitor their heart rate. Extra fluids are not indicated. Salt substitutes contain potassium.

A patient from Africa is being treated for East African trypanosomiasis. The nurse is preparing to administer a first dose of suramin sodium [Germanin]. To prevent serious side effects, which action will the nurse take? a. Administer a small test dose before giving the drug. b. Ask the patient to lie flat while the drug is given. c. Give a high-fat snack with the drug to minimize gastric upset. d. Suggest to the provider that liver function tests (LFTs) be performed.

ANS: A Suramin sodium can cause severe side effects, including a shock-like syndrome with IV dosing. A small test dose should be given initially to make sure this is not likely. Patients receiving pentamidine should be supine while receiving that drug. It is not necessary to provide a fatty snack with suramin or to obtain LFTs before administration.

3. A patient will be taking bismuth subsalicylate (Pepto-Bismol) to control diarrhea. When reviewing the patient's other ordered medications, the nurse recognizes that which medication or medication class will interact significantly with the Pepto-Bismol? a. Hypoglycemic drugs b. Antibiotics c. Acetaminophen (Tylenol) d. Antidepressants

ANS: A Taking hypoglycemic drugs with an adsorbent such as bismuth subsalicylate may result in decreased absorption of the hypoglycemic drugs. The other options are incorrect. DIF: COGNITIVE LEVEL: Understanding (Comprehension) REF: p. 816 TOP: NURSING PROCESS: Implementation

1. When monitoring a patient who is taking a low-dose cholinergic drug, the nurse will watch for which cardiovascular effect? a. Bradycardia b. Tachycardia c. Vasoconstriction d. Palpitations

ANS: A The cardiovascular effects of low doses of cholinergic drugs are decreased heart rate (not tachycardia) and vasodilation. Palpitations are not effects of cholinergic drugs. DIF: COGNITIVE LEVEL: Understanding (Comprehension) REF: p. 323 TOP: NURSING PROCESS: Evaluation

8. A nurse explains to a nursing student why opioid antidiarrheal medications are classified as drugs with little or no abuse potential. Which statement by the student indicates a need for further teaching? a. "Formulations for the treatment of diarrhea have very short half-lives." b. "Opioid antidiarrheal drugs contain other drugs with unpleasant side effects at higher doses." c. "Some opioid antidiarrheal drugs do not cross the blood-brain barrier." d. "Some opioid antidiarrheal medications are not water soluble and cannot be given parenterally."

ANS: A The half-life of the opioid antidiarrheal drugs is the same as that of the opioid analgesics. The formulations of opioid antidiarrheal medications that are classified with low abuse potential are often combined with atropine, which has unpleasant side effects at higher doses. Some opioid antidiarrheal drugs are formulated so that they do not cross the blood-brain barrier. Others are not water soluble and therefore cannot be dissolved and injected.

11. During treatment of a patient who has brain cancer, the nurse hears the oncologist mention that the patient has reached the "nadir." The nurse knows that this term means which of these? a. The lowest level of neutrophils reached during therapy. b. The highest level of neutrophils reached during therapy. c. The point at which the adverse effects of chemotherapy will stop. d. The point at which the cytotoxic action against cancer cells is the highest.

ANS: A The lowest neutrophil count reached after a course of chemotherapy is known as the nadir. The other options are incorrect. DIF: COGNITIVE LEVEL: Understanding (Comprehension) REF: p. 717 TOP: NURSING PROCESS: Implementation

4. An infertile couple begins treatment with menotropins. To enhance the therapeutic effects of the medication, the nurse will advise the patients to have intercourse: a. before hCG injection and the following 2 to 3 days. b. the morning of the hCG injection and later that evening. c. the evening of the injection and the following day. d. the evening of the injection and the following morning.

ANS: A The nurse should advise the patients to have intercourse the evening before the injection and on the following 2 to 3 days (during the probable period of ovulation). Having intercourse the morning of the hCG injection and later that evening will not enhance the effects of menotropins. Avoiding intercourse the evening of the injection and on the following day will not enhance the effects of menotropins. Having intercourse the evening of the injection and on the following morning will not enhance the effects of menotropins. PTS: 1 DIF: Cognitive Level: Application REF: pp. 771-772

1. The nurse is administering insulin lispro (Humalog) and will keep in mind that this insulin will start to have an effect within which time frame? a. 15 minutes b. 1 to 2 hours c. 80 minutes d. 3 to 5 hours

ANS: A The onset of action for insulin lispro is 15 minutes. The peak plasma concentration is 1 to 2 hours; the elimination half-life is 80 minutes; and the duration of action is 3 to 5 hours. DIF: COGNITIVE LEVEL: Remembering (Knowledge) REF: p. 516 TOP: NURSING PROCESS: Implementation

A patient who was injured at home is brought to the emergency department. The nurse caring for this patient notes a respiratory rate of 32 breaths per minute and a heart rate of 90 beats per minute. The injuries are minor, but the patient is inconsolable and hysterical. The nurse expects that initial management will include: a. administering a gas mixture of 5% carbon dioxide (CO2). b. providing 100% oxygen via nasal cannula. c. giving sodium bicarbonate IV. d. providing sedatives to calm the patient.

ANS: A The patient is at risk for respiratory alkalosis as a result of hyperventilation, and giving the patient a gas mixture containing CO2 will help correct the alkalosis. Administering oxygen would worsen the problem. Sodium bicarbonate is given to correct metabolic acidosis. Sedatives may be necessary if initial measures fail.

A male patient reports decreased libido and the nurse notes galactorrhea during a physical assessment. The nurse will report these findings to the provider and will anticipate an order for which medication? a.Cabergoline b.Conivaptan c.Dopamine d.Prolactin

ANS: A The patient is showing signs of prolactin hypersecretion. Cabergoline is a dopamine agonist and is used to inhibit prolactin release. Conivaptan is used to treat hypernatremia. Dopamine is not given. Prolactin would make the condition worse.

7. A patient with allergic rhinitis is taking a compound product of loratadine/pseudoephedrine [Claritin-D] every 12 hours. The patient complains of insomnia. The nurse notes that the patient is restless and anxious. The patient's heart rate is 90 beats per minute, and the blood pressure is 130/85 mm Hg. The nurse will contact the provider to: a. discuss using an intranasal glucocorticoid and loratadine [Claritin]. b. report acute toxicity caused by pseudoephedrine. c. suggest using an agent with a sympathomimetic drug only. d. suggest using a topical decongestant to minimize systemic symptoms.

ANS: A This patient is showing central nervous system (CNS) and cardiovascular side effects of the pseudoephedrine. A better option would be to use single-ingredient products for each symptom; an intranasal glucocorticoid and an oral antihistamine are considered first-line treatments. This patient is demonstrating adverse effects but not acute toxicity. Using a sympathomimetic agent would increase the adverse effects, because pseudoephedrine is a sympathomimetic drug. Topical decongestants are not first-line drugs for allergic rhinitis.

7. A postpartum patient is receiving methylergonovine to prevent hemorrhage. The nurse assesses the patient and notes a heart rate of 76 beats per minute, a respiratory rate of 16 breaths per minute, and a blood pressure of 120/80 mm Hg. The patient's uterus is firm. The patient complains of mild to moderate cramping. Bleeding is minimal. What will the nurse do? a. Continue to monitor and tell the patient to report increased cramping. b. Notify the provider of possible uterine hypertonicity. c. Notify the provider of the patient's blood pressure. d. Request an order to administer the methylergonovine intravenously.

ANS: A This patient is showing signs of a desired response to the drug, so the nurse should continue to monitor. Increased cramping may be a sign of overdose, so the patient should know to notify the nurse if this occurs. A firm uterus with minimal bleeding is the desired effect. The blood pressure is within normal limits. There is no indication for giving this drug intravenously, because that method poses significant risks and is indicated only with severe hemorrhage. PTS: 1 DIF: Cognitive Level: Application REF: p. 784

4. The nurse is caring for a patient who is taking a vasodilator that dilates capacitance vessels. The nurse will expect which effect in this patient? a. Decrease in cardiac work b. Increase in cardiac output c. Increase in tissue perfusion d. Increase in venous return

ANS: A Vasodilators that dilate capacitance vessels, or veins, lead to a decrease in venous return to the heart, which reduces preload and the force of ventricular contraction. The resultant effect is a decrease in cardiac work. With a decrease in ventricular contraction, cardiac output is reduced, as is tissue perfusion. Dilation of veins causes a decrease in venous return.

1. A nurse is caring for a patient who is receiving verapamil [Calan] for hypertension and digoxin [Lanoxin] for heart failure. The nurse will observe this patient for: a. AV blockade. b. gingival hyperplasia. c. migraine headaches. d. reflex tachycardia.

ANS: A Verapamil and digoxin both suppress impulse conduction through the AV node; when the two drugs are used concurrently, the risk of AV blockade is increased. Gingival hyperplasia can occur in rare cases with verapamil, but it is not an acute symptom. Verapamil can be used to prevent migraine, and its use for this purpose is under investigation. Verapamil and digoxin both suppress the heart rate. Nifedipine causes reflex tachycardia.

4. The nurse is assessing a patient who is malnourished and has a history of poor nutrition. The patient reports difficulty seeing at night. This patient is likely to be deficient in which fat-soluble vitamin? a. A (retinol) b. D c. E (alpha-tocopherol) d. K

ANS: A Vitamin A is needed for dark adaptation; night blindness often is the first indication of deficiency. Night blindness is not a sign of deficiency of vitamins D, E, or K. PTS: 1 DIF: Cognitive Level: Application REF: pp. 988-989

A patient at increased risk for thromboembolic disorders will begin taking a progestin-only oral contraceptive. Which statement by the patient indicates understanding of how this oral contraceptive works? a. "I will need to use backup contraception if I miss a pill." b. "Irregular bleeding is an indication that I should stop using this drug." c. "The mini-pill is safer than combination OCs and is just as effective." d. "The progestin-only mini-pill will prevent me from ovulating."

ANS: A When a patient taking a progestin-only OC misses one or more pills, backup contraception should always be used for at least 2 days. Irregular bleeding occurs but is not an indication to stop using the drug, although many women do because of the inconvenience. The mini-pill is slightly safer than combination OCs but not as effective. The mini-pill is a weak inhibitor of ovulation.

2. An infertile patient has received two 5-day courses of clomiphene [Clomid] to help her conceive. An ultrasound 1 week after the last dose reveals that follicular maturation has occurred without ovulation. The nurse expects the next step for this patient to be: a. administration of human chorionic gonadotropin (hCG). b. an order for cabergoline to be given twice weekly. c. evaluation of ovarian and pituitary function. d. repeating clomiphene once daily for 5 days.

ANS: A When treatment with clomiphene has failed to produce ovulation after two courses, hCG is used to promote ovulation after follicular maturation has occurred. Cabergoline is used to treat hyperprolactinemia when excessive prolactin secretion occurs. Ovarian and pituitary function should be evaluated before a patient begins treatment with clomiphene. If ovulation had occurred, another cycle of clomiphene would be indicated. PTS: 1 DIF: Cognitive Level: Application REF: pp. 770-771 | pp. 772-773

Which types of drugs are used to treat inflammatory bowel disease (IBD)? (Select all that apply.) a. Aminosalicylates b. Glucocorticoids c. Immunomodulators d. Opioid antidiarrheals e. Sulfonamide antibiotics

ANS: A, B, C Five types of drugs are used to treat IBD: aminosalicylates, glucocorticoids, immunosuppressants, immunomodulators, and antibiotics such as metronidazole and ciprofloxacin. Opioid antidiarrheal drugs and sulfonamide antibiotics are not used for this purpose.

1. A patient is taking a sulfonylurea medication for new-onset type 2 diabetes mellitus. When reviewing potential adverse effects during patient teaching, the nurse will include information about which of these effects? (Select all that apply.) a. Hypoglycemia b. Nausea c. Diarrhea d. Weight gain e. Peripheral edema

ANS: A, B, D The most common adverse effect of the sulfonylureas is hypoglycemia, the degree to which depends on the dose, eating habits, and presence of hepatic or renal disease. Another predictable adverse effect is weight gain because of the stimulation of insulin secretion. Other adverse effects include skin rash, nausea, epigastric fullness, and heartburn. DIF: COGNITIVE LEVEL: Applying (Application) REF: p. 509 TOP: NURSING PROCESS: Implementation

3. During an intravenous infusion of calcium, the nurse carefully monitors the patient for symptoms of hypercalcemia. Which are symptoms of hypercalcemia? (Select all that apply.) a. Anorexia b. Nausea and vomiting c. Diarrhea d. Constipation e. Cardiac irregularities f. Drowsiness

ANS: A, B, D, E Symptoms of hypercalcemia include anorexia, nausea, vomiting, and constipation. Long-term excessive calcium intake can result in severe hypercalcemia, which can cause cardiac irregularities, delirium, and coma. The other options are incorrect. DIF: COGNITIVE LEVEL: Applying (Application) REF: p. 853 TOP: NURSING PROCESS: Implementation

1. Levothyroxine (Synthroid) has been prescribed for a patient with hypothyroidism. The nurse reviews the patient's current medications for potential interactions. Which of these drugs or drug classes interact with levothyroxine? (Select all that apply.) a. Phenytoin (Dilantin) b. Estrogens c. Beta blockers d. Warfarin (Coumadin) e. Penicillins f. Iron supplements

ANS: A, B, D, F Drug interactions with thyroid preparations include phenytoin, cholestyramine, antacids, calcium salts, iron products, estrogens, and warfarin (see Table 31-3). The other options are not correct. DIF: COGNITIVE LEVEL: Applying (Application) REF: p. 493 TOP: NURSING PROCESS: Planning

2. The nurse expects that a patient is experiencing undersecretion of adrenocortical hormones when which conditions are found upon assessment? (Select all that apply.) a. Dehydration b. Weight loss c. Steroid psychosis d. Increased potassium levels e. Increased blood glucose levels f. Decreased serum sodium levels

ANS: A, B, D, F The undersecretion (hyposecretion) of adrenocortical hormones causes a condition known as Addison's disease, which is associated with decreased blood sodium and glucose levels, increased potassium levels, dehydration, and weight loss. Steroid psychosis is an effect of glucocorticoid excess. DIF: COGNITIVE LEVEL: Applying (Application) REF: p. 524 TOP: NURSING PROCESS: Assessment

1. A pregnant patient is 1 week post-term and will receive intravenous oxytocin. Which will the nurse evaluate before starting the infusion? (Select all that apply.) a. Cervical ripening b. Fetal monitoring c. Renal function d. Respiratory function e. Uterine activity

ANS: A, B, E Before labor can be safely induced, cervical ripening must occur. Baseline data must include full maternal and fetal status. Maternal renal function and respiratory function are not part of the baseline assessment. PTS: 1 DIF: Cognitive Level: Analysis REF: p. 783 | p. 786

A patient with hepatitis B begins treatment with adefovir [Hepsera] and asks the nurse how long the drug therapy will last. The nurse will tell the patient that the medication will need to be taken for: a.a lifetime. b.an indefinite, prolonged period of time. c.48 weeks. d.until nephrotoxicity occurs

ANS: B Current guidelines recommend treatment only for patients at highest risk; it is unknown whether treatment should continue lifelong. Treatment is usually prolonged, without a specific period of time. Nephrotoxicity is common but is not the deciding factor when determining length of effective treatment.

The nurse is preparing to discharge a patient with HIV who will continue to take enfuvirtide [Fuzeon] at home. The nurse is providing patient education about the medication. What information about the administration of enfuvirtide is most appropriate for the patient? a.The importance of injecting the drug into two alternating sites daily b.How to reconstitute and self-administer a subcutaneous injection c.The importance of taking the drug with high doses of vitamin E d.Likely drug interactions between enfuvirtide and other antiretroviral drugs

ANS: B The most appropriate information about the administration of enfuvirtide is how to reconstitute and self-administer a subcutaneous injection. The medication should never be injected into the same site or just between two sites. Vitamin E is not indicated for this medication. Enfuvirtide does not appear to cause significant drug interactions.

12. A nurse is providing teaching to a nursing student about to care for a woman with irritable bowel syndrome with diarrhea (IBS-D) who is receiving alosetron [Lotronex]. Which statement by the student indicates a need for further teaching? a. "I should evaluate the patient's abdomen for distension and bowel sounds." b. "Patients with diverticulitis and IBS-C may take this drug." c. "This drug can cause ischemic colitis in some patients." d. "This drug is given only to women with severe IBS-D."

ANS: B Alosetron is approved for use in women only with diarrhea-predominant IBS; it is contraindicated in patients with diverticulitis. Constipation can be a severe adverse effect, so patients should be assessed for signs of constipation, such as abdominal distension and diminished bowel sounds. Alosetron can cause ischemic colitis. Alosetron is approved for use in women with IBS-D regardless of the severity of the disease.

6. A glucocorticoid is prescribed for a patient. The nurse checks the patient's medical history knowing that glucocorticoid therapy is contraindicated in which disorder? a. Cerebral edema b. Peptic ulcer disease c. Tuberculous meningitis d. Chronic obstructive pulmonary disease

ANS: B Contraindications to the administration of glucocorticoids include drug allergy and may include cataracts, glaucoma, peptic ulcer disease, mental health problems, and diabetes mellitus. The other options are indications for glucocorticoids. DIF: COGNITIVE LEVEL: Understanding (Comprehension) REF: p. 526 TOP: NURSING PROCESS: Assessment

15. The nurse is preparing to administer the contraceptive form of medroxyprogesterone (Depo-Provera). What route is appropriate? a. Subcutaneous b. Intramuscular c. Vaginal d. Transdermal

ANS: B Depo-Provera is a progestin-only injectable contraceptive that is given by the intramuscular route. The other options are incorrect. DIF: COGNITIVE LEVEL: Understanding (Comprehension) REF: p. 549 TOP: NURSING PROCESS: Planning

6. During the administration of finasteride (Proscar), the nurse must remember which important precaution? a. It must be taken on an empty stomach. b. It must not be handled by pregnant women. c. It is given by deep intramuscular injection to avoid tissue irritation. d. The patient needs to be warned that alopecia is a common adverse effect.

ANS: B Finasteride must not be handled by pregnant women because of its teratogenic effects. It is taken orally and without regard to meals. The other options are incorrect. DIF: COGNITIVE LEVEL: Applying (Application) REF: p. 558 TOP: NURSING PROCESS: Implementation

9. A patient receives an injection of human chorionic gonadotropin after follicular maturation has been induced with another agent. The patient comes to the clinic 2 days later complaining of headache, irritability, and fatigue. What will the nurse do? a. Perform a urine pregnancy test. b. Reassure the patient that these are known adverse effects. c. Request an order for a serum estrogen level. d. Review the patient's abdominal ultrasound.

ANS: B Headache, irritability, and fatigue are known adverse effects of hCG. A urine pregnancy test is not indicated, because it is too soon for the patient to be pregnant. A serum estrogen level and an abdominal ultrasound are not indicated. PTS: 1 DIF: Cognitive Level: Application REF: pp. 772-773

A patient has been taking a progestin-only, or "minipill," OC for 3 months and reports spotting and irregular menstrual cycles. The nurse will: a. question the patient about any possible missed doses of the pill. b. reassure the patient that this is normal with this form of contraception. c. recommend that she take a pregnancy test to rule out pregnancy. d. suggest that she use a backup form of contraception until these symptoms resolve.

ANS: B Patients taking the progestin-only OC may expect irregular bleeding, including spotting and irregular periods. This symptom does not indicate lack of compliance with the regimen. A pregnancy test is not indicated. It is not necessary to use a backup form of contraception.

2. The nurse notes in a patient's medication history that the patient is taking pilocarpine (Pilocar). Based on this finding, the nurse interprets that the patient has which disorder? a. Anticholinergic poisoning b. Glaucoma c. Bladder atony d. Myasthenia gravis

ANS: B Pilocarpine is a direct-acting drug that is used topically to reduce intraocular pressure in patients with glaucoma. The other disorders are not indications for pilocarpine. DIF: COGNITIVE LEVEL: Applying (Application) REF: p. 323 TOP: NURSING PROCESS: Assessment

A patient is about to begin treatment for latent tuberculosis with a short course of daily rifampin. The patient asks why rifapentine [Priftin] cannot be used, because it can be given twice weekly. What will the nurse tell this patient about rifapentine? a. It is more toxic than rifampin. b. It is not approved for treatment of latent TB. c. It is not well absorbed and thus not as effective. d. It will stain contact lenses orange.

ANS: B Rifapentine is indicated only for pulmonary TB. Rifapentine's toxicity is similar to that of rifampin. Rifapentine is well absorbed. Both drugs stain contact lenses.

1. The nurse is reviewing the classes of antidysrhythmic drugs. Amiodarone (Cordarone) is classified on the Vaughan Williams classification as a class III drug, which means it works by which mechanism of action? a. Blocking slow calcium channels b. Prolonging action potential duration c. Blocking sodium channels and affecting phase 0 d. Decreasing spontaneous depolarization and affecting phase 4

ANS: B Vaughan Williams class III drugs (amiodarone, dronedarone, sotalol, ibutilide, and dofetilide) increase the action potential duration by prolonging repolarization in phase 3. The other answers are incorrect. DIF: COGNITIVE LEVEL: Understanding (Comprehension) REF: p. 396 TOP: NURSING PROCESS: Implementation

6. A patient tells the nurse that he likes to eat large amounts of garlic "to help lower his cholesterol levels naturally." The nurse reviews his medication history and notes that which drug has a potential interaction with the garlic? a. Acetaminophen (Tylenol) b. Warfarin (Coumadin) c. Digoxin (Lanoxin) d. Phenytoin (Dilantin)

ANS: B When using garlic, it is recommended to avoid any other drugs that may interfere with platelet and clotting function. These drugs include antiplatelet drugs, anticoagulants, nonsteroidal anti-inflammatory drugs, and aspirin. The other drugs listed do not have known interactions with garlic. DIF: COGNITIVE LEVEL: Understanding (Comprehension) REF: p. 437 TOP: NURSING PROCESS: Planning

A nurse counsels a patient with diabetes who is starting therapy with an alpha-glucosidase inhibitor. The patient should be educated about the potential for which adverse reactions? (Select all that apply.) a. Hypoglycemia b. Flatulence c. Elevated iron levels in the blood d. Fluid retention e. Diarrhea

ANS: B, E Acarbose and miglitol frequently cause flatulence, cramps, abdominal distention, borborygmus (rumbling bowel sounds), and diarrhea. Acarbose also can cause liver damage. Neither acarbose nor miglitol causes hypoglycemia, elevated iron levels, or fluid retention.

1. The nurse is providing patient teaching for a patient who is starting antitubercular drug therapy. Which of these statements should be included? (Select all that apply.) a. "Take the medications until the symptoms disappear." b. "Take the medications at the same time every day." c. "You will be considered contagious during most of the illness and must take precautions to avoid spreading the disease." d. "Stop taking the medications if you have severe adverse effects." e. "Avoid alcoholic beverages while on this therapy." f. "If you notice reddish-brown or reddish-orange urine, stop taking the drug and contact your doctor right away." g. "If you experience a burning or tingling in your fingers or toes, report it to your prescriber immediately." h. "Oral contraceptives may not work while you are taking these drugs, so you will have to use another form of birth control."

ANS: B, E, G, H Medications for tuberculosis must be taken on a consistent schedule to maintain blood levels. Medication therapy for tuberculosis may last up to 24 months, long after symptoms disappear, and patients are infectious during the early part of the treatment. Compliance with antitubercular drug therapy is key, so if symptoms become severe, the prescriber should be contacted for an adjustment of the drug therapy. The medication must not be stopped. Because of potential liver toxicity, patients on this drug therapy must not drink alcohol. Discoloration of the urine is an expected adverse effect, and patients need to be warned about it beforehand. Burning or tingling in the fingers or toes may indicate that peripheral neuropathy is developing, and the prescriber needs to be notified immediately. A second form of birth control must be used because antitubercular drug therapy makes oral contraceptives ineffective. DIF: COGNITIVE LEVEL: Applying (Application) REF: pp. 667-668 TOP: NURSING PROCESS: Implementation

An immunocompromised child is exposed to chickenpox and the provider orders valacyclovir [Valtrex] to be given orally three times daily. The nurse will contact the provider to change this order for which reason? a.Valacyclovir is not used as varicella prophylaxis. b.The dosage is too high for this indication. c.The drug may cause serious adverse effects in immunocompromised patients. d.Valacyclovir is not approved for use in children.

ANS: C Valacyclovir is approved for use for varicella in immunocompetent children. In immunocompromised patients, it has produced a syndrome known as thrombotic thrombocytopenic purpura/hemolytic uremia syndrome (TTP/HUS). The dosage is fine for immunocompetent children.

1. When a patient is on aminoglycoside therapy, the nurse will monitor the patient for which indicators of potential toxicity? a. Fever b. White blood cell count of 8000 cells/mm3 c. Tinnitus and dizziness d. Decreased blood urea nitrogen (BUN) levels

ANS: C Dizziness, tinnitus, hearing loss, or a sense of fullness in the ears could indicate ototoxicity, a potentially serious toxicity in a patient. Nephrotoxicity is indicated by rising blood urea nitrogen and creatinine levels. Fever may be indicative of the patient's infection; a white blood cell count of 7000 cells/mm3 is within the normal range of 5000 to 10,000 cells/mm3. DIF: COGNITIVE LEVEL: Applying (Application) REF: p. 627 TOP: NURSING PROCESS: Evaluation

6. During an infusion of albumin, the nurse monitors the patient closely for the development of which adverse effect? a. Hypernatremia b. Fluid volume deficit c. Fluid volume overload d. Transfusion reaction

ANS: C During the infusion of albumin, the development of fluid volume overload must be monitored by the nurse, especially in those at risk for heart failure. The other options are incorrect. DIF: COGNITIVE LEVEL: Applying (Application) REF: p. 477 TOP: NURSING PROCESS: Evaluation

5. A patient is receiving oxytocin (Pitocin) to induce labor. During administration of this medication, the nurse will also implement which action? a. Giving magnesium sulfate along with the oxytocin b. Administering the medication in an intravenous (IV) bolus c. Administering the medication with an IV infusion pump d. Monitoring fetal heart rate and maternal vital signs every 6 hours

ANS: C Oxytocin is infused via an infusion pump, not via an IV bolus. Magnesium sulfate is not administered with oxytocin. Fetal heart rate and maternal vital signs should be monitored continuously. DIF: COGNITIVE LEVEL: Applying (Application) REF: p. 549 TOP: NURSING PROCESS: Implementation

11. A patient has received an overdose of intravenous heparin, and is showing signs of excessive bleeding. Which substance is the antidote for heparin overdose? a. Vitamin E b. Vitamin K c. Protamine sulfate d. Potassium chloride

ANS: C Protamine sulfate is a specific heparin antidote and forms a complex with heparin, completely reversing its anticoagulant properties. Vitamin K is the antidote for warfarin (Coumadin) overdose. The other options are incorrect. DIF: COGNITIVE LEVEL: Applying (Application) REF: p. 416 TOP: NURSING PROCESS: Implementation

8. A nurse provides teaching to a woman who is taking tranexamic acid [Lysteda] for menorrhagia. Which statement by the patient indicates a need for further teaching? a. "I may experience back pain or muscle cramps while taking this drug." b. "I may take two tablets 3 times daily for up to 5 days with each period." c. "I should take an oral contraceptive to prevent pregnancy while taking this drug." d. "I should take this medication with food to improve absorption of the drug."

ANS: C The greatest concern with tranexamic acid (TA) is venous or arterial thrombosis. Women taking TA should not take oral contraceptives, which would increase the risk of thrombosis. Back pain and muscle cramps are known adverse effects. Women may take up to two tablets 3 times a day for up to 5 days with each period. The medication can be taken with or without food, but bioavailability increases in the presence of food. PTS: 1 DIF: Cognitive Level: Application REF: p. 785

2. A nurse is discussing heart failure with a group of nursing students. Which statement by a student reflects an understanding of how compensatory mechanisms can compound existing problems in patients with heart failure? a. "An increase in arteriolar tone to improve tissue perfusion can decrease resistance." b. "An increase in contractility to increase cardiac output can cause pulmonary edema." c. "When the heart rate increases to increase cardiac output, it can prevent adequate filling of the ventricles." d. "When venous tone increases to increase ventricular filling, an increase in arterial pressure occurs."

ANS: C The heart rate increases to improve cardiac output, but it may prevent adequate ventricular filling. An increase in arteriole tone improves tissue perfusion but also increases both the resistance to the pumping of the heart and the cardiac workload. Increased contractility helps improve cardiac output but is detrimental because it increases the oxygen demand of the heart. An increase in venous tone improves ventricular filling, but as the ventricles fail, blood can back up and cause pulmonary edema.

9. A patient with hypertension will begin taking an alpha1 blocker. What will the nurse teach this patient? a. A persistent cough is a known side effect of this drug. b. Eat foods rich in potassium while taking this drug. c. Move slowly from sitting to standing when taking this drug. d. Report shortness of breath while taking this drug.

ANS: C The most disturbing side effect of alpha blockers is orthostatic hypotension. Patients taking these drugs should be cautioned to stand up slowly to avoid lightheadedness or falls. A persistent cough is a common side effect of ACE inhibitors. It is not necessary to increase dietary potassium intake when taking this drug. Shortness of breath may occur in individuals with asthma who are taking beta blockers.

1. A patient is to receive testosterone therapy via a transdermal patch. He asks the nurse, "Why am I getting a patch? Can't I just take a pill?" Which response by the nurse is correct? a. "The patch reduces the incidence of side effects." b. "If you don't take the patch, you will have to have injections instead." c. "The patch allows for better absorption of the medication." d. "You will only have to change the patch weekly."

ANS: C The transdermal form allows for better absorption of testosterone because of its high first-pass effect. Oral forms are poorly absorbed, and the transdermal form is preferable to an injection and is preferred for hormonal replacement therapy. The patch is changed daily. DIF: COGNITIVE LEVEL: Applying (Application) REF: p. 555 TOP: NURSING PROCESS: Implementation

An older adult patient who lives alone and is somewhat forgetful has an overactive bladder (OAB) and reports occasional constipation. The patient has tried behavioral therapy to treat the OAB without adequate results. Which treatment will the nurse anticipate for this patient? a.Oxybutynin short-acting syrup b.Oxybutynin [Ditropan XL] extended-release tablets c.Oxybutynin [Oxytrol] transdermal patch d.Percutaneous tibial nerve stimulation (PTNS)

ANS: C The transdermal patch is applied weekly and may be the best option for a patient who is more likely to forget to take a daily medication. The transdermal preparation has fewer side effects than the systemic dose, so it is less likely to increase this patient's constipation. The syrup has a high incidence of dry mouth and other anticholinergic side effects. The extended-release tablets must be given daily, and this patient may not remember to take them. PTNS is used after behavioral and drug therapies have failed.

1. A patient who is receiving a final dose of intravenous (IV) cephalosporin begins to complain of pain and irritation at the infusion site. The nurse observes signs of redness at the IV insertion site and along the vein. What is the nurse's priority action? a. Apply warm packs to the arm, and infuse the medication at a slower rate. b. Continue the infusion while elevating the arm. c. Select an alternate intravenous site and administer the infusion more slowly. d. Request central venous access.

ANS: C These signs indicate thrombophlebitis. The nurse should select an alternative IV site and administer the infusion more slowly. The IV should not be continued in the same site, because necrosis may occur. A central line would be indicated only for long-term administration of antibiotics.

A healthcare worker who is asymptomatic has a screening TST result of 10 mm of induration during a pre-employment physical. What will the nurse reading this test tell the patient? a. "This is a negative test, so you are cleared for employment." b. "You have latent TB and will need to take isoniazid for 6 to 9 months." c. "You need to have a chest radiograph and a sputum culture." d. "You will begin taking a four-drug regimen to treat tuberculosis."

ANS: C This patient has a moderate risk and a positive skin test result. Before beginning treatment for latent TB, active TB must be ruled out with chest X-rays and sputum cultures. This is not a negative test result in a moderate-risk individual. Latent TB needs to be confirmed with diagnostic testing. Treatment will not begin until tests have confirmed the diagnosis.

A patient who is a chronic alcoholic is admitted to the hospital. Admission laboratory work reveals a magnesium level of 1.2 mEq/L. The prescriber orders intravenous magnesium sulfate in a 10% solution at a rate of 10 mL/min. What will the nurse do? a. Administer the IV dose as ordered and have calcium gluconate on hand. b. Administer the IV dose and make preparations for mechanical ventilation. c. Hold the IV dose until the infusion rate has been clarified with the provider. d. Request an order for renal function tests before administering the IV dose

ANS: C This patient has hypomagnesemia and should be given magnesium sulfate intravenously. The percent of magnesium in solution is correct; however, magnesium should not be infused faster than 1.5 mL/min, so the nurse is correct to question the rate of infusion. Calcium gluconate should be available when magnesium is given, but the nurse needs to clarify the rate of infusion first. Mechanical ventilation is necessary with excessive magnesium. Renal function tests are not indicated.

6. A patient will be receiving a thrombolytic drug as part of the treatment for acute myocardial infarction. The nurse explains to the patient that this drug is used for which purpose? a. To relieve chest pain b. To prevent further clot formation c. To dissolve the clot in the coronary artery d. To control bleeding in the coronary vessels

ANS: C Thrombolytic drugs lyse, or dissolve, thrombi. They are not used to prevent further clot formation or to control bleeding. As a result of dissolving of the thrombi, chest pain may be relieved, but that is not the primary purpose of thrombolytic therapy. DIF: COGNITIVE LEVEL: Understanding (Comprehension) REF: p. 414 TOP: NURSING PROCESS: Implementation

A clinic patient who has cirrhosis of the liver develops hypervolemic hpernatremia. Which medication will the nurse expect the provider to order? a.Conivaptan [Vaprisol] b.Desmopressin [DDAVP] c.Tolvaptan [Samsca] d.Vasopressin [Pitressin]

ANS: C Tolvaptan is an oral vasopressin antagonist indicated for reducing hypernatremia in patients with euvolemic or hypervolemic hypernatremia. Conivaptan has the same indications, but it is administered IV for hospitalized patients. Desmopressin and vasopressin are antidiuretic hormone replacement medications that will make the condition worse.

1. A patient who is diagnosed with shingles is taking topical acyclovir, and the nurse is providing instructions about adverse effects. The nurse will discuss which adverse effects of topical acyclovir therapy? a. Insomnia and nervousness b. Temporary swelling and rash c. Burning when applied d. This medication has no adverse effects.

ANS: C Transient burning may occur with topical application of acyclovir. The other options are incorrect. DIF: COGNITIVE LEVEL: Understanding (Comprehension) REF: p. 646 TOP: NURSING PROCESS: Implementation

A nurse working in a family planning clinic is teaching a class on intrauterine devices (IUDs). Which patient should be advised against using an IUD for contraception? a. A 45-year-old married woman with four children b. A 30-year-old monogamous married woman c. An 18-year-old woman with multiple sexual partners d. A 35-year-old woman with a history of rosacea

ANS: C Women at risk for sexually transmitted diseases (STDs) should not use an IUD, because the risk of infection is higher. Women who have multiple sexual partners are especially at risk for STDs. Monogamous married women are less apt to contract STDs. Women with rosacea can use an IUD.

1. When teaching a patient who will be receiving antihistamines, the nurse will include which instructions? (Select all that apply.) a. "Antihistamines are generally safe to take with over-the-counter medications." b. "Take the medication on an empty stomach to maximize absorption of the drug." c. "Take the medication with food to minimize gastrointestinal distress." d. "Drink extra fluids if possible." e. "Antihistamines may cause restlessness and disturbed sleep." f. "Avoid activities that require alertness until you know how adverse effects are tolerated."

ANS: C, D, F Antihistamines should be taken with food, even though this slightly reduces the absorption of the drug, so as to minimize the gastrointestinal upset that can occur. Over-the-counter medications must not be taken with an antihistamine unless approved by the physician because of the serious drug interactions that may occur. Drinking extra fluids will help to ease the removal of secretions, and activities that require alertness, such as driving, must not be engaged in until the patient knows how he or she responds to the sedating effects of antihistamines. DIF: COGNITIVE LEVEL: Applying (Application) REF: p. 575 TOP: NURSING PROCESS: Implementation

A patient is taking digoxin [Lanoxin] and develops a dysrhythmia. The nurse reports this finding to the prescriber, who will most likely order what? (Select all that apply.) a. Amiodarone b. Diltiazem c. Phenytoin [Dilantin] d. Quinidine e. Serum electrolytes

ANS: C, E Phenytoin is an antiseizure medication used to treat digoxin-induced dysrhythmias. Because digoxin-induced dysrhythmias can be caused by hypokalemia, it is appropriate to evaluate the serum electrolyte levels. Amiodarone, diltiazem, and quinidine increase digoxin levels.

A patient who is taking didanosine [Videx] reports nausea, vomiting, and abdominal pain. What will the nurse recommend to this patient? a."Take the drug with food to minimize these side effects." b."Stop taking the drug immediately and resume taking it once your symptoms subside." c."Take the medication in the evening to avoid experiencing these kinds of symptoms." d."You will need laboratory tests to determine if these are serious effects of the drug."

ANS: D As with all NRTIs, pancreatitis may occur and may manifest as nausea, vomiting, and abdominal pain. The patient will need evaluation of serum amylase, triglycerides, and calcium. Taking the drug with food or at a different time of day are not indicated. It is not correct to discontinue the drug and to resume it when symptoms subside, since pancreatitis may be fatal.

5. A nursing student asks a nurse working in the newborn nursery how vitamin K deficiency is treated in newborns. Which response by the nurse is correct? a. "A newborn infant's intestinal flora will produce vitamin K." b. "If coagulation studies show a bleeding disorder, oral menadione is given." c. "Menadione is given intravenously shortly after delivery." d. "Phytonadione is given intramuscularly immediately after delivery."

ANS: D A normal infant is born vitamin K deficient, and all infants are given an IM injection of phytonadione immediately after delivery. Newborns have not developed the intestinal flora needed to produce sufficient amounts of vitamin K. Menadione is not used in infants because of the increased risk of hyperbilirubinemia associated with its use. PTS: 1 DIF: Cognitive Level: Application REF: p. 991

1. A patient has a new order for the adrenergic drug doxazosin (Cardura). When providing education about this drug, the nurse will include which instructions? a. "Weigh yourself daily, and report any weight loss to your prescriber." b. "Increase your potassium intake by eating more bananas and apricots." c. "The impaired taste associated with this medication usually goes away in 2 to 3 weeks." d. "Be sure to lie down after taking the first dose, because first-dose hypotension may make you dizzy."

ANS: D A patient who is starting doxazosin should take the first dose while lying down because there is a first-dose hypotensive effect with this medication. The other options are incorrect. DIF: COGNITIVE LEVEL: Analyzing (Analysis) REF: p. 354 TOP: NURSING PROCESS: Implementation

13. A patient asks how she can consume optimal amounts of folate, because she is trying to get pregnant. The nurse will advise this patient to do what? a. Eat foods naturally high in folate. b. Stop worrying, because folate is present in many foods. c. Take synthetic folate on an empty stomach. d. Take synthetic folate in addition to foods high in folate.

ANS: D All women of child-bearing age need 400 to 800 mg of synthetic folate in addition to dietary sources to help prevent neural tube defects in the fetus. The bioavailability of synthetic folate is increased in the presence of food. Dietary folate is not sufficient to prevent neural tube defects, even when these foods are consumed in increased amounts. Taking folate on an empty stomach reduces the amount absorbed. PTS: 1 DIF: Cognitive Level: Application REF: p. 994

11. A 79-year-old patient is taking a diuretic for treatment of hypertension. This patient is very independent and wants to continue to live at home. The nurse will know that which teaching point is important for this patient? a. He should take the diuretic with his evening meal. b. He should skip the diuretic dose if he plans to leave the house. c. If he feels dizzy while on this medication, he needs to stop taking it and take potassium supplements instead. d. He needs to take extra precautions when standing up because of possible orthostatic hypotension and resulting injury from falls.

ANS: D Caution must be exercised in the administration of diuretics to the older adults because they are more sensitive to the therapeutic effects of these drugs and are more sensitive to the adverse effects of diuretics, such as dehydration, electrolyte loss, dizziness, and syncope. Taking the diuretic with the evening meal may disrupt sleep because of nocturia. Doses should never be skipped or stopped without checking with the prescriber. DIF: COGNITIVE LEVEL: Applying (Application) REF: p. 460 TOP: NURSING PROCESS: Implementation

11. A patient has a skin infection and the culture reveals methicillin-resistant Staphylococcus aureus (MRSA). What is an appropriate treatment for this patient? a. Cefaclor b. Cefazolin c. Cefotaxime d. Ceftaroline

ANS: D Ceftaroline is a fifth-generation cephalosporin with a spectrum similar to third-generation cephalosporins but also with activity against MRSA. Cefaclor is a second-generation cephalosporin. Cefazolin is a first-generation cephalosporin. Cefotaxime is a third-generation cephalosporin.

8. The nurse is caring for a patient who will begin receiving intravenous ciprofloxacin [Cipro] to treat pyelonephritis. The nurse learns that the patient has a history of myasthenia gravis. Which action by the nurse is correct? a. Administer the ciprofloxacin and monitor the patient for signs of muscle weakness. b. Ask the provider whether the ciprofloxacin can be given orally. c. Request an order for concurrent administration of metronidazole [Flagyl]. d. Suggest that the provider order a different antibiotic for this patient.

ANS: D Ciprofloxacin and other fluoroquinolones can exacerbate muscle weakness in patients with myasthenia gravis and should not be given to these patients. It is not correct to administer the drug and monitor for this effect. Giving the drug by a different route will not alter this effect. Metronidazole is given when C. difficile occurs.

6. The nurse is reviewing the medication orders for a patient who will be receiving gentamicin therapy. Which other medication or medication class, if ordered, would be a potential interaction concern? a. Calcium channel blockers b. Phenytoin c. Proton pump inhibitors d. Loop diuretics

ANS: D Concurrent use of aminoglycosides, such as gentamicin, with loop diuretics increases the risk for ototoxicity. The other drugs and drug classes do not cause interactions. DIF: COGNITIVE LEVEL: Applying (Application) REF: p. 627 TOP: NURSING PROCESS: Assessment

1. A patient, newly diagnosed with hypothyroidism, receives a prescription for a thyroid hormone replacement drug. The nurse assesses for which potential contraindication to this drug? a. Infection b. Diabetes mellitus c. Liver disease d. Recent myocardial infarction

ANS: D Contraindications to thyroid preparations include known drug allergy to a given drug product, recent myocardial infarction, adrenal insufficiency, and hyperthyroidism. The other options are incorrect. DIF: COGNITIVE LEVEL: Understanding (Comprehension) REF: p. 493 TOP: NURSING PROCESS: Assessment

8. A patient has been started on therapy of a continuous infusion of lidocaine after receiving a loading dose of the drug. The nurse will monitor the patient for which adverse effect? a. Drowsiness b. Nystagmus c. Dry mouth d. Convulsions

ANS: D Convulsions are possible if lidocaine reaches toxic levels. The other options are not adverse effects of lidocaine. DIF: COGNITIVE LEVEL: Applying (Application) REF: p. 401 TOP: NURSING PROCESS: Evaluation

5. The nurse will prepare to give which preparation to a newborn upon arrival in the nursery after delivery? a. Vitamin B1 (thiamine) b. Vitamin D (calciferol) c. Folic acid d. Vitamin K (AquaMEPHYTON)

ANS: D Deficiency in vitamin K can be seen in newborns because of malabsorption attributed to inadequate amounts of bile. AquaMEPHYTON is given as a single intramuscular dose for infants upon arrival in the nursery. DIF: COGNITIVE LEVEL: Understanding (Comprehension) REF: p. 847 TOP: NURSING PROCESS: Implementation

2. The nurse recognizes that use of estrogen drugs is contraindicated in which patient? a. A patient who has atrophic vaginitis b. A patient who has inoperable prostate cancer c. A woman who has just given birth and wants to prevent postpartum lactation d. A woman with a history of thrombophlebitis

ANS: D Estrogenic drugs are contraindicated in people who have active thromboembolic disorders and in those with histories of thromboembolic disease. Atrophic vaginitis and inoperable prostate cancer are potential indications for estrogen therapy. Estrogen is not used to prevent lactation. DIF: COGNITIVE LEVEL: Understanding (Comprehension) REF: p. 537 TOP: NURSING PROCESS: Assessment

A child with Prader-Willi syndrome (PWS) has short stature, and the provider is considering treatment with growth hormone. Which aspect of this child's history should the nurse report to the provider? a.Behavior problems b.Low muscle tone c.Mental impairment d.Sleep apnea

ANS: D Growth hormone must be avoided in patients with PWS who are severely obese, who have a history of upper airway obstruction or sleep apnea, or who have severe respiratory impairment because of a risk of sudden death associated with these disorders. Behavior problems, low muscle tone, and mental impairment are commonly associated with PWS and are not contraindications to treatment with growth hormone

1. Two patients arrive at the clinic; one is a young boy with sickle cell anemia, and another is a 57-year-old woman with early stages of Hodgkin's disease. The nurse notices that both patients need the same vaccine. What vaccine would that be? a. Varicella virus vaccine (Varivax) b. Herpes zoster vaccine (Zostavax) c. Hepatitis B virus vaccine, inactivated (Recombivax HB) d. Haemophilus influenzae type b (Hib) vaccine

ANS: D H. influenzae type b conjugate vaccine is usually given to patients with one of these disorders: sickle cell anemia, an immunodeficiency syndrome, Hodgkin's disease, and others. The other options are incorrect. DIF: COGNITIVE LEVEL: Applying (Application) REF: p. 788 TOP: NURSING PROCESS: Assessment

A patient with strongyloidiasis is being treated with ivermectin [Stromectol]. What will the nurse expect to teach this patient? a. A Mazotti-type reaction is likely. b. Hypotension is a common side effect of the drug. c. The drug should be taken every 6 to 12 months until symptoms have cleared. d. The drug should be taken with water for 1 to 2 days.

ANS: D Ivermectin is given for 1 to 2 days to treat strongyloidiasis. A Mazotti-type reaction occurs when ivermectin is used to treat onchocerciasis. Hypotension is not a common side effect. Ivermectin is given every 6 to 12 months when treating onchocerciasis.

1. A pregnant patient is treated with trimethoprim/sulfamethoxazole (TMP/SMZ) (Bactrim) for a urinary tract infection at 34 weeks' gestation. A week later, the woman delivers her infant prematurely. The nurse will expect to monitor the infant for: a. birth defects. b. hypoglycemia. c. rash. d. scleral jaundice.

ANS: D Kernicterus is a disorder in newborns caused by deposition of bilirubin in the brain. Sulfonamides promote kernicterus by displacing bilirubin from plasma proteins. Sulfonamides should not be given to infants under 2 months of age or to pregnant women near term. Jaundice is an early sign of elevated bilirubin, so the nurse should assess for scleral jaundice. Sulfonamides do not cause birth defects or hypoglycemia. Serious rashes may occur but are not the primary concern in the newborn.

7. A provider orders intravenous moxifloxacin [Avelox] for a patient who has sinusitis. Before administering the drug, the nurse will review this patient's chart for: a. a history of asthma. b. concurrent use of digoxin. c. concurrent use of warfarin. d. recent serum electrolyte levels.

ANS: D Moxifloxacin prolongs the QT interval and poses a risk of serious dysrhythmias. Patients with hypokalemia have an increased risk, so serum electrolyte levels should be monitored. Having a history of asthma is not significant. Moxifloxacin does not alter digoxin or warfarin levels.

A patient is experiencing toxic side effects from atropine, including delirium and hallucinations. Which medication will the nurse expect to administer? a.Donepezil [Aricept] b.Edrophonium [Reversol] c.Neostigmine [Prostigmin] d.Physostigmine

ANS: D Physostigmine is the drug of choice for treating poisoning from atropine and other drugs that cause muscarinic blockade. Donepezil is used to treat Alzheimer's disease. Edrophonium is used to distinguish between a myasthenic crisis and a cholinesterase inhibitor overdose. Neostigmine does not cross the blood-brain barrier and would not effectively treat this patient's CNS symptoms.

3. The nurse will assess the patient for which potential contraindication to antitubercular therapy? a. Glaucoma b. Anemia c. Heart failure d. Hepatic impairment

ANS: D Results of liver function studies (e.g., bilirubin level, liver enzyme levels) need to be assessed because isoniazid and rifampin may cause hepatic impairment; severe liver dysfunction is a contraindication to these drugs. In addition, the patient's history of alcohol use needs to be assessed. DIF: COGNITIVE LEVEL: Understanding (Comprehension) REF: p. 662 TOP: NURSING PROCESS: Assessment

5. A patient with hypertension is taking furosemide [Lasix] for congestive heart failure. The prescriber orders digoxin to help increase cardiac output. What other medication will the nurse expect to be ordered for this patient? a. Bumetanide [Bumex] b. Chlorothiazide [Diuril] c. Hydrochlorothiazide [HydroDIURIL] d. Spironolactone [Aldactone]

ANS: D Spironolactone is used in conjunction with furosemide because of its potassium-sparing effects. Furosemide can contribute to hypokalemia, which can increase the risk of fatal dysrhythmias, especially with digoxin administration. The other diuretics listed are all potassium-wasting diuretics.

4. A nurse preparing to administer morning medications notes that a patient with a history of hypertension has been prescribed spironolactone [Aldactone]. The nurse assesses the patient and notes dyspnea, bilateral crackles, and pitting edema in both feet. Which intervention is appropriate? a. Administer the medications as ordered. b. Ask the patient about the use of salt substitutes. c. Contact the provider to request an order for serum electrolytes. d. Request an order for furosemide [Lasix].

ANS: D Spironolactone takes up to 48 hours for its effects to develop, so it should not be used when the patient needs immediate diuresis. This patient has shortness of breath, crackles, and edema and needs a short-acting diuretic, such as furosemide. Asking the patient about the use of salt substitutes is not indicated. The patient does not need assessment of serum electrolytes.

9. A patient has been taking levothyroxine (Synthroid) for more than one decade for primary hypothyroidism. Today she calls because she has a cousin who can get her the same medication in a generic form from a pharmaceutical supply company. Which is the nurse's best advice? a. "This would be a great way to save money." b. "There's no difference in brands of this medication." c. "This should never be done; once you start with a certain brand, you must stay with it." d. "It's better not to switch brands unless we check with your doctor."

ANS: D Switching brands of levothyroxine during treatment can destabilize the course of treatment. Thyroid function test results need to be monitored more carefully when switching products. DIF: COGNITIVE LEVEL: Analyzing (Analysis) REF: p. 496 TOP: NURSING PROCESS: Implementation

A patient is beginning treatment for active tuberculosis (TB) in a region with little drug-resistant TB. Which treatment regimen will be used initially? a. Isoniazid and pyrazinamide b. Isoniazid, pyrazinamide, and ethambutol c. Rifampin, pyrazinamide, and ethambutol d. Isoniazid, rifampin, pyrazinamide, and ethambutol

ANS: D The induction phase of treatment for patients in a region without drug resistance is the same as for patients who are human immunodeficiency virus (HIV) negative or HIV positive and includes isoniazid, rifampin, pyrazinamide, and ethambutol. It is not correct to begin with two drugs. The three-drug regimen is used for inductions in areas with resistance to either isoniazid or rifampin.

2. The nurse is giving intravenous nitroglycerin to a patient who has just been admitted because of an acute myocardial infarction. Which statement is true regarding the administration of the intravenous form of this medication? a. The solution will be slightly colored green or blue. b. The intravenous form is given by bolus injection. c. It can be given in infusions with other medications. d. Non-polyvinylchloride (non-PVC) plastic intravenous bags and tubing must be used.

ANS: D The non-PVC infusion kits are used to avoid absorption and/or uptake of the nitrate by the intravenous tubing and bag and/or decomposition of the nitrate. The medication is given by infusion via an infusion pump and not with other medications. It is not given by bolus injection. If the parenteral solution is discolored blue or green, it should be discarded. DIF: COGNITIVE LEVEL: Applying (Application) REF: p. 372 TOP: NURSING PROCESS: Implementation

1. A patient is taking intravenous aminophylline for a severe exacerbation of chronic obstructive pulmonary disease. The nurse will assess for which therapeutic response? a. Increased sputum production b. Increased heart rate c. Increased respiratory rate d. Increased ease of breathing

ANS: D The therapeutic effects of bronchodilating drugs such as xanthine derivatives include increased ease of breathing. The other responses are incorrect. DIF: COGNITIVE LEVEL: Applying (Application) REF: p. 593 TOP: NURSING PROCESS: Evaluation

8. A nurse is caring for a patient who recently immigrated from a Third World country. The patient is thin and appears malnourished. The nurse notes that the patient has loose and missing teeth, gingivitis, and bleeding gums. The patient has multiple sores and ecchymoses. The nurse will expect the provider to order: a. cyanocobalamin (vitamin B12). b. high-dose nicotinic acid. c. intramuscular thiamine for 1 to 2 weeks. d. intravenous ascorbic acid.

ANS: D This patient has scurvy with acute, severe symptoms and needs intravenous vitamin C. The patient does not have signs of vitamin B12 deficiency, niacin deficiency, or thiamine deficiency. PTS: 1 DIF: Cognitive Level: Application REF: p. 992

11. A patient has a new prescription for the transdermal form of scopolamine. The nurse knows that this form of scopolamine is used for which condition? a. Angina b. Chronic pain c. Hypertension d. Motion sickness

ANS: D Transdermal scopolamine (Transderm-Scop) is a patch that can be applied just behind the ear 4 to 5 hours before travel for the prevention of motion sickness. The other options are incorrect. DIF: COGNITIVE LEVEL: Applying (Application) REF: p. 335 TOP: NURSING PROCESS: Planning

3. A child is diagnosed with rickets. The nurse knows that this child is most likely deficient in which vitamin? a. Niacin (nicotinic acid) b. Thiamin (vitamin B1) c. Vitamin C (ascorbic acid) d. Vitamin D

ANS: D Vitamin D plays a critical role in the regulation of the metabolism of calcium and phosphorus. Classical effects of deficiency are rickets in children and osteomalacia in adults. Niacin, thiamin, and vitamin C do not play a role in the prevention of rickets. PTS: 1 DIF: Cognitive Level: Analysis REF: p. 990

14. A patient who is taking azathioprine [Imuran] to prevent rejection of a renal transplant develops gout and the provider orders allopurinol. The nurse will contact the provider to discuss: a. decreasing the allopurinol dose. b. decreasing the azathioprine dose. c. increasing the allopurinol dose. d. increasing the azathioprine dose.

B Allopurinol delays conversion of mercaptopurine to inactive products and increases the risk of azathioprine toxicity. Patients taking the two concurrently should have an approximate 70% reduction in the azathioprine dose. Altering the allopurinol dose is not indicated.

A female patient who begins taking spironolactone [Aldactone] as an adjunct to furosemide [Lasix] complains that her voice is deepening. What will the nurse do? a. Contact the provider to obtain an order for a complete blood count (CBC) and liver function tests. b. Explain that this drug binds with receptors for steroid hormones, causing this effect. c. Report this side effect to the provider and request another medication for this patient. d. Teach the patient to report any associated cough, which may indicate a more severe side effect.

B Spironolactone binds with receptors for aldosterone and with other steroid hormones, causing side effects such as gynecomastia, menstrual irregularities, impotence, hirsutism, and deepening of the voice. There is no indication for a CBC or for liver function tests in this patient with this side effect. Deepening of the voice is not an indication for withdrawal of the drug. This side effect is not associated with cough.

7. A nurse is teaching a patient who is about to undergo allograft transplantation of the liver. Which statement by the patient indicates understanding of the post-transplant medications? a. "Immunosuppressants help reduce the risk of postoperative infection." b. "I will need to have periodic laboratory work to assess for toxicity." c. "I will need to take immunosuppressants until all signs of organ rejection are gone." d. "These drugs will prevent organ rejection."

B To prevent toxicity from high drug levels or organ rejection from low levels, blood levels of immunosuppressants should be checked periodically. Immunosuppressants do not prevent infection; they increase the risk of infection. Patients must take immunosuppressants for life. Immunosuppressants do not prevent organ rejection; they help minimize the risk.

The nurse is beginning a heparin infusion for a patient with evolving stroke. The baseline activated partial thromboplastin time (aPTT) is 40 seconds. Which aPTT value indicates that a therapeutic dose has been achieved? A. 50 B. 70 C. 90 D. 110

B. 70 The therapeutic level of heparin is achieved when the aPTT reaches 1.5 to 2 times normal. Thus, a range of 60 to 80 seconds would be appropriate for this patient.

10. The nurse is preparing to administer basiliximab [Simulect] to a patient to prevent acute rejection. By which route will the nurse administer the drug? a. Oral b. Intramuscular c. Intravenous d. Transdermal

C Basiliximab is used to prevent acute rejection and is administered intravenously.

When monitoring for the therapeutic effects of intranasal desmopressin (DDAVP) in a patient who has diabetes insidious, which assessment finding will the nurse look for as an indication that the medication therapy is successful? A. Increased insulin levels B. Decreased diarrhea C. Improved nasal patency D. Decreased thirst

D. Decreased thirst

C. reduces inflammation in the airway

During a teaching session for a patient who will be receiving a new prescription for the LTRA montelukast (singulair), the nurse will tell the patient that the drug has which therapeutic effects A. improves respiratory drive B. loosens and removes thickened secretions C. reduces inflammation in the airway D. stimulates immediate bronchodilators

34. Adverse effects for adrenal drugs of this system include: - Fragile skin, petechiae, ecchymosis, facial erythema, poor wound healing, hirsutism, urticaria A. Cardiovascular B. CNS C. Endocrine D. GI E. Integumentary F. Musculoskeletal G. Ocular H. Other

E

B. dizziness C. "hangover" effect D. drowsiness F. dry mouth

The nurse is giving an antihistamine and will observe the patient for which side effects? Select all that apply.... A. hypertension B. dizziness C. "hangover" effect D. drowsiness E. tachycardia F. dry mouth

C. intranasal steroids

The nurse is providing education to a patient with a history of chronic nasal congestion secondary to allergic rhinitis. Which class of medications would the nurse anticipate the provider would recommend for the patient to use on a long-term basis? A. Anititussives B. Antihistamines C. Intranasal steroids D. Expectorants

B. Neurologic status

What system assessment will the nurse monitor in a patient due to a known common side effect of traditional antihistamines? A. Cardiac status B. Neurologic status C. Respiratory status D. Gastrointestinal status

ANS: D Low potassium levels can enhance paralysis, whereas high potassium levels can reduce paralysis. Because electrolyte status can influence the depth of neuromuscular blockade, it is important to maintain normal electrolyte balance. There are no indications that sodium, calcium, and/or magnesium have effects similar to those of potassium in relation to neuromuscular blocking agents

While preparing a patient for a procedure in which a neuromuscular blocking agent will be used, the nurse reviews the patient's preprocedure laboratory values. Which abnormality would cause the most concern? a.Hyponatremia b.Hypercalcemia c.Hypomagnesemia d.Hypokalemia

In administering nitroglycerin to a patient with angina, the nurse correlates its therapeutic effects primarily with what? A) Peripheral venous dilation B) Coronary artery relaxation C) Coronary vein constriction D) Peripheral arterial constriction

a Nitroglycerin works directly on the smooth muscles in the vessel walls, leading to vasodilation. Nitroglycerin works primarily on the veins and decreases cardiac preload and, ultimately, stress to the heart.

A patient is being treated for a systemic fungal infection with amphotericin B [Abelcet] and will be discharged home from the hospital to complete every other day infusions of the medication for 6 to 8 weeks. The nurse provides discharge teaching before dismissal. Which statement by the patient indicates a need for further teaching? a. "I may experience headaches and pain in my lower back, legs, and abdomen." b. "I may need to take potassium supplements while taking this drug." c. "I should take acetaminophen and diphenhydramine before each infusion." d. "I will need to have blood drawn for serum creatinine and BUN levels every 3 to 4 days."

a. "I may experience headaches and pain in my lower back, legs, and abdomen."

A nurse is teaching a group of parents about the role of testosterone in puberty for boys. To evaluate the group's understanding, the nurse asks, "What physiologic effects related to testosterone can you expect to see in your sons?" Which responses indicate an understanding of the role of testosterone in male puberty? (Select all that apply.) a. "It promotes skeletal muscle growth." b. "It increases height and weight." c. "It delays epiphyseal closure." d. "It causes a high-pitched voice." e. "It causes acne."

a. "It promotes skeletal muscle growth." b. "It increases height and weight." e. "It causes acne."

Which superficial mycosis is generally treated with oral antifungal agents? a. Tinea capitis b. Tinea corporis c. Tinea cruris d. Tinea pedis

a. Tinea capitis

A patient has a Pseudomonas aeruginosa infection that is sensitive to aminoglycosides, and the prescriber orders gentamicin. The patient tells the nurse that a friend received amikacin [Amikin] for a similar infection and wonders why amikacin was not ordered. What will the nurse tell the patient? a. "Amikacin is given when infectious agents are resistant to other aminoglycosides." b. "Amikacin is more vulnerable to inactivation by bacterial enzymes." c. "Amikacin is a narrow-spectrum drug and will probably not work for this infection." d. "Gentamicin is less toxic to the ears and the kidneys."

a. "Amikacin is given when infectious agents are resistant to other aminoglycosides." Resistance to amikacin is uncommon at this point; to minimize the emergence of amikacin-resistant bacteria, this drug is reserved for infections in which resistance to other aminoglycosides has developed. Amikacin is the least susceptible to inactivation by bacterial enzymes. Amikacin is a broad-spectrum antibiotic. All aminoglycosides are ototoxic and nephrotoxic.

A nurse is explaining to nursing students why a cephalosporin is used in conjunction with an aminoglycoside for a patient with an infection. Which statement by a student indicates understanding of the teaching? a. "Cephalosporins enhance the actions of aminoglycosides by weakening bacterial cell walls." b. "Cephalosporins prevent neuromuscular blockade associated with aminoglycosides." c. "Cephalosporins prolong the postantibiotic effects of the aminoglycosides so doses can be decreased." d. "Cephalosporins reduce bacterial resistance to aminoglycosides."

a. "Cephalosporins enhance the actions of aminoglycosides by weakening bacterial cell walls." Cephalosporins, penicillins, and vancomycin can be used in conjunction with aminoglycosides; these drugs weaken the bacterial cell wall and enhance the bactericidal actions of aminoglycosides. Cephalosporins do not prevent neuromuscular blockade. They do not prolong the postantibiotic effects of aminoglycosides. They do not affect bacterial resistance.

A patient is admitted to the unit for treatment for an infection. The patient receives IV amikacin [Amikin] twice a day. When planning for obtaining a peak aminoglycoside level, when should the nurse see that the blood is drawn? a. 30 minutes after the IV infusion is complete b. 1 hour after the IV infusion is complete c. 1 hour before administration of the IV infusion d. A peak level is not indicated with twice-daily dosing.

a. 30 minutes after the IV infusion is complete When divided daily doses are used, blood samples for measurement of peak levels are drawn 1 hour after IM injection and 30 minutes after completion of an IV infusion. This medication is administered IV, so blood draws must follow 30 minutes after infusion to obtain peak levels. Measurement of peak levels is unnecessary only when a single daily dose is used.

Which patients may receive gentamicin safely? (Select all that apply.) a. A 6-day-old newborn b. A 5-year-old child c. A pregnant woman d. A woman breast-feeding an infant e. An elderly patient with renal disease

a. A 6-day-old newborn b. A 5-year-old child d. A woman breast-feeding an infant Aminoglycosides may be safely given to infants younger than 8 days old and to children and adolescents. Use during pregnancy is not recommended because they may harm the fetus. Gentamicin is probably safe during lactation. Gentamicin is used with caution in older patients with reduced renal function

A patient who is taking gentamicin and a cephalosporin for a postoperative infection requests medication for mild postsurgical pain. The nurse will expect to administer which of the following medications? a. Acetaminophen b. Aspirin c. Ibuprofen d. Morphine

a. Acetaminophen Gentamicin and cephalosporins are both nephrotoxic. This patient should avoid taking other potentially nephrotoxic drugs. Acetaminophen is not nephrotoxic and may be given for mild pain. Aspirin and ibuprofen are both nephrotoxic. Morphine is not nephrotoxic but is not indicated for mild pain.

A patient is diagnosed with an infection caused by Staphylococcus aureus, and the prescriber orders intravenous gentamicin and penicillin (PCN). Both drugs will be given twice daily. What will the nurse do? a. Administer gentamicin, flush the line, and then give the penicillin. b. Give the gentamicin intravenously and the penicillin intramuscularly. c. Infuse the gentamicin and the penicillin together to prevent fluid overload. d. Request an order to change the penicillin to vancomycin.

a. Administer gentamicin, flush the line, and then give the penicillin. Gentamicin should not be infused with penicillins in the same solution, because PCN inactivates gentamicin; therefore, the nurse should give one first, flush the line, and then give the other. The nurse cannot give a drug IM when it is ordered IV without an order from the prescriber. These two drugs should not be infused in the same solution. There is no indication for changing the PCN to vancomycin; that should be done for serious infections.

A patient who has been receiving intravenous gentamicin for several days reports having had a headache for 2 days. The nurse will request an order to: a. discontinue the gentamicin. b. obtain a gentamicin trough before the next dose is given c. give an analgesic to control headache discomfort. d. obtain renal function tests to evaluate for potential nephrotoxicity.

a. discontinue the gentamicin. A persistent headache may be a sign of developing ototoxicity, and since ototoxicity is largely irreversible, gentamicin should be withdrawn at the first sign of developing ototoxicity. A gentamicin trough should be obtained before the next dose is given when high gentamicin levels are suspects. Analgesics are not indicated until a serious cause of the headache has been ruled out. A headache is an early sign of ototoxicity, not nephrotoxicity.

Preparation for the NCLEX® Examination Questions 7. Which client statement demonstrates understanding of teaching by the nurse regarding the use of histamine 2 - receptor antagonists? a. "Because I am taking this medication, it is OK for me to eat spicy foods." b. "Smoking decreases the effects of this medication, so I should look into cessation programs." c. "I should take this medication 1 hour after each meal to maximally decrease gastric acidity." d. "I should increase bulk and fluids in my diet to prevent constipation."

b "Smoking decrease the effect of this medication, so I should look into cessation programs." Clients taking histamine 2 -receptor - blocking drugs should avoid spicy foods, extremes in temperatures, alcohol, and smoking. Diarrhea, not constipation, is a GI adverse effect. Whereas cimetidine should be taken with meals, famotidine can be taken without regard to meals.

NCLEX EXAMINATION REVIEW QUESTIONS 3. Which is the correct action when the nurse is administering sucralfate? a. Giving the drug with meals b. Giving the drug on an empty stomach c. Instructing the patient to restrict fluids d. Waiting 30 minutes before administering other drugs

b Giving the drug on an empty stomach

Preparation for the NCLEX® Examination Questions 6. What is the mechanism of action for Famotidine (Pepcid)? a. It forms a protective coating against gastric acid, pepsin, and bile salts. b. It competes with histamine for binding sites on the parietal cells c. It irreversibly binds to the hydrogen-potassium-adenosine triphosphate (ATPas) pump. d. It causes a decrease in stomach pH, reducing stomach acidity.

b It competes with histamine for binding sites on the parietal cells. Histamine receptor-blocking drugs decrease gastric acid by competing with histamine for binding sites on the parietal cells.

Preparation for the NCLEX® Examination Questions 8. How will the nurse describe the action of proton pump inhibitors (PPIs)? a. They form a protective barrier that can be thought of as a liquid bandage. b. They irreversibly bind to the hydrogen-potassium-ATPase pump. c. They compete with histamine for binding sites on the parietal cells d. They help to neutralize acid secretions to promote gastric mucosal defensive mechanisms.

b They irreversibly bind to the hydrogen-potassium-ATPase pump. PPIs work to block the final step in the acid-secreting mechanisms of the proton pump. They do this by irreversibly binding to the ATPase pump, H+/K+/ATPase, the enzyme for this step.

A patient is taking oral ketoconazole [Nizoral] for a systemic fungal infection. The nurse reviews the medication administration record and notes that the patient is also taking omeprazole [Prilosec] for reflux disease. What action should the nurse take? a. Administer the omeprazole 1 hour before the ketoconazole. b. Administer the omeprazole at least 2 hours after the ketoconazole. c. Confer with the prescriber about a potential hazardous interaction. d. The nurse should not administer omeprazole to a patient receiving ketoconazole.

b. Administer the omeprazole at least 2 hours after the ketoconazole.

A nurse is caring for a patient who is receiving amphotericin B [Abelcet] for a systemic fungal infection. In spite of receiving diphenhydramine and acetaminophen before initiation of treatment, the patient has fever and chills with rigors. The nurse will contact the provider to discuss the addition of which drug? a. Aspirin b. Dantrolene c. Hydrocortisone d. Omeprazole

b. Dantrolene

A patient with a history of congestive heart failure and renal impairment has esophageal candidiasis. Which antifungal agent would the nurse anticipate giving to this patient? a. Amphotericin B [Abelcet] b. Fluconazole [Diflucan] c. Itraconazole [Sporanox] d. Voriconazole [Vfend]

b. Fluconazole [Diflucan]

A patient is receiving intravenous voriconazole [Vfend]. Shortly after the infusion starts, the patient tells the nurse, "Colors look different, and the light hurts my eyes." What will the nurse do? a. Observe the patient closely for the development of hallucinations. b. Reassure the patient that these effects will subside in about 30 minutes. c. Stop the infusion and notify the provider of CNS toxicity. d. Tell the patient that this is an irreversible effect of the drug.

b. Reassure the patient that these effects will subside in about 30 minutes.

The nurse is providing patient education about glucocorticoid therapy to a patient preparing to be discharged home. Which statement made by the patient best demonstrates understanding of glucocorticoid therapy? a. "I will take the entire dose early with breakfast." b. "I may take two-thirds of the dose in the morning and one-third in the afternoon." c. "I will divide the dose in half and take half in the morning and half in the evening." d. "I will take a dose with each meal."

b. "I may take two-thirds of the dose in the morning and one-third in the afternoon." Patients should be instructed to follow the prescribed dosing schedule. Some prescribers recommend dividing the daily dose by taking two-thirds in the morning and one-third in the evening. Other prescribers recommend taking the entire daily dose at bedtime. Taking the dose at breakfast, dividing the dose in half, and taking a dose with each meal are not recommended dosing schedules.

The nurse is providing patient education for a postmenopausal patient who is considering EPT. Which risks associated with EPT should the nurse discuss with the patient? (Select all that apply.) a. Increased colon cancer b. Stroke c. Deep vein thrombosis d. Ovarian cancer e. Decreased bone density

b. Stroke c. Deep vein thrombosis d. Ovarian cancer Risk factors for EPT include stroke, deep vein thrombosis, and ovarian cancer. A decreased risk of colon cancer is associated with EPT. EPT preserves bone mineral density.

A patient who has been taking gentamicin for 5 days reports a headache and dizziness. What will the nurse do? a. Request an order for a gentamicin peak level. b. Suspect ototoxicity and notify the prescriber. c. Tell the patient to ask for help with ambulation. d. Tell the patient to report any tinnitus.

b. Suspect ototoxicity and notify the prescriber. Headache and dizziness are signs of ototoxicity, and the prescriber should be notified. A peak level is not indicated; it is more important to know the trough level. Telling the patient to ask for help with ambulation and to report tinnitus should both be done but neither one is the priority nursing action.

An older adult patient with chronic obstructive pulmonary disease (COPD) develops bronchitis. The patient has a temperature of 39.5°C. The nurse will expect the provider to: a. obtain a sputum culture and wait for the results before prescribing an antibiotic. b. order empiric antibiotics while waiting for sputum culture results. c. treat symptomatically, because antibiotics are usually ineffective against bronchitis. d. treat the patient with more than one antibiotic without obtaining cultures.

b. order empiric antibiotics while waiting for sputum culture results. Patients with severe infections should be treated while culture results are pending. If a patient has a severe infection or is at risk of serious sequelae if treatment is not begun immediately, it is not correct to wait for culture results before beginning treatment. Until a bacterial infection is ruled out, treating symptomatically is not indicated. Treating without obtaining cultures is not recommended.

The nurse correlates the pain associated with exertional angina with what? A) Coronary artery spasm B) Coronary artery thrombosis C) Partial coronary artery occlusion D) Complete coronary artery obstruction

c Decreased blood flow, with a resultant decrease in oxygen delivery, is the cause of pain with exertional angina. At rest the patient usually does not have pain because at rest the coronary perfusion is adequate. The decreased blood flow is most often caused by fatty deposits on the arterial wall. With complete occlusion, the patient could experience myocardial tissue death resulting from a lack of perfusion.

NCLEX EXAMINATION REVIEW QUESTIONS 8. The nurse is preparing to administer the first dose of misoprostol (Cytotec) for a patient who has been diagnosed with a gastric ulcer. What condition would be a contraindication to this medication? a. Hypothyroidism b. Type 2 diabetes mellitus c. Pregnancy d. Hypertension

c Pregnancy

A nurse is discussing intravenous amphotericin B treatment with a nursing student who is about to care for a patient with a systemic fungal infection. Which statement by the student indicates a need for further teaching? a. "A test dose of amphotericin B may be given to assess the patient's reaction." b. "If I see any precipitate in the IV solution, I should stop the infusion immediately." c. "Infusions of amphotericin B should be administered over 1 to 2 hours." d. "The IV site should be rotated frequently to reduce the risk of phlebitis."

c. "Infusions of amphotericin B should be administered over 1 to 2 hours."

A nursing student asks a nurse about flucytosine [Ancobon]. Which statement by the nurse is correct? a. "Flucytosine has a broad antifungal spectrum." b. "Irreversible neutropenia and thrombocytopenia may occur with this drug." c. "Resistance is common with this medication." d. "Severe hepatic injury is common and limits this drug's use."

c. "Resistance is common with this medication."

A child has ringworm of the scalp. A culture of the lesion reveals a dermatophytic infection. The nurse teaching the child's parents about how to treat this infection will include which statement? a. "Adverse effects of the medication include itching, burning, and erythema." b. "Apply the topical medication daily until at least 1 week after the rash is gone." c. "Your child will need to take this oral medication for 6 to 8 weeks." d. "You will use an antifungal shampoo to treat this infection."

c. "Your child will need to take this oral medication for 6 to 8 weeks."

A 12-year-old male patient diagnosed with hypogonadism will begin testosterone injections. What will the nurse include when teaching the family about this therapy? a. Annual x-rays of the hands and wrists are necessary to monitor epiphyseal closure. b. Gynecomastia may occur and is a common side effect. c. Injections are given every 2 to 4 weeks for 3 to 4 years. d. Use of this drug may lead to prostate cancer later in life.

c. Injections are given every 2 to 4 weeks for 3 to 4 years.

A patient will begin taking an immunosuppressant medication. The nurse learns that the patient has a history of frequent candidal infections. The nurse will expect the provider to order which drug as prophylaxis? a. Fluconazole [Diflucan] b. Ketoconazole c. Posaconazole [Noxafil] d. Voriconazole [Vfend]

c. Posaconazole [Noxafil]

A female patient who swims competitively admits to using anabolic steroids to increase her muscle mass and improve her performance. What will the nurse tell this patient? a. Anabolic steroids may actually regulate her periods. b. Breast size may increase as a result of this drug use. c. The risk of liver disease will increase. d. Voice changes and facial hair will reverse when she stops the drug.

c. The risk of liver disease will increase.

A patient is receiving tobramycin 3 times daily. The provider has ordered a trough level with the 8:00 AM dose. The nurse will ensure that the level is drawn at what time? a. 4:00 AM b. 7:00 AM c. 7:45 AM d. 8:45 AM

c. 7:45 AM When a patient is receiving divided doses of an aminoglycoside, the trough level should be drawn just before the next dose; therefore, 7:45 AM would be the appropriate time. It would not be appropriate to draw a trough at the other times listed.

A patient shows signs and symptoms of conjunctivitis. Which aminoglycoside would the nurse expect to be ordered? a. Amikacin [Amikin] b. Kanamycin [Kantrex] c. Neomycin [Neomycin] d. Paromomycin [Humatin]

c. Neomycin [Neomycin] Neomycin is used for topical treatment of infections of the eye, ear, and skin. Amikacin, kanamycin, and paromomycin are not topical treatments and are not indicated for eye infections.

A patient has come to the clinic with tinea corporis, and the prescriber has ordered clotrimazole. When educating the patient about this medication, the nurse will include which statement? a. "Apply the medication over the entire body twice daily for 2 weeks." b. "Sun exposure will minimize the drug's effects." c. "This drug is effective after a single application." d. "Use the medication for at least 1 week after the symptoms have cleared."

d. "Use the medication for at least 1 week after the symptoms have cleared."

A nurse is obtaining a history and reviewing the chart of an adult male patient who has been taking oral androgens. Which assessment would warrant notifying the provider? a. Acne and increased facial hair b. Breast enlargement c. Increased libido d. Nausea, anorexia, and fatigue

d. Nausea, anorexia, and fatigue

A male patient is being treated for benign prostatic hyperplasia (BPH) by another provider but cannot remember which drug he is taking. He comes to the clinic seeking treatment for erectile dysfunction and receives a prescription for sildenafil [Viagra]. What will the nurse teach this patient? a. "Sildenafil is not safe to take in conjunction with finasteride [Proscar]." b. "Sildenafil is safe with any medication for benign prostatic hyperplasia." c. "To be safe, you should have a transurethral resection before starting sildenafil." d. "You should not take sildenafil if you are taking silodosin [Rapaflo]."

d. "You should not take sildenafil if you are taking silodosin [Rapaflo]." Sildenafil should not be taken with alpha1-adrenergic antagonists, such as silodosin, because of the risk of severe decreases in blood pressure. Finasteride is not an alpha1-adrenergic antagonist and may be taken with sildenafil. Not all medications for BPH are safe to take with sildenafil. Invasive treatments for BPH are used when symptoms are severe.

The nurse is caring for a patient on a medical-surgical unit who has a fever of unknown origin. The prescriber has ordered a broad-spectrum antibiotic. Which intervention is the priority? a. Administering the antibiotic immediately b. Administering antipyretics as soon as possible c. Delaying administration of the antibiotic until the culture results are available d. Obtaining all cultures before the antibiotic is administered

d. Obtaining all cultures before the antibiotic is administered It is essential that samples of exudates and body fluids (in this case, blood cultures) be obtained for culture before initiation of treatment. Administration of the antibiotic is important but not more important than obtaining specimens for culture. Antipyretics may be indicated, but the priority is obtaining specimens for culture. Treatment may be initiated before the test results are available.

A patient who has been taking 25 mg of hydrocortisone each morning for several months reports feeling fatigued late in the day each day. What will the nurse tell the patient to discuss with the provider? a. Adding a mineralocorticoid to the drug regimen b. Assessing serum electrolytes to check for toxicity c. Increasing the dose to 50 mg daily d. Splitting the daily dose into a morning and an afternoon dose

d. Splitting the daily dose into a morning and an afternoon dose Patients generally take hydrocortisone once daily in the morning to mimic the body's natural cortisol release cycle. However, some patients develop fatigue late in the day. These patients may split the daily dose, taking two-thirds in the morning and one-third in the afternoon or early evening. Mineralocorticoids are given to maintain intravascular volume and regulate sodium, potassium, and hydrogen, so adding this drug would not help with fatigue. Fatigue is not a sign of steroid toxicity. Increasing the dose is not indicated.

An older adult patient is diagnosed with hypothyroidism. The initial free T4 level is 0.5 mg/dL, and the TSH level is 8 microunits/mL. The prescriber orders levothyroxine [Levothroid] 100 mcg/day PO. What will the nurse do? a. Administer the medication as ordered. b. Contact the provider to discuss giving the levothyroxine IV. c. Request an order to give desiccated thyroid (Armour Thyroid). d. Suggest that the provider lower the dose.

d. Suggest that the provider lower the dose. In older adult patients, initial dosing of levothyroxine should start low and be increased gradually. A typical starting dose for an elderly patient is 25 to 50 mcg/day. It is not correct to administer the medication without questioning the provider. Unless the patient has signs of myxedema, there is no need to give the medication IV. Desiccated thyroid is no longer used except in patients who have been taking it long term.

A patient who is at risk for osteoporosis will begin taking the selective estrogen receptor modulator raloxifene [Evista]. Which statement will the nurse include when teaching this patient about the medication? a. Raloxifene reduces the risk of thromboembolism. b. The drug is associated with an increased risk of breast cancer. c. Use of this drug increases the risk of endometrial carcinoma. d. Vasomotor symptoms are a common side effect of this drug.

d. Vasomotor symptoms are a common side effect of this drug. Raloxifene can induce hot flashes in patients taking the drug. It increases the risk for thromboembolism. It protects against breast cancer and does not pose a risk of uterine cancer.

Contraindications: Thyroid Drugs

pregnancy A drugs, hypersensitivity, adrenal insufficiency, previous myocardial infarction, or hyperthyroidism

Indications: Antithyroid Drugs

treat hyperthyroidism and prevent the surge in thyroid hormones that occurs after the surgical treatment of or during radioactive iodine therapy for hyperthyroidism or thyroid cancer **long-term use of these drugs during treatment of hyperthyroidism like during Graves' disease (several years) may induce spontaneous remission

Potassium-sparing diuretics MOA

-Work in collecting ducts and distal convoluted tubules -Interfere with sodium-potassium exchange -Competitively bind to aldosterone receptors -Block resorption of sodium and water usually induced by aldosterone

Loop diuretics MOA

-act directly on the ascending limb of the loop of henle to inhibit chloride and sodium resorption -increase renal prostaglandins, resulting in the dilation of blood vessels and reduced peripheral vascular resistance -useful in tx of edema

Patient teaching

-thyroid replacement drugs best taken 1/2-1 hour before breakfast on an empty stomach to enhance absorption orally, maintain constant hormone levels, and help prevent insomnia -do not abruptly discontinue, lifelong therapy is usually the norm -importance of keeping follow-up visits to monitor thyroid hormone levels, CBC, and liver function studies -brands of thyroid replacement drugs cannot be interchanged -all thyroid tablets must be protected from light -Antithyroid drugs taken better with meals or a snack, given at the same time every day to maintain consistent blood levels of drug -consult with physician before taking any OTC drugs -avoid eating foods high in iodine with antithyroid drugs

A. epinephrine

A patient is experiencing status asthmacticus. Th nurse will prepare to administer which drug first? A. epinephrine B. methylprednisolone ( soul-medrol) C. cromolyn (Intal) D. montelukast (singulair)

D. This drug often causes a "hangover"effect

A patient has been self-medicating with diphenydramine (Bendadryl) to help her sleep. She calls the nurse to ask "why do I feel so tired during the day after I take this pill?" Which statement by the nurse is correct? A. You are probably getting too much sleep B. You are taking too much of the drug C. This drug is not really meant to help people sleep D. This drug often causes a "hangover"effect

6. The nurse should question a prescription for aminoglutethimide (Cytadren) in a patient with which condition? A. Addison's disease B. Adrenal malignancy C. Cushing's syndrome D. Metastatic breast cancer

A

A patient who has been taking an antihypertensive medication for several years is recovering from a myocardial infarction. The prescriber changes the patient's medication to an ACE inhibitor. The patient asks the nurse why a new drug is necessary. What is the nurse's response? a. "ACE inhibitors can prevent or reverse pathologic changes in the heart's structure." b. "ACE inhibitors help lower LDL cholesterol and raise HDL cholesterol." c. "ACE inhibitors increase venous return to the heart, improving cardiac output." d. "ACE inhibitors regulate electrolytes that affect the cardiac rhythm."

A ACE inhibitors have many advantages over other antihypertensive medications, the most important of which is their ability to prevent or reverse pathologic changes in the heart and reduce the risk of cardiac mortality caused by hypertension. They are useful in patients with high low-density lipoprotein (LDL) or low high-density lipoprotein (HDL) cholesterol, but they do not directly affect this comorbidity. They reduce venous return to the heart, thereby reducing right heart size. They do not alter serum electrolyte levels.

9. A nursing student asks the nurse how antibodies provide immune suppression. The nurse responds by telling the student that antibodies: a. block T-cell function. b. boost immune responses. c. reduce proliferation of B cells. d. suppress interferon production.

A Antibodies can be directed against components of the immune system to suppress the immune response and block T-cell function. They inhibit rather than boost immune responses. They do not reduce the proliferation of B cells or suppress interferon production.

4. A patient with a history of lung transplantation is admitted for treatment for a respiratory infection. The patient has been taking cyclosporine [Sandimmune], prednisone, and azathioprine [Imuran] for 8 months. The provider has ordered azithromycin [Zithromax] to treat the infection and acetaminophen [Tylenol] as needed for fever. The nurse will contact the provider to: a. ask whether a different antibiotic can be used. b. ask that the prednisone be discontinued until the infection clears. c. suggest increasing the dose of cyclosporine. d. suggest using ibuprofen instead of acetaminophen.

A Macrolide antibiotics, such as azithromycin, can inhibit cyclosporine metabolism, leading to increased levels of the drug. This patient needs either a reduced dose of cyclosporine or a different antibiotic. There is no indication for discontinuing the prednisone during treatment. The dose of cyclosporine would need to be reduced, because azithromycin leads to increased drug levels. There is no contraindication to using acetaminophen.

A pregnant patient with fever, flank pain, and chills has a history of two previous bladder infections before getting pregnant. She is allergic to several antibiotics. She reports having taken methenamine successfully in the past. What will the nurse tell her? a. "This agent is not effective against infections of the upper urinary tract." b. "This antiseptic agent is safe for use during pregnancy and has no drug resistance." c. "This drug is linked to many serious birth defects and is not recommended during pregnancy." d. "You will need to take this medication with meals to avoid gastric upset."

A Methenamine is safe for use during pregnancy and would be an excellent choice for this patient if she had a lower urinary tract infection. However, it is not an effective agent for upper urinary tract infection, because it is a prodrug that must break down into ammonia and formaldehyde to be effective. There is not enough time for formaldehyde to form in the kidneys, so it is not effective in the upper tract. Nitrofurantoin is linked to serious birth defects and also must be given with food to prevent gastrointestinal problems.

Before giving methenamine [Hiprex] to a patient, it is important for the nurse to review the patient's history for evidence of which problem? a. Elevated blood urea nitrogen and creatinine b. History of reactions to antibiotic agents c. Possibility of pregnancy d. Previous resistance to antiseptic agents

A Methenamine should not be given to patients with renal impairment, because crystalluria can occur. There is no cross-reactivity between methenamine and antibiotic agents. Methenamine is safe for use during pregnancy. There is no organism drug resistance to methenamine.

A patient has a positive urine culture 1 week after completion of a 3-day course of antibiotics. The nurse anticipates that the prescriber will: a. begin a 2-week course of antibiotics. b. evaluate for a structural abnormality of the urinary tract. c. initiate long-term prophylaxis with low-dose antibiotics. d. treat the patient with intravenous antibiotics.

A Patients who develop a subsequent urinary tract infection after treatment are treated in a stepwise fashion, beginning with a longer course of antibiotics. The next steps would be to begin a 4- to 6-week course of therapy, followed by a 6-month course of therapy if that is unsuccessful. If urinary tract infections are thought to be caused by other complicating factors, an evaluation for structural abnormalities may be warranted. Unless the infections are severe or are complicated, intravenous antibiotics are not indicated.

A patient with a history of renal calculi has fever, flank pain, and bacteriuria. The nurse caring for this patient understands that it is important for the provider to: a. begin antibiotic therapy after urine culture and sensitivity results are available. b. give prophylactic antibiotics for 6 weeks after the acute infection has cleared. c. initiate immediate treatment with broad-spectrum antibiotics. d. refer the patient for intravenous antibiotics and hospitalization.

A Patients with renal calculi are more likely to have complicated urinary tract infections that have less predictable microbiologic etiologies. Because the symptoms are mild, it is important first to obtain a culture and sensitivity to assist with antibiotic selection. If symptoms worsen, a broad-spectrum antibiotic may be started until sensitivity information is available. Intravenous antibiotics are indicated for severe pyelonephritis. Long-term prophylaxis is not indicated unless this patient develops frequent reinfection.

A nurse administers an ACE inhibitor to a patient who is taking the drug for the first time. What will the nurse do? a. Instruct the patient not to get up without assistance. b. Make sure the patient takes a potassium supplement. c. Report the presence of a dry cough to the prescriber. d. Request an order for a diuretic to counter the side effects of the ACE inhibitor.

A Severe hypotension can result with the first dose of an ACE inhibitor. The patient should be discouraged from getting up without assistance. Potassium supplements are contraindicated. A dry cough is an expected side effect that eventually may cause a patient to discontinue the drug; however, it is not a contraindication to treatment. Diuretics can exacerbate hypotension and should be discontinued temporarily when a patient starts an ACE inhibitor.

1. A patient has undergone liver transplantation. The provider orders cyclosporine [Sandimmune], prednisone, and sirolimus [Rapamune]. What will the nurse do? a. Question the order for sirolimus. b. Request an order for a serum glucose level. c. Request an order for a macrolide antibiotic. d. Suggest changing the cyclosporine to tacrolimus.

A Sirolimus is given to prevent rejection in renal transplantation; it has no proof of efficacy in patients with heart, lung, or liver transplants. A serum glucose level is not indicated; patients taking repaglinide for diabetes should be monitored closely while taking cyclosporine. Although antibiotic prophylaxis may be necessary, macrolide antibiotics increase the level of cyclosporine. Tacrolimus is more toxic than cyclosporine.

A 20-year-old female patient has suprapubic discomfort, pyuria, dysuria, and bacteriuria greater than 100,000/mL of urine. Which are the most likely diagnosis and treatment? a. Uncomplicated lower urinary tract infection treatable with short-course therapy b. Complicated lower urinary tract infection treatable with single-dose therapy c. Uncomplicated upper urinary tract infection requiring 14 days of oral antibiotics d. Complicated upper urinary tract infection requiring parenteral antibiotics

A These are symptoms of uncomplicated cystitis, which is a lower urinary tract infection that can be treated with a short course of antibiotics. Short-course therapy is more effective than single-dose therapy and is preferred. A complicated lower urinary tract infection would be associated with some predisposing factor, such as renal calculi, an obstruction to the flow of urine, or an indwelling catheter. Upper urinary tract infections often include severe flank pain, fever, and chills.

B. dextromethorphan (Robitussin-DM) oral solution

A 94-year-old patient has a severe dry cough. He has coughed so hard that the muscles in his chest are hurting. He is unsteady on his feet and slightly confused. Which drug would be the best choice for this patient's cough? A. benzonatate (Tessalon Perles) capsules B. dextromethorphan (Robitussin-DM) oral solution C. codeine cough syrup D. guaifenesin (Mucinex)

A nurse identifies therapeutic goals for androgen therapy for which patients? (Select all that apply.) A male patient with prostate cancer A female patient with breast cancer A male patient with hypogonadism A female patient with breast engorgement A female patient with acquired immunodeficiency syndrome (AIDS)

A male patient with hypogonadism A female patient with breast engorgement A female patient with acquired immunodeficiency syndrome (AIDS)

ANS: A Malignant hyperthermia is a rare but potentially fatal adverse reaction that can be triggered by succinylcholine. Muscle pain is an expected adverse effect of succinylcholine that is thought to be caused by the muscle contractions that occur during induction and does not indicate malignant hyperthermia; analgesics can be given, but this effect usually occurs 10 to 12 hours after the procedure and is not an emergency. Dantrolene is given because it slows metabolic activity in skeletal muscles, resulting in decreased heat production. Succinylcholine should be stopped immediately if malignant hyperthermia occurs. Cold saline, given intravenously, often is necessary to lower a patient's temperature.

A nurse is reviewing nursing actions for emergency treatment of malignant hyperthermia with a group of nursing students. Which statement by a student indicates a need for further teaching? a."Analgesics should be given to help with muscle pain." b."Dantrolene helps to slow the metabolic activity in skeletal muscles." c."If malignant hyperthermia occurs, the succinylcholine must be stopped immediately." d."The patient may need an intravenous infusion of cold saline."

ANS: B, D, E Neuromuscular blocking agents are used to produce muscle paralysis in procedures including ECT, intubation and ventilation, and surgery. They do not produce analgesia. They are not used to treat malignant hyperthermia

A nurse is teaching a group of nursing students about neuromuscular blocking agents. For what may these agents be used? (Select all that apply.) a.Analgesia b.Electroshock therapy c.Malignant hyperthermia d.Mechanical ventilation e.Surgery

B. sustained use of nasal decongestants over several days may result in rebound congestion.

A patient complains of worsening nasal congestion despite the use of oxymetazoline (Afrin) nasal spray every 2 to 4 hours for the past 5 days. The nurse's response is based on knowledge that A. oxymetazoline is not an effective nasal decongestant. B. sustained use of nasal decongestants over several days may result in rebound congestion. C. oxymetazoline should be administered in an hourly regimen for severe congestion. D. the patient is probably displaying an idiosyncratic reaction to oxymetazoline.

A. Antihistamines

A patient is making an appointment for allergy testing. The nurse instructs the patient not take what class of medications at least 4 days before allergy testing? A. Antihistamines B. Decongestants C. Bronchodilators D. Antitussives

B. Five minutes

A patient is prescribed two different types of inhaled medications for treatment of chronic obstructive pulmonary disease (COPD). After administering the first medication, how long should the nurse wait to administer the second medication? A. One minute B. Five minutes C. Ten minutes D. Fifteen minutes

ANS: D Omeprazole needs to be taken before meals, and an entire capsule must be taken whole, not crushed, chewed, opened, or dissolved in liquid when treating GERD. This medication is used on a long-term basis to maintain healing.

A patient is taking omeprazole (Prilosec) for the treatment of gastroesophageal reflux disease (GERD). The nurse will include which statement in the teaching plan about this medication? a."Take this medication once a day after breakfast." b."You will be on this medication for only 2 weeks for treatment of the reflux disease." c."The medication may be dissolved in a liquid for better absorption." d."The entire capsule must be taken whole, not crushed, chewed, or opened."

A. a beta agonist (albuterol)

A patient who has a history of asthma is experiencing acute episode of SOB and needs to take medication for immediate relief. The nurse will choose which medication that is appropriate for this situation? A. a beta agonist (albuterol) B. an leukotriene receptor antagonist (montelukast) C. a corticosteroid (fluticasone) D. anticholinergic (ipratropium)

A. corticosteroid

A patient who has asthma may be prescribed which type of inhaled drug for its antiinflammatory effects? A. corticosteroid B. anticholinergic C. xanthine derivative D. beta adrenergic

ANS: B Electroconvulsive therapy (ECT) is effective to treat severe depression with benefits deriving strictly from the effects of electroshock on the brain and not on the convulsive movements that accompany electroshock. Because these convulsive movements can cause harm, succinylcholine is given to prevent them. Giving the medication does not reduce the dose needed for effective treatment or prolong the convulsions. It is given prior to the procedure to prevent convulsions, not during the procedure to stop them.

A patient who will undergo electroconvulsive therapy to treat severe depression will be given succinylcholine just prior to the procedure. When teaching the patient about the procedure the nurse will explain that the succinylcholine is given for what purpose? a.To minimize the amount of electroshock treatment required b.To prevent harm caused by electroshock-induced convulsions c.To prolong the therapeutic electroshock-induced convulsions d.To stop the electroshock-induced convulsions once therapeutic effects have occurred

ANS: A The antibiotic clarithromycin is active against H. pylori and is used in combination with omeprazole to eradicate the bacteria. First-line therapy against H. pylori includes a 10- to 14-day course of a proton pump inhibitor such as omeprazole plus the antibiotics clarithromycin and either amoxicillin or metronidazole, or a combination of a proton pump inhibitor, bismuth subsalicylate, and the antibiotics tetracycline and metronidazole. Many different combinations are used.

A patient will be taking a 2-week course of combination therapy with omeprazole (Prilosec) and another drug for a peptic ulcer caused by Helicobacter pylori. The nurse expects a drug from which class to be ordered with the omeprazole? a.An antibiotic b.A nonsteroidal antiinflammatory drug c.An antacid d.An antiemetic

ANS: C Atracurium is used for muscle relaxation during surgery. Unlike other drugs in this class, it can cause hypotension secondary to antihistamine release. Cholinesterase inhibitors can reverse this reaction. Drugs in this class do not cause CNS depression. Dantrolene is the agent used to treat malignant hyperthermia, which is triggered by succinylcholine. Atracurium is eliminated primarily by plasma cholinesterase and not by the renal or hepatic routes.

A patient will receive atracurium [Tracrium] during surgery for neuromuscular blockade. The nurse caring for this patient will correctly perform which of the following actions? a.Assess the patient for signs of CNS depression. b.Ensure that dantrolene is available in case of a serious adverse reaction. c.Monitor the patient's blood pressure closely. d.Review the preprocedure laboratory values related to renal and hepatic function.

D. guaifenesin (Mucinex)

A patient with a tracheostomy developed pneumonia. It is very difficult for the patient to cough up the thick, dry secretions he has developed. The nurse identifies which drug as being most effective in helping this patient? A. benzonatate (Tessalon Perles) capsules B. dextromethorphan (Robitussin-DM) oral solution C. codeine cough syrup D. guaifenesin (Mucinex)

Nitrofurantoin [Macrodantin] is prescribed for an adolescent female patient with acute cystitis. What should the nurse include in the teaching for this patient? (Select all that apply.) a. "Make sure you tell your prescriber if you might be pregnant." b. "If you experience any tingling or numbness, stop taking the drug and call the clinic immediately." c. "Headaches and drowsiness can occur and are mild side effects." d. "You should not take sulfonamides while taking this medication." e. "Your urine may have a brown tinge while you are taking this drug."

A, B, C, E Nitrofurantoin is linked to serious birth defects and is not recommended during pregnancy. Tingling and numbness indicate neuropathy, which is a serious and often irreversible side effect. Central nervous system (CNS) side effects usually are mild and reversible. Sulfonamides cannot be taken with methenamine, because they can cause crystalluria. Patients' urine may have a brown tinge while they are taking this drug.

A nurse should establish which outcomes on the care plan for a patient taking oral terbutaline? (Select all that apply.) A. Absence of tachycardia B. No reports of chest pain C. Less irritation of gum tissue D. Decrease in bronchospasm E. No jaundice or dark urine

A. Absence of tachycardia B. No reports of chest pain D. Decrease in bronchospasm Terbutaline is an oral beta2 agonist. It promotes bronchodilation through activation of beta2 receptors in the lung, which relieves bronchospasm. It also stimulates the beta receptors of the heart and can result in tachycardia and chest pain if dosing is excessive. Irritated mucosal tissue of the gums, jaundice, and dark urine are not adverse effects of the medication.

Which are advantages of a dry-powder inhaler (DPI) over a metered-dose inhaler (MDI)? (Select all that apply.) A. More of the drug is delivered to the lungs and less to the oropharynx. B. Use of a spacer is not necessary. C. Less propellant is needed to deliver the medication. D. Less hand-lung coordination is required. E. DPIs pose no environmental risk.

A. More of the drug is delivered to the lungs and less to the oropharynx. B. Use of a spacer is not necessary. D. Less hand-lung coordination is required. E. DPIs pose no environmental risk. DPIs deliver more drug to the lungs (20% of the total released, compared to 10% for MDIs). Spacers are not necessary with DPIs; they are recommended for use with MDIs to increase the delivery of drug to the lungs rather than the oropharyngeal mucosa. DPIs do not require the hand-lung coordination needed with MDIs. DPIs present no environmental hazard, because no propellant is required for delivery.

Using a stepwise approach to managing asthma, a nurse teaches a patient who is at step 1 to use albuterol MDI [Proventil] at which of these times? A. Whenever needed (PRN) as a quick-relief agent B. Twice daily combined with an inhaled glucocorticoid C. Only with a long-acting beta2 agonist (LABA) D. If nighttime awakenings occur more than 2 days a week

A. Whenever needed (PRN) as a quick-relief agent For patients at step 1 in the stepwise approach to managing asthma, albuterol is a short-acting beta2 agonist (SABA) used only PRN to relieve ongoing asthma attacks and prevent exercise-induced bronchospasm. For patients at step 1, no long-term control medications are taken. A patient is at a higher step than 1 in the stepwise approach if the patient requires a daily inhaled glucocorticoid or LABA or awakens at night more often than 2 days a week. For patients at steps 2 to 6, albuterol is considered a quick-relief medication taken PRN.

1. A child will be receiving diphenhydramine (Benadryl), 5 mg/kg/day, in divided doses, every 6 hours. The child weighs 80 pounds. Identify how many milligrams of medication will the child receive with each dose. (record answer using one decimal place) _______

ANS: DIF: COGNITIVE LEVEL: Applying (Application) REF: N/A TOP: NURSING PROCESS: Implementation

1. A patient is to receive hydrochlorothiazide (HydroDIURIL) via a percutaneous endoscopic gastrostomy (PEG) tube. The order reads, "Give hydrochlorothiazide, 25 mg, per PEG tube once daily." The medication is available in a liquid form, 50 mg/5 mL. Identify how many milliliters will the nurse administer for each dose. _______

ANS: DIF: COGNITIVE LEVEL: Applying (Application) REF: N/A TOP: NURSING PROCESS: Implementation

1. A patient is to receive vasopressin (Pitressin) 5 units subcut BID. The medication is available in a vial that contains 20 units/mL. Identify how many milliliters will the nurse draw up for this dose. (record answer using two decimal places) _______

ANS: DIF: COGNITIVE LEVEL: Applying (Application) REF: N/A TOP: NURSING PROCESS: Implementation

1. The medication order reads, "Give simvastatin (Zocor) 10 mg daily at bedtime, PO." The medication is available in 20-mg tablets. Identify how many tablets will the nurse administer to the patient. _______

ANS: DIF: COGNITIVE LEVEL: Applying (Application) REF: N/A TOP: NURSING PROCESS: Implementation

2. An elderly patient has had hip surgery and will be receiving heparin, 4000 units subcutaneously every 12 hours. The heparin is available in vials of 5000 units/mL. Calculate how much heparin the nurse will administer for this dose. _______

ANS: DIF: COGNITIVE LEVEL: Applying (Application) REF: N/A TOP: NURSING PROCESS: Implementation

2. An intravenous piggyback (IVPB) antibiotic needs to infuse over 90 minutes. The IVPB bag contains 150 mL. Calculate the setting for the infusion pump. _______

ANS: DIF: COGNITIVE LEVEL: Applying (Application) REF: N/A TOP: NURSING PROCESS: Implementation

1. A patient has a metered-dose inhaler that contains 200 actuations ('puffs'), and it does not have a dose counter. He is to take two puffs two times a day. If he does not take any extra doses, identify how many days will this inhaler last at the prescribed dose. _______

ANS: 50 days Note the number of doses in the canister, and then calculate the number of days that the canister will last. For this question, assuming that two puffs are taken two times a day, and the inhaler has a capacity of 200 inhalations. Two puffs two times a day equal four inhalations per day. Four divided into 200 yields 50; that is, the inhaler will last approximately 50 days. DIF: COGNITIVE LEVEL: Applying (Application) REF: p. 594 TOP: NURSING PROCESS: Implementation

1. A patient is to receive medroxyprogesterone (Depo-Provera) 700 mg weekly, intramuscularly, as part of palliative therapy for endometrial cancer. The medication is available in vials of 400 mg/mL. Identify how many milliliters will the nurse draw up and administer with each injection. (record answer using one decimal place) _______

ANS: 1.8 mL DIF: COGNITIVE LEVEL: Applying (Application) REF: N/A TOP: NURSING PROCESS: Implementation

12. A woman who lives in Seattle is preparing to take a plane trip to London. She has been taking the SERM raloxifene (Evista) for 6 months. The nurse will provide which instructions to this patient? a. She needs to stop taking the drug at least 72 hours before the trip. b. She must remember to take this drug with a full glass of water each morning. c. She will not take the drug while traveling on the plane. d. No change in how the drug is taken will be needed.

ANS: A A patient taking a SERM must be informed to discontinue the drug 72 hours before and during prolonged immobility so as to prevent the development of a thrombosis. DIF: COGNITIVE LEVEL: Applying (Application) REF: p. 549 TOP: NURSING PROCESS: Implementation

2. A 65-year-old patient who receives glucocorticoids for arthritis is admitted to the hospital for treatment of a urinary tract infection. The prescriber has ordered intravenous ciprofloxacin [Cipro]. Before administering the third dose of this drug, the nurse reviews the bacterial culture report and notes that the causative organism is Escherichia coli. The bacterial sensitivity report is pending. The patient complains of right ankle pain. What will the nurse do? a. Withhold the dose of ciprofloxacin and notify the provider of the patient's symptoms. b. Instruct the patient to exercise the right foot and ankle to minimize the pain. c. Question the patient about the consumption of milk and any other dairy products. d. Request an order to increase this patient's dose of glucocorticoids.

ANS: A A rare but serious adverse effect associated with fluoroquinolones is tendon rupture, and those at highest risk are children, patients older than 60 years, transplant patients, and any patients taking glucocorticoids. Any pain in either heel should be reported and the drug should be discontinued. Patients should be instructed not to exercise until tendonitis has been ruled out. Dairy products can reduce the absorption of ciprofloxacin, so this is not a concern with this patient. Because the pain may be caused by tendonitis associated with ciprofloxacin, it is not correct to request an increase in the glucocorticoid dosing.

2. A patient with diabetes develops hypertension. The nurse will anticipate administering which type of medication to treat hypertension in this patient? a. ACE inhibitors b. Beta blockers c. Direct-acting vasodilators d. Thiazide diuretics

ANS: A ACE inhibitors slow the progression of kidney injury in diabetic patients with renal damage. Beta blockers can mask signs of hypoglycemia and must be used with caution in diabetics. Direct-acting vasodilators are third-line drugs for chronic hypertension. Thiazide diuretics promote hyperglycemia.

3. A nurse is teaching a patient who is about to undergo direct-current (DC) cardioversion to treat atrial flutter. The patient has been taking verapamil and warfarin for 6 months. Which statement by the patient indicates understanding of the teaching? a. "I may need long-term therapy with another cardiac medication after the procedure." b. "I should stop taking warfarin a few days before the procedure." c. "I will need to take a beta blocker after the procedure to prevent recurrence of atrial flutter." d. "I will not have to take antidysrhythmia medications after the procedure."

ANS: A After cardioversion for atrial flutter, patients may continue to need long-term therapy with either a class IC agent or a class III agent to prevent recurrence. Patients undergoing DC cardioversion need to take warfarin 3 to 4 weeks before the procedure and for several weeks afterward. Beta blockers are not indicated for postprocedural prophylaxis. Class IC and class III agents are antidysrhythmic drugs.

9. A patient has heart failure and is taking an ACE inhibitor. The patient has developed fibrotic changes in the heart and vessels. The nurse expects the provider to order which medication to counter this development? a. Aldosterone antagonist b. Angiotensin II receptor blocker (ARB) c. Beta blocker d. Direct renin inhibitor (DRI)

ANS: A Aldosterone antagonists are added to therapy for patients with worsening symptoms of HF. Aldosterone promotes myocardial remodeling and myocardial fibrosis, so aldosterone antagonists can help with this symptom. ARBs are given for patients who do not tolerate ACE inhibitors. Beta blockers do not prevent fibrotic changes. DRIs are not widely used.

5. A patient is receiving aminoglycoside therapy and will be receiving a beta-lactam antibiotic as well. The patient asks why two antibiotics have been ordered. What is the nurse's best response? a. "The combined effect of both antibiotics is greater than each of them alone." b. "One antibiotic is not strong enough to fight the infection." c. "We have not yet isolated the bacteria, so the two antibiotics are given to cover a wide range of microorganisms." d. "We can give a reduced amount of each one if we give them together."

ANS: A Aminoglycosides are often used in combination with other antibiotics, such as beta-lactams or vancomycin, in the treatment of various infections because the combined effect of the two antibiotics is greater than that of either drug alone. DIF: COGNITIVE LEVEL: Analyzing (Analysis) REF: p. 626 TOP: NURSING PROCESS: Implementation

8. A female patient is receiving palliative therapy with androgen hormones as part of treatment for inoperable breast cancer. The nurse will discuss with the patient which potential body image changes that may occur as adverse effects? a. Hirsutism and acne b. Weight gain c. Flushing and hot flashes d. Alopecia and body odor

ANS: A Androgens used for cancer treatment, such as fluoxymesterone and testolactone, can cause menstrual irregularities, virilization of female, gynecomastia, hirsutism, acne, anxiety, headache, and nausea. The patient needs to be told of these effects before therapy begins. The other options are incorrect. DIF: COGNITIVE LEVEL: Applying (Application) REF: p. 738 TOP: NURSING PROCESS: Planning

8. The nurse is reviewing the medication list of a patient who will be starting androgen therapy. Which drug classes, if taken with androgens, may have an interaction with them? a. Oral anticoagulants b. Nitrates c. Beta blockers d. Proton pump inhibitors

ANS: A Androgens, when used with oral anticoagulants, can significantly increase or decrease anticoagulant activity. The other options are incorrect. DIF: COGNITIVE LEVEL: Understanding (Comprehension) REF: p. 558 TOP: NURSING PROCESS: Planning

14. A patient with heart failure who has been given digoxin [Lanoxin] daily for a week complains of nausea. Before giving the next dose, the nurse will: a. assess the heart rate (HR) and give the dose if the HR is greater than 60 beats per minute. b. contact the provider to report digoxin toxicity. c. request an order for a decreased dose of digoxin. d. review the serum electrolyte values and withhold the dose if the potassium level is greater than 3.5 mEq/L.

ANS: A Anorexia, nausea, and vomiting are the most common adverse effects of digoxin and should cause nurses to evaluate for more serious signs of toxicity. If the HR is greater than 60 beats per minute, the dose may be given. Nausea by itself is not a sign of toxicity. A decreased dose is not indicated. A serum potassium level less than 3.5 mEq/L is an indication for withholding the dose.

7. When applying transdermal nitroglycerin patches, which instruction by the nurse is correct? a. "Rotate application sites with each dose." b. "Use only the chest area for application sites." c. "Temporarily remove the patch if you go swimming." d. "Apply the patch to the same site each time."

ANS: A Application sites for transdermal nitroglycerin patches need to be rotated. Apply the transdermal patch to any nonhairy area of the body; the old patch should first be removed. The patch may be worn while swimming, but if it does come off, it should be replaced after the old site is cleansed. DIF: COGNITIVE LEVEL: Applying (Application) REF: p. 374 TOP: NURSING PROCESS: Implementation

19. The nurse is teaching a group of patients about management of diabetes. Which statement about basal dosing is correct? a. "Basal dosing delivers a constant dose of insulin." b. "With basal dosing, you can eat what you want and then give yourself a dose of insulin." c. "Glargine insulin is given as a bolus with meals." d. "Basal-bolus dosing is the traditional method of managing blood glucose levels."

ANS: A Basal-bolus therapy is the attempt to mimic a healthy pancreas by delivering basal insulin constantly as a basal, and then as needed as a bolus. Glargine insulin is used as a basal dose, not as a bolus with meals. Basal-bolus therapy is a newer therapy; historically, sliding-scale coverage was implemented. DIF: COGNITIVE LEVEL: Applying (Application) REF: p. 508 TOP: NURSING PROCESS: Implementation

1. A patient is being treated for infertility. An examination reveals cervical mucus that is scant, thick, and sticky. The nurse suspects that the patient has been taking which medication? a. Clomiphene [Clomid] b. Estrogen c. Follicle-stimulating hormone (FSH) d. Progesterone

ANS: A Because clomiphene has antiestrogenic actions, it can force the production of scant and viscous cervical mucus, which interferes with conception. Estrogen is given to create a favorable cervical mucous environment that is more hospitable to sperm. FSH and progesterone do not alter cervical mucus in this way. PTS: 1 DIF: Cognitive Level: Application REF: pp. 770-771

7. A 62-year-old man is to receive lidocaine as treatment for a symptomatic dysrhythmia. Upon assessment, the nurse notes that he has a history of alcoholism and has late-stage liver failure. The nurse will expect which adjustments to his drug therapy? a. The dosage will be reduced by 50%. b. A diuretic will be added to the lidocaine. c. The lidocaine will be changed to an oral dosage form. d. An increased dosage of lidocaine will be prescribed so as to obtain adequate blood levels.

ANS: A Because lidocaine is metabolized primarily by the liver, a reduction of the dosage by 50% may be necessary in cases of liver failure or cirrhosis. Lidocaine does not come in oral form. DIF: COGNITIVE LEVEL: Applying (Application) REF: p. 401 TOP: NURSING PROCESS: Planning

6. A prescriber has ordered cefoxitin for a patient who has an infection caused by a gram-negative bacteria. The nurse taking the medication history learns that the patient experienced a maculopapular rash when taking amoxicillin [Amoxil] several years earlier. What will the nurse do? a. Administer the cefoxitin and observe for any side effects. b. Give the cefoxitin and have epinephrine and respiratory support available. c. Request an order for a different, nonpenicillin, noncephalosporin antibiotic. d. Request an order to administer a skin test before giving the cefoxitin.

ANS: A Because of structural similarities between penicillins (PCNs) and cephalosporins, a few patients allergic to one drug type will be allergic to the other drug type, although this is rare. For patients with mild PCN allergy, such as rash, cephalosporins can be used with minimal concern, so it is correct to administer the drug and monitor for side effects. It is unnecessary to prepare for anaphylaxis, to give another class of drug, or to administer a skin test.

A patient who is taking drugs to treat HIV has tuberculosis and has been on a four-drug regimen for 3 months without improvement in symptoms. Which drug will the nurse anticipate that the provider will add to this patient's regimen? a. bedaquiline [Sirturo] b. capreomycin [Capastat Sulfate] c. ethionamide [Trecator] d. pyridoxine

ANS: A Bedaquiline is a newer, highly effective anti-TB drug that does not accelerate the metabolism of HIV drugs and is sued for multi-drug resistant TB. Capreomycin is a second-line drug used for drug-resistant TB, but is not as effective as bedaquiline. Ethionamide is a second-line drug that is less well tolerated of all anti-TB agents and is used only when there is no alternative. Pyridoxine is given to prevent peripheral neuritis in patients taking isoniazid.

6. A patient with hypertension with a blood pressure of 168/110 mm Hg begins taking hydrochlorothiazide and verapamil. The patient returns to the clinic after 2 weeks of drug therapy, and the nurse notes a blood pressure of 140/85 mm Hg and a heart rate of 98 beats per minute. What will the nurse do? a. Notify the provider and ask about adding a beta blocker medication. b. Reassure the patient that the medications are working. c. Remind the patient to move slowly from sitting to standing. d. Request an order for an electrocardiogram.

ANS: A Beta blockers are often added to drug regimens to treat reflex tachycardia, which is a common side effect of lowering blood pressure, caused by the baroreceptor reflex. The patient's blood pressure is responding to the medications, but the tachycardia warrants treatment. Reminding the patient to move slowly from sitting to standing is appropriate with any blood pressure medication, but this patient has reflex tachycardia, which must be treated. An electrocardiogram is not indicated.

8. A nurse is discussing how beta blockers work to decrease blood pressure with a nursing student. Which statement by the student indicates a need for further teaching? a. "Beta blockers block the actions of angiotensin II." b. "Beta blockers decrease heart rate and contractility." c. "Beta blockers decrease peripheral vascular resistance." d. "Beta blockers decrease the release of renin."

ANS: A Beta blockers reduce the release of renin by blockade of beta1 receptors on juxtaglomerular cells in the kidney, which reduces angiotensin II-mediated vasoconstriction, but do not block the actions of angiotensin II directly. Beta blockers decrease heart rate and cardiac contractility, decrease peripheral vascular resistance, and decrease the release of renin.

A prescriber has ordered bethanechol [Urecholine] for a postoperative patient who has urinary retention. The nurse reviews the patient's chart before giving the drug. Which part of the patient's history would be a contraindication to using this drug? a.Asthma as a child b.Gastroesophageal reflux c.Hypertension d.Hypothyroidism

ANS: A Bethanechol is contraindicated in patients with active or latent asthma, because activation of muscarinic receptors in the lungs causes bronchoconstriction. It increases the tone and motility of the gastrointestinal (GI) tract and is not contraindicated in patients with reflux. It causes vasodilation and would actually lower blood pressure in a hypertensive patient. It causes dysrhythmias in hyperthyroid patients.

4. The nurse is preparing for a community education program on hypertension. Which of these parameters determine the regulation of arterial blood pressure? a. Cardiac output and systemic vascular resistance b. Heart rate and peripheral resistance c. Blood volume and renal blood flow d. Myocardial contractility and arteriolar constriction

ANS: A Blood pressure is determined by the product of cardiac output and systemic vascular resistance. The other options are incorrect. DIF: COGNITIVE LEVEL: Understanding (Comprehension) REF: p. 343 TOP: NURSING PROCESS: General

6. A patient who is receiving intravenous ciprofloxacin for pneumonia develops diarrhea. A stool culture is positive for Clostridium difficile. The nurse will expect the provider to: a. add metronidazole [Flagyl]. b. increase the dose of ciprofloxacin. c. restrict dairy products. d. switch to gemifloxacin.

ANS: A C. difficile is resistant to fluoroquinolones; metronidazole is the drug of choice to treat this infection. Metronidazole is lethal only to anaerobic organisms, so the ciprofloxacin should be continued to treat the pneumonia. Increasing the dose of ciprofloxacin is not indicated, because C. difficile is resistant to ciprofloxacin. Gemifloxacin is approved for use in respiratory infections.

5. A nursing student is discussing with a nurse the plan of care for a patient about to undergo a third round of chemotherapy with cisplatin. Which statement by the nursing student about the treatment of CINV is correct? a. "Aprepitant [Emend] will be necessary to treat CINV caused by cisplatin." b. "Antiemetics are most effective if given just as the chemotherapy is finished." c. "Lorazepam probably would not be helpful for this patient." d. "This patient will need intravenous antiemetics for best effects.

ANS: A CINV caused by cisplatin is maximal 48 to 72 hours after dosing and can persist for 6 to 7 days, so an antiemetic such as aprepitant, which treats delayed emesis, is an important part of antiemetic therapy. Antiemetics should be started 30 minutes before initiation of chemotherapy. Lorazepam should be given to this patient, because the patient has had some experience with chemotherapy and is likely to have anticipatory emesis. There is no benefit to IV over oral dosing unless the patient has ongoing emesis.

1. Which two classes of antidysrhythmic drugs have nearly identical cardiac effects? a. Beta blockers and calcium channel blockers b. Beta blockers and potassium channel blockers c. Calcium channel blockers and sodium channel blockers d. Sodium channel blockers and potassium channel blockers

ANS: A Calcium channel blockade has the same impact on cardiac action potentials as does beta blockade, so these agents have nearly identical effects on cardiac function; that is, they reduce automaticity in the SA node, delay conduction through the AV node, and reduce myocardial contractility. Potassium channel blockers act by delaying repolarization of fast potentials. Sodium channel blockers block sodium channels to slow impulse conduction in the atria, ventricles, and His-Purkinje system.

4. A 74-year-old professional golfer has chest pain that occurs toward the end of his golfing games. He says the pain usually goes away after one or two sublingual nitroglycerin tablets and rest. What type of angina is he experiencing? a. Classic b. Variant c. Unstable d. Prinzmetal's

ANS: A Classic, or chronic stable, angina is triggered by either exertion or stress and usually subsides within 15 minutes with either rest or drug therapy. DIF: COGNITIVE LEVEL: Understanding (Comprehension) REF: p. 363 TOP: NURSING PROCESS: Assessment

1. A nurse is taking a history on a clinic patient who reports being constipated. Upon further questioning, the nurse learns that the patient's last stool was 4 days ago, that it was of normal, soft consistency, and that the patient defecated without straining. The patient's abdomen is not distended, and bowel sounds are present. The patient reports usually having a stool every 1 to 2 days. What will the nurse do? a. Ask about recent food and fluid intake. b. Discuss the use of polyethylene glycol [MiraLax]. c. Recommend a bulk laxative. d. Suggest using a bisacodyl [Dulcolax] suppository.

ANS: A Constipation cannot necessarily be defined by the frequency of bowel movements, because this varies from one individual to another. Constipation is defined in terms of a variety of symptoms, including hard stools, infrequent stools, excessive straining, prolonged effort, and unsuccessful or incomplete defecation. A common cause of constipation is diet, especially fluid and fiber intake; therefore, when changes in stool patterns occur, patients should be questioned about food and fluid intake. Because this patient has only more infrequent stools and is not truly constipated, laxatives are not indicated.

6. A patient's medication order indicates that he is to receive a dose of cosyntropin (Cortrosyn). The nurse is aware that this drug is used to diagnose which condition? a. Adrenocortical insufficiency b. Diabetes insipidus c. Myasthenia gravis d. Pituitary dwarfism

ANS: A Cosyntropin is used for the diagnosis of adrenocortical insufficiency. The other options are incorrect. DIF: COGNITIVE LEVEL: Understanding (Comprehension) REF: p. 485 TOP: NURSING PROCESS: Planning

1. A nurse is preparing to administer an antibiotic to a patient with methicillin-resistant Staphylococcus aureus (MRSA). The nurse would expect the healthcare provider to order which antibiotic? a. Daptomycin [Cubicin] b. Levofloxacin [Levaquin] c. Norfloxacin [Noroxin] d. Ciprofloxacin [Cipro]

ANS: A Daptomycin is active against MRSA. Levofloxacin and norfloxacin are not approved to treat MRSA. Ciprofloxacin is a poor choice for staphylococcal infections, including MRSA.

10. The nurse is discussing the use of adsorbents such as bismuth subsalicylate (Pepto-Bismol) with a patient who has diarrhea. The nurse will warn the patient about which possible adverse effects? a. Dark stools and blue gums b. Urinary hesitancy c. Drowsiness and dizziness d. Blurred vision and headache

ANS: A Dark stools and blue gums are two of the possible adverse effects of bismuth subsalicylate (see Table 51-2). The other adverse effects listed may occur with the use of other antidiarrheal drugs. DIF: COGNITIVE LEVEL: Understanding (Comprehension) REF: p. 815 TOP: NURSING PROCESS: Assessment

1. A patient is receiving doxorubicin (Adriamycin) as part of treatment for ovarian cancer. Which nursing diagnosis is related to this antineoplastic drug? a. Decreased cardiac output related to the adverse effect of cardiotoxicity b. Ineffective breathing pattern related to the adverse effect of pulmonary toxicity c. Risk for injury related to the effects of neurotoxicity (ataxia, numbness of hands and feet) d. Impaired urinary elimination pattern related to hyperuricemia

ANS: A Decreased cardiac output related to the adverse effect of cardiotoxicity is a nursing diagnosis related to doxorubicin because adverse effects of doxorubicin include liver and cardiovascular toxicities. The other options are incorrect. DIF: COGNITIVE LEVEL: Applying (Application) REF: p. 742 TOP: NURSING PROCESS: Nursing Diagnosis

A nurse teaches a nursing student about the differences between esmopressin (DDAVP) and vasopressin [Pitressin]. Which statement by the student indicates a need for further teaching? a."Desmopressin has a shorter duration of action than vasopressin." b."Desmopressin is easier to administer than vasopressin." c."Vasopressin can be used in cardiac resuscitation." d."Vasopressin can cause serious adverse cardiovascular effects."

ANS: A Desmopressin has a long duration of action, which is the reason it is preferred for the treatment of diabetes insipidus. It can be administered intranasally, so it is easier to administer than vasopressin. Vasopressin has hemodynamic effects that can be beneficial during cardiac resuscitation but that also can cause serious adverse cardiovascular effects.

7. A nurse is obtaining a medication history on a newly admitted patient, who reports taking minoxidil for hypertension. Admission vital signs reveal a heart rate of 78 beats per minute and a blood pressure of 120/80 mm Hg. What is an important part of the initial assessment for this patient? a. Evaluating ankle edema b. Monitoring for nausea and vomiting c. Noting the presence of hypertrichosis d. Obtaining a blood glucose

ANS: A Fluid retention is a common and serious adverse effect of minoxidil, because it can lead to cardiac decompensation. If present, a diuretic is indicated. Nausea and vomiting may occur with this drug but is not a serious side effect. Hypertrichosis occurs in about 80% of patients taking the drug, but its effects are cosmetic and not life threatening. It may be important to monitor the blood glucose level in some patients, because the drug can alter glucose tolerance, but this effect is not as serious as fluid retention.

2. When a male patient is receiving androgen therapy, the nurse will monitor for signs of excessive androgens such as: a. fluid retention. b. dehydration. c. restlessness. d. visual changes.

ANS: A Fluid retention is an undesirable effect of androgens. The other options are incorrect. DIF: COGNITIVE LEVEL: Applying (Application) REF: p. 557 TOP: NURSING PROCESS: Assessment

4. A patient is taking guaifenesin (Humibid) as part of treatment for a sinus infection. Which instruction will the nurse include during patient teaching? a. Force fluids to help loosen and liquefy secretions. b. Report clear-colored sputum to the prescriber. c. Avoid driving a car or operating heavy machinery because of the sedating effects. d. Report symptoms that last longer than 2 days.

ANS: A Forcing fluids helps to loosen and liquefy secretions. The patient must be fully aware that any fever, chest tightness, change in sputum from clear to colored, difficult or noisy breathing, activity intolerance, or weakness needs to be reported. The patient must also report to the prescriber a fever of higher than 100.4° F (38° C) or symptoms that last longer than 3 to 4 days. Decongestants do not cause sedation, and therefore the patient does not need to avoid driving a car or operating heavy machinery. DIF: COGNITIVE LEVEL: Applying (Application) REF: p. 575 TOP: NURSING PROCESS: Implementation

1. A patient is brought to the emergency department with shortness of breath, a respiratory rate of 30 breaths per minute, intercostal retractions, and frothy, pink sputum. The nurse caring for this patient will expect to administer which drug? a. Furosemide [Lasix] b. Hydrochlorothiazide [HydroDIURIL] c. Mannitol [Osmitrol] d. Spironolactone [Aldactone]

ANS: A Furosemide, a potent diuretic, is used when rapid or massive mobilization of fluids is needed. This patient shows severe signs of congestive heart failure with respiratory distress and pulmonary edema and needs immediate mobilization of fluid. Hydrochlorothiazide and spironolactone are not indicated for pulmonary edema, because their diuretic effects are less rapid. Mannitol is indicated for patients with increased intracranial pressure and must be discontinued immediately if signs of pulmonary congestion or heart failure occur.

5. A 75-year-old woman with type 2 diabetes has recently been placed on glipizide (Glucotrol), 10 mg daily. She asks the nurse when the best time would be to take this medication. What is the nurse's best response? a. "Take this medication in the morning, 30 minutes before breakfast." b. "Take this medication in the evening with a snack." c. "This medication needs to be taken after the midday meal." d. "It does not matter what time of day you take this medication."

ANS: A Glipizide is taken in the morning, 30 minutes before breakfast. When taken at this time, it has a longer duration of action, causing a constant amount of insulin to be released. This may be beneficial in controlling blood glucose levels throughout the day. DIF: COGNITIVE LEVEL: Applying (Application) REF: p. 511 TOP: NURSING PROCESS: Implementation

4. The nurse is teaching a patient about the adverse effects of fertility drugs such as clomiphene (Clomid). Which is a potential adverse effect of this drug? a. Headache b. Drowsiness c. Dysmenorrhea d. Hypertension

ANS: A Headache is one of the possible adverse effects of the fertility drugs. They may also cause vomiting, restlessness, and urticaria. Drowsiness, dysmenorrhea, and hypertension are not potential adverse effects. See Table 34-5 for other adverse effects. DIF: COGNITIVE LEVEL: Understanding (Comprehension) REF: p. 544 TOP: NURSING PROCESS: Assessment

A patient arrives in the emergency department complaining of muscle weakness and drowsiness. The nurse notes a heart rate of 80 beats per minute, a respiratory rate of 18 breaths per minute, and a blood pressure of 90/50 mm Hg. The electrocardiogram reveals an abnormal rhythm. The nurse will question the patient about which over-the-counter medication? a. Antacids b. Aspirin c. Laxatives d. Potassium supplements

ANS: A Hypermagnesemia can occur when patients are taking magnesium-containing antacids. Symptoms include muscle weakness, sedation, hypotension, and ECG changes. Aspirin would cause metabolic acidosis. Laxatives can contribute to hypokalemia. Potassium supplements would cause hyperkalemia.

3. A patient has three separate blood pressure (BP) readings of 120/100 mm Hg, 138/92 mm Hg, and 126/96 mm Hg. Which category describes this patient's BP? a. Hypertension b. Isolated systolic hypertension c. Normal d. Prehypertension

ANS: A Hypertension is defined as systolic BP over 140 mm Hg or diastolic BP over 90 mm Hg. When systolic and diastolic BP fall into different categories, classification is based on the higher category. This patient has a hypertensive diastolic BP. Isolated systolic hypertension occurs if the systolic BP is greater than 140 mm Hg with a diastolic BP less than 90 mm Hg. Because this patient has an elevated diastolic BP, it is not considered normal. Prehypertension occurs with a systolic BP of 120 to 139 mm Hg or a diastolic BP of 80 to 89 mm Hg.

5. A patient arrives in the emergency department with severe chest pain. The patient reports that the pain has been occurring off and on for a week now. Which assessment finding would indicate the need for cautious use of nitrates and nitrites? a. Blood pressure of 88/62 mm Hg b. Apical pulse rate of 110 beats/min c. History of renal disease d. History of a myocardial infarction 2 years ago

ANS: A Hypotension is a possible contraindication to the use of nitrates because the medications may cause the blood pressure to decrease. The other options are incorrect. DIF: COGNITIVE LEVEL: Analyzing (Analysis) REF: p. 364 TOP: NURSING PROCESS: Assessment

7. An adolescent patient recently attended a health fair and had a serum glucose test. The patient telephones the nurse and says, "My level was 125 mg/dL. Does that mean I have diabetes?" What is the nurse's most accurate response? a. "Unless you were fasting for longer than 8 hours, this does not necessarily mean you have diabetes." b. "At this level, you probably have diabetes. You will need an oral glucose tolerance test this week." c. "This level is conclusive evidence that you have diabetes." d. "This level is conclusive evidence that you do not have diabetes."

ANS: A If a person has not fasted for 8 hours, a blood sugar level of 125 mg/dL would be considered normal, because it is less than 200 mg/dL for a random sampling. Also, a person must have positive outcomes on two separate days to be diagnosed with diabetes. This patient does not need to have an oral glucose tolerance test, because the 125 mg/dL reading is so far below 200 mg/dL, which would require further work-up. No conclusive evidence indicates that this patient has diabetes, because the random sample value is so low, and the patient has not had two separate tests on different days. However, this also is not conclusive evidence that the patient does not have diabetes.

7. A patient, newly diagnosed with hypothyroidism, has received a prescription for thyroid replacement therapy. The nurse will instruct the patient to take this medication at which time of day? a. In the morning b. With the noon meal c. With the evening meal d. At bedtime

ANS: A If possible, it is best to administer thyroid drugs taken once daily in the morning so as to decrease the likelihood of insomnia that may result from evening dosing. DIF: COGNITIVE LEVEL: Applying (Application) REF: p. 496 TOP: NURSING PROCESS: Implementation

4. An 8-year-old child has been diagnosed with true pituitary dwarfism and is being treated with somatropin. In follow-up visits, the nurse will monitor for which expected outcome? a. Increased growth b. Decreased urinary output c. Increased muscle strength d. Increased height when the child reaches puberty

ANS: A In patients for whom somatropin is indicated, increased growth is expected. The other options are incorrect. DIF: COGNITIVE LEVEL: Understanding (Comprehension) REF: p. 488 TOP: NURSING PROCESS: Evaluation

15. A patient with a recent onset of nephrosclerosis has been taking an ACE inhibitor and a thiazide diuretic. The patient's initial blood pressure was 148/100 mm Hg. After 1 month of drug therapy, the patient's blood pressure is 130/90 mm Hg. The nurse will contact the provider to discuss: a. adding a calcium channel blocker to this patient's drug regimen. b. lowering doses of the antihypertensive medications. c. ordering a high-potassium diet. d. adding spironolactone to the drug regimen.

ANS: A In patients with renal disease, the goal of antihypertensive therapy is to lower the blood pressure to 130/80 mm Hg or less. Adding a third medication is often indicated. Lowering the dose of the medications is not indicated because the patient's blood pressure is not in the target range. Adding potassium to the diet and using a potassium-sparing diuretic are contraindicated.

1. The nurse is reviewing conditions caused by nutrient deficiencies. Conditions such as infantile rickets, tetany, and osteomalacia are caused by a deficiency in which vitamin or mineral? a. Vitamin D b. Vitamin C c. Zinc d. Cyanocobalamin (vitamin B12)

ANS: A Infantile rickets, tetany, and osteomalacia are all a result of long-term vitamin D deficiency. The other options are incorrect. DIF: COGNITIVE LEVEL: Remembering (Knowledge) REF: p. 845 TOP: NURSING PROCESS: Assessment

12. A hospitalized patient has a blood pressure of 145/96 mm Hg. The nurse caring for this patient notes that the blood pressure the day before was 132/98 mm Hg. The patient reports ambulatory blood pressure readings of 136/98 and 138/92 mm Hg. The patient has a history of a previous myocardial infarction and has adopted a lifestyle that includes use of the DASH diet and regular exercise. What will the nurse do? a. Notify the provider and discuss ordering a beta blocker for this patient. b. Notify the provider and suggest a thiazide diuretic as initial therapy. c. Order a diet low in sodium and high in potassium for this patient. d. Recheck the patient's blood pressure in 4 hours to verify the result.

ANS: A Initial drug selection is determined by the presence or absence of a compelling indication or comorbid condition. This patient has a history of MI; beta blockers are indicated for patients with preexisting heart disease. Thiazide diuretics are first-line drugs of choice in patients without compelling indications. The patient is already consuming a DASH diet; closer monitoring of sodium or potassium will not help lower blood pressure. The patient has a record of hypertension, so it is unnecessary to recheck the blood pressure to verify the condition.

8. A patient is being discharged to home on a single daily dose of a diuretic. The nurse instructs the patient to take the dose at which time so it will be least disruptive to the patient's daily routine? a. In the morning b. At noon c. With supper d. At bedtime

ANS: A It is better to take the diuretic medication early in the morning to prevent urination during the night. Taking the diuretic at the other times may cause nighttime urination and disrupt sleep. DIF: COGNITIVE LEVEL: Applying (Application) REF: p. 461 TOP: NURSING PROCESS: Implementation

8. A 30-year-old woman is in the clinic for her yearly gynecologic exam and asks the nurse about the "new vaccine that prevents HPV." She wants to receive the papillomavirus vaccine (Gardasil). Which response by the nurse is most appropriate? a. "For women, the recommended age for this vaccine is 13 to 26 years of age." b. "We will need to make sure you are not pregnant first." c. "There will be a total of three injections." d. "I will check with your health care provider and then get the first dose of the vaccine ready."

ANS: A It is important to make sure that a patient receiving Gardasil is not pregnant and that the patient knows that there are a total of three injections, but this particular patient is too old to receive the vaccine. The guidelines recommend the vaccine for women 13 to 26 years of age. DIF: COGNITIVE LEVEL: Applying (Application) REF: p. 790 TOP: NURSING PROCESS: Implementation

3. Mannitol (Osmitrol) has been ordered for a patient with acute renal failure. The nurse will administer this drug using which procedure? a. Intravenously, through a filter b. By rapid intravenous bolus c. By mouth in a single morning dose d. Through a gravity intravenous drip with standard tubing

ANS: A Mannitol is administered via intravenous infusion through a filter because of possible crystallization. It is not available in oral form. The other options are incorrect. DIF: COGNITIVE LEVEL: Applying (Application) REF: p. 456 TOP: NURSING PROCESS: Implementation

6. A patient who was in a motor vehicle accident sustained a severe head injury and is brought into the emergency department. The provider orders intravenous mannitol [Osmitrol]. The nurse knows that this is given to: a. reduce intracranial pressure. b. reduce renal perfusion. c. reduce peripheral edema. d. restore extracellular fluid.

ANS: A Mannitol is an osmotic diuretic that is used to reduce intracranial pressure by relieving cerebral edema. The presence of mannitol in blood vessels in the brain creates an osmotic force that draws edematous fluid from the brain into the blood. Mannitol can also be used to increase renal perfusion. It can cause peripheral edema and is not used to restore extracellular fluid.

5. After receiving a nebulizer treatment with a beta agonist, the patient complains of feeling slightly nervous and wonders if her asthma is getting worse. What is the nurse's best response? a. "This is an expected adverse effect. Let me take your pulse." b. "The next scheduled nebulizer treatment will be skipped." c. "I will notify the physician about this adverse effect." d. "We will hold the treatment for 24 hours."

ANS: A Nervousness, tremors, and cardiac stimulation are possible and expected adverse effects of beta agonists. The other options are incorrect responses. DIF: COGNITIVE LEVEL: Applying (Application) REF: p. 583 TOP: NURSING PROCESS: Assessment

6. An adult who has been self-medicating, using nutritional therapy for an elevated cholesterol level, complains of repeated episodes of flushing. The nurse suspects that the patient has been taking: a. niacin. b. thiamine. c. riboflavin. d. pyridoxine.

ANS: A Niacin is used to reduce cholesterol levels. When taken in large doses, nicotinic acid can cause vasodilation, with resultant flushing, dizziness, and nausea. Flushing is not a side effect of thiamine, riboflavin, or pyridoxine because they do not cause vasodilation. PTS: 1 DIF: Cognitive Level: Application REF: pp. 992-993

A patient has just purchased a 1-year supply of 28-day-cycle oral contraceptives. She tells the nurse she wishes she had planned things better, because she has calculated that her period will begin during her upcoming honeymoon. What will the nurse suggest? a. She should discard the inert pills and start a new pack during the honeymoon. b. She should discontinue the oral contraceptives and use an alternative form of birth control. c. She should discuss a prescription for an extended-cycle product with her provider. d. She should discuss DMPA (Depo-Provera) injections in addition to the OC with her provider.

ANS: A OC users can achieve an extended-cycle schedule by discarding the inert pills and beginning a new pack for up to four cycles. It is not necessary to discontinue OCs. Because this woman has already purchased a 1-year supply, using the 28-day-cycle product as described is appropriate. If this woman opts for a long-term product, she should discontinue the OC.

16. A patient wants to try an oral soy product to relieve perimenopausal symptoms. The nurse will assess the patient's medication history for which potential drug interaction? a. Thyroid replacement therapy b. Oral anticoagulant therapy c. Nonsteroidal anti-inflammatory drugs d. Beta blockers

ANS: A Orally administered soy may interfere with thyroid hormone absorption, so concurrent use must be avoided. The other options are incorrect. DIF: COGNITIVE LEVEL: Applying (Application) REF: p. 536 TOP: NURSING PROCESS: Assessment

9. The nurse is caring for an older adult patient after a right hip open reduction internal fixation (ORIF). The patient is taking an opioid every 6 hours as needed for pain. The nurse discusses obtaining an order from the prescriber for which medication? a. Docusate sodium [Colace] b. GoLYTELY c. Lactulose d. Polyethylene glycol [MiraLax]

ANS: A Oxycodone can be constipating. The patient needs something prophylactically, such as docusate sodium, that can be taken daily to prevent constipation. In addition, the patient's mobility is limited, which can further increase the risk of constipation. GoLYTELY is not indicated for constipation; it is used for cleansing the bowel before diagnostic procedures. Lactulose is not indicated; it typically is used for reducing ammonia levels in hepatic encephalopathy. Polyethylene glycol is indicated for occasional constipation; no information suggests that the patient is constipated.

A hospitalized patient is being treated for tuberculosis with a drug regimen that includes pyrazinamide. The patient complains of pain in the knee and shoulder joints. The nurse will contact the provider to request an order for: a. ibuprofen. b. renal function tests. c. discontinuation of the pyrazinamide. d. measurement of uric acid levels.

ANS: A Polyarthralgias occur in 40% of patients during the initial phase of treatment with pyrazinamide and can be managed with a nonsteroidal anti-inflammatory drug (NSAID). Renal function tests and uric acid levels are not indicated. Unless the pain cannot be managed with NSAIDs, there is no need to reduce the dose of pyrazinamide.

13. The nurse recognizes that the risk of osteoporosis is higher in an individual with which risk factor? a. White or Asian race b. African-American race c. History of participation in active sports d. Obesity

ANS: A Risk factors for postmenopausal osteoporosis include white or Asian descent, slender body build, early estrogen deficiency, smoking, alcohol consumption, low-calcium diet, sedentary lifestyle, and family history of osteoporosis. DIF: COGNITIVE LEVEL: Remembering (Knowledge) REF: p. 541 TOP: NURSING PROCESS: Assessment

3. A 45-year-old man has received a series of immunizing drugs in preparation for a trip to a developing country. Within hours, his wife brings him to the emergency department because he has developed edema of the face, tongue, and throat and is having trouble breathing. The nurse suspects that, based on the patient's history and symptoms, he is experiencing which condition? a. Serum sickness b. Cross-sensitivity c. Thrombocytopenic purpura d. Adenopathy

ANS: A Serum sickness sometimes occurs after repeated injections of equine (horse)-made immunizing drugs and is characterized by edema of the face, tongue, and throat; rash; urticaria; fever; flushing; dyspnea; and other symptoms. DIF: COGNITIVE LEVEL: Applying (Application) REF: p. 787 TOP: NURSING PROCESS: Evaluation

A patient will be discharged from the hospital with a prescription for TMP/SMZ (Bactrim). When providing teaching for this patient, the nurse will tell the patient that it will be important to: a. drink 8 to 10 glasses of water each day. b. eat foods that are high in potassium. c. take the medication with food. d. take folic acid supplements.

ANS: A TMP/SMZ can injure the kidneys, because it causes deposition of sulfonamide crystals in the kidneys. Patients should be advised to drink 8 to 10 glasses of water a day to maintain a urine flow of 1200 mL in adults. Trimethoprim can cause hyperkalemia, so consuming extra potassium is unnecessary. The medication should be taken on an empty stomach. It is not necessary to consume extra folic acid, because mammalian cells use dietary folate and do not have to synthesize it; it is the process of folic acid synthesis that is altered by sulfonamides.

1. The nurse working on a high-acuity medical-surgical unit is prioritizing care for four patients who were just admitted. Which patient should the nurse assess first? a. The NPO patient with a blood glucose level of 80 mg/dL who just received 20 units of 70/30 Novolin insulin b. The patient with a pulse of 58 beats per minute who is about to receive digoxin [Lanoxin] c. The patient with a blood pressure of 136/92 mm Hg who complains of having a headache d. The patient with an allergy to penicillin who is receiving an infusion of vancomycin [Vancocin]

ANS: A The NPO patient with hypoglycemia who just received 70/30 Novolin insulin takes priority, because this patient needs to consume a good source of glucose immediately or perhaps the NPO status will be discontinued for this shift. The digoxin may be withheld for the patient with a pulse of 58 beats per minute, but this is not a priority action. The patient with a headache needs to be followed up, but because the blood pressure is 136/92 mm Hg, the headache is probably not caused by hypertension. The patient with an allergy to penicillin will not have a reaction to the vancomycin.

7. During interleukin drug therapy, a patient is showing signs of severe fluid retention, with increasing dyspnea and severe peripheral edema. The next dose of the interleukin is due now. Which action will the nurse take next? a. Hold the drug, and notify the prescriber. b. Give the drug, and notify the prescriber. c. Give the drug along with acetaminophen and diphenhydramine (Benadryl). d. Monitor the patient for 2 hours, and then give the drug if the patient's condition improves.

ANS: A The fluid retention that may occur with interleukin therapy is reversible; if therapy is stopped, the prescriber must be notified. The other options are incorrect. DIF: COGNITIVE LEVEL: Applying (Application) REF: p. 760 TOP: NURSING PROCESS: Implementation

8. During therapy with amphotericin B, the nurse will monitor the patient for known adverse effects that would be reflected by which laboratory result? a. Serum potassium level of 2.7 mEq/L b. Serum potassium level of 5.8 mEq/L c. White blood cell count of 7000 cells/mm3 d. Platelet count of 300,000/ microliter

ANS: A The nurse needs to monitor for hypokalemia, a possible adverse effect of amphotericin B. The other options are incorrect. DIF: COGNITIVE LEVEL: Analyzing (Analysis) REF: p. 673 TOP: NURSING PROCESS: Implementation

6. When monitoring a patient who is taking hydrochlorothiazide (HydroDIURIL), the nurse notes that which drug is most likely to cause a severe interaction with the diuretic? a. Digitalis b. Penicillin c. Potassium supplements d. Aspirin

ANS: A There is an increased risk for digitalis toxicity in the presence of hypokalemia, which may develop with hydrochlorothiazide therapy. Potassium supplements are often prescribed with hydrochlorothiazide therapy to prevent hypokalemia. The other options do not have interactions with hydrochlorothiazide. DIF: COGNITIVE LEVEL: Applying (Application) REF: p. 458 TOP: NURSING PROCESS: Implementation

6. A female patient with essential hypertension is being treated with hydralazine 25 mg twice daily. The nurse assesses the patient and notes a heart rate of 96 beats per minute and a blood pressure of 110/72 mm Hg. The nurse will request an order to: a. administer a beta blocker. b. administer a drug that dilates veins. c. reduce the dose of hydralazine. d. give the patient a diuretic.

ANS: A This patient is showing signs of reflex tachycardia, so a beta blocker is indicated to slow the heart rate. Patients with heart failure who take hydralazine often require the addition of isosorbide dinitrate, which also dilates veins. There is no indication for reducing the dose of hydralazine. A diuretic can be given with hydralazine if sodium and water retention is present.

9. A nurse is caring for a woman during the perinatal and postnatal period. To stop postpartum hemorrhage, how will the nurse administer oxytocin [Pitocin] to this patient? a. 20 units in 1000 mL of IV solution after delivery of the placenta b. 20 units intramuscularly prior to delivery of the placenta c. 40 units intramuscularly after delivery of the placenta d. 60 units per rectum prior to delivery of the placenta

ANS: A To stop postpartum hemorrhage, oxytocin is administered either IV or IM following delivery of the placenta. If given IV, the dose is 10 to 40 units in 1000 mL of IV fluid. The IM dose is 10 units given as a single injection, also after delivery of the placenta. Misoprostol [Cytotec] is given rectally. PTS: 1 DIF: Cognitive Level: Application REF: p. 783

7. A patient will be discharged home to complete treatment with intravenous cefotetan with the assistance of a home nurse. The home care nurse will include which instruction when teaching the patient about this drug treatment? a. Abstain from alcohol consumption during therapy. b. Avoid dairy products while taking this drug. c. Take an antihistamine if a rash occurs. d. Use nonsteroidal anti-inflammatory drugs (NSAIDs), not acetaminophen, for pain.

ANS: A Two cephalosporins, including cefotetan, can induce a state of alcohol intolerance and cause a disulfiram-like reaction when alcohol is consumed; therefore, patients should be advised to avoid alcohol. It is not necessary to avoid dairy products. Patients who experience a rash should report this to their provider. Cefotetan can also promote bleeding, so drugs that inhibit platelet aggregation should be avoided.

13. Which statement is correct about the contrast between acarbose and miglitol? a. Miglitol has not been associated with hepatic dysfunction. b. With miglitol, sucrose can be used to treat hypoglycemia. c. Miglitol is less effective in African Americans. d. Miglitol has no gastrointestinal side effects.

ANS: A Unlike acarbose, miglitol has not been associated with hepatic dysfunction. Sucrose should not be used to treat hypoglycemia with miglitol. Miglitol is more effective in African American patients. Miglitol has gastrointestinal side effects.

5. A provider has ordered ceftriaxone 4 gm once daily for a patient with renal impairment. What will the nurse do? a. Administer the medication as prescribed. b. Contact the provider to ask about giving the drug in divided doses. c. Discuss increasing the interval between doses with the provider. d. Discuss reducing the dose with the provider.

ANS: A Unlike other cephalosporins, ceftriaxone is eliminated largely by the liver, so dosage reduction is unnecessary in patients with renal impairment. Giving the drug in divided doses, increasing the interval between doses, and reducing the dose are not necessary.

5. A patient who took an overdose of verapamil has been treated with gastric lavage and a cathartic. The emergency department nurse assesses the patient and notes a heart rate of 50 beats per minute and a blood pressure of 90/50 mm Hg. The nurse will anticipate: a. administering intravenous norepinephrine (NE) and atropine and glucagon. b. assisting with direct-current (DC) cardioversion. c. placing the patient in an upright position. d. preparing to administer a beta blocker.

ANS: A Verapamil toxicity can cause bradycardia and hypotension. Atropine and glucagon should be given to treat bradycardia and NE for hypotension. DC cardioversion is indicated for ventricular tachydysrhythmias, which this patient does not have. Patients with hypotension should be placed in Trendelenburg's position. Beta blockers will only exacerbate these effects.

13. Which two-drug regimen would be appropriate for a patient with hypertension who does not have other compelling conditions? a. Hydrochlorothiazide and nadolol b. Hydralazine and minoxidil c. Spironolactone and amiloride d. Trichlormethiazide and hydrochlorothiazide

ANS: A When two or more drugs are used to treat hypertension, each drug should come from a different class. Hydrochlorothiazide is a diuretic and nadolol is a beta blocker, so this choice is appropriate. Hydralazine and minoxidil are vasodilators. Spironolactone and amiloride are potassium-sparing diuretics. Trichlormethiazide and hydrochlorothiazide are both thiazide diuretics.

18. A patient has been taking digoxin [Lanoxin] 0.25 mg, and furosemide [Lasix] 40 mg, daily. Upon routine assessment by the nurse, the patient states, "I see yellow halos around the lights." The nurse should perform which action based on this assessment? a. Check the patient for other symptoms of digitalis toxicity. b. Withhold the next dose of furosemide. c. Continue to monitor the patient for heart failure. d. Document the findings and reassess in 1 hour.

ANS: A Yellow halos around lights indicate digoxin toxicity. The use of furosemide increases the risk of hypokalemia, which in turn potentiates digoxin toxicity. The patient should also be assessed for headache, nausea, and vomiting, and blood should be drawn for measurement of the serum digoxin level. The nurse should not withhold the dose of furosemide until further assessment is done, including measurement of a serum digoxin level. No evidence indicates that the patient is in worsening heart failure. Documentation of findings is secondary to further assessment and prevention of digoxin toxicity.

Which are therapeutic uses for verapamil? (Select all that apply.) a. Angina of effort b. Cardiac dysrhythmias c. Essential hypertension d. Sick sinus syndrome e. Suppression of preterm labor

ANS: A, B, C Verapamil is used to treat both vasospastic angina and angina of effort. It slows the ventricular rate in patients with atrial flutter, atrial fibrillation, and paroxysmal supraventricular tachycardia. It is a first-line drug for the treatment of essential hypertension. It is contraindicated in patients with sick sinus syndrome. Nifedipine has investigational uses in suppressing preterm labor.

1. Which patient-teaching instructions are appropriate for a patient taking an antidysrhythmic drug? (Select all that apply.) a. "Do not chew or crush extended-release forms of medication." b. "Take the medication with food if gastrointestinal distress occurs." c. "If a dose is missed, the missed dose should be taken along with the next dose that is due to be taken." d. "Take the medications with an antacid if gastrointestinal distress occurs." e. "Limit or avoid the use of caffeine." f. "The presence of a capsule in the stool should be reported to the physician immediately."

ANS: A, B, E Appropriate teaching instructions for a patient taking an antidysrhythmic drug include: do not chew or crush extended-release forms; if gastrointestinal distress occurs, take the drug with food; and limit or avoid the use of caffeine. Do not double medication doses or take medications with an antacid. The presence of a portion of a capsule or tablet in the stool is actually the wax matrix that carried the medication, which has been absorbed. The physician does not need to be notified. DIF: COGNITIVE LEVEL: Applying (Application) REF: pp. 408-409 TOP: NURSING PROCESS: Implementation

Besides the cost of administering a given drug, which are considerations when a provider selects a cephalosporin to treat an infection? (Select all that apply.) a. Adverse effects b. Antimicrobial spectrum c. Brand name d. Manufacturer e. Pharmacokinetics

ANS: A, B, E Cephalosporins tend to be more alike than different, but some differences make some preferable to others. Adverse effects, antimicrobial spectrum, and drug pharmacokinetics affect a prescriber's decision to select one drug over another. The agent's brand name and manufacturer should not have anything to do with drug selection.

1. A patient will be taking dabigatran (Pradaxa) as part of treatment for chronic atrial fibrillation. Which statements about dabigatran are true? (Select all that apply.) a. The dose of dabigatran is reduced in patients with decreased renal function. b. Bleeding is the most common adverse effect. c. Potassium chloride is given as an antidote in cases of overdose. d. Dabigatran levels are monitored by measuring prothrombin time/international normalized ratio (PT/INR) results. e. This drug is a prodrug and becomes activated in the liver.

ANS: A, B, E Dabigatran is excreted extensively in the kidneys, and the dose is dependent upon renal function. The normal dose is 150 mg twice daily, but it must be reduced to 75 mg twice daily if creatinine clearance is less than 30 mL/min. The most common and serious side effect is bleeding. Dabigatran is a prodrug that becomes activated in the liver. There is no antidote to dabigatran. The other options are incorrect. DIF: COGNITIVE LEVEL: Applying (Application) REF: p. 417 TOP: NURSING PROCESS: Implementation

Which patients may receive hydralazine to treat hypertension? (Select all that apply.) a. A 1-month-old infant b. A 5-year-old child c. A pregnant woman d. A mother breast-feeding a newborn e. An older adult

ANS: A, B, E Hydralazine may be used in infants as young as one month of age, in children, and in older adults. Hydralazine is labeled pregnancy category C and data is lacking regarding transmission of hydralazine in breast-feeding women, so benefits should outweigh risks.

Besides having diuretic effects for patients with congestive heart failure, thiazides are also used to treat what? (Select all that apply.) a. Diabetes insipidus b. Hepatic failure c. Increased intracranial pressure d. Intraocular pressure e. Postmenopausal osteoporosis

ANS: A, B, E Thiazide diuretics have the paradoxical effect of reducing urine output in patients with diabetes insipidus. They can also be used to mobilize edema associated with liver disease. They promote tubular reabsorption of calcium, which may reduce the risk of osteoporosis in postmenopausal women. Mannitol is used to treat edema that causes increased intracranial pressure and intraocular pressure.

Which are indications for discontinuing growth hormone (GH) therapy in children with documented growth hormone deficiency? (Select all that apply.) a.Epiphyseal closure has begun. b.A satisfactory adult height has been achieved. c.Serum GH levels have been normal for 12 consecutive months. d.Normal serum growth hormone levels occur. e.The child no longer responds to the hormone.

ANS: A, B, E Treatment with GH for a child with a documented GH deficiency may continue until a satisfactory adult height has been achieved, if a child does not show a response to the hormone, or until epiphyseal closure begins. Treatment does not continue for life, and GH should not be administered during or after closure of the epiphyses. Treatment does not depend on growth hormone levels; these are measured to ensure therapeutic dosing.

2. The nurse is reviewing the information about the herpes zoster vaccine (Zostavax) before administering the dose. Which statements about the vaccine are true? (Select all that apply.) a. It is a one-time vaccine. b. The vaccine is recommended for patients 50 years of age and older. c. The vaccine is given to children to prevent chickenpox. d. It is used to prevent postherpetic neuralgia. e. It is contraindicated in patients who have already had shingles. f. The vaccine is used to prevent reactivation of the zoster virus that causes shingles.

ANS: A, B, F Zoster vaccine (Zostavax) is used to prevent shingles; it also prevents reactivation of the zoster virus that causes shingles. It is given to patients 50 years of age and older, and it is a one-time vaccine. It is not given to prevent chickenpox or given to children. It does not prevent postherpetic neuralgia, and it can be given to patients who have already had shingles. DIF: COGNITIVE LEVEL: Understanding (Comprehension) REF: p. 790 TOP: NURSING PROCESS: Implementation

2. Which are indications for early induction of labor? (Select all that apply.) a. Abruptio placentae b. Active genital herpes infection c. Gestational hypertension d. Premature rupture of the membranes e. Umbilical cord prolapse

ANS: A, C, D Indications for early induction of labor include abruptio placentae, gestational hypertension, and premature rupture of the membranes. Active genital herpes infection and umbilical cord prolapse are contraindications to induction, even at term, because they can involve significant harm to the fetus. PTS: 1 DIF: Cognitive Level: Analysis REF: p. 781

1. Methotrexate is ordered for a patient with a malignant tumor, and the nurse is providing education about self-care after the chemotherapy is given. Which statements by the nurse are appropriate for the patient receiving methotrexate? (Select all that apply.) a. Report unusual bleeding or bruising. b. Hair loss is not expected with this drug. c. Prepare for hair loss. d. Avoid areas with large crowds or gatherings. e. Avoid foods that are too hot or too cold or rough in texture. f. Restrict fluid intake to reduce nausea and vomiting.

ANS: A, C, D, E Counsel patients who are taking methotrexate to expect hair loss and to report any unusual bleeding or bruising. Because of the possibility of infection, avoid areas with large crowds or gatherings. Foods that are too hot or too cold or rough in texture may be irritating to the oral mucosa. Fluid intake is to be encouraged to prevent dehydration. DIF: COGNITIVE LEVEL: Applying (Application) REF: pp. 731-732 TOP: NURSING PROCESS: Implementation

A patient asks about the effectiveness of various birth control methods. The nurse should inform her that the most effective methods are what? (Select all that apply.) a. Intrauterine devices (IUDs) b. Spermicides c. Intramuscular medroxyprogesterone acetate d. Etonogestrel subdermal implants e. Male or female sterilization

ANS: A, C, D, E The most effective methods of birth control are IUDs, intramuscular medroxyprogesterone acetate [Depo-Provera], implants, and sterilization. Spermicides are not one of the most effective methods of birth control.

1. The nurse is administering intravenous vancomycin (Vancocin) to a patient who has had gastrointestinal surgery. Which nursing measures are appropriate? (Select all that apply.) a. Monitoring serum creatinine levels b. Restricting fluids while the patient is on this medication c. Warning the patient that a flushed feeling or facial itching may occur d. Instructing the patient to report dizziness or a feeling of fullness in the ears e. Reporting a trough drug level of 11 mcg/mL and holding the drug f. Reporting a trough drug level of 24 mcg/mL and holding the drug

ANS: A, C, D, F Constant monitoring for drug-related neurotoxicity, nephrotoxicity, ototoxicity, and superinfection remain critical to patient safety. Monitor for nephrotoxicity by monitoring serum creatinine levels. Ototoxicity may be indicated if the patient experiences dizziness or a feeling of fullness in the ears, and these symptoms must be reported immediately. Vancomycin infusions may cause red man syndrome, which is characterized by flushing of the neck and face and a decrease in blood pressure. In addition, adequate hydration (at least 2 L of fluids every 24 hours unless contraindicated) is most important to prevent nephrotoxicity. Optimal trough blood levels of vancomycin are 10 to 20 mcg/mL; therefore, the drug should not be administered when there is a trough level of 24 mcg/mL. DIF: COGNITIVE LEVEL: Applying (Application) REF: p. 633 | p. 638 TOP: NURSING PROCESS: Evaluation

A patient will be taking amiodarone [Cordarone]. Which baseline tests are necessary before this medication is started? (Select all that apply.) a. Chest radiograph and pulmonary function tests b. Complete blood count with differential c. Ophthalmologic examination d. Renal function tests e. Thyroid function tests

ANS: A, C, E Amiodarone has many potential toxic side effects, including pulmonary toxicity, ophthalmic effects, and thyroid toxicity, so these systems should be evaluated at baseline and periodically while the patient is taking the drug. A complete blood count is not indicated. Renal function tests are not indicated.

2. When giving chemotherapy as cancer treatment, the nurse recognizes that toxicity to rapidly growing normal cells also occurs. Which rapidly growing normal cells are also harmed by chemotherapy? (Select all that apply.) a. Bone marrow cells b. Retinal cells c. Hair follicle cells d. Nerve myelin cells e. Gastrointestinal (GI) mucous membrane cells

ANS: A, C, E Chemotherapy toxicities generally stem from the fact that chemotherapy drugs affect rapidly dividing cells—both harmful cancer cells and healthy, normal cells. Three types of rapidly dividing human cells are the cells of hair follicles, GI tract cells, and bone marrow cells. The other options are incorrect. DIF: COGNITIVE LEVEL: Understanding (Comprehension) REF: p. 725 TOP: NURSING PROCESS: Implementation

Which medications are included in first-line therapy for heart failure? (Select all that apply.) a. Agents that inhibit the renin-angiotensin-aldosterone system (RAAS) b. Aldosterone antagonists c. Beta blockers d. Cardiac glycosides e. Diuretics

ANS: A, C, E For routine therapy, heart failure is treated with agents that inhibit the RAAS, beta blockers, and diuretics. Aldosterone antagonists and cardiac glycosides are not first-line drugs for HF.

1. The nurse is providing patient education for a patient taking an oral contraceptive. Which drugs may cause interactions with oral contraceptives? (Select all that apply.) a. Cephalexin (Keflex) b. Guaifenesin (Robitussin) c. Warfarin (Coumadin) d. Ibuprofen (Motrin) e. Theophylline (Uniphyl)

ANS: A, C, E Patients must be educated about the need to use alternative birth control methods for at least 1 month during and after taking any of these drugs: antibiotics (especially penicillins and cephalosporins); barbiturates; isoniazid; and rifampin. The effectiveness of other drugs, such as anticonvulsants, beta blockers, hypnotics, antidiabetic drugs, warfarin, theophylline, tricyclic antidepressants, and vitamins, may be reduced when they are taken with oral contraceptives. DIF: COGNITIVE LEVEL: Applying (Application) REF: p. 540 TOP: NURSING PROCESS: Planning

A patient is admitted to the hospital after several days of vomiting and diarrhea. After an initial bolus of isotonic (0.9%) sodium chloride solution, the prescriber orders dextrose 5% in normal saline (D5NS) with 20 mEq potassium chloride to infuse at a maintenance rate. What should the nurse review before implementing this order? (Select all that apply.) a. Electrocardiogram b. Arterial blood gas levels c. Serum electrolyte levels d. Serum glucose level e. Urine output

ANS: A, C, E Patients receiving potassium should be monitored for cardiovascular toxicity; an ECG before and during administration can help monitor for this adverse effect. Serum electrolyte levels should be reviewed to make sure the patient is not already hyperkalemic. Because potassium is excreted via the kidneys, it is important to determine that renal function is intact. Intravenous potassium should never be given if the patient has not voided. Patients who are hyperkalemic can be treated with sodium bicarbonate to increase pH and insulin to promote uptake of potassium by cells, but it is not necessary to evaluate the blood gas or glucose levels before administering potassium.

1. Which of the following vitamins can be stored by the body? (Select all that apply.) a. Vitamin E (alpha-tocopherol) b. Vitamin C (ascorbic acid) c. Cyanocobalamin (vitamin B12) d. Folic acid e. Vitamin D

ANS: A, C, E The fat-soluble vitamins (A, D, E, and K) are stored in massive amounts in the body. Of the water-soluble vitamins, B12 can be stored; all others must be replenished with frequent ingestion. PTS: 1 DIF: Cognitive Level: Comprehension REF: pp. 987-988

2. The nurse is instructing a male patient about application of transdermal testosterone gel (AndroGel). Which body location is preferred for this medication? (Select all that apply.) a. Back b. Chest c. Thigh d. Scrotum e. Abdomen f. Upper arms

ANS: A, C, E, F AndroGel is applied to the skin of the back, abdomen, upper arms, or thighs. Testoderm patches are applied to the scrotal skin. DIF: COGNITIVE LEVEL: Applying (Application) REF: p. 558 TOP: NURSING PROCESS: Implementation

1. The nurse is reviewing vitamin therapy in preparation for a nutrition class. Which statements are accurate regarding vitamin C (ascorbic acid)? (Select all that apply.) a. Vitamin C is important in the maintenance of bone, teeth, and capillaries. b. Vitamin C is essential for night vision. c. Vitamin C is important for tissue repair. d. Vitamin C is found in animal sources such as dairy products and meat. e. Vitamin C is found in tomatoes, strawberries, and broccoli. f. Vitamin C is also known as the "sunshine vitamin." g. Vitamin C deficiency is known as scurvy.

ANS: A, C, E, G These statements are true of vitamin C. Vitamin A is essential for night vision, and vitamin D is known as the sunshine vitamin. With the exception of liver, meat and dairy products are not sources of vitamin C. DIF: COGNITIVE LEVEL: Applying (Application) REF: p. 852 TOP: NURSING PROCESS: Implementation

1. The nurse is providing education about the use of sublingual nitroglycerin tablets. She asks the patient, "What would you do if you experienced chest pain while mowing your yard? You have your bottle of sublingual nitroglycerin with you." Which actions by the patient are appropriate in this situation? (Select all that apply.) a. Stop the activity, and lie down or sit down. b. Call 911 immediately. c. Call 911 if the pain is not relieved after taking one sublingual tablet. d. Call 911 if the pain is not relieved after taking three sublingual tablets in 15 minutes. e. Place a tablet under the tongue. f. Place a tablet in the space between the gum and cheek. g. Take another sublingual tablet if chest pain is not relieved after 5 minutes, up to three total.

ANS: A, C, E, G With sublingual forms, the medication is taken at the first sign of chest pain, not delayed until the pain is severe. The patient needs to sit down or lie down and take one sublingual tablet. According to current guidelines, if the chest pain or discomfort is not relieved in 5 minutes, after 1 dose, the patient (or family member) must call 911 immediately. The patient can take one more tablet while awaiting emergency care and may take a third tablet 5 minutes later, but no more than a total of three tablets. These guidelines reflect the fact that angina pain that does not respond to nitroglycerin may indicate a myocardial infarction. The sublingual dose is placed under the tongue, and the patient needs to avoid swallowing until the tablet has dissolved. Placing a tablet between the gum and cheek is the buccal route. DIF: COGNITIVE LEVEL: Analyzing (Analysis) REF: p. 372 TOP: NURSING PROCESS: Implementation

2. What are the potential beneficial effects of taking vitamin E (alpha-tocopherol) supplements? (Select all that apply.) a. Delayed progression of macular degeneration b. Improved blood clotting c. Lowered risk of developing Alzheimer's disease d. Reduced risk of cardiovascular disease e. Reduced risk of colds in elderly patients

ANS: A, E Studies to determine the potential benefits of vitamin E supplements show a decreased risk of colds in elderly patients when vitamin E is combined with vitamin C, beta-carotene, zinc, and copper, and a potential delay in the progression of macular degeneration. Vitamin E may actually increase the risk of bleeding. Studies have not shown a decrease in the risk of Alzheimer's disease or cardiovascular disease. PTS: 1 DIF: Cognitive Level: Analysis REF: pp. 990-991

1. When teaching a patient about antihypertensive drug therapy, which statements by the nurse are correct? (Select all that apply.) a. "You need to have your blood pressure checked once a week and keep track of the readings." b. "If you notice that the symptoms have gone away, you should be able to stop taking the drug." c. "An exercise program may be helpful in treating hypertension, but let's check with your doctor first." d. "If you experience severe side effects, stop the medicine and let us know at your next office visit." e. "Most over-the-counter decongestants are compatible with antihypertensive drugs." f. "Please continue taking the medication, even if you are feeling better."

ANS: A, C, F Keeping a record of weekly blood pressure checks helps to monitor the effectiveness of the therapy. Remind the patient not to stop taking the medication just because he or she is feeling better. Abruptly stopping the medication may lead to rebound hypertension. Therapy is often lifelong, even though symptoms may improve. Many over-the-counter drugs, especially decongestants, have serious interactions with antihypertensive drugs. The patient needs to consult his or her prescriber before taking any other medication. DIF: COGNITIVE LEVEL: Analyzing (Analysis) REF: p. 359 TOP: NURSING PROCESS: Assessment

1. The nurse is reviewing the health history of a new patient who may need immunizations. Active immunizations are usually contraindicated in which patients? (Select all that apply.) a. Patients with a febrile illness b. Children younger than 1 year of age c. Elderly patients d. Patients who are immunosuppressed e. Those receiving cancer chemotherapy

ANS: A, D, E Contraindications to the administration of immunizing drugs include a history of reactions to or serious adverse effects resulting from the drugs, and patients who are already immunosuppressed (patients with AIDS and patients receiving chemotherapy). Immunizations are best deferred until after a febrile illness. Children younger than 1 year of age and the elderly may receive immunizing drugs. DIF: COGNITIVE LEVEL: Applying (Application) REF: p. 787 TOP: NURSING PROCESS: Assessment

1. The nurse notes in a patient's medication history that the patient is taking the synthetic androgen danazol (Danocrine). Indications for danazol include which conditions? (Select all that apply.) a. Endometriosis b. Decreased sexual libido c. Postpartum breast engorgement d. Fibrocystic breast disease in women e. Hereditary angioedema f. Metastatic breast cancer

ANS: A, D, E Danazol is used to treat hereditary angioedema and to treat women who have endometriosis or fibrocystic breast disease. The other options are incorrect. DIF: COGNITIVE LEVEL: Applying (Application) REF: p. 555 TOP: NURSING PROCESS: Planning

1. The nurse is assessing a patient who is receiving chemotherapy with an alkylating drug. Which assessment findings would be considered indications of an oncologic emergency? (Select all that apply.) a. Dry, "scratchy," or "swollen" throat b. Loss of hair c. Decreased red blood cell count d. White patches in the mouth or throat e. Temperature of 100.7° F (38.2° C) f. Decreased urine output

ANS: A, D, E, F Indications of an oncologic emergency include fever and/or chills with a temperature higher than 100.5° F (38.1° C); new sores or white patches in the mouth or throat; changes in bladder function or patterns; dry, burning, "scratchy," or "swollen" throat; and other signs and symptoms (see Box 46-4). The prescriber must be contacted immediately if any of the listed signs or symptoms occur. Loss of hair and decreased red blood cell count (a result of bone marrow suppression) are expected effects of chemotherapy. DIF: COGNITIVE LEVEL: Analyzing (Analysis) REF: p. 744 TOP: NURSING PROCESS: Assessment

1. The nurse is reviewing the indications for atropine sulfate. Atropine is appropriate for which of these patients? (Select all that apply.) a. A patient who has suddenly developed symptomatic bradycardia with a heart rate of 32 beats/min b. A patient who has suddenly developed symptomatic tachycardia with a heart rate of 180 beats/min c. A patient with severe narrow-angle glaucoma d. A patient who is about to have surgery e. A patient newly diagnosed with myasthenia gravis f. A patient with anticholinesterase inhibitor poisoning

ANS: A, D, F Anticholinergic drugs are used for symptomatic bradycardia and certain other cardiac conditions. It is given preoperatively to control secretions during surgery and is used as an antidote for anticholinesterase inhibitor poisoning. The other options are contraindications to the use of atropine. DIF: COGNITIVE LEVEL: Analyzing (Analysis) REF: p. 334 TOP: NURSING PROCESS: Planning

2. A patient tells the nurse that she wants to start taking the herbal product goldenseal to improve her health. The nurse will assess for which potential cautions or contraindications to goldenseal? (Select all that apply.) a. Taking a proton-pump inhibitor b. Nasal congestion c. Hypothyroidism d. Hypertension e. Sinus infections f. Pregnancy

ANS: A, D, F Goldenseal is contraindicated in patients with acute or chronic gastrointestinal disorders and during pregnancy (because it has uterine stimulant properties). It should be used with caution by those with cardiovascular disorders. Potential drug interactions may occur with gastric acid suppressors such as antacids, histamine H2 blockers (e.g., ranitidine), and proton pump inhibitors (e.g., omeprazole), theoretically because of their reduced effectiveness resulting from the acid-promoting effect of the herb, and with antihypertensive drugs (because of the vasoconstrictive activity of the herb). Goldenseal is potentially useful for sinus infections and chronic nasal allergies. DIF: COGNITIVE LEVEL: Applying (Application) REF: p. 567 TOP: NURSING PROCESS: Assessment

2. A patient will be receiving nitrofurantoin (Macrodantin) treatment for a urinary tract infection. The nurse is reviewing the patient's history and will question the nitrofurantoin order if which disorder is present in the history? (Select all that apply.) a. Liver disease b. Coronary artery disease c. Hyperthyroidism d. Type 1 diabetes mellitus e. Chronic renal disease

ANS: A, E Nitrofurantoin is contraindicated in cases of known drug allergy and also in cases of significant renal function impairment, because the drug concentrates in the urine. Because adverse effects include hepatotoxicity, which is rare but often fatal, the nurse should also question the order if liver disease is present. The other options are not contraindications. DIF: COGNITIVE LEVEL: Applying (Application) REF: p. 632 TOP: NURSING PROCESS: Assessment

A nurse is caring for a patient with HIV/acquired immunodeficiency syndrome (AIDS) who has taken protease inhibitors for 18 months. The patient has been diagnosed with active tuberculosis. Which drug(s) would the nurse expect to be added to the treatment plan of a patient with HIV/AIDS? (Select all that apply.) a. Isoniazid b. Rifampin [Rifadin] c. Pyrazinamide [PZA] d. Ethambutol [Myambutol] e. Rifabutin [Mycobutin]

ANS: A,C,D,E Isoniazid, pyrazinamide, ethambutol, and rifabutin could be included in the treatment of a patient with HIV/AIDS. Rifampin is contraindicated as part of the new antituberculosis regimen, because it would reduce the effects of the protease inhibitors.

Which patient should begin treatment for tuberculosis? a. A patient with HIV and a tuberculin skin test result of a 4-mm region of induration b. A recent immigrant from a country with a high prevalence of TB with a 10-mm region of induration c. A patient with no known risk factors who has a job-related tuberculin skin test result of a 12-mm area of induration d. An intravenous drug abuser with a tuberculin skin test result of a 5-mm region of induration

ANS: B testbanks_and_xanax The immigrant is considered to be at moderate risk, meaning that a 10-mm area of induration on a tuberculin skin test (TST) is considered a positive result. After being evaluated for active TB, this patient should be treated for latent TB. A patient with HIV is considered high risk, but this patient has a negative TST result of less than 5 mm. For a low-risk patient receiving a screening TST for a job, the area of induration must be 15 mm or greater to be considered a positive result. An IV drug abuser is in the moderate-risk category; an area of induration of 10 mm or greater is needed to be considered a positive TST result.

3. While assessing a patient who is receiving intravenous digitalis, the nurse recognizes that the drug has a negative chronotropic effect. How would this drug effect be evident in the patient? a. Decreased blood pressure b. Decreased heart rate c. Decreased conduction d. Decreased ectopic beats

ANS: B A negative chronotropic effect results in a decreased heart rate; this is one effect of cardiac glycosides. The other options are incorrect. DIF: COGNITIVE LEVEL: Applying (Application) REF: p. 381 TOP: NURSING PROCESS: Evaluation

10. A patient has had recent mechanical heart valve surgery and is receiving anticoagulant therapy. While monitoring the patient's laboratory work, the nurse interprets that the patient's international normalized ratio (INR) level of 3 indicates that: a. the patient is not receiving enough warfarin to have a therapeutic effect. b. the patient's warfarin dose is at therapeutic levels. c. the patient's intravenous heparin dose is dangerously high. d. the patient's intravenous heparin dose is at therapeutic levels.

ANS: B A normal INR (without warfarin) is 1.0. A therapeutic INR for patients who have had mechanical heart valve surgery ranges from 2.5 to 3.5, with a middle value of 3. DIF: COGNITIVE LEVEL: Applying (Application) REF: p. 419 TOP: NURSING PROCESS: Implementation

10. After a severe auto accident, a patient has been taken to the trauma unit and has an estimated blood loss of more than 30% of his blood volume. The nurse prepares to administer which product? a. Albumin b. Whole blood c. Packed red blood cells d. Fresh frozen plasma

ANS: B A patient who has lost a massive amount (over 25%) of blood volume would receive whole blood. PRBCs are given to increase the oxygen-carrying capacity in patients with anemia, in patients with substantial hemoglobin deficits, and in patients who have lost up to 25% of their total blood volume. A patient with a coagulation disorder or a clotting-factor deficiency would receive fresh frozen plasma; albumin is used to expand fluid volume. DIF: COGNITIVE LEVEL: Applying (Application) REF: p. 469 TOP: NURSING PROCESS: Planning

3. A nurse is educating the staff nurses about ketoacidosis. To evaluate the group's understanding, the nurse asks, "Which sign or symptom would not be consistent with ketoacidosis?" The group gives which correct answer? a. Blood glucose level of 600 mg/dL b. Blood glucose level of 60 mg/dL c. Acidosis d. Ketones in the urine

ANS: B A patient with diabetic ketoacidosis (DKA) has a high glucose level (at least 500 mg/dL or higher); therefore, a glucose level of 60 mg/dL would not be consistent with DKA. A blood glucose level of 600 mg/dL, acidosis, and ketones in the urine are consistent with DKA.

The nurse is teaching a patient who is newly diagnosed with myasthenia gravis about managing the disease. Which is important when teaching this patient about adverse effects of cholinesterase inhibitors? a."There are very few serious side effects associated with your medication." b."It is important to notify your provider if you have excessive saliva." c."If you experience urinary urgency, you may need to increase your dose." d."Excessive sweating is a minor side effect and will improve over time."

ANS: B Accumulation of ACh at muscarinic receptors can result in excessive salivation, bradycardia, sweating, increased gastric secretions, increased tone and motility of the GI tract, and urinary urgency. These responses may have to be suppressed with atropine. In toxic doses, accumulation of ACh may produce depolarizing neuromuscular blockade with paralysis of respiratory muscles. Patients should be taught to report any of these symptoms to their provider.

14. A nurse is discussing adenosine with a nursing student. Which statement by the student indicates a need for further teaching? a. "Adenosine acts by suppressing action potentials in the SA and AV nodes." b. "Adenosine can be used to prevent paroxysmal supraventricular tachycardia and Wolff-Parkinson-White syndrome." c. "Adenosine has a half-life that lasts only a few seconds and must be given intravenously." d. "Adenosine is not effective for treating atrial fibrillation, atrial flutter, or ventricular dysrhythmias."

ANS: B Adenosine is used to terminate paroxysmal supraventricular tachycardia (SVT) and Wolff-Parkinson-White (WPW) syndrome, not to prevent symptoms. Adenosine suppresses action potentials in the SA and AV nodes. Because it has a very short half-life of 1.5 to 10 seconds, it must be given IV bolus, as close to the heart as possible. Adenosine is not active against atrial fibrillation, atrial flutter, or ventricular dysrhythmias.

1. The nurse is administering adrenal drugs to a patient. Which action by the nurse is appropriate for this patient? a. Administering oral drugs on an empty stomach to maximize absorption b. Rinsing the oral cavity after using corticosteroid inhalers c. Administering the corticosteroids before bedtime to minimize adrenal suppression d. Discontinuing the medication immediately if weight gain of 5 pounds or more in 1 week occurs

ANS: B After the patient has used the corticosteroid inhalers, cleaning the oral cavity helps to prevent possible oral fungal infections from developing. Adrenal drugs need be taken with meals to minimize gastrointestinal upset and in the mornings to minimize adrenal suppression, and they need to be discontinued by weaning, not abruptly. DIF: COGNITIVE LEVEL: Applying (Application) REF: p. 530 TOP: NURSING PROCESS: Implementation

A nurse is teaching a group of nursing students about irreversible cholinesterase inhibitors. Which statement by a student indicates understanding of the teaching? a."Diazepam is given to reverse inhibition of cholinesterase when overdose occurs." b."Irreversible cholinesterase inhibitors are rapidly absorbed by all routes." c."These agents are often used to treat glaucoma." d."Toxic doses of these agents produce an anticholinergic crisis."

ANS: B Almost all of these agents are highly lipid soluble, making them readily absorbed from all routes of administration. Diazepam is used during overdose but only to control seizures and not to reverse effects. The only clinical use for these agents is to treat glaucoma, but this use is limited. Toxic doses of these agents produce a cholinergic, not an anticholinergic, crisis.

6. A patient asks the nurse about the difference between diphenoxylate with atropine (Lomotil) and the over-the-counter drug loperamide (Imodium). Which response by the nurse is correct? a. "Lomotil acts faster than Imodium." b. "Imodium does not cause physical dependence." c. "Lomotil is available in suppository form." d. "Imodium is a natural antidiarrheal drug."

ANS: B Although the drug exhibits many characteristics of the opiate class, physical dependence on loperamide has not been reported. All antidiarrheal drugs are orally administered. The other options are incorrect. DIF: COGNITIVE LEVEL: Applying (Application) REF: p. 817 TOP: NURSING PROCESS: Planning

A nurse is helping a nursing student who is administering a medication to a patient with myasthenia gravis. Which statement by the student indicates the need for further teaching? a."I will ask the patient to sip some water before giving the medication." b."I will let the patient sleep after giving the medication, because rest is important." c."I will record muscle strength assessments before and after I give the medication." d."I will report excessive salivation to the patient's prescriber."

ANS: B An important assessment indicating the medication's effectiveness is evaluating a patient's ability to raise the eyelids. Letting the patient sleep, even though rest is important, would prevent the nurse from making this assessment. To make sure the patient can safely swallow a pill, the nurse must evaluate swallowing ability by asking the patient to take a sip of water. Muscle strength assessments before and after medication administration help the nurse evaluate the medication's effectiveness. Excessive salivation may be a sign of drug toxicity and should be reported.

1. When monitoring a patient who has diabetes and is receiving a carbonic anhydrase inhibitor for edema, the nurse will monitor for which possible adverse effect? a. Metabolic alkalosis b. Elevated blood glucose c. Hyperkalemia d. Mental alertness

ANS: B An undesirable effect of carbonic anhydrase inhibitors is that they elevate the blood glucose level and cause glycosuria in diabetic patients. They induce metabolic acidosis, making their usefulness limited. In addition, hypokalemia and drowsiness may occur. DIF: COGNITIVE LEVEL: Applying (Application) REF: p. 453 TOP: NURSING PROCESS: Evaluation

4. A 12-month-old infant has received an MMR II (measles, mumps, and rubella virus vaccine), and her mother calls the clinic that afternoon to ask about helping her fussy infant to "feel better." What will the nurse suggest? a. Apply an ice pack to the injection site. b. Apply warm compresses to the injection site. c. Observe the site for further swelling and redness. d. Bring the infant in to the emergency department for an immediate examination.

ANS: B Applying warm compresses to the injection site and using acetaminophen (not aspirin, which carries the risk for Reye's syndrome) should help to relieve the discomfort. The other options are incorrect. DIF: COGNITIVE LEVEL: Applying (Application) REF: p. 794 TOP: NURSING PROCESS: Implementation

2. If a patient is taking fluconazole (Diflucan) with an oral anticoagulant, the nurse will monitor for which possible interaction? a. Reduced action of oral anticoagulants b. Increased effects of oral anticoagulants c. Hypokalemia d. Decreased effectiveness of the antifungal drug

ANS: B Azole antifungal drugs increase the effects of oral anticoagulants. As a result, increased bleeding may occur. The other options are incorrect. DIF: COGNITIVE LEVEL: Applying (Application) REF: p. 675 TOP: NURSING PROCESS: Assessment

7. The nurse is administering a combination of three different antineoplastic drugs to a patient who has metastatic breast cancer. Which statement best describes the rationale for combination therapy? a. There will be less nausea and vomiting. b. Increased cancer-cell killing will occur. c. The drugs will prevent metastasis. d. Combination therapy reduces the need for radiation therapy.

ANS: B Because drug-resistant cells commonly develop, exposure to multiple drugs with multiple mechanisms and sites of action will destroy more subpopulations of cells. The other options are incorrect. DIF: COGNITIVE LEVEL: Understanding (Comprehension) REF: p. 715 TOP: NURSING PROCESS: Planning

A patient collapses after running a marathon on a hot day and is brought to the emergency department to be treated for dehydration. The nurse will expect to provide which therapy? a. Intravenous hypertonic fluids given slowly over several hours b. Intravenous hypotonic fluids administered in stages c. Intravenous isotonic fluids given as a rapid bolus d. Oral electrolyte replacement fluids with potassium

ANS: B Because this patient is experiencing fluid volume loss as the result of excessive sweating, this is most likely hypertonic dehydration, in which loss of water exceeds loss of electrolytes. This should be treated with a hypotonic solution or with fluids that contain no solutes at all. Initial treatment may consist of having the patient drink water. When intravenous therapy is provided, volume replenishment should occur in stages. Hypertonic fluids are used to treat hypotonic contraction, usually caused by excessive sodium loss through the kidneys as the result of diuretic therapy. Isotonic fluids are used to treat isotonic contraction, which is generally caused by vomiting and diarrhea. An oral electrolyte solution would only increase the hypertonicity; if oral rehydration is used in this case, the patient should drink plain water.

Bethanechol [Urecholine] is used to treat urinary retention but is being investigated for use in which other condition? a.Gastric ulcers b.Gastroesophageal reflux c.Hypotension d.Intestinal obstruction

ANS: B Bethanechol is being investigated for treatment for gastroesophageal reflux disease (GERD) because of its effects on esophageal motility and the lower esophageal sphincter. Bethanechol stimulates acid secretion and could intensify ulcer formation. Bethanechol can cause hypotension. Because bethanechol increases the motility and tone of intestinal smooth muscle, the presence of an obstruction could lead to bowel rupture

9. A patient is taking bismuth subsalicylate [Pepto-Bismol] to prevent diarrhea. The nurse performing an assessment notes that the patient's tongue is black. What will the nurse do? a. Assess further for signs of gastrointestinal (GI) bleeding. b. Reassure the patient that this is an expected side effect of this drug. c. Request an order for liver function tests to evaluate for hepatotoxicity. d. Withhold the drug, because this is a sign of bismuth overdose.

ANS: B Bismuth subsalicylate can cause blackening of the tongue and stools, an expected side effect. This finding does not indicate GI bleeding, hepatotoxicity, or drug overdose.

11. The nurse is providing patient teaching about the oral bisphosphonate alendronate (Fosamax). Which statement by the patient indicates a good understanding of when this drug should be taken? a. "I will take it in the evening just before bedtime." b. "I will take it in the morning with an 8-ounce glass of water." c. "I will take it with the first bite of the morning meal." d. "I will take it between meals on an empty stomach."

ANS: B Bisphosphonates must be taken in the morning, with 6 to 8 ounces of plain water, to prevent esophageal erosion. In addition, the patient must sit upright for 30 minutes after taking them. DIF: COGNITIVE LEVEL: Applying (Application) REF: p. 549 TOP: NURSING PROCESS: Implementation

6. A nurse provides teaching for a patient who will begin taking procainamide [Procanbid] for long-term suppression of a dysrhythmia. Which statement by the patient indicates a need for further teaching? a. "I need to take this drug at evenly spaced intervals around the clock." b. "I may have increased bruising, but this is a temporary side effect." c. "I should report pain and swelling in my joints when taking this drug." d. "I will need to have blood tests at regular intervals while taking this drug."

ANS: B Blood dyscrasias are a rare but potentially fatal side effect of procainamide and are an indication for withdrawing the drug. Procainamide should be taken around the clock at evenly spaced intervals. Lupus-like symptoms may occur; inflammation of the joints is one manifestation and should be reported so that antinuclear antibody (ANA) titers can be monitored. Because of the risk of lupus-like symptoms and blood dyscrasias, blood tests need to be done weekly at first and then periodically thereafter.

8. The nurse is preparing to transfuse a patient with a unit of packed red blood cells (PRBCs). Which intravenous solution is correct for use with the PRBC transfusion? a. 5% dextrose in water (D5W) b. 0.9% sodium chloride (NS) c. 5% dextrose in 0.45% sodium chloride (D5NS) d. 5% dextrose in lactated Ringer's solution (D5LR)

ANS: B Blood products should be given only with normal saline 0.9% because D5W will also cause hemolysis of the blood product. DIF: COGNITIVE LEVEL: Applying (Application) REF: p. 479 TOP: NURSING PROCESS: Implementation

7. A patient wants to prevent problems with constipation and asks the nurse for advice about which type of laxative is safe to use for this purpose. Which class of laxative is considered safe to use on a long-term basis? a. Emollient laxatives b. Bulk-forming laxatives c. Hyperosmotic laxatives d. Stimulant laxatives

ANS: B Bulk-forming laxatives are the only laxatives recommended for long-term use. Stimulant laxatives are the most likely of all the laxative classes to cause dependence. The other options are incorrect. DIF: COGNITIVE LEVEL: Applying (Application) REF: p. 819 TOP: NURSING PROCESS: Planning

A patient who is taking a combination oral contraceptive begins taking carbamazepine. After several weeks, the patient tells the nurse she has begun experiencing spotting during her cycle. What will the nurse tell her to do? a. Change to condoms instead of oral contraceptives. b. Discuss an oral contraceptive with increased estrogen. c. Request a decreased dose of carbamazepine. d. Stop taking the oral contraceptive immediately.

ANS: B Carbamazepine induces hepatic cytochrome P450 and thus accelerates the metabolism of oral contraceptives. Spotting is a sign of reduced OC blood levels; patients experiencing this symptom may need an increased estrogen dose. If the dose of OC is not changed, the woman may use condoms along with the OC. Reducing the dose of carbamazepine is not correct. Discontinuing the OC immediately is not correct.

14. A nurse receives an order to administer castor oil to a patient. Which action by the nurse is correct? a. Administer the medication at bedtime. b. Chill the medication and mix it with fruit juice. c. Provide teaching about home use of this medication. d. Teach the patient that the effects will occur slowly.

ANS: B Castor oil has an unpleasant taste that can be improved by chilling it and mixing it with fruit juice. The medication acts quickly and should not be given at bedtime. It is only used when prompt evacuation of intestinal contents is needed, as for radiological procedures, so the patient will not be instructed in home use of the medication and should be taught that the effects will be immediate.

3. The nurse is teaching a class about the various chemotherapy drugs. Which of these statements explains why alkylating drugs are also called "cell cycle-nonspecific drugs"? a. They are cytotoxic during a specific cell cycle. b. They are cytotoxic in any phase of the cell cycle. c. They are effective against several types of neoplasms. d. They are more highly differentiated than cell cycle-specific drugs.

ANS: B Cell cycle-nonspecific drugs kill cancer cells during any phase of the growth cycle, whereas cell cycle-specific drugs kill cancer cells during specific phases of the cell growth cycle. The other options are incorrect. DIF: COGNITIVE LEVEL: Understanding (Comprehension) REF: p. 734 TOP: NURSING PROCESS: Evaluation

9. A patient who has been diagnosed with Sjögren's syndrome will be given cevimeline for the treatment of xerostomia. The nurse will monitor for what therapeutic effect? a. Reduction of salivation b. Stimulation of salivation c. Reduction of gastrointestinal peristalsis d. Improvement of fine-motor control

ANS: B Cevimeline is a direct-acting cholinergic drug that is used to stimulate salivation in patients who have xerostomia (dry mouth), one of the manifestations of Sjögren's syndrome. The other options are incorrect. DIF: COGNITIVE LEVEL: Applying (Application) REF: p. 323 TOP: NURSING PROCESS: Implementation

10. In preparation for eye surgery, the nurse monitors for which desired drug effect in a patient who is receiving a cholinergic-blocking eyedrop medication? a. Miosis b. Mydriasis c. Increased intraocular fluid production d. Enhanced tear production

ANS: B Cholinergic-blocking eyedrops cause dilation of the pupil (mydriasis) and paralysis of the ocular lens (cycloplegia), both of which are important for eye surgery. The other options are incorrect. DIF: COGNITIVE LEVEL: Applying (Application) REF: p. 332 TOP: NURSING PROCESS: Evaluation

2. When giving cisplatin (Platinol-AQ), the nurse is aware that the major dose-limiting effect of this drug is which condition? a. Alopecia b. Kidney damage c. Cardiotoxicity d. Stomatitis

ANS: B Cisplatin may cause nephrotoxicity, and the patient's renal function must be monitored closely while on this drug. Ensuring hydration will help to prevent nephrotoxicity. DIF: COGNITIVE LEVEL: Understanding (Comprehension) REF: p. 735 TOP: NURSING PROCESS: Implementation

6. A nurse is explaining to a nursing student how clomiphene [Clomid] works to improve fertility. Which statement by the student indicates a need for further teaching? a. "Clomiphene blocks estrogen receptors to cause increased secretion of gonadotropins." b. "Clomiphene directly stimulates the ovary, causing follicular maturation and ovulation." c. "If follicular maturation is the only result of clomiphene therapy, human chorionic gonadotropin may be given." d. "If the pituitary gland cannot produce LH and FSH, clomiphene will not be effective."

ANS: B Clomiphene blocks receptors for estrogen; it does not directly stimulate the ovary. When estrogen receptors are blocked, the pituitary increases production of the gonadotropins LH and FSH. Clomiphene may produce follicular maturation without ovulation, and hCG will be given to produce ovulation. Without a pituitary capable of producing LH and FSH, clomiphene will not be effective. PTS: 1 DIF: Cognitive Level: Application REF: pp. 770-771

12. A 7-year-old child has a cough, runny nose, congestion, and fever, and the parents ask the nurse to recommend an over-the-counter product. Which response by the nurse is correct? a. "Any product will be effective when combined with vitamin C and zinc." b. "It is best to use single-agent medications to treat individual symptoms." c. "The fever indicates that your child may need an antibiotic; you should call your provider." d. "You should ask your provider to prescribe a combination product that will treat multiple symptoms."

ANS: B Combination medications may provide ingredients that are not needed or may provide ingredients that are either excessive or subtherapeutic. It is best to use single-agent drugs to treat individual symptoms. The efficacy of vitamin C and zinc for treating colds in children has not been established. Fever may accompany viral respiratory infections and not necessarily bacterial infections that need an antibiotic.

8. A patient asks the nurse about the uses of echinacea. Which use will the nurse include in the response? a. Memory enhancement b. Boosting the immune system c. Improving mood d. Promoting relaxation

ANS: B Common uses of echinacea include stimulation of the immune system, antisepsis, treatment of viral infections and influenza-like respiratory tract infections, and promotion of the healing of wounds and chronic ulcerations. The other options are incorrect. DIF: COGNITIVE LEVEL: Understanding (Comprehension) REF: p. 566 TOP: NURSING PROCESS: Implementation

12. The nurse is preparing to give a potassium supplement. Which laboratory test should be checked before the patient receives a dose of potassium? a. Complete blood count b. Serum potassium level c. Serum sodium level d. Liver function studies

ANS: B Contraindications to potassium replacement products include hyperkalemia from any cause. It is important to know the patient's electrolyte levels before beginning electrolyte replacement therapy. Giving potassium supplements to a patient whose serum potassium levels are already high may cause worsening of the hyperkalemia. The other options are incorrect. DIF: COGNITIVE LEVEL: Applying (Application) REF: p. 471 TOP: NURSING PROCESS: Planning

13. A patient with a history of chronic obstructive pulmonary disease (COPD) and type 2 diabetes has been treated for pneumonia for the past week. The patient has been receiving intravenous corticosteroids as well as antibiotics as part of his therapy. At this time, the pneumonia has resolved, but when monitoring the blood glucose levels, the nurse notices that the level is still elevated. What is the best explanation for this elevation? a. The antibiotics may cause an increase in glucose levels. b. The corticosteroids may cause an increase in glucose levels. c. His type 2 diabetes has converted to type 1. d. The hypoxia caused by the COPD causes an increased need for insulin.

ANS: B Corticosteroids can antagonize the hypoglycemic effects of insulin, resulting in elevated blood glucose levels. The other options are incorrect. DIF: COGNITIVE LEVEL: Analyzing (Analysis) REF: p. 515 TOP: NURSING PROCESS: Evaluation

14. A nurse is discussing the use of immunosuppressants for the treatment of inflammatory bowel disease (IBD) with a group of nursing students. Which statement by a student indicates understanding of the teaching? a. "Azathioprine [Imuran] helps induce rapid remission of IBD." b. "Cyclosporine [Sandimmune] can be used to induce remission of IBD." c. "Cyclosporine [Sandimmune] does not have serious adverse effects." d. "Methotrexate is used long term to maintain remission of IBD."

ANS: B Cyclosporine can be given intravenously to induce rapid remission of IBD. Azathioprine has delayed onset of effects up to 6 months and is not used to induce rapid remission. Cyclosporine is a toxic compound that can cause renal impairment, neurotoxicity, and immune suppression. Methotrexate is used to promote short-term remission.

3. A patient will receive oral ciprofloxacin [Cipro] to treat a urinary tract infection. The nurse provides teaching for this patient. Which statement by the patient indicates a need for further teaching? a. "I may have abdominal pain and nausea, but these are usually mild." b. "I should take this medication with food or milk to improve absorption." c. "I should stop taking the medication immediately if I experience heel pain." d. "I will need to use sunscreen every time I go outdoors."

ANS: B Dairy products inhibit the absorption of ciprofloxacin, so they should be avoided. Abdominal pain and nausea and vomiting are common and usually mild. Patients should stop taking the drug if heel pain occurs until tendonitis has been ruled out. Photosensitivity can occur, so sunscreen should be used.

1. The nurse notes in a patient's medication history that the patient has been taking desmopressin (DDAVP). Based on this finding, the nurse interprets that the patient has which disorder? a. Diabetes mellitus b. Diabetes insipidus c. Adrenocortical insufficiency d. Carcinoid tumor

ANS: B Desmopressin is used to prevent or control polydipsia (excessive thirst), polyuria, and dehydration in patients with diabetes insipidus. The symptoms are caused by a deficiency of endogenous antidiuretic hormone. The other options are incorrect. DIF: COGNITIVE LEVEL: Understanding (Comprehension) REF: p. 485 TOP: NURSING PROCESS: Assessment

12. A patient asks a nurse why he cannot use digoxin [Lanoxin] for his heart failure, because both of his parents used it for HF. The nurse will explain that digoxin is not the first-line therapy for which reason? a. It causes tachycardia and increases the cardiac workload. b. It does not correct the underlying pathology of heart failure. c. It has a wide therapeutic range that makes dosing difficult. d. It may actually shorten the patient's life expectancy.

ANS: B Digoxin improves cardiac output, alters electrical effects, and helps to decrease sympathetic outflow from the central nervous system (CNS) through its neurohormonal effects; however, it does not alter the underlying pathology of heart failure or prevent cardiac remodeling. Digoxin causes bradycardia and increases the cardiac workload by increasing contractility. It has a narrow therapeutic range and many adverse effects. Digoxin does not improve life expectancy; in women it may actually shorten life expectancy.

2. A patient with atrial fibrillation is taking verapamil [Calan]. The patient has read about the drug on the Internet and wants to know why a drug that affects the rate of ventricular contraction is used to treat an abnormal atrial contraction. What will the nurse tell the patient? a. "Drugs that treat ventricular dysrhythmias help to restore normal sinus rhythm." b. "Atrial dysrhythmias can have life-threatening effects on ventricular function." c. "Treating ventricular dysrhythmias helps prevent the likelihood of stroke." d. "When ventricular contraction slows, atrial contraction is also slowed."

ANS: B Dysrhythmic activity in the atria does not significantly reduce cardiac output but can be dangerous when dysrhythmic impulses cross the AV node, causing ventricular dysrhythmias, which can be life threatening. Treating ventricular dysrhythmia helps improve ventricular pumping. These drugs do not restore normal sinus rhythm. To prevent stroke, an anticoagulant, such as warfarin, is used. Slowing ventricular contraction does not affect the rate of atrial contraction. Restoring normal sinus rhythm requires cardioversion, short-term treatment with amiodarone or sotalol, or RF ablation of the dysrhythmia source.

A patient with myasthenia gravis who is taking a cholinesterase inhibitor is being admitted to the intensive care unit and is on mechanical ventilation. The prescriber has ordered a challenge dose of edrophonium to distinguish between a myasthenic crisis and a medication overdose. The nurse will expect to do what? a.Administer neostigmine if muscle strength decreases. b.Be prepared to administer atropine and oxygen if muscle weakness increases. c.Give a second dose of edrophonium if no improvement is seen. d.Give pralidoxime [Protopam] if cholinergic symptoms worsen.

ANS: B Edrophonium is an ultra-short-acting cholinesterase inhibitor used to distinguish between a myasthenic crisis and a cholinesterase inhibitor overdose, because weakness or paralysis can occur with both. If the symptoms are intensified and muscle weakness increases, the crisis is cholinergic, meaning that the patient is having toxic effects from the cholinesterase inhibitor and needs atropine as an antidote. Neostigmine would be given if the patient shows improved muscle strength, because that would indicate that the patient's symptoms are the result of a myasthenic crisis. A second dose of edrophonium would only worsen the symptoms. Pralidoxime is used to treat poisoning caused by irreversible cholinesterase inhibitors.

6. A patient has an order for the monoclonal antibody adalimumab (Humira). The nurse notes that the patient does not have a history of cancer. What is another possible reason for administering this drug? a. Severe anemia b. Rheumatoid arthritis c. Thrombocytopenia d. Osteoporosis

ANS: B Monoclonal antibodies are used for the treatment of cancer, rheumatoid arthritis and other inflammatory diseases, multiple sclerosis, and organ transplantation. DIF: COGNITIVE LEVEL: Understanding (Comprehension) REF: p. 757 TOP: NURSING PROCESS: Assessment

10. A patient, diagnosed with lymphoma, has an allergy to one of the proposed chemotherapy drugs. The tumor has not responded to other types of treatment. The nurse expects the oncologist to follow which course of treatment? a. The physician will choose another drug to use. b. The chemotherapy will be given along with supportive measures to treat a possible allergic reaction. c. The patient will receive reduced doses of chemotherapy for a longer period of time. d. The chemotherapy cannot be given because of the patient's allergy.

ANS: B Even if a patient has a known allergic reaction to a given antineoplastic medication, the urgency of treating the patient's cancer may still necessitate administering the medication and then treating any allergic symptoms with supportive medications, such as antihistamines, corticosteroids, and acetaminophen. DIF: COGNITIVE LEVEL: Applying (Application) REF: p. 717 TOP: NURSING PROCESS: Planning

10. A patient with a cough has been advised to use guaifenesin. The patient asks the nurse to explain the purpose of the drug. The nurse will explain that guaifenesin: a. dries secretions to help suppress coughing so patients can rest. b. helps stimulate the flow of secretions to increase cough productivity. c. helps to relieve chest pain associated with a cough. d. stimulates the body's natural immune responses.

ANS: B Expectorants stimulate the flow of respiratory tract secretions to improve cough productivity. Guaifenesin does not dry secretions, because it does not have anticholinergic effects. Guaifenesin does not alleviate pain associated with cough. Guaifenesin does not stimulate immune responses.

3. A patient is concerned about the body changes that have resulted from long-term prednisone therapy for the treatment of asthma. Which effect of this drug therapy would be present to support the nursing diagnosis of disturbed body image? a. Weight loss b. Weight gain c. Pale skin color d. Hair loss

ANS: B Facial erythema, weight gain, hirsutism, and "moon face" (characteristic of Cushing's syndrome) are possible body changes that may occur with long-term prednisone therapy. DIF: COGNITIVE LEVEL: Analyzing (Analysis) REF: p. 527 TOP: NURSING PROCESS: Nursing Diagnosis

4. A patient is concerned about the adverse effects of the fibric acid derivative she is taking to lower her cholesterol level. Which is an adverse effect of this class of medication? a. Constipation b. Diarrhea c. Joint pain d. Dry mouth

ANS: B Fibric acid derivatives may cause nausea, vomiting, diarrhea, drowsiness, and dizziness. Other effects are listed in Table 27-8. The other options are not adverse effects of fibric acid derivatives. DIF: COGNITIVE LEVEL: Understanding (Comprehension) REF: p. 445 TOP: NURSING PROCESS: Implementation

3. A patient is receiving finasteride (Proscar) for treatment of benign prostatic hyperplasia. The nurse will tell him that a possible effect of this medication is: a. alopecia. b. increased hair growth. c. urinary retention. d. increased prostate size.

ANS: B Finasteride is given to reduce prostate size in men with benign prostatic hyperplasia. It has been noted that men taking this medication experience increased hair growth. The other options are incorrect. DIF: COGNITIVE LEVEL: Understanding (Comprehension) REF: p. 557 TOP: NURSING PROCESS: Planning

3. A woman complains of burning on urination and increased frequency. The patient has a history of frequent urinary tract infections (UTIs) and is going out of town in 2 days. To treat the infection quickly, the nurse would expect the healthcare provider to order: a. aztreonam [Azactam]. b. fosfomycin [Monurol]. c. trimethoprim/sulfamethoxazole [Bactrim]. d. vancomycin [Vancocin].

ANS: B Fosfomycin has been approved for single-dose therapy of UTIs in women. Vancomycin and aztreonam are not indicated for UTIs. Bactrim is indicated for UTIs, but administration of a single dose is not therapeutic.

8. A patient with multiple sclerosis will be starting therapy with an immunosuppressant drug. The nurse expects that which drug will be used? a. Azathioprine (Imuran) b. Glatiramer acetate (Copaxone) c. Daclizumab (Zenapax) d. Sirolimus (Rapamune)

ANS: B Glatiramer acetate and fingolimod are the only immunosuppressants currently indicated for reduction of the frequency of relapses (exacerbations) in a type of multiple sclerosis known as relapsing-remitting multiple sclerosis. DIF: COGNITIVE LEVEL: Understanding (Comprehension) REF: p. 771 TOP: NURSING PROCESS: Planning

4. A patient is receiving high doses of methotrexate and is experiencing severe bone marrow suppression. The nurse expects which intervention to be ordered with this drug to reduce this problem? a. A transfusion of whole blood b. Leucovorin rescue c. Therapy with filgrastim (Neupogen) d. Administration of allopurinol (Zyloprim)

ANS: B High-dose methotrexate is associated with bone marrow suppression, and it is always given in conjunction with the rescue drug leucovorin, which is an antidote for folic acid antagonists. Basically, leucovorin rescues the healthy cells from methotrexate. The other options are incorrect. DIF: COGNITIVE LEVEL: Understanding (Comprehension) REF: p. 719 TOP: NURSING PROCESS: Implementation

11. The nurse notes in the patient's medication orders that the patient will be taking ibutilide (Corvert). Based on this finding, the nurse interprets that the patient has which disorder? a. Ventricular ectopy b. Atrial fibrillation c. Supraventricular tachycardia d. Bradycardia

ANS: B Ibutilide (Corvert) is one of two class III antidysrhythmic drugs available for rapid conversion of these atrial fibrillations and atrial flutters into normal sinus rhythm. DIF: COGNITIVE LEVEL: Understanding (Comprehension) REF: p. 404 TOP: NURSING PROCESS: Planning

7. A 73-year-old male patient is in the clinic for a yearly physical and is asking for a prescription for sildenafil (Viagra). He has listed on his health history that he is taking a nitrate for angina. The nurse is aware that which problem may occur if sildenafil is taken with a nitrate? a. Significant increase in pulse rate b. Significant decrease in blood pressure c. Increased risk of bleeding d. Reduced effectiveness of the sildenafil

ANS: B In patients with pre-existing cardiovascular disease, especially those on nitrates, erectile dysfunction drugs such as sildenafil lower blood pressure substantially, potentially leading to more serious adverse events. The other options are incorrect. DIF: COGNITIVE LEVEL: Applying (Application) REF: p. 558 TOP: NURSING PROCESS: Assessment

3. A patient who is having her sixth child has gone beyond term, and her labor is being induced with oxytocin [Pitocin]. The patient is having increased frequency, duration, and intensity of contractions. The nurse will interrupt the oxytocin infusion if what occurs? a. Contractions occur every 2 to 3 minutes. b. Individual contractions last 2 minutes. c. Mild to moderate pain occurs with uterine contractions. d. Resting intrauterine pressure is greater than 10 to 15 mm Hg.

ANS: B Induction of labor in patients of high parity (five or more pregnancies) carries a high risk of uterine rupture, and oxytocin should be used with great caution. Oxytocin infusions should be interrupted for contractions that last longer than 1 minute. Contractions that occur more often than every 2 to 3 minutes are a concern. Mild to moderate pain is normal with contractions. A resting intrauterine pressure greater than 15 to 20 mm Hg is a concern. PTS: 1 DIF: Cognitive Level: Application REF: p. 783

4. A patient asks the nurse what type of medications would be most effective for treating seasonal and perennial rhinitis. Which response by the nurse is correct? a. Pseudoephedrine [Sudafed] b. Fluticasone propionate [Fluticasone] c. Loratadine [Claritin] d. Intranasal cromolyn sodium [Atrovent]

ANS: B Intranasal glucocorticoids, such as fluticasone propionate, are the most effective drugs for prevention and treatment, because they prevent or suppress all the major symptoms of allergic rhinitis (congestion, rhinorrhea, sneezing, nasal itching, and erythema). Pseudoephedrine is an oral sympathomimetic used to reduce nasal congestion associated with allergic rhinitis. It has no effect on other symptoms. Loratadine, an oral antihistamine, reduces sneezing, rhinorrhea, and nasal itching only and is less effective than intranasal glucocorticoids. Intranasal cromolyn sodium is moderately effective in the treatment of allergic rhinitis, but the benefits are much less than those of intranasal glucocorticoids.

8. What is ipratropium bromide [Atrovent]? a. A cholinergic agent used for perennial rhinitis b. An anticholinergic used for allergic rhinitis and colds c. A medication that is used only in patients with asthma d. A drug that is inappropriate for use in patients with allergic rhinitis

ANS: B Ipratropium bromide is an anticholinergic that is indicated for allergic rhinitis, asthma, and the common cold. The drug reduces rhinorrhea. Ipratropium bromide is an anticholinergic. In addition to asthma, ipratropium bromide can be used for allergic rhinitis and the common cold. Ipratropium bromide can be used for allergic rhinitis.

A patient arrives in the emergency department after becoming dehydrated. Based on the patient's history, the provider determines that isotonic dehydration has occurred. Which solution will the nurse expect to infuse to treat this patient? a. 0.45% sodium chloride in sterile water b. 0.9% sodium chloride in sterile water c. 3% sodium chloride in sterile water d. 5% dextrose solution

ANS: B Isotonic dehydration should be treated with an isotonic solution of 0.9% NaCl in sterile water. A 0.45% solution or 5% dextrose is used to treat hypertonic dehydration. A 3% NaCl solution is used to treat hypotonic dehydration.

10. The nurse is reviewing the mechanism of action of cholinergic drugs. The undesired effects of cholinergic drugs come from the stimulation of which receptors? a. Muscarinic b. Nicotinic c. Cholinergic d. Ganglionic

ANS: B Many of the undesirable adverse effects are due to nicotinic receptor stimulation. The desired effects come from muscarinic receptor stimulation. The other options are incorrect. DIF: COGNITIVE LEVEL: Understanding (Comprehension) REF: p. 323 TOP: NURSING PROCESS: Implementation

8. A patient receiving menotropins [Repronex] for infertility comes to the clinic for evaluation on the ninth day of treatment. Her serum estrogen level is 200 pg/mL per maturing follicle. An ultrasound reveals that follicles have enlarged to 22 mm. The patient complains of left lower abdominal pain. What will the nurse do? a. Administer human chorionic gonadotropin, because the ovary has ripened. b. Assess for abdominal fullness and distension and auscultate lung sounds. c. Inform the patient that the pain is associated with ovulation. d. Recommend ibuprofen for pain and administer the next dose of menotropins.

ANS: B Menotropins can cause ovarian hyperstimulation syndrome, which can cause sudden enlargement of the ovaries. When it occurs rapidly and is accompanied by ascites, pleural effusion, and pain, the patient should be hospitalized and the menotropins should be withdrawn. Although the follicular size and estrogen level indicate a ripened ovary, the first priority is to assess for ovarian hyperstimulation syndrome, because this patient reports pain. Ovulation will not occur until hCG is given, so this pain is not associated with ovulation. Until ovarian hyperstimulation syndrome has been ruled out, it is not appropriate to administer the next dose of menotropins. PTS: 1 DIF: Cognitive Level: Application REF: pp. 771-772

15. A patient has been diagnosed with metabolic syndrome and is started on the biguanide metformin (Glucophage). The nurse knows that the purpose of the metformin, in this situation, is which of these? a. To increase the pancreatic secretion of insulin b. To decrease insulin resistance c. To increase blood glucose levels d. To decrease the pancreatic secretion of insulin

ANS: B Metformin decreases glucose production by the liver; decreases intestinal absorption of glucose; and improves insulin receptor sensitivity in the liver, skeletal muscle, and adipose tissue, resulting in decreased insulin resistance. The other options are incorrect. DIF: COGNITIVE LEVEL: Understanding (Comprehension) REF: p. 508 TOP: NURSING PROCESS: Planning

17. The nurse is preparing to administer methylnaltrexone (Relistor), a peripherally acting opioid antagonist. This drug is appropriate for which patient? a. A patient with diarrhea b. A terminally ill patient who has opioid-induced constipation c. A patient who is scheduled for a colonoscopy d. A patient who will be having colon surgery in the morning

ANS: B Methylnaltrexone is approved only for terminally ill (hospice) patients who have opioid-induced constipation. The other options are incorrect. DIF: COGNITIVE LEVEL: Applying (Application) REF: p. 819 TOP: NURSING PROCESS: Planning

11. A child who has perennial allergic rhinitis has been using an intranasal glucocorticoid. The provider has ordered montelukast [Singulair] to replace the glucocorticoid, because the child has frequent nosebleeds. When teaching this child's parents about montelukast, the nurse will include which statement? a. "Montelukast is also effective for treating infectious rhinitis." b. "Montelukast may cause behavior changes in your child." c. "Montelukast will treat both congestion and rhinitis." d. "Montelukast works best when combined with a topical decongestant."

ANS: B Montelukast can cause rare but serious neuropsychiatric effects in patients, and parents should be warned of this possibility. It is not useful for treating infectious rhinitis. It does not affect congestion. It is not necessary to add a topical decongestant when using this drug for allergic rhinitis.

7. A patient who has been on long-term corticosteroid therapy has had surgery to correct an abdominal hernia. The nurse keeps in mind that which potential effect of this medication may have the most impact on the patient's recovery? a. Hypotension b. Delayed wound healing c. Muscle weakness d. Osteoporosis

ANS: B Muscle weakness and osteoporosis may also result from long-term therapy, but delayed wound healing would have the most impact on the patient's recovery from abdominal surgery at this time. Hypertension, not hypotension, may result from long-term corticosteroid therapy. DIF: COGNITIVE LEVEL: Applying (Application) REF: p. 526 TOP: NURSING PROCESS: Planning

9. The nurse will monitor for myopathy (muscle pain) when a patient is taking which class of antilipemic drugs? a. Niacin b. HMG-CoA reductase inhibitors c. Fibric acid derivatives d. Bile acid sequestrants

ANS: B Myopathy (muscle pain) is a clinically important adverse effect that may occur with HMG-CoA reductase inhibitors. It may progress to a serious condition known as rhabdomyolysis. Patients receiving statin therapy need to be advised to report any unexplained muscular pain or discomfort to their health care providers immediately. The other drugs and drug classes do not cause muscle pain or myopathy. DIF: COGNITIVE LEVEL: Applying (Application) REF: p. 441 TOP: NURSING PROCESS: Assessment

A patient who has myasthenia gravis will be taking neostigmine [Prostigmin]. What will the nurse emphasize when teaching this patient about the medication? a."Stop taking the drug if you have diarrhea." b."Take a supplementary dose before exercise." c."Use atropine if you have excessive salivation." d."Withhold the dose if ptosis occurs."

ANS: B Neostigmine doses must be adjusted continually, and patients usually need supplemental doses before exertion; therefore, patients must be taught how to modify doses as needed. Increased gastrointestinal (GI) secretions can cause loose stools; this is a known adverse effect that does not warrant stopping the drug. Atropine will help with excessive salivation but should not be used routinely, because it can mask the early signs of anticholinesterase overdose. Ptosis is one of the symptoms of myasthenia gravis and is an indication for taking neostigmine.

10. The nurse notes in a patient's medical record that nesiritide (Natrecor) has been ordered. Based on this order, the nurse interprets that the patient has which disorder? a. Atrial fibrillation b. Acutely decompensated heart failure with dyspnea at rest c. Systolic heart failure d. Chronic, stable heart failure

ANS: B Nesiritide is indicated for the treatment of acutely decompensated heart failure with dyspnea at rest. Digoxin is used for the treatment of atrial fibrillation and systolic heart failure. Nesiritide is not indicated for chronic, stable heart failure. DIF: COGNITIVE LEVEL: Understanding (Comprehension) REF: p. 380 TOP: NURSING PROCESS: Assessment

14. A nursing student asks a nurse about the recommendations for vitamin supplements to prevent cancer. Which statement by the student indicates a need for further teaching? a. "Foods rich in antioxidant vitamins may confer protection by other means." b. "Observational studies provide clear proof that some antioxidants reduce the risk of cancer." c. "Randomized controlled studies have not demonstrated cancer prevention benefits." d. "Some vitamins have an increased cancer risk associated with too low and too high amounts."

ANS: B No clear evidence indicates that antioxidant vitamins lead to a reduced risk of cancer. Some observational studies show a link between the consumption of foods high in antioxidants and lower cancer rates, but this link may be related to other components of these diets, such as the high fiber also found in such foods. Some vitamins are linked to higher cancer rates when consumed in both too-low and too-high amounts. PTS: 1 DIF: Cognitive Level: Analysis REF: p. 990

17. The nurse is administering oxytocin (Pitocin). Which situation is an indication for the use of oxytocin? a. Decreased fetal heart rate and movements b. Stimulation of contractions during labor c. Cervical ripening near term in pregnant patients d. To reverse premature onset of labor

ANS: B Oxytocin is used to induce labor at or near full-term gestation and to enhance labor when uterine contractions are weak and ineffective. DIF: COGNITIVE LEVEL: Applying (Application) REF: p. 545 TOP: NURSING PROCESS: Implementation

10. A patient will undergo a colonoscopy, and the provider has ordered sodium phosphate as a bowel cleanser before the procedure. The nurse reviews the patient's chart and notes that the patient's creatinine clearance and blood urea nitrogen are both elevated. What will the nurse do? a. Reduce the amount of fluid given with the laxative to prevent fluid retention. b. Request an order to give polyethylene glycol and electrolytes (PEG-ELS) instead. c. Suggest that the patient reduce the dietary sodium intake. d. Suggest using a suppository laxative instead.

ANS: B PEG-ELS solutions provide an isosmotic solution and do not cause dehydration or electrolyte imbalance. They are safe to use in patients with renal impairment or cardiovascular disease. This patient's laboratory values suggest renal impairment. Sodium phosphate products can cause kidney damage; giving them with less fluid only increases this possibility. Reducing the dietary intake of sodium is not recommended. Suppositories are not effective bowel cleansing agents.

9. The nurse is preparing to transfuse a patient with a unit of packed red blood cells (PRBCs). Which patient would be best treated with this transfusion? a. A patient with a coagulation disorder b. A patient with severe anemia c. A patient who has lost a massive amount of blood after an accident d. A patient who has a clotting-factor deficiency

ANS: B PRBCs are given to increase the oxygen-carrying capacity in patients with anemia, in patients with substantial hemoglobin deficits, and in patients who have lost up to 25% of their total blood volume. Patients with coagulation disorder or clotting-factor deficiency would receive fresh frozen plasma; a patient who has lost a massive amount of blood would receive whole blood. DIF: COGNITIVE LEVEL: Analyzing (Analysis) REF: p. 469 TOP: NURSING PROCESS: Implementation

17. An 88-year-old patient with heart failure has progressed to Stage D and is hospitalized for the third time in a month. The nurse will expect to discuss which topic with the patient's family? a. Antidysrhythmic medications b. End-of-life care c. Heart transplantation d. Implantable mechanical assist devices

ANS: B Patients in Stage D heart failure have advanced structural heart disease. For eligible patients, the best long-term solution is heart transplantation, but this patient probably is not eligible, given his advanced age. End-of-life care should be discussed. Antidysrhythmic medications are not indicated and may make symptoms worse. An implantable mechanical assist device is used in patients awaiting heart transplantation.

6. A 19-year-old student was diagnosed with hypothyroidism and has started thyroid replacement therapy with levothyroxine (Synthroid). After 1 week, she called the clinic to report that she does not feel better. Which response from the nurse is correct? a. "It will probably require surgery for a cure to happen." b. "The full therapeutic effects may not occur for 3 to 4 weeks." c. "Is it possible that you did not take your medication as instructed?" d. "Let's review your diet; it may be causing absorption problems."

ANS: B Patients need to understand that it may take up to 3 to 4 weeks to see the full therapeutic effects of thyroid drugs. The other options are incorrect. DIF: COGNITIVE LEVEL: Applying (Application) REF: p. 497 TOP: NURSING PROCESS: Evaluation

3. Which nursing diagnosis is appropriate for a patient who has started aminoglycoside therapy? a. Constipation b. Risk for injury (renal damage) c. Disturbed body image related to gynecomastia d. Imbalanced nutrition, less than body requirements, related to nausea

ANS: B Patients on aminoglycoside therapy have an increased risk for injury caused by nephrotoxicity. The other options are incorrect. DIF: COGNITIVE LEVEL: Applying (Application) REF: p. 635 TOP: NURSING PROCESS: Nursing Diagnosis

4. A patient is taking fludrocortisone (Florinef) for Addison's disease, and his wife is concerned about all of the problems that may occur with this therapy. When teaching them about therapy with this drug, the nurse will include which information? a. It may cause severe postural hypotension. b. It needs to be taken with food or milk to minimize gastrointestinal upset. c. The medication needs to be stopped immediately if nausea or vomiting occurs. d. Weight gain of 5 pounds or more in 1 week is an expected adverse effect.

ANS: B Patients receiving fludrocortisone need to take it with food or milk to minimize gastrointestinal upset; weight gain of 5 pounds or more in 1 week needs to be reported to the physician; abrupt withdrawal is not recommended because it may precipitate an adrenal crisis. Adverse effects are related to the fluid retention and may include heart failure and hypertension. DIF: COGNITIVE LEVEL: Applying (Application) REF: p. 531 TOP: NURSING PROCESS: Implementation

A patient is taking a combination oral contraceptive (OC) and tells the nurse that she is planning to undergo knee replacement surgery in 2 months. What will the nurse recommend for this patient? a. The patient should ask her provider about an OC with less progestin. b. The patient should discuss an alternate method of birth control prior to surgery. c. The patient should request an OC containing less estrogen after surgery. d. The patient should take the OC at bedtime after her surgery to reduce side effects.

ANS: B Patients taking an OC who undergo surgery in which immobilization increases the risk of postoperative thrombosis should stop taking the OC at least 4 weeks prior to surgery. The patient should discuss an alternate method of birth control with her provider. Estrogen, not progestin, increases the risk of thrombosis. The estrogen-containing OC should be stopped 4 weeks prior to surgery. Taking the OC at bedtime does not decrease the risk.

5. When administering cyclosporine, the nurse notes that allopurinol is also ordered for the patient. What is a potential result of this drug interaction? a. Reduced adverse effects of the cyclosporine b. Increased levels of cyclosporine and toxicity c. Reduced uric acid levels d. Reduced nephrotoxic effects of cyclosporine

ANS: B The allopurinol may cause increased levels of cyclosporine, and toxicity may result. The other options are incorrect. DIF: COGNITIVE LEVEL: Applying (Application) REF: p. 774 TOP: NURSING PROCESS: Planning

1. A nurse provides teaching to a patient with allergic rhinitis who will begin using an intranasal glucocorticoid. Which statement by the patient indicates understanding of the teaching? a. "If the glucocorticoid causes burning or itching, I should use it every other day." b. "I should use a decongestant if necessary before using the glucocorticoid." c. "I should use the glucocorticoid whenever I have symptoms." d. "I will probably develop systemic effects from the topical glucocorticoid."

ANS: B Patients using intranasal glucocorticoids should be taught to use a decongestant to unblock nasal passages if needed before using the medication. Intranasal glucocorticoids should be used regularly on a daily basis to achieve optimal effects and not every other day or as needed. Systemic effects from intranasal glucocorticoids can occur but are not likely.

6. The nurse is reviewing infection-prevention measures with a patient who is receiving antineoplastic drug therapy. Which statement by the patient indicates the need for further teaching? a. "I will avoid those who have recently had a vaccination." b. "I will eat only fresh fruits and vegetables." c. "I will report a sore throat, cough, or low-grade temperature." d. "It is important for both my family and me to practice good hand washing."

ANS: B Patients who are neutropenic and susceptible to infections need to adhere to a low-microbe diet by washing fresh fruits and vegetables and making sure foods are well cooked. The other options are correct. DIF: COGNITIVE LEVEL: Applying (Application) REF: p. 729 TOP: NURSING PROCESS: Implementation

1. When monitoring a patient's response to interferon therapy, the nurse notes that the major dose-limiting factor for interferon therapy is which condition? a. Diarrhea b. Fatigue c. Anxiety d. Nausea and vomiting

ANS: B Patients who receive interferon therapy may experience flu-like symptoms: fever, chills, headache, malaise, myalgia, and fatigue. Fatigue is the major dose-limiting factor for interferon therapy. Patients taking high dosages become so exhausted that they are commonly confined to bed. DIF: COGNITIVE LEVEL: Understanding (Comprehension) REF: p. 754 TOP: NURSING PROCESS: Evaluation

A patient with congestive heart failure is admitted to the hospital. During the admission assessment, the nurse learns that the patient is taking a thiazide diuretic. The nurse notes that the admission electrolyte levels include a sodium level of 142 mEq/L, a chloride level of 95 mEq/L, and a potassium level of 3 mEq/L. The prescriber has ordered digoxin to be given immediately. What will the nurse do initially? a. Give the digoxin and maintain close cardiac monitoring. b. Hold the digoxin and report the laboratory values to the provider. c. Hold the thiazide diuretic and give the digoxin. d. Request an order for an electrocardiogram (ECG).

ANS: B Potassium depletion is common with thiazide diuretics, and hypokalemia is especially dangerous for patients receiving digoxin, because the drug can precipitate a fatal dysrhythmia and digoxin toxicity. The provider should be notified of the serum potassium level so that it can be corrected before the digoxin is administered. Giving the digoxin could produce a fatal adverse effect, so this is not an appropriate course of action. Holding the thiazide diuretic will not correct the potassium deficiency. An ECG is not the initial priority.

5. A patient who is taking propylthiouracil (PTU) for hyperthyroidism wants to know how this medicine works. Which explanation by the nurse is accurate? a. It blocks the action of thyroid hormone. b. It slows down the formation of thyroid hormone. c. It destroys overactive cells in the thyroid gland. d. It inactivates already existing thyroid hormone in the bloodstream.

ANS: B Propylthiouracil impedes the formation of thyroid hormone but has no effect on already existing thyroid hormone. The other options are incorrect. DIF: COGNITIVE LEVEL: Understanding (Comprehension) REF: p. 494 TOP: NURSING PROCESS: Implementation

4. A patient will be discharged on quinidine sulfate (Quinidex) extended-release tablets for the treatment of ventricular ectopy. The nurse will include which information in the teaching plan? a. The medication should be stopped once the cardiac symptoms subside. b. Signs of cinchonism, such as tinnitus, loss of hearing, or slight blurring of vision, may occur. c. It is important to use sunscreen products when outside because of increased photosensitivity. d. If any tablet or capsule is visible in the stool, contact the prescriber immediately.

ANS: B Quinidine, a cinchona alkaloid, may cause the symptoms of cinchonism, including tinnitus, loss of hearing, slight blurring of vision, and gastrointestinal upset. The medication will need to be continued even after symptoms subside, or the symptoms may return. Tablets or capsules that are visible in the stool are actually the wax matrices that contained the drug; the medication is extracted while in the intestines. Photosensitivity occurs with class III drugs, not with quinidine (class Ia). DIF: COGNITIVE LEVEL: Applying (Application) REF: p. 401 TOP: NURSING PROCESS: Implementation

8. The nurse is caring for a patient who is receiving vancomycin [Vancocin]. The nurse notes that the patient is experiencing flushing, rash, pruritus, and urticaria. The patient's heart rate is 120 beats per minute, and the blood pressure is 92/57 mm Hg. The nurse understands that these findings are consistent with: a. allergic reaction. b. red man syndrome. c. rhabdomyolysis. d. Stevens-Johnson syndrome.

ANS: B Rapid infusion of vancomycin can cause flushing, rash, pruritus, urticaria, tachycardia, and hypotension, a collection of symptoms known as red man syndrome. Rhabdomyolysis is not associated with the administration of vancomycin. The patient's symptoms may seem to indicate an allergic reaction, but this is specifically red man syndrome. The symptoms are not those of Stevens-Johnson syndrome, which manifests as blisters or sores (or both) on the lips and mucous membranes after exposure to the sun.

12. The nurse is preparing to administer insulin intravenously. Which statement about the administration of intravenous insulin is true? a. Insulin is never given intravenously. b. Only regular insulin can be administered intravenously. c. Insulin aspart or insulin lispro can be administered intravenously, but there must be a 50% dose reduction. d. Any form of insulin can be administered intravenously at the same dose as that is ordered for subcutaneous administration.

ANS: B Regular insulin is the usual insulin product to be dosed via intravenous bolus, intravenous infusion, or even intramuscularly. These routes, especially the intravenous infusion route, are often used in cases of diabetic ketoacidosis, or coma associated with uncontrolled type 1 diabetes. DIF: COGNITIVE LEVEL: Applying (Application) REF: p. 516 TOP: NURSING PROCESS: Implementation

17. The nurse is reviewing a patient's medication list and notes that sitagliptin (Januvia) is ordered. The nurse will question an additional order for which drug or drug class? a. Glitazone b. Insulin c. Metformin (Glucophage) d. Sulfonylurea

ANS: B Sitagliptin is indicated for management of type 2 diabetes either as monotherapy or in combination with metformin, a sulfonylurea, or a glitazone, but not with insulin. DIF: COGNITIVE LEVEL: Applying (Application) REF: p. 512 TOP: NURSING PROCESS: Implementation

8. During a follow-up visit, a patient who has been on estrogen therapy admits that she has continued to smoke cigarettes. The nurse will remind the patient that smoking while on estrogen may lead to increased: a. incidence of nausea. b. risk for thrombosis. c. levels of triglycerides. d. tendency to bleed during menstruation.

ANS: B Smoking should be avoided during estrogen therapy because it adds to the risk for thrombosis formation. The other options are incorrect. DIF: COGNITIVE LEVEL: Applying (Application) REF: p. 537 TOP: NURSING PROCESS: Planning

9. A patient with chronic congestive heart failure has repeated hospitalizations in spite of ongoing treatment with hydrochlorothiazide [HydroDIURIL] and digoxin. The prescriber has ordered spironolactone [Aldactone] to be added to this patient's drug regimen, and the nurse provides education about this medication. Which statement by the patient indicates understanding of the teaching? a. "I can expect improvement within a few hours after taking this drug." b. "I need to stop taking potassium supplements." c. "I should use salt substitutes to prevent toxic side effects." d. "I should watch closely for dehydration."

ANS: B Spironolactone is a potassium-sparing diuretic used to counter the potassium-wasting effects of hydrochlorothiazides. Patients taking potassium supplements are at risk for hyperkalemia when taking this medication, so they should be advised to stop the supplements. Spironolactone takes up to 48 hours to have effects. Salt substitutes contain high levels of potassium and are contraindicated. Spironolactone is a weak diuretic, so the risk of dehydration is not increased.

13. The nurse is providing education to a patient with ulcerative colitis who is being treated with sulfasalazine [Azulfidine]. What statement by the patient best demonstrates understanding of the action of sulfasalazine? a. "It treats the infection that triggers the condition." b. "It reduces the inflammation." c. "It enhances the immune response." d. "It increases the reabsorption of fluid."

ANS: B Sulfasalazine reduces the inflammation seen with ulcerative colitis; this statement indicates understanding. Although similar to sulfonamides, sulfasalazine is not used to treat infections; further teaching is needed. Sulfasalazine does not enhance the immune response or increase the reabsorption of fluid; further teaching is needed.

10. When teaching a patient who is beginning antilipemic therapy about possible drug-food interactions, the nurse will discuss which food? a. Oatmeal b. Grapefruit juice c. Licorice d. Dairy products

ANS: B Taking HMG-CoA reductase inhibitors with grapefruit juice may cause complications. Components in grapefruit juice inactivate CYP3A4 in both the liver and intestines. This enzyme plays a key role in statin metabolism. The presence of grapefruit juice in the body may therefore result in sustained levels of unmetabolized statin drug, which increases the risk for major drug toxicity, possibly leading to rhabdomyolysis. The other foods do not interact with these drugs. DIF: COGNITIVE LEVEL: Understanding (Comprehension) REF: p. 441 TOP: NURSING PROCESS: Implementation

2. A patient is taking a xanthine derivative as part of treatment for chronic obstructive pulmonary disease. The nurse will monitor for which adverse effects associated with the use of xanthine derivatives? a. Diarrhea b. Palpitations c. Bradycardia d. Drowsiness

ANS: B The common adverse effects of the xanthine derivatives include nausea, vomiting, and anorexia. In addition, gastroesophageal reflux has been observed to occur during sleep in patients taking these drugs. Cardiac adverse effects include sinus tachycardia, extrasystole, palpitations, and ventricular dysrhythmias. Transient increased urination and hyperglycemia are other possible adverse effects. DIF: COGNITIVE LEVEL: Remembering (Knowledge) REF: p. 585 TOP: NURSING PROCESS: Evaluation

17. The nurse is caring for a pregnant patient who is in labor. The woman reports having had mild preeclampsia with a previous pregnancy. The nurse notes that the woman has a blood pressure of 168/102 mm Hg. The nurse will contact the provider to request an order for which drug? a. Angiotensin-converting enzyme (ACE) inhibitor b. Hydralazine (Apresoline) c. Magnesium sulfate d. Sodium nitroprusside

ANS: B The drug of choice for lowering blood pressure in a patient with severe preeclampsia is hydralazine. Sodium nitroprusside is not indicated. Magnesium sulfate is given as prophylaxis against seizures but does not treat hypertension. ACE inhibitors are contraindicated because of their potential for fetal harm.

11. A patient reports taking an oral bisacodyl laxative [Dulcolax] for several years. The provider has suggested discontinuing the laxative, but the patient is unsure how to do this. The nurse will tell the patient to: a. stop taking the oral laxative and use a suppository until normal motility resumes. b. stop taking the laxative immediately and expect no stool for several days. c. switch to a bulk-forming laxative, such as methylcellulose [Metamucil]. d. withdraw from the laxative slowly to avoid a rebound constipation effect.

ANS: B The first step in breaking the laxative habit is abrupt cessation of laxative use. Bowel movements will be absent for several days after laxative withdrawal. Using a suppository, a bulk-forming laxative, or tapering the laxative only prolongs the habit and prevents normal function from returning.

8. A patient who has recently started therapy on a statin drug asks the nurse how long it will take until he sees an effect on his serum cholesterol. Which statement would be the nurse's best response? a. "Blood levels return to normal within a week of beginning therapy." b. "It takes 6 to 8 weeks to see a change in cholesterol levels." c. "It takes at least 6 months to see a change in cholesterol levels." d. "You will need to take this medication for almost a year to see significant results."

ANS: B The maximum extent to which lipid levels are lowered may not occur until 6 to 8 weeks after the start of therapy. The other responses are incorrect. DIF: COGNITIVE LEVEL: Applying (Application) REF: p. 440 TOP: NURSING PROCESS: Implementation

2. A pregnant patient who is taking ondansetron [Zofran] for morning sickness tells the nurse she is experiencing headache and dizziness. What will the nurse tell her? a. It is not safe to take this drug during pregnancy. b. These are common side effects of ondansetron. c. She should stop taking the ondansetron immediately. d. She should report these adverse effects to her provider.

ANS: B The most common side effects of ondansetron are headache, diarrhea, and dizziness. Ondansetron is used off-label to treat morning sickness. These side effects do not indicate a need to stop taking the drug or to report the side effects to the provider.

6. A patient has a digoxin level of 1.1 ng/mL. Which interpretation by the nurse is correct? a. It is below the therapeutic level. b. It is within the therapeutic range. c. It is above the therapeutic level. d. It is at a toxic level.

ANS: B The normal therapeutic drug level of digoxin is between 0.5 and 2 ng/mL. The other options are incorrect. DIF: COGNITIVE LEVEL: Analyzing (Analysis) REF: p. 383 TOP: NURSING PROCESS: Evaluation

14. A patient who is taking the bisphosphonate alendronate (Fosamax) has been instructed to lie flat in bed for 2 days after having ophthalmic surgery. Which intervention is appropriate at this time? a. She will continue to take the alendronate with water. b. She cannot take the alendronate until she can sit up for 30 minutes. c. She can take the medication with breakfast. d. She will stop taking the medication 72 hours before her surgery.

ANS: B The nurse must emphasize that the patient should remain upright in either a standing or sitting position for approximately 30 minutes after taking a bisphosphonate so as to help prevent esophageal erosion or irritation. Because this patient will be required to lie flat in bed for 2 days after the surgery, the prescriber will need to be notified that the patient cannot take the medication during this time. DIF: COGNITIVE LEVEL: Applying (Application) REF: p. 549 TOP: NURSING PROCESS: Implementation

13. A nurse is preparing to administer digoxin [Lanoxin] to a patient. The patient's heart rate is 62 beats per minute, and the blood pressure is 120/60 mm Hg. The last serum electrolyte value showed a potassium level of 5.2 mEq/L. What will the nurse do? a. Contact the provider to request an increased dose of digoxin. b. Give the dose of digoxin and notify the provider of the potassium level. c. Request an order for a diuretic. d. Withhold the dose and notify the provider of the heart rate.

ANS: B The patient's serum potassium level is above normal limits, but only slightly. An elevated potassium level can reduce the effects of digoxin, so there is no risk of toxicity. There is no indication that an increased dose of digoxin is needed. There is no indication for a diuretic. The heart rate is acceptable; doses should be withheld if the heart rate is less than 60 beats per minute.

21. The insulin order reads, "Give 10 units of NPH insulin and 5 units of regular insulin, subcut, every morning before breakfast." Choose the proper syringe for this injection. a. b. c. d.

ANS: B The proper syringe for insulin injection is the insulin syringe, which is marked in units. The other syringes listed are not correct for use with insulin because they are not marked in units. DIF: COGNITIVE LEVEL: Analyzing (Analysis) REF: p. 506 TOP: NURSING PROCESS: Planning

3. A patient is in an urgent care center with an acute asthma attack. The nurse expects that which medication will be used for initial treatment? a. An anticholinergic such as ipratropium (Atrovent) b. A short-acting beta2 agonist such as albuterol (Proventil) c. A long-acting beta2 agonist such as salmeterol (Serevent) d. A corticosteroid such as fluticasone (Flovent)

ANS: B The short-acting beta2 agonists are commonly used during the acute phase of an asthmatic attack to reduce airway constriction quickly and to restore airflow to normal levels. The other drugs listed are not appropriate for acute asthma attacks. Anticholinergic drugs and long-acting beta2 agonists are used to prevent attacks; corticosteroids are used to reduce airway inflammation. DIF: COGNITIVE LEVEL: Applying (Application) REF: p. 582 TOP: NURSING PROCESS: Planning

7. A patient who experiences motion sickness is about to go on a cruise. The prescriber orders transdermal scopolamine [Transderm Scop]. The patient asks the nurse why an oral agent is not ordered. The nurse will explain that the transdermal preparation: a. can be applied as needed at the first sign of nausea. b. has less intense anticholinergic effects than the oral form. c. is less sedating than the oral preparation. d. provides direct effects, because it is placed close to the vestibular apparatus of the ear.

ANS: B The transdermal system of scopolamine is preferred, because it may have less intense anticholinergic effects than oral or subcutaneous dosing. Antinausea medications for motion sickness are more effective if given prophylactically than after symptoms begin. Sedation side effects are similar with all forms. Placement near the ear does not cause the medication to absorb directly into the vestibular apparatus.

16. When administering morning medications for a newly admitted patient, the nurse notes that the patient has an allergy to sulfa drugs. There is an order for the sulfonylurea glipizide (Glucotrol). Which action by the nurse is correct? a. Give the drug as ordered 30 minutes before breakfast. b. Hold the drug, and check the order with the prescriber. c. Give a reduced dose of the drug with breakfast. d. Give the drug, and monitor for adverse effects.

ANS: B There is a potential for cross-allergy in patients who are allergic to sulfonamide antibiotics. Although such an allergy is listed as a contraindication by the manufacturer, most clinicians do prescribe sulfonylureas for such patients. The order needs to be clarified. DIF: COGNITIVE LEVEL: Applying (Application) REF: p. 509 TOP: NURSING PROCESS: Implementation

6. A patient with a history of alcohol abuse has been admitted for severe weakness and malnutrition. The nurse will prepare to administer which vitamin preparation to prevent Wernicke's encephalopathy? a. Vitamin B3 (niacin) b. Vitamin B1 (thiamine) c. Vitamin B6 (pyridoxine) d. Folic acid

ANS: B Thiamine is necessary for the treatment of a variety of thiamine deficiencies, including Wernicke's encephalopathy. The other options are incorrect. DIF: COGNITIVE LEVEL: Understanding (Comprehension) REF: p. 848 TOP: NURSING PROCESS: Planning

An adolescent female patient with multiple sexual partners asks a nurse about birth control methods. The patient tells the nurse she tried oral contraceptives once but often forgot to take her pills. The nurse will recommend discussing which contraception with the provider? a. An intrauterine device with a spermicide b. DMPA (Depo-Provera) and condoms c. Tubal ligation and condoms d. Progestin-only oral contraceptives

ANS: B This patient has demonstrated a previous history of nonadherence, so a long-acting contraceptive would be more effective for her. Because she has multiple sexual partners, she should use a condom for protection against STDs. An IUD is not indicated for her; patients with multiple sexual partners who use IUDs are at greater risk for STDs. Tubal ligation carries surgical risks and should not be used by young women, because it is irreversible. Progestin-only oral contraceptives must be taken every day.

6. A pregnant patient is being induced with oxytocin, which has been infusing for 1 hour. The initial rate was 6 milliunits/min, and the rate now is 18 milliunits/min. The nurse notes regular contractions occurring every 3 minutes, each lasting 35 seconds. The nurse will: a. increase the rate of infusion by 1 to 2 milliunits/min every 15 to 40 minutes. b. increase the rate of infusion by 3 to 6 milliunits/min every 15 to 40 minutes. c. interrupt the infusion and continue to monitor the patient before restarting. d. interrupt the infusion and notify the provider of potential oxytocin toxicity.

ANS: B This patient is being induced with a high-dose regimen of oxytocin, which starts at 6 milliunits/min and is increased by 3 to 6 milliunits/min every 15 to 40 minutes. The frequency and rate of her contractions are within normal limits, so the nurse can continue to increase the rate by 3 to 6 milliunits/min every 15 to 40 minutes until contractions last longer than 1 minute or occur more frequently than every 2 to 3 minutes. Increasing the rate of infusion by 1 to 2 milliunits/min every 15 to 40 minutes is part of the low-dose regimen. Because the frequency and duration of contractions have not reached optimum level, there is no need to interrupt the infusion. There are no signs of oxytocin toxicity. PTS: 1 DIF: Cognitive Level: Application REF: p. 783 | p. 786

Which laxatives may be used safely in infants? (Select all that apply.) a. Castor oil b. Docusate sodium (Colace) c. Glycerin suppositories d. Lactulose e. Milk of magnesia

ANS: B, C, D Docusate, lactulose, and glycerin suppositories have been used to treat constipation safely in infants. Castor oil and milk of magnesia are not.

6. A patient admitted to the hospital has been using phenylephrine nasal spray [Neo-Synephrine], 2 sprays every 4 hours, for a week. The patient complains that the medication is not working, because the nasal congestion has increased. What will the nurse do? a. Request an order for an oral decongestant to replace the intranasal phenylephrine. b. Request an order for an intranasal glucocorticoid to be used while the phenylephrine is withdrawn. c. Tell the patient to increase the dose of phenylephrine to 4 sprays every 4 hours. d. Tell the patient to stop using the phenylephrine and begin using an intranasal antihistamine.

ANS: B This patient is experiencing rebound congestion, which develops when topical sympathomimetics are used for longer than a few days. Abrupt withdrawal can stop the cycle of rebound congestion but is uncomfortable, so using an intranasal glucocorticoid, beginning one week before discontinuing the decongestant, while withdrawing the decongestant, is recommended. An oral decongestant is not recommended. Increasing the dose of the intranasal decongestant will only compound the problem of rebound congestion. Stopping the intranasal decongestant will only increase the congestion; using an intranasal antihistamine will not help with congestion.

A patient received atropine intravenously before surgery. The recovery room nurse notes that the patient is delirious upon awakening and has a heart rate of 96 beats per minute, a respiratory rate of 22 breaths per minute, and a blood pressure of 110/78 mm Hg. The nurse notifies the anesthesiologist, who will order: a.activated charcoal to minimize intestinal absorption of the antimuscarinic agent. b.an acetylcholinesterase inhibitor to compete with the antimuscarinic agent at receptors. c.an antipsychotic medication to treat the patient's central nervous system symptoms. d.ipratropium bromide [Atrovent] to counter the respiratory effects of the antimuscarinic agent.

ANS: B This patient is showing signs of antimuscarinic toxicity, caused by the atropine given during surgery. The most effective antidote is physostigmine, which inhibits acetylcholinesterase, allowing acetylcholine to build up at cholinergic junctions and compete with the antimuscarinic agent for receptor binding. Activated charcoal is useful only if an antimuscarinic agent has been ingested, because it impedes absorption from the GI tract. Because this patient's psychotic symptoms are caused by an antimuscarinic agent, physostigmine should be given to treat the cause; an antipsychotic medication would only treat the symptom. Ipratropium bromide is an antimuscarinic agent and would only compound the effects. This patient's respiratory rate is only mildly elevated.

10. A patient who is malnourished has scaling, cracked skin on the arms and face. The patient is irritable, anxious, and has difficulty sleeping. The patient complains of soreness of the tongue and mouth. When teaching this patient about vitamin therapy for this disorder, the nurse will tell the patient to report which side effects? a. Abdominal cramps and diarrhea b. Flushing, dizziness, and nausea c. Migraine headaches d. Numbness of feet and hands

ANS: B This patient shows signs of niacin deficiency. Side effects of niacin deficiency include flushing, dizziness, and nausea secondary to vasodilation. Abdominal cramps, diarrhea, migraines, and numbness of extremities are not side effects of niacin deficiency. PTS: 1 DIF: Cognitive Level: Analysis REF: pp. 992-993

6. The nurse is reviewing the use of uterine tocolytics, such as indomethacin (Indocin). Which statement best describes the indication for these drugs? a. Prevention of preterm labor in the 15th week of pregnancy b. Prevention of preterm labor in the 22nd week of pregnancy c. Stimulation of contractions in prolonged labor d. Stimulation of ovulation as part of infertility treatments

ANS: B Tocolytics relax uterine smooth muscles and stop the uterus from contracting and are used along with nonpharmacologic measures to prevent preterm labor between 20 and 37 weeks of pregnancy. DIF: COGNITIVE LEVEL: Applying (Application) REF: p. 546 TOP: NURSING PROCESS: Planning

2. The nurse is preparing a plan of care for a patient undergoing therapy with vitamin A. Which nursing diagnosis is appropriate for this patient? a. Impaired tissue integrity related to vitamin deficiency b. Risk for injury related to night blindness caused by vitamin deficiency c. Impaired physical mobility (muscle weakness) related to vitamin deficiency d. Acute confusion related to vitamin deficiency

ANS: B Vitamin A deficiency causes night blindness, so risk for injury is an appropriate nursing diagnosis. The other nursing diagnoses are not appropriate for patients receiving vitamin A. DIF: COGNITIVE LEVEL: Applying (Application) REF: p. 857 TOP: NURSING PROCESS: Nursing Diagnosis

7. A patient with a severe fungal infection has orders for voriconazole (Vfend). The nurse is reviewing the patient's medical record and would be concerned if which assessment finding is noted? a. Decreased breath sounds in the lower lobes b. History of cardiac dysrhythmias c. History of type 2 diabetes d. Potassium level of 4.0 mEq/L

ANS: B Voriconazole is contraindicated when co-administered with certain other drugs metabolized by the cytochrome P-450 enzyme 3A4 (e.g., quinidine) because of the risk for inducing serious cardiac dysrhythmias. DIF: COGNITIVE LEVEL: Applying (Application) REF: p. 675 TOP: NURSING PROCESS: Assessment

A patient has been experiencing side effects with a combination oral contraceptive, and her provider has ordered a different combination product. The nurse will instruct the patient to do what? a. Begin taking the new product immediately. b. Change products at the beginning of her next cycle. c. Stop taking the old OC 1 week before starting the new OC. d. Use an alternate method of contraception for 1 month before starting the new OC.

ANS: B When changing one combination OC for another, the change is best made at the beginning of a new cycle. It is not correct to begin taking the new product immediately; to stop the old product 1 week before starting the new product; or to use an alternate method of birth control between regimens.

4. A nurse is teaching a nursing student what is meant by "generations" of cephalosporins. Which statement by the student indicates understanding of the teaching? a. "Cephalosporins are assigned to generations based on their relative costs to administer." b. "Cephalosporins have increased activity against gram-negative bacteria with each generation." c. "First-generation cephalosporins have better penetration of the cerebrospinal fluid." d. "Later generations of cephalosporins have lower resistance to destruction by beta-lactamases."

ANS: B With each progression from first-generation agents to fifth-generation agents, the cephalosporins show increased activity against gram-negative organisms, increased resistance to destruction by beta-lactamases, and increased ability to reach the CSF. Cost is not a definitive factor. First-generation drugs have less penetration of the CSF. Resistance to destruction by beta-lactamases increases with increasing generations.

10. A newly admitted patient has orders for a zinc supplement. The nurse reviews the patient's medical history and concludes that the zinc is ordered for which reason? a. To treat pellagra b. To aid in wound healing c. To treat osteomalacia d. As an antidote for anticoagulant overdose

ANS: B Zinc plays a crucial role in the enzymatic metabolic reactions involving both proteins and carbohydrates. This makes it especially important for normal tissue growth and repair. It therefore also has a major role in wound healing. Vitamin B3 (niacin) is used to treat pellagra; vitamin D is used to treat osteomalacia; and vitamin K is used as an antidote for anticoagulant overdose. DIF: COGNITIVE LEVEL: Applying (Application) REF: p. 856 TOP: NURSING PROCESS: Planning

2. The nurse is monitoring for adverse effects in a patient who is receiving an amiodarone (Cordarone) infusion. Which are adverse effects for amiodarone? (Select all that apply.) a. Tachycardia b. Constipation c. Chest pain d. QT prolongation e. Headache f. Hypotension g. Blue-gray coloring of the skin on the face, arms, and neck

ANS: B, D, F, G There are numerous adverse effects of amiodarone, including pulmonary toxicity, thyroid disorders, bradycardia, hypotension, SA node dysfunction, QT prolongation, blue-gray coloring of the skin (face, arms, and neck), constipation, and others. Tachycardia, chest pain, and headache are not adverse effects of amiodarone therapy. DIF: COGNITIVE LEVEL: Applying (Application) REF: p. 399 TOP: NURSING PROCESS: Evaluation

1. The nurse is reviewing medications for the treatment of asthma. Which drugs are used for acute asthma attacks? (Select all that apply.) a. Salmeterol (Serevent) inhaler b. Albuterol (Proventil) nebulizer solution c. Epinephrine d. Montelukast (Singulair) e. Fluticasone (Flovent) Rotadisk inhaler

ANS: B, C Albuterol (a short-acting beta2 agonist) and epinephrine (a beta1 and beta2 agonist) are used for acute bronchospasms. Salmeterol is a long-acting beta2 agonist that is indicated for maintenance treatment, not acute episodes. Fluticasone is an inhaled corticosteroid; montelukast is a leukotriene receptor antagonist (LTRA). These types of medications are used for asthma prophylaxis. DIF: COGNITIVE LEVEL: Applying (Application) REF: p. 591 TOP: NURSING PROCESS: Implementation

A nursing student is caring for a patient with community-acquired pneumonia (CAP) who also has a methicillin-resistant Staphylococcus aureus (MRSA) skin infection. The prescriber has ordered daptomycin [Cubicin]. The nurse encourages the student to approach the provider to request a different antibiotic because of which facts about daptomycin? (Select all that apply.) a. It causes significant renal impairment. b. It is approved only for bloodstream and skin infections. c. It increases the risk of serious cardiorespiratory events. d. It is more likely to produce resistant strains of bacteria. e. It is not effective against MRSA infections.

ANS: B, C Daptomycin has been approved only for bloodstream and skin infections caused by S. aureus, including MRSA infections, and would not be indicated for a CAP infection. In patients with CAP, daptomycin poses a higher risk of serious cardiorespiratory side effects. Daptomycin does not cause renal impairment and is no more likely to produce bacterial resistance than other antibiotics. Daptomycin can be used to treat MRSA infections.

1. Antilipemic drug therapy is prescribed for a patient, and the nurse is providing instructions to the patient about the medication. Which instructions will the nurse include? (Select all that apply.) a. Limit fluid intake to prevent fluid overload. b. Eat extra servings of raw vegetables and fruit. c. Report abnormal or unusual bleeding or yellow discoloration of the skin. d. Report the occurrence of muscle pain immediately. e. Drug interactions are rare with antilipemics. f. Take the drug 1 hour before or 2 hours after meals to maximize absorption.

ANS: B, C, D Instructions need to include preventing constipation by encouraging a diet that is plentiful in raw vegetables, fruit, and bran. Forcing fluids (up to 3000 mL/day unless contraindicated) may also help to prevent constipation. Notify the prescriber if there are any new or troublesome symptoms, abnormal or unusual bleeding, yellow discoloration of the skin, or muscle pain. These drugs are highly protein bound; therefore, they interact with many drugs. Taking these drugs with food may help to reduce gastrointestinal distress. DIF: COGNITIVE LEVEL: Applying (Application) REF: p. 448 TOP: NURSING PROCESS: Planning

Which patients would be candidates for the use of dronabinol [Marinol] to treat nausea and vomiting? (Select all that apply.) a. A patient with a history of a psychiatric disorder b. A patient with acquired immunodeficiency syndrome (AIDS)-induced anorexia c. A patient with chemotherapy-induced nausea and vomiting d. A patient with nausea who has used marijuana in the past e. A patient with postoperative nausea and vomiting

ANS: B, C, D Two cannabinoids have been approved for medical use in the United States for AIDS-induced anorexia and CINV. Because these drugs do not produce the same "high" that smoking marijuana does, their use is not contraindicated in patients who have used marijuana in the past. They do cause potentially unpleasant effects, such as temporal disintegration, dissociation, depersonalization, and dysphoria, so they are contraindicated in patients with psychiatric disorders. They are not approved for postoperative nausea and vomiting.

2. The patient asks the nurse about taking large doses of vitamin C to improve her immunity to colds. "It's just a vitamin, right? What can happen?" Which responses by the nurse are correct? (Select all that apply.) a. "Vitamin C is harmless because it is a water-soluble vitamin." b. "Large doses of vitamin C can cause nausea, vomiting, headache, and abdominal cramps." c. "Keep in mind that if you suddenly stop taking these large doses, you might experience symptoms similar to scurvy." d. "Studies have shown that vitamin C has little value in preventing the common cold." e. "Vitamin C acidifies the urine, which can lead to the formation of kidney stones." f. "Large doses of vitamin C may delay wound healing."

ANS: B, C, D, E Vitamin C is usually nontoxic unless excessive dosages are consumed. Large doses (megadoses) can produce nausea, vomiting, headache, and abdominal cramps, and they acidify the urine, which can result in the formation of kidney stones. Furthermore, individuals who discontinue taking excessive daily doses of ascorbic acid can experience scurvy-like symptoms. Studies have shown that megadoses of vitamin C have little or no value as prophylaxis against the common cold. Vitamin C is required for several important metabolic activities, including collagen synthesis and the maintenance of connective tissue and tissue repair. DIF: COGNITIVE LEVEL: Applying (Application) REF: p. 852 TOP: NURSING PROCESS: Implementation

1. The nurse is reviewing the uses of oral laxatives. Which conditions are general contraindications to or cautions about the use of oral laxatives? (Select all that apply.) a. Irritable bowel syndrome b. Undiagnosed abdominal pain c. Nausea and vomiting d. Fecal impaction e. Ingestion of toxic substances f. Acute surgical abdomen

ANS: B, C, D, F Cautious use of laxatives is recommended in the presence of these: acute surgical abdomen; appendicitis symptoms, such as abdominal pain, nausea, and vomiting; intestinal obstruction; and undiagnosed abdominal pain. Oral laxatives must not be used with fecal impaction; mineral oil enemas are indicated for fecal impaction. The other options are indications for laxative use. DIF: COGNITIVE LEVEL: Applying (Application) REF: p. 820 TOP: NURSING PROCESS: Assessment

Which order for potassium (KCl) would the nurse question? (Select all that apply.) a. NS with 20 mEq KCl to start after patient voids b. NS with 60 mEq KCl for a patient with a serum potassium of 3.2 mEq/L c. K-Dur, 1 tablet daily for a patient in diabetic ketoacidosis d. K-Dur, 1 tablet with a full glass of water e. Potassium chloride, 10 mEq rapid IV push

ANS: B, C, E Patients receiving potassium should not receive potassium in solution diluted at more than 40 mEq/L. Diabetic ketoacidosis causes decreased cellular uptake of potassium, and patients in whom acidosis is present should also receive sodium bicarbonate if they need potassium. IV potassium should not be infused faster than 10 mEq/hr. NS with 20 mEq KCl is an appropriate solution for a patient with intact renal function. Oral potassium should be given with a full glass of water to minimize gastrointestinal (GI) effects.

1. A cholinergic drug is prescribed for a patient. The nurse checks the patient's medical history, knowing that this drug is contraindicated in which disorders? (Select all that apply.) a. Bladder atony b. Gastrointestinal obstruction c. Bradycardia d. Alzheimer's disease e. Hypotension f. Chronic obstructive pulmonary disease

ANS: B, C, E, F Contraindications to the use of cholinergic drugs include gastrointestinal or genitourinary obstruction, bradycardia, hypotension, and chronic obstructive pulmonary disease. The other options are possible indications for cholinergic drugs. DIF: COGNITIVE LEVEL: Applying (Application) REF: p. 324 TOP: NURSING PROCESS: Assessment

2. The nurse is providing instructions to a patient who has a new prescription for a corticosteroid metered-dose inhaler. Which statement by the patient indicates that further instruction is needed? (Select all that apply.) a. "I will rinse my mouth with water after using the inhaler and then spit out the water." b. "I will gargle after using the inhaler and then swallow." c. "I will clean the plastic inhaler casing weekly by removing the canister and then washing the casing in warm soapy water. I will then let it dry before reassembling." d. "I will use this inhaler for asthma attacks." e. "I will continue to use this inhaler, even if I am feeling better." f. "I will use a peak flow meter to measure my response to therapy."

ANS: B, D The inhaled corticosteroid is a maintenance drug used to prevent asthma attacks; it is not indicated for acute asthma attacks. Rinsing the mouth with water is appropriate and necessary to prevent oral fungal infections; the water is not to be swallowed after rinsing. The patient needs to be given instructions about keeping the inhaler clean, including removing the canister from the plastic casing weekly and washing the casing in warm soapy water. Once the casing is dry, the canister and mouthpiece may be put back together and the cap applied. The glucocorticoid may predispose the patient to oral fungal overgrowth, thus the need for implicit instructions about cleaning inhaling devices. Use of a peak flow meter assists in monitoring the patient's response to therapy. The medication needs to be taken as ordered every day, regardless of whether the patient is feeling better. DIF: COGNITIVE LEVEL: Applying (Application) REF: p. 594 TOP: NURSING PROCESS: Implementation

1. When a patient is experiencing digoxin toxicity, which clinical situation would necessitate the use of digoxin immune Fab (Digifab)? (Select all that apply.) a. The patient reports seeing colorful halos around lights. b. The patient's serum potassium level is above 5 mEq/L. c. The patient is experiencing nausea and anorexia. d. The patient is experiencing severe sinus bradycardia that does not respond to cardiac pacing. e. The patient has received an overdose of greater than 10 mg of digoxin. f. The patient reports fatigue and headaches.

ANS: B, D, E Clinical situations that would require the use of digoxin immune Fab in a patient with digoxin toxicity include serum potassium level above 5 mEq/L, severe sinus bradycardia that does not respond to cardiac pacing, or an overdose of more than 10 mg of digoxin. Seeing colorful halos around lights and experiencing nausea, anorexia, fatigue, and headaches are potential adverse effects of digoxin therapy but are not necessarily reasons for digoxin immune Fab treatment. DIF: COGNITIVE LEVEL: Analyzing (Analysis) REF: p. 382 TOP: NURSING PROCESS: Assessment

1. During diuretic therapy, the nurse monitors the fluid and electrolyte status of the patient. Which assessment findings are symptoms of hyponatremia? (Select all that apply.) a. Red, flushed skin b. Lethargy c. Decreased urination d. Hypotension e. Stomach cramps f. Elevated temperature

ANS: B, D, E Hyponatremia is manifested by lethargy, hypotension, stomach cramps, vomiting, diarrhea, and seizures. The other options are symptoms of hypernatremia. DIF: COGNITIVE LEVEL: Understanding (Comprehension) REF: p. 472 TOP: NURSING PROCESS: Evaluation

2. A patient has started azathioprine (Imuran) therapy as part of renal transplant surgery. The nurse will monitor for which expected adverse effect of azathioprine therapy? (Select all that apply.) a. Tremors b. Leukopenia c. Diarrhea d. Thrombocytopenia e. Hepatotoxicity f. Fluid retention

ANS: B, D, E Leukopenia is an expected adverse effect of azathioprine therapy, as are thrombocytopenia and hepatotoxicity. The other options are incorrect. DIF: COGNITIVE LEVEL: Applying (Application) REF: p. 773 TOP: NURSING PROCESS: Evaluation

A patient newly diagnosed with diabetes expresses concern about losing her vision. Which interventions should be included in the plan of care to reduce this risk? (Select all that apply.) a. Initiation of reliable contraception to prevent pregnancy b. Ways to reduce hyperglycemic episodes c. Use of a prokinetic drug (eg, metoclopramide) d. Smoking cessation e. Emphasis on the importance of taking antihypertensive drugs consistently

ANS: B, D, E Tighter glycemic control and fewer hyperglycemic episodes have been correlated with fewer complications. Smoking cessation and strict compliance with any drugs prescribed for hypertension also help reduce the risk of complications. The use of contraception or a prokinetic drug is not related to a reduced risk of vision loss in patients with diabetes.

1. The nurse follows which procedures when giving intravenous (IV) cyclosporine? (Select all that apply.) a. Administering it as a single IV bolus injection to minimize adverse effects b. Using an infusion pump to administer this medication c. Monitoring the patient for potential delayed adverse effects, which may be severe d. Monitoring the patient closely for the first 30 minutes for severe adverse effects e. Checking blood levels periodically during cyclosporine therapy f. Performing frequent oral care during therapy

ANS: B, D, E, F Cyclosporine is infused intravenously with an infusion pump, not as an IV bolus. Monitor the patient closely for the first 30 minutes for adverse effects, especially for allergic reactions, and monitor blood levels periodically to ensure therapeutic, not toxic, levels of the medication. Perform oral hygiene frequently to prevent dry mouth and subsequent infections. DIF: COGNITIVE LEVEL: Applying (Application) REF: p. 778 TOP: NURSING PROCESS: Implementation

1. The nurse is administering an amphotericin B infusion. Which actions by the nurse are appropriate? (Select all that apply.) a. Administering the medication by rapid IV infusion b. Discontinuing the drug immediately if the patient develops tingling and numbness in the extremities c. If adverse effects occur, reducing the IV rate gradually until they subside d. Using an infusion pump for IV therapy e. Monitoring the IV site for signs of phlebitis and infiltration f. Administering premedication for fever and nausea g. Knowing that the intravenous solution for amphotericin B will be cloudy h. Knowing that muscle twitching may indicate hypokalemia

ANS: B, D, E, F If the patient develops tingling and numbness in the extremities (paresthesias), discontinue the drug immediately. An infusion pump is necessary for the infusion, and the nurse will monitor the IV site for signs of phlebitis and infiltration. Premedication to reduce the adverse effects of fever, malaise, and nausea may be ordered. The IV solution must be clear and without precipitates; and muscle weakness, not twitching, may indicate hypokalemia. The medication must be administered at the rate recommended and stopped, not slowed, if adverse reactions occur. DIF: COGNITIVE LEVEL: Applying (Application) REF: p. 673 TOP: NURSING PROCESS: Implementation

Which patient(s) should be tested for latent TB? (Select all that apply.) a. A day care employee b. A worker in a mycobacteriology laboratory c. A patient with cardiovascular disease d. A patient with diabetes mellitus e. An immigrant from a country where TB is prevalent

ANS: B,D,E Testing should be limited to people who are at high risk of either having acquired TB recently or who might be progressing from latent to active TB. People who work in mycobacteriology laboratories work with TB and are at risk. Patients with some debilitating diseases, such as diabetes mellitus, are at risk. Immigrants from countries where TB is prevalent are at risk. Employees of day care centers and patients with cardiovascular disease are not at increased risk.

6. A patient with type 1 diabetes recently became pregnant. The nurse plans a blood glucose testing schedule for her. What is the recommended monitoring schedule? a. Before each meal and before bed b. In the morning for a fasting level and at 4:00 PM for the peak level c. Six or seven times a day d. Three times a day, along with urine glucose testing

ANS: C A pregnant patient with type 1 diabetes must have frequent blood sugar monitoring (eg, six or seven times a day) to manage both the patient and the fetus so that no teratogenic effects occur. Monitoring the blood sugar level before meals and at bedtime is not significant enough to provide the necessary glycemic control. Morning and 4:00 PM monitoring is not enough to provide glycemic control. Urine glucose testing is not sensitive enough to aid glycemic control, and monitoring three times a day is not enough.

9. A patient has received a prescription for a 2-week course of antifungal suppositories for a vaginal yeast infection. She asks the nurse if there is an alternative to this medication, saying, "I don't want to do this for 2 weeks!" Which is a possibility in this situation? a. A single dose of a vaginal antifungal cream. b. A one-time infusion of amphotericin B. c. A single dose of a fluconazole (Diflucan) oral tablet. d. There is no better alternative to the suppositories.

ANS: C A single oral dose of fluconazole may be used to treat vaginal candidiasis. The other options are incorrect. DIF: COGNITIVE LEVEL: Applying (Application) REF: p. 674 TOP: NURSING PROCESS: Planning

3. A patient has been diagnosed with angina and will be given a prescription for sublingual nitroglycerin tablets. When teaching the patient how to use sublingual nitroglycerin, the nurse will include which instruction? a. Take up to 5 doses at 15-minute intervals for an angina attack. b. If the tablet does not dissolve quickly, chew the tablet for maximal effect. c. If the chest pain is not relieved after one tablet, call 911 immediately. d. Wait 1 minute between doses of sublingual tablets, up to 3 doses.

ANS: C According to current guidelines, if the chest pain or discomfort is not relieved in 5 minutes, after 1 dose, the patient (or family member) must call 911 immediately. The patient may take one more tablet while awaiting emergency care and may take a third tablet 5 minutes later, but no more than a total of three tablets. The sublingual dose is placed under the tongue, and the patient needs to avoid swallowing until the tablet has dissolved. DIF: COGNITIVE LEVEL: Applying (Application) REF: p. 373 TOP: NURSING PROCESS: Implementation

18. An older adult female patient is receiving the progestin drug megestrol (Megace). Which is the most likely reason megestrol is ordered for this patient? a. Migraine headaches b. Osteoporosis c. Appetite stimulant d. Reduction of hot flashes

ANS: C Megestrol can cause appetite stimulation and weight gain, and therefore is used in the management of anorexia, cachexia, or unexplained substantial weight loss in patients with acquired immunodeficiency syndrome (AIDS) and in patients with cancer. The other options are incorrect. DIF: COGNITIVE LEVEL: Applying (Application) REF: p. 539 TOP: NURSING PROCESS: Planning

6. The nurse is preparing to administer adenosine (Adenocard) to a patient who is experiencing an acute episode of paroxysmal supraventricular tachycardia. When giving this medication, which is important to remember? a. The onset of action occurs in 5 minutes. b. The medication must be given as a slow intravenous (IV) push. c. Asystole may occur for a few seconds after administration. d. The medication has a long half-life, and therefore duration of action is very long.

ANS: C Adenosine has an extremely short half-life of less than 10 seconds; its onset occurs within 1 minute; and it must be given as a fast IV push injection. In addition, a very brief episode of asystole may occur after administration. DIF: COGNITIVE LEVEL: Applying (Application) REF: p. 405 TOP: NURSING PROCESS: Implementation

12. A patient who is known to be a heavy drinker is brought to the emergency department with ataxia and confusion. The patient cannot remember the events of the previous day. The examination reveals nystagmus, and the patient reports having double vision. The nurse will expect to administer which vitamin to this patient? a. Ascorbic acid (vitamin C) IV b. Intramuscular pyridoxine (vitamin B6) c. Intravenous thiamine (vitamin B1) d. Nicotinic acid (niacin) PO

ANS: C Alcoholics who are malnourished have a form of thiamine deficiency called Wernicke-Korsakoff syndrome, which is characterized by nystagmus, diplopia, ataxia, confusion, and short-term memory loss. Parenteral thiamine is indicated for treatment. Ascorbic acid is given to treat vitamin C deficiency, or scurvy. Pyridoxine is given for vitamin B6 deficiency, also common in alcoholics, but this patient does not have the symptoms of seborrheic dermatitis and peripheral neuropathy. Nicotinic acid is used for niacin deficiency, which is characterized by severe dry, rough skin. PTS: 1 DIF: Cognitive Level: Application REF: p. 993

13. A laxative has been ordered for a patient. The nurse checks the patient's medical history and would be concerned if which condition is present? a. High ammonia levels due to liver failure b. Diverticulosis c. Abdominal pain of unknown origin d. Chronic constipation

ANS: C All categories of laxatives share the same general contraindications and precautions, including avoidance in cases of drug allergy and the need for cautious use in the presence of these: acute surgical abdomen; appendicitis symptoms such as abdominal pain, nausea, and vomiting; fecal impaction (mineral oil enemas excepted); intestinal obstruction; and undiagnosed abdominal pain. The other options are possible indications for laxatives. DIF: COGNITIVE LEVEL: Applying (Application) REF: p. 820 TOP: NURSING PROCESS: Implementation

A nurse working in the emergency department is assigned to a child who is arriving by ambulance after being involved in a spill of organophosphate insecticides. What will the nurse expect to be the initial priority for treating this child? a.Administering diphenhydramine to control secretions b.Giving diazepam to control seizures c.Providing mechanical ventilation and oxygen d.Reporting the spill to the Environmental Protection Agency

ANS: C All of these courses of action may be part of the treatment. However, the initial concern is to prevent death from apnea caused by laryngospasm, bronchoconstriction, and paralysis leading to apnea; therefore, maintaining an airway is the most important consideration. Pralidoxime is the specific antidote and should be given next. Atropine is used to control secretions. Diazepam is given when seizures occur. Reporting the accident to the proper authorities is not a priority during a life-threatening emergency.

18. A patient has been treated with alosetron (Lotronex) for severe irritable bowel syndrome (IBS) for 2 weeks. She calls the clinic and tells the nurse that she has been experiencing constipation for 3 days. The nurse will take which action? a. Advise the patient to increase intake of fluids and fiber. b. Advise the patient to hold the drug for 2 days. c. Instruct the patient to stop taking the drug and to come to the clinic right away to be evaluated. d. Instruct the patient to continue the alosetron and to take milk of magnesia for the constipation.

ANS: C Alosetron must be discontinued immediately if constipation or signs of ischemic colitis occur. The other options are incorrect. DIF: COGNITIVE LEVEL: Analyzing (Analysis) REF: p. 823 TOP: NURSING PROCESS: Implementation

8. The nurse is monitoring drug levels for a patient who is receiving theophylline. The most recent theophylline level was 22 mcg/mL, and the nurse evaluates this level to be: a. below the therapeutic level. b. at a therapeutic level. c. above the therapeutic level. d. at a toxic level.

ANS: C Although the optimal level may vary from patient to patient, most standard references have suggested that the therapeutic range for theophylline blood level is 10 to 20 mcg/mL. However, most clinicians now advise levels between 5 and 15 mcg/mL. DIF: COGNITIVE LEVEL: Applying (Application) REF: p. 585 TOP: NURSING PROCESS: Evaluation

5. A patient is in the intensive care unit because of an acute myocardial infarction. He is experiencing severe ventricular dysrhythmias. The nurse will prepare to give which drug of choice for this dysrhythmia? a. Diltiazem (Cardizem) b. Verapamil (Calan) c. Amiodarone (Cordarone) d. Adenosine (Adenocard)

ANS: C Amiodarone (Cordarone) is the drug of choice for ventricular dysrhythmias according to the Advanced Cardiac Life Support guidelines. The other drugs are not used for acute ventricular dysrhythmias. DIF: COGNITIVE LEVEL: Applying (Application) REF: p. 403 TOP: NURSING PROCESS: Planning

6. A patient who has been taking verapamil [Calan] for hypertension complains of constipation. The patient will begin taking amlodipine [Norvasc] to avoid this side effect. The nurse provides teaching about the difference between the two drugs. Which statement by the patient indicates that further teaching is needed? a. "I can expect dizziness and facial flushing with nifedipine." b. "I should notify the provider if I have swelling of my hands and feet." c. "I will need to take a beta blocker to prevent reflex tachycardia." d. "I will need to take this drug once a day."

ANS: C Amlodipine produces selective blockade of calcium channels in blood vessels with minimal effects on the heart. Reflex tachycardia is not common, so a beta blocker is not indicated to prevent this effect. Dizziness and facial flushing may occur. Peripheral edema may occur and should be reported to the provider. Amlodipine is given once daily.

8. The nurse notes in the patient's medication orders that the patient will be starting anticoagulant therapy. What is the primary goal of anticoagulant therapy? a. Stabilizing an existing thrombus b. Dissolving an existing thrombus c. Preventing thrombus formation d. Dilating the vessel around a clot

ANS: C Anticoagulants prevent thrombus formation but do not dissolve or stabilize an existing thrombus, nor do they dilate vessels around a clot. DIF: COGNITIVE LEVEL: Understanding (Comprehension) REF: p. 414 TOP: NURSING PROCESS: Implementation

2. During a blood transfusion, the patient begins to have chills and back pain. What is the nurse's priority action? a. Observe for other symptoms. b. Slow the infusion rate of the blood. c. Discontinue the infusion immediately, and notify the prescriber. d. Tell the patient that these symptoms are a normal reaction to the blood product.

ANS: C Because of the possibility of a transfusion reaction, the infusion should be discontinued immediately and the prescriber notified. The intravenous line should be kept patent with isotonic normal saline solution infusing at a slow rate, and the health care institution's protocol for transfusion reactions should always be followed. The other options are inappropriate actions. DIF: COGNITIVE LEVEL: Applying (Application) REF: p. 477 TOP: NURSING PROCESS: Implementation

10. A patient with endometriosis is being treated with the gonadotropin-releasing hormone (GnRH) agonist leuprolide [Lupron Depot]. A nurse is teaching the patient about the drug. Which statement by the patient indicates understanding of the teaching? a. "I can continue to take the medication if I get pregnant." b. "I can expect the medication to cure my symptoms." c. "I may have menopausal-like symptoms when taking this medication." d. "I will need to take the medication for several years."

ANS: C Because the GnRH agonists deprive the ovary of the stimulation to produce estrogen, women may experience symptoms associated with menopause, including hot flashes, vaginal dryness, decreased libido, mood changes, and headache. GnRH agonists are teratogenic and should not be used during pregnancy. The medication does not produce a cure; symptoms return in up to 50% of women after discontinuation of the drug. GnRH agonists should be discontinued after 6 months of treatment because of the risk of osteoporosis. PTS: 1 DIF: Cognitive Level: Application REF: p. 774

A patient is taking a combination oral contraceptive (OC) and reports breast tenderness, edema, and occasional nausea. What will the nurse recommend? a. The patient should ask her provider about an OC with less progestin. b. The patient should discuss an alternate method of birth control. c. The patient should request an OC containing less estrogen. d. The patient should take the OC at bedtime to reduce side effects.

ANS: C Breast tenderness, edema, and nausea are associated with estrogen; women experiencing these side effects may benefit from an OC with a lower estrogen dose. Lowering the progestin will not decrease these symptoms. It is not necessary to change birth control methods if side effects can be managed by altering the estrogen dose. Taking the OC at bedtime will not affect the symptoms.

2. When administering a bulk-forming laxative, the nurse instructs the patient to drink the medication mixed in a full 8-ounce glass of water. Which statement best explains the rationale for this instruction? a. The water acts to stimulate bowel movements. b. The water will help to reduce the bulk of the intestinal contents. c. These laxatives may cause esophageal obstruction if taken with insufficient water. d. The water acts as a lubricant to produce bowel movements.

ANS: C Bulk-forming drugs increase water absorption, which results in greater total volume (bulk) of the intestinal contents. Bulk-forming laxatives tend to produce normal, formed stools. Their action is limited to the gastrointestinal tract, so there are few, if any, systemic effects. However, they need to be taken with liberal amounts of water to prevent esophageal obstruction and fecal impaction. DIF: COGNITIVE LEVEL: Applying (Application) REF: p. 820 TOP: NURSING PROCESS: Implementation

5. A patient's provider has recommended a bulk-forming laxative for occasional constipation. Which statement by the patient indicates understanding of the teaching about this agent? a. "I can take this medication long term." b. "I should not take this drug if I have diverticulitis." c. "I should take each dose with a full glass of water." d. "This drug can cause severe diarrhea."

ANS: C Bulk-forming laxatives provide insoluble substances that swell in water to both soften and increase the size of the fecal mass. Patients should be taught to take the dose with a full glass of water. Laxatives in general are not recommended for long-term use. Bulk-forming laxatives are safe for patients with diverticulitis. They are often used to treat diarrhea, because they help form the fecal mass.

6. A patient is infected by invasive aspergillosis, and the medical history reveals that the patient has not been able to tolerate several antifungal drugs. The nurse anticipates an order for which medication to treat this infection? a. Fluconazole (Diflucan) b. Micafungin (Mycamine) c. Caspofungin (Cancidas) d. Nystatin (Mycostatin)

ANS: C Caspofungin is used for treating severe infection by Aspergillus species (invasive aspergillosis) in patients who are intolerant of or refractory to other drugs. DIF: COGNITIVE LEVEL: Understanding (Comprehension) REF: p. 674 TOP: NURSING PROCESS: Planning

10. A patient is taking clonidine for hypertension and reports having dry mouth and drowsiness. What will the nurse tell the patient? a. Beta blockers can reverse these side effects. b. Discontinue the medication immediately and notify the provider. c. Drink extra fluids and avoid driving when drowsy. d. Notify the provider if symptoms persist after several weeks.

ANS: C Clonidine can cause dry mouth and sedation; patients should be warned to drink extra fluids and avoid driving. Beta blockers do not reverse these drug side effects. Discontinuing clonidine abruptly can cause severe rebound hypertension. These are common side effects that do not abate over time.

19. A 60-year-old African American patient has a blood pressure of 120/80 mm Hg and reports a family history of hypertension. The patient has a body mass index of 22.3. The patient reports consuming alcohol occasionally. Which therapeutic lifestyle change will the nurse expect to teach this patient? a. Alcohol cessation b. Potassium supplementation c. Sodium restriction d. Weight loss

ANS: C Current recommendations for African Americans to prevent hypertension include sodium restriction of less than 1500 mg/day, although this is under investigation. The patient has a normal BMI and does not consume alcohol excessively, so weight loss and alcohol cessation are not necessary. Potassium supplements are not indicated; patients should be advised to consume foods high in potassium.

9. A parent asks a nurse to recommend an intranasal decongestant for a 6-year-old child. Which response by the nurse is correct? a. "Decongestants are too sedating for children and should not be used." b. "Decongestants should not be given to children under 7 years old." c. "Decongestant drops are recommended instead of decongestant sprays." d. "Decongestant sprays should be used no longer than 5 to 10 days."

ANS: C Decongestant drops are recommended for children, because the number of drops can be controlled precisely. When sprays are used, the amount given is not well controlled. Decongestants cause CNS excitation. Decongestants may be given to children over the age of 4 years. Intranasal decongestants should not be used for longer than 5 days.

1. When giving dextromethorphan, the nurse understands that this drug suppresses the cough reflex by which mechanism of action? a. Causing depression of the central nervous system b. Anesthetizing the stretch receptors c. Having direct action on the cough center d. Decreasing the viscosity of the bronchial secretions

ANS: C Dextromethorphan suppresses the cough reflex through a direct action on the cough center. The other options are incorrect. DIF: COGNITIVE LEVEL: Understanding (Comprehension) REF: p. 572 TOP: NURSING PROCESS: Implementation

18. The nurse is teaching a group of nursing students about dofetilide [Tikosyn] to treat dysrhythmias. Which statement by a student indicates understanding of the teaching? a. "Dofetilide is a first-line medication to treat atrial flutter or atrial fibrillation." b. "Dofetilide carries a lower risk of causing torsades de pointes than other drugs." c. "Dofetilide is used for patients with severe symptoms of atrial dysrhythmias." d. "Dofetilide may be safely used as outpatient therapy to treat atrial fibrillation."

ANS: C Dofetilide is used for patients with severe, highly symptomatic atrial dysrhythmias because of its higher than usual risk of torsades de pointes. It is used only when other medications have not been effective. It is used for hospitalized patients with close ECG monitoring.

8. A patient has been taking donepezil (Aricept) for 2 weeks as part of the treatment for early stages of Alzheimer's disease. Her daughter calls the prescriber's office and is upset because "Mother has not improved one bit!" Which response by the nurse is appropriate? a. "Increase the dosage to twice daily." b. "It takes time for the cure to take effect." c. "It may take up to 6 weeks to see an improvement." d. "Take the medication on an empty stomach for improved absorption."

ANS: C Donepezil therapy is not a cure for Alzheimer's disease, but it may help to improve symptoms in the early stages. It may take up to 6 weeks to see improvement. The family should be taught that the medication must be taken exactly as ordered and with meals, and the medication should not be abruptly stopped or the dosage increased without the physician's approval because of the possibility of serious complications. DIF: COGNITIVE LEVEL: Applying (Application) REF: p. 328 TOP: NURSING PROCESS: Evaluation

2. During a routine checkup, a patient states that she is unable to take the prescribed antihistamine because of one of its most common adverse effects. The nurse suspects that which adverse effect has been bothering this patient? a. Constipation b. Abdominal cramps c. Drowsiness d. Decreased libido

ANS: C Drowsiness is usually the chief complaint of people who take antihistamines. DIF: COGNITIVE LEVEL: Understanding (Comprehension) REF: p. 567 TOP: NURSING PROCESS: Assessment

11. A patient calls the clinic to speak to the nurse about taking an herbal product that contains ginkgo (Ginkgo biloba) to "help my memory." He states that he has read much information about the herbal product. Which statement by the patient indicates a need for further education? a. "I know the FDA has not approved this herbal product, but I'd like to try it to see if it helps my memory." b. "I need to watch for possible side effects, such as headaches, or stomach or intestinal upset." c. "I will take aspirin or ibuprofen (Motrin) if I have a headache." d. "Ginkgo may cause increased bleeding, so I'll have to be careful when doing yard work."

ANS: C Drug interactions may occur between the taking of aspirin and nonsteroidal anti-inflammatory drugs and the taking of ginkgo. The other options are incorrect. DIF: COGNITIVE LEVEL: Analyzing (Analysis) REF: p. 326 TOP: NURSING PROCESS: Evaluation

3. A gardener needs a decongestant because of seasonal allergy problems and asks the nurse whether he should take an oral form or a nasal spray. Which of these is a benefit of orally administered decongestants? a. Immediate onset b. A more potent effect c. Lack of rebound congestion d. Shorter duration

ANS: C Drugs administered by the oral route produce prolonged decongestant effects, but the onset of action is more delayed and the effect less potent than those of decongestants applied topically. However, the clinical problem of rebound congestion associated with topically administered drugs is almost nonexistent with oral dosage forms. DIF: COGNITIVE LEVEL: Understanding (Comprehension) REF: p. 570 TOP: NURSING PROCESS: Planning

5. A patient who has received chemotherapy has a steadily decreasing white blood cell count. The chemotherapy will end on Tuesday afternoon. The oncologist has mentioned that a colony-stimulating factor will be started soon. The nurse knows that the appropriate time to start this medication is when? a. While the patient is still receiving chemotherapy b. Two hours after the chemotherapy ends c. Wednesday afternoon, 24 hours after the chemotherapy ends d. In 2 to 4 days, after the white blood cells have reached their nadir

ANS: C Drugs that are given to enhance the activity of bone marrow cells interfere directly with the action of myelosuppressive cancer therapy. For this reason, therapy with colony-stimulating factors usually begins 24 hours after the chemotherapy has been completed. DIF: COGNITIVE LEVEL: Applying (Application) REF: p. 752 TOP: NURSING PROCESS: Planning

2. When teaching about hypoglycemia, the nurse will make sure that the patient is aware of the early signs of hypoglycemia, including: a. hypothermia and seizures. b. nausea and diarrhea. c. confusion and sweating. d. fruity, acetone odor to the breath.

ANS: C Early symptoms of hypoglycemia include the central nervous system manifestations of confusion, irritability, tremor, and sweating. Hypothermia and seizures are later symptoms of hypoglycemia. The other options are incorrect. DIF: COGNITIVE LEVEL: Understanding (Comprehension) REF: p. 514 TOP: NURSING PROCESS: Implementation

5. A patient has been instructed to take one enteric-coated low-dose aspirin a day as part of therapy to prevent strokes. The nurse will provide which instruction when providing patient teaching about this medication? a. Aspirin needs to be taken on an empty stomach to ensure maximal absorption. b. Low-dose aspirin therapy rarely causes problems with bleeding. c. Take the medication with 6 to 8 ounces of water and with food. d. Coated tablets may be crushed if necessary for easier swallowing.

ANS: C Enteric-coated aspirin is best taken with 6 to 8 ounces of water and with food to help decrease gastrointestinal upset. Enteric-coated tablets should not be crushed. Risk for bleeding increases with aspirin therapy, even at low doses. DIF: COGNITIVE LEVEL: Applying (Application) REF: p. 431 TOP: NURSING PROCESS: Implementation

14. An elderly patient who has type 2 diabetes has a history of severe hypoglycemia. The patient's spouse asks the nurse what the optimum A1c level is for the patient. Which is correct? a. Between 6.5 and 7.0 b. Below 7.0 c. Below 8.0 d. Between 7.0 and 8.5

ANS: C For patients with a history of severe hypoglycemia and those with a limited life expectancy or advanced micro- and macrovascular complications, the target A1c level should be below 8.0. For most other patients with diabetes, the target is 7.0 and below.

11. Insulin glargine is prescribed for a hospitalized patient who has diabetes. When will the nurse expect to administer this drug? a. Approximately 15 to 30 minutes before each meal b. In the morning and at 4:00 PM c. Once daily at bedtime d. After meals and at bedtime

ANS: C Glargine insulin is indicated for once daily subcutaneous administration to treat adults and children with type 1 diabetes and adults with type 2 diabetes. According to the package labeling, the once-daily injection should be given at bedtime. Glargine insulin should not be given more than once a day, although some patients require bid dosing to achieve a full 24 hours of basal coverage.

A nurse is providing teaching for a nondiabetic adult who develops growth hormone deficiency and who will begin treatment with somatropin [Humatrope]. Which statement by the patient indicates understanding of the teaching? a."Intramuscular dosing is more effective than subcutaneous dosing." b."I will have increased muscle mass and strength as well as increased height." c."I will need to monitor my blood pressure frequently while taking this drug." d."I will need to take insulin while using this, because it causes hyperglycemia."

ANS: C Growth hormone in adults causes an increase in systolic blood pressure, so patients should be taught to monitor blood pressure while taking the drug. Subcutaneous dosing is as effective as IM dosing and is preferred, because it is less painful. Although muscle mass will increase, strength and height will not. Growth hormone is diabetogenic but causes significant problems in patients with preexisting diabetes

7. The nurse is reviewing instructions for a patient with type 2 diabetes who also takes insulin injections as part of the therapy. The nurse asks the patient, "What should you do if your fasting blood glucose is 47 mg/dL?" Which response by the patient reflects a correct understanding of insulin therapy? a. "I will call my doctor right away." b. "I will give myself the regular insulin." c. "I will take an oral form of glucose." d. "I will rest until the symptoms pass."

ANS: C Hypoglycemia can be reversed if the patient eats glucose tablets or gel, corn syrup, or honey, or drinks fruit juice or a nondiet soft drink or other quick sources of glucose, which must always be kept at hand. She should not wait for instructions from her physician, nor delay taking the glucose by resting. The regular insulin would only lower her blood glucose levels more. DIF: COGNITIVE LEVEL: Applying (Application) REF: p. 518 TOP: NURSING PROCESS: Implementation

4. A patient is receiving intravenous promethazine [Phenergan] 25 mg for postoperative nausea and vomiting. What is an important nursing action when giving this drug? a. Giving the dose as an IV push over 3 to 5 minutes b. Infusing the dose with microbore tubing and an infusion pump c. Observing the IV insertion site frequently for patency d. Telling the patient to report dry mouth and sedation

ANS: C If IV administration must be done with this drug, it should be given through a large-bore, freely flowing line. The site should be monitored closely for local burning or pain or any sign of extravasation, which can cause abscess formation, tissue necrosis, and gangrene requiring amputation. Giving the medication as a rapid IV push or through microbore tubing does not adequately slow the infusion or dilute the drug. Dry mouth and sedation are expected side effects of this drug and are not dangerous.

4. What is the nurse's priority action if extravasation of an antineoplastic drug occurs during intravenous (IV) administration? a. Reduce the infusion rate. b. Discontinue the IV, and apply warm compresses. c. Stop the infusion immediately, but leave the IV catheter in place. d. Change the infusion to normal saline, and inject the area with hydrocortisone.

ANS: C If extravasation is suspected, administration of the drug must be stopped immediately but the IV catheter left in place and the appropriate antidote instilled through the existing IV tube, after which the needle may be removed. The other options are incorrect. DIF: COGNITIVE LEVEL: Applying (Application) REF: p. 736 TOP: NURSING PROCESS: Implementation

8. A patient is receiving irinotecan (Camptosar), along with other antineoplastic drugs, as treatment for ovarian cancer. The nurse will monitor for which potentially life-threatening adverse effect that is associated with this drug? a. Severe stomatitis b. Bone marrow suppression c. Delayed-onset cholinergic diarrhea d. Immediate and severe nausea and vomiting

ANS: C In addition to producing hematologic adverse effects, irinotecan has been associated with severe diarrhea, known as cholinergic diarrhea, which may occur during infusions. Delayed diarrhea may occur 2 to 10 days after infusion of irinotecan. It is recommended that this condition be treated with atropine unless use of that drug is strongly contraindicated. This diarrhea can be severe and even life threatening. DIF: COGNITIVE LEVEL: Understanding (Comprehension) REF: p. 723 TOP: NURSING PROCESS: Evaluation

11. The nurse counseling a couple who cannot conceive learns that female infertility problems have been ruled out. What will the nurse expect to tell this couple? a. "If medication is necessary for male infertility, treatment will last a few months." b. "Male infertility is usually treatable with a combination of medications." c. "Most male infertility is not linked to known endocrine disorders." d. "Treatment with sildenafil [Viagra] will be necessary to improve fertility."

ANS: C In most cases, infertility in males in not associated with an identifiable endocrine disorder, and with the exception of infertility associated with erectile dysfunction, male infertility is generally unresponsive to drugs. In rare cases, failure to produce sperm is related to insufficient gonadotropin secretion and hCG may be used, but therapy is prolonged and may be required for up to 3 to 4 years. Treatment with sildenafil is necessary only when ED has been identified. PTS: 1 DIF: Cognitive Level: Analysis REF: p. 769

4. A patient who has had abdominal surgery has been discharged on a cholinergic drug to assist in increasing gastrointestinal peristalsis. The nurse will teach this patient to look for which therapeutic effect? a. Decreased pulse rate b. Abdominal cramping c. Passage of flatus d. Decreased urge to void

ANS: C In patients suffering a decrease in GI peristalsis postoperatively, taking a cholinergic drug should result in an increase in bowel sounds, the passage of flatus, and the occurrence of bowel movements that indicate increased GI peristalsis. DIF: COGNITIVE LEVEL: Applying (Application) REF: p. 328 TOP: NURSING PROCESS: Evaluation

6. A factory worker has been admitted to the emergency department after an industrial accident involving organophosphate insecticides. The nurse will prepare to administer which drug? a. Pilocarpine (Salagen) b. Bethanechol (Urecholine) c. Physostigmine (Antilirium) d. Tacrine (Cognex)

ANS: C Indirect-acting drugs such as physostigmine inhibit acetylcholinesterase, thus reversing the neuromuscular blockade produced by anticholinergic poisoning as well as poisoning by irreversible cholinesterase inhibitors such as the organophosphates and carbamates, common classes of insecticides. The other drugs listed are not appropriate antidotes. DIF: COGNITIVE LEVEL: Understanding (Comprehension) REF: p. 323 TOP: NURSING PROCESS: Planning

11. A patient in the emergency department was showing signs of hypoglycemia and had a fingerstick glucose level of 34 mg/dL. The patient has just become unconscious. What is the nurse's next action? a. Have the patient eat glucose tablets. b. Have the patient consume fruit juice, a nondiet soft drink, or crackers. c. Administer intravenous glucose (50% dextrose). d. Call the lab to order a fasting blood glucose level.

ANS: C Intravenous glucose raises blood glucose levels when the patient is unconscious and unable to take oral forms of glucose. DIF: COGNITIVE LEVEL: Analyzing (Analysis) REF: p. 514 TOP: NURSING PROCESS: Implementation

12. A patient with a history of chronic alcohol abuse has been admitted to the unit with cirrhosis. Upon review of the patient's laboratory test results, the nurse notes that the patient's ammonia level is elevated at 218 mcg/dL. What medication should the nurse prepare to administer? a. 0.9% NS b. Docusate sodium [Colace] c. Lactulose d. Polyethylene glycol [MiraLax]

ANS: C Lactulose is the only laxative known to lower ammonia levels in patients with portal hypertension and hepatic encephalopathy secondary to liver disease. No information suggests that the patient needs fluid or electrolyte replacement. Docusate sodium and polyethylene glycol are not effective at lowering ammonia levels.

7. A 6-year-old child has frequent constipation. The nurse provides teaching after the parent asks the nurse why the provider recommended using laxatives only when needed. Which statement by the parent indicates a need for further teaching? a. "Children who take laxatives regularly can become dehydrated." b. "Chronic laxative use can cause electrolyte imbalances." c. "Frequent use of laxatives can cause diverticulitis." d. "The normal reflex to defecate can be inhibited with overuse of laxatives."

ANS: C Laxatives do not cause diverticulitis, although some laxatives can aggravate this condition. Chronic use of laxatives can cause dehydration and electrolyte imbalances and can suppress the normal defecation reflex.

5. While recovering from surgery, a 74-year-old woman started taking a stimulant laxative, senna (Senokot), to relieve constipation caused by the pain medications. Two weeks later, at her follow-up appointment, she tells the nurse that she likes how "regular" her bowel movements are now that she is taking the laxative. Which teaching principle is appropriate for this patient? a. She needs to be sure to take this medication with plenty of fluids. b. It is important to have a daily bowel movement to promote bowel health. c. Long-term use of laxatives often results in decreased bowel tone and may lead to dependency. d. She needs to switch to glycerin suppositories to continue having daily bowel movements.

ANS: C Long-term use of laxatives may lead to dependency. Patients need to be taught that daily bowel movements are not necessary for bowel health. DIF: COGNITIVE LEVEL: Applying (Application) REF: p. 819 TOP: NURSING PROCESS: Implementation

5. A patient who has been on methotrexate therapy is experiencing mild pain. The patient is asking for aspirin for the pain. The nurse recognizes that which of these is true in this situation? a. The aspirin will aggravate diarrhea. b. The aspirin will masks signs of infection. c. Aspirin can lead to methotrexate toxicity. d. The aspirin will cause no problems for the patient on methotrexate.

ANS: C Methotrexate interacts with weak organic acids, such as aspirin, and can lead to toxicity by displacing the methotrexate from protein-binding sites. DIF: COGNITIVE LEVEL: Applying (Application) REF: p. 720 TOP: NURSING PROCESS: Implementation

8. A patient with diabetes develops ventricular tachycardia and is in the hospital for evaluation of this condition. The nurse reviews the history and learns that the patient takes mexiletine [Mexitil] for pain caused by peripheral neuropathy. What should the nurse do? a. Discuss common side effects associated with taking mexiletine with cardiac agents. b. Understand that this drug will help with both peripheral neuropathy and dysrhythmias. c. Notify the provider to request that another drug be used for peripheral neuropathy pain. d. Request an order for renal function and hepatic function tests.

ANS: C Mexiletine is an antidysrhythmic medication that can also cause dysrhythmias. It is used to treat the pain associated with peripheral neuropathy in diabetic patients, but it is contraindicated in diabetic patients with heart disease, and so it should be stopped now that this patient has developed a heart disorder. Because it is contraindicated, the nurse will not teach the patient about side effects with other agents. It can exacerbate cardiac symptoms, so it should not be used to treat dysrhythmias in diabetic patients. There is no indication for tests of renal and hepatic function.

A nurse is discussing various ways to obtain a medical abortion with a patient. Which statement by the patient best demonstrates understanding of mifepristone (RU 486) [Mifeprex]? "This drug is most effective if I use it: a. before the first missed menstrual period." b. the day after unprotected intercourse." c. within 7 weeks of conception." d. immediately after ovulation."

ANS: C Mifepristone is most effective if it is used within 7 weeks of conception. The timing specified in the other responses is incorrect.

3. A patient calls the clinic office saying that the cholestyramine (Questran) powder he started yesterday clumps and sticks to the glass when he tries to mix it. The nurse will suggest what method for mixing this medication for administration? a. Mix the powder in a carbonated soda drink to dissolve it faster. b. Add the powder to any liquid, and stir vigorously to dissolve it quickly. c. Mix the powder with food or fruit, or at least 4 to 6 ounces of fluid. d. Sprinkle the powder into a spoon and take it dry, followed by a glass of water.

ANS: C Mix the powder with food or at least 4 to 6 ounces of fluid. The powder may not mix completely at first, but patients should be sure to mix the dose as much as possible and then dilute any undissolved portion with additional fluid. The powder should be dissolved for at least 1 full minute. Powder and granule dosages are never to be taken in dry form. DIF: COGNITIVE LEVEL: Applying (Application) REF: p. 447 TOP: NURSING PROCESS: Implementation

2. A patient asks about his cancer treatment with monoclonal antibodies. The nurse tells him that which is the major advantage of treating certain cancers with monoclonal antibodies? a. They will help the patient improve more quickly than will other antineoplastic drugs. b. They are more effective against metastatic tumors. c. Monoclonal antibodies target certain tumor cells and bypass normal cells. d. There are fewer incidences of opportunistic infections with monoclonal antibodies.

ANS: C Monoclonal antibodies can target cancer cells specifically and have minimal effects on healthy cells, unlike conventional cancer treatments. As a result, there are fewer adverse effects when compared to traditional antineoplastic therapy. The other options are incorrect. DIF: COGNITIVE LEVEL: Understanding (Comprehension) REF: p. 755 TOP: NURSING PROCESS: Planning

2. The nurse is reviewing principles of immunization. What type of immunization occurs when antibodies pass from mother to infant during breastfeeding or through the placenta during pregnancy? a. Artificial active immunization b. Attenuating immunization c. Natural passive immunization d. Artificial passive immunization

ANS: C Natural passive immunization occurs when antibodies are transferred from the mother to her infant in breast milk or through the bloodstream via the placenta during pregnancy. Artificial active immunization causes an antigen-antibody response and stimulates the body's defenses to resist any subsequent exposures. Passive immunization is conferred by bypassing the host's immune system and injecting the person with antiserum or concentrated antibodies obtained from other humans or animals; this gives the host direct means of fighting off an invading microorganism. The host's immune system therefore does not have to manufacture these antibodies. DIF: COGNITIVE LEVEL: Understanding (Comprehension) REF: p. 784 TOP: NURSING PROCESS: Planning

4. The nurse is administering one of the lipid formulations of amphotericin B. When giving this drug, which concept is important to remember? a. The lipid formulations may be given in oral form. b. The doses are much lower than the doses of the older drugs. c. The lipid formulations are associated with fewer adverse effects than the older drugs. d. There is no difference in cost between the newer and older forms.

ANS: C Newer lipid formulations of amphotericin B have been developed in an attempt to decrease the incidence of its adverse effects and increase its efficacy. However, the lipid formulations are more costly. DIF: COGNITIVE LEVEL: Understanding (Comprehension) REF: p. 673 TOP: NURSING PROCESS: Implementation

7. Niacin is prescribed for a patient who has hyperlipidemia. The nurse checks the patient's medical history, knowing that this medication is contraindicated in which disorder? a. Renal disease b. Cardiac disease c. Liver disease d. Diabetes mellitus

ANS: C Niacin, unlike certain other B-complex vitamins, has additional contraindications besides drug allergy. They include liver disease, severe hypotension, arterial hemorrhage, and active peptic ulcer disease. The other options are incorrect. DIF: COGNITIVE LEVEL: Applying (Application) REF: p. 849 TOP: NURSING PROCESS: Assessment

9. A patient with hyperlipidemia has been told by the provider to take extra niacin. The nurse will tell the patient to: a. increase servings of poultry, fish, and cereals. b. take nicotinamide supplements. c. take nicotinic acid supplements. d. take tryptophan supplements.

ANS: C Nicotinic acid is given in high doses to treat hyperlipidemia. Increasing dietary niacin is not sufficient to treat hyperlipidemia. Nicotinamide does not affect plasma lipoproteins. Dietary tryptophan is converted by the body into nicotinic acid, but not in sufficient amounts to treat hyperlipidemia. PTS: 1 DIF: Cognitive Level: Application REF: pp. 992-993

15. A patient with Crohn's disease will begin receiving an initial infusion of infliximab [Remicade]. The nurse explains how this drug works to treat this disease. Which statement by the patient indicates a need for further teaching? a. "I may have an increased risk of infections, such as tuberculosis, when taking infliximab." b. "I should report chills, fever, itching, and shortness of breath while receiving the infusion." c. "This drug sometimes provides a complete cure of inflammatory bowel disease." d. "I will take the second dose in 2 weeks, the third dose in 6 weeks, and then a dose every 8 weeks thereafter."

ANS: C None of the drugs used to treat obstructive bowel disease (OBD) are curative. Patients taking immunomodulators, such as infliximab, have an increased risk of infection, especially opportunistic infections such as TB. Infusion reactions may occur and include chills, fever, itching, and shortness of breath. The induction regimen is 5 mg/kg infused at 0, 2, and 6 weeks, followed by a maintenance regimen every 8 weeks.

9. During chemotherapy, a patient develops severe diarrhea caused by a vasoactive intestinal peptide-secreting tumor (VIPoma). The nurse expects to administer which drug for this problem? a. Dexrazoxane (Zinecard) b. Allopurinol (Zyloprim) c. Octreotide (Sandostatin) d. Bismuth subsalicylate (Pepto-Bismol)

ANS: C Octreotide (Sandostatin) is used for the management of a cancer-related condition called carcinoid crisis and treatment of the severe diarrhea caused by vasoactive intestinal peptide-secreting tumors (VIPomas). The other options are incorrect. DIF: COGNITIVE LEVEL: Applying (Application) REF: p. 740 TOP: NURSING PROCESS: Planning

1. A nurse caring for a patient who is undergoing a third round of chemotherapy is preparing to administer ondansetron [Zofran] 30 minutes before initiation of the chemotherapy. The patient tells the nurse that the ondansetron did not work as well the last time as it had the first time. What will the nurse do? a. Administer the ondansetron at the same time as the chemotherapy. b. Contact the provider to suggest using high-dose intravenous dolasetron [Anzemet]. c. Request an order to administer dexamethasone with the ondansetron. d. Suggest to the provider that loperamide [Lomotil] be given with the ondansetron.

ANS: C Ondansetron is a serotonin receptor antagonist; drugs in this class are the most effective drugs available for suppressing nausea and vomiting associated with anticancer drugs. The drug is even more effective when combined with dexamethasone. For best effect, ondansetron should be given 30 minutes before beginning chemotherapy. Dolasetron is similar to ondansetron, but when given intravenously in high doses, it is associated with fatal dysrhythmias. Loperamide is used to treat diarrhea.

1. A patient is taking enalapril [Vasotec]. The nurse understands that patients taking this type of drug for heart failure need to be monitored carefully for: a. hypernatremia. b. hypertension. c. hyperkalemia. d. hypokalemia.

ANS: C One of the principal effects of angiotensin-converting enzyme (ACE) inhibitors is hyperkalemia, which is due to decreased aldosterone release arising from blockage of angiotensin II. There is no indication that careful monitoring of sodium for increased levels is indicated. Vasotec is indicated for heart failure, not hypertension. The drug therapy should be monitored to ascertain its effectiveness, but hyperkalemia is the main concern. Hyperkalemia, not hypokalemia, is a concern because of the decreased aldosterone release that occurs with blockage of angiotensin II.

4. A patient who has received chemotherapy has a critically low platelet count. The nurse expects which drug or drug class to be used to stimulate platelet cell production? a. Filgrastim (Neupogen) b. Interferons c. Oprelvekin (Neumega) d. Epoetin alfa (Epogen)

ANS: C Oprelvekin (Neumega) stimulates bone marrow cells, specifically megakaryocytes, which eventually form platelets. The other options are incorrect. DIF: COGNITIVE LEVEL: Applying (Application) REF: p. 753 TOP: NURSING PROCESS: Planning

7. The nurse checks the patient's laboratory work prior to administering a dose of vancomycin (Vancocin) and finds that the trough vancomycin level is 24 mcg/mL. What will the nurse do next? a. Administer the vancomycin as ordered. b. Hold the drug, and administer 4 hours later. c. Hold the drug, and notify the prescriber. d. Repeat the test to verify results.

ANS: C Optimal blood levels of vancomycin are a trough level of 10 to 20 mcg/mL. Measurement of peak levels is no longer routinely recommended, and only trough levels are commonly monitored. Blood samples for measurement of trough levels are drawn immediately before administration of the next dose. Because of the increase in resistant organisms, many clinicians use a trough level of 15 to 20 mcg/mL as their goal. These trough levels mean that even just before the next dose is due, when drug levels should be low, the drug levels are actually too high. DIF: COGNITIVE LEVEL: Analyzing (Analysis) REF: p. 633 TOP: NURSING PROCESS: Evaluation

7. A patient will begin taking iron supplements to treat anemia. The nurse will recommend that the patient take the iron with which food to facilitate absorption? a. Cereal b. Dairy products c. Orange juice d. Red meats

ANS: C Orange juice is a good source of vitamin C, and vitamin C facilitates the absorption of iron. Cereals often are fortified with iron but do not facilitate its absorption. Calcium interferes with the absorption of iron. Red meats are a natural source of iron. PTS: 1 DIF: Cognitive Level: Analysis REF: p. 992

11. A patient who is severely anemic also has acute heart failure with severe edema due to fluid overload. The prescriber wants to raise the patient's hemoglobin and hematocrit levels. The nurse anticipates that the patient will receive which blood product? a. Fresh frozen plasma b. Albumin c. Packed red blood cells (PRBCs) d. Whole blood

ANS: C PRBCs are given to increase the oxygen-carrying capacity in a patient with anemia, in a patient with substantial hemoglobin deficits, and in a patient who has lost up to 25% of total blood volume. A patient with a coagulation disorder or a clotting-factor deficiency would receive fresh frozen plasma; a patient who has lost a massive amount of blood would receive whole blood. DIF: COGNITIVE LEVEL: Applying (Application) REF: p. 469 TOP: NURSING PROCESS: Planning

7. A patient is taking gentamicin [Garamycin] and furosemide [Lasix]. The nurse should counsel this patient to report which symptom? a. Frequent nocturia b. Headaches c. Ringing in the ears d. Urinary retention

ANS: C Patients taking furosemide should be advised that the risk of furosemide-induced hearing loss can be increased when other ototoxic drugs, such as gentamicin, are also taken. Patients should be told to report tinnitus, dizziness, or hearing loss. Nocturia may be an expected effect of furosemide. Headaches are not likely to occur with concomitant use of gentamicin and furosemide. Urinary retention is not an expected side effect.

1. The nurse is providing teaching for a patient who is to receive estrogen replacement therapy. Which statement is correct to include in the teaching session? a. "If you miss a dose, double-up on the next dose." b. "There's no need to be concerned about breast lumps or bumps that occur." c. "Be sure to report any weight gain of 5 pounds or more per week." d. "Take the medication on an empty stomach to enhance absorption."

ANS: C Patients taking oral estrogen therapy should report weight gain of 5 pounds or more per week to a physician. The other statements are not true for estrogen replacement therapy. DIF: COGNITIVE LEVEL: Applying (Application) REF: p. 550 TOP: NURSING PROCESS: Implementation

3. A patient with renal disease is scheduled for a colonoscopy. Before the procedure, the nurse will anticipate administering: a. glycerin suppository. b. magnesium hydroxide (MOM). c. polyethylene glycol and electrolytes. d. sodium phosphate.

ANS: C Polyethylene glycol (PEG) plus electrolytes (ELS) is one of two bowel cleansers used before colonoscopy to clear the bowel. PEG-ELS products are preferred, because unlike sodium phosphate, they are isotonic and do not increase the likelihood of dehydration and electrolyte imbalance. Glycerin suppositories and magnesium hydroxide are not used for bowel cleansing.

9. The nurse administering the phosphodiesterase inhibitor milrinone (Primacor) recognizes that this drug will have a positive inotropic effect. Which result reflects this effect? a. Increased heart rate b. Increased blood vessel dilation c. Increased force of cardiac contractions d. Increased conduction of electrical impulses across the heart

ANS: C Positive inotropic drugs increase myocardial contractility, thus increasing the force of cardiac conduction. Positive chronotropic drugs increase the heart rate. Positive dromotropic drugs increase the conduction of electrical impulses across the heart. Blood vessel dilation is not affected. DIF: COGNITIVE LEVEL: Applying (Application) REF: p. 377 TOP: NURSING PROCESS: Assessment

10. A patient has a diagnosis of primary hypothyroidism. Which statement accurately describes this problem? a. The hypothalamus is not secreting thyrotropin-releasing hormone (TRH); therefore, thyroid-stimulating hormone (TSH) is not released from the pituitary gland. b. The pituitary gland is dysfunctional and is not secreting TSH. c. The abnormality is in the thyroid gland itself. d. The abnormality is caused by an insufficient intake of iodine.

ANS: C Primary hypothyroidism stems from an abnormality in the thyroid gland itself and occurs when the thyroid gland is not able to perform one of its many functions. Secondary hypothyroidism begins at the level of the pituitary gland and results from reduced secretion of TSH. TSH is needed to trigger the release of the T3 and T4 stored in the thyroid gland. Tertiary hypothyroidism is caused by a reduced level of the TRH from the hypothalamus. This reduced level, in turn, reduces TSH and thyroid hormone levels. DIF: COGNITIVE LEVEL: Understanding (Comprehension) REF: p. 492 TOP: NURSING PROCESS: Assessment

15. A patient is taking a thiazide diuretic for hypertension and quinidine to treat a dysrhythmia. The prescriber orders digoxin 0.125 mg to improve this patient's cardiac output. The nurse should contact the provider to request: a. adding spironolactone [Aldactone]. b. reducing the dose of digoxin. c. discontinuing the quinidine. d. giving potassium supplements.

ANS: C Quinidine can cause plasma levels of digoxin to rise; concurrent use of quinidine and digoxin is contraindicated. There is no indication for adding spironolactone unless this patient's potassium level is elevated. The dose of digoxin ordered is a low dose. Potassium supplements are contraindicated with digoxin.

6. When hanging a new infusion bag of a chemotherapy drug, the nurse accidentally spills a small amount of the solution onto the floor. Which action by the nurse is appropriate? a. Let it dry, and then mop the floor. b. Wipe the area with a disposable paper towel. c. Use a spill kit to clean the area. d. Ask the housekeeping department to clean the floor.

ANS: C Special spill kits are employed to clean up even the smallest chemotherapy spills. These precautions are necessary to protect the health care provider from the cytotoxic effects of these drugs. DIF: COGNITIVE LEVEL: Applying (Application) REF: p. 741 TOP: NURSING PROCESS: Implementation

5. A sanitation worker has experienced a needle stick by a contaminated needle that was placed in a trash can. The employee health nurse expects that which drug will be used to provide passive immunity to hepatitis B infection? a. Haemophilus influenzae type b (Hib) b. Varicella virus vaccine (Varivax) c. Hepatitis B immunoglobulin (BayHep B) d. Hepatitis B virus vaccine (inactivated) (Recombivax HB)

ANS: C Recombivax HB promotes active immunity to hepatitis B infection in people who are considered to be at high risk for potential exposure to the virus, whereas hepatitis B immunoglobulin provides passive immunity for the prophylaxis and postexposure treatment of people exposed to hepatitis B virus or HBs-Ag-positive materials, such as blood, plasma, or serum. Hib and Varivax vaccines are not appropriate for this situation. DIF: COGNITIVE LEVEL: Applying (Application) REF: p. 791 TOP: NURSING PROCESS: Implementation

4. A nurse is preparing to assist a nursing student to administer intravenous verapamil to a patient who also receives a beta blocker. The nurse asks the nursing student to discuss the plan of care for this patient. Which statement by the student indicates a need for further teaching? a. "I will check to see when the last dose of the beta blocker was given." b. "I will monitor vital signs closely to assess for hypotension." c. "I will monitor the heart rate frequently to assess for reflex tachycardia." d. "I will prepare to administer intravenous norepinephrine if necessary."

ANS: C Reflex tachycardia is not an expected effect; the greater risks are cardiosuppression and bradycardia. Because beta blockers and verapamil have the same effects on the heart, there is a risk of excessive cardiosuppression. To minimize this risk, the two drugs should be given several hours apart. Hypotension may occur and should be treated with IV norepinephrine.

A nurse is caring for a child whose respirations are shallow and marked by a prolonged expiratory phase. The nurse auscultates wheezes and poor air movement bilaterally. The child's respiratory rate is 26 breaths per minute, and the oxygen saturation is 89%. What does the nurse suspect? a.Metabolic acidosis b. Metabolic alkalosis c. Respiratory acidosis d. Respiratory alkalosis

ANS: C Respiratory acidosis results from retention of CO2 secondary to hypoventilation caused either by depression of the medullary respiratory center in the central nervous system (CNS) or by a pathologic lung condition, such as asthma. This child has shallow respirations with evidence of airway obstruction and poor ventilation. Metabolic acidosis occurs in chronic renal failure, loss of bicarbonate with severe diarrhea, metabolic disorders, and ingestion of acids such as salicylates; hyperventilation is a compensatory mechanism. Metabolic alkalosis is usually caused by excessive loss of gastric acid or by ingestion of alkalinizing salts; hypoventilation is a compensatory mechanism. Respiratory alkalosis is produced by hyperventilation and can occur in any condition that increases the respiratory rate and depth.

11. Which B vitamin deficiency is associated with cheilosis, glossitis, vascularization of the cornea, and itchy dermatitis of the scrotum and vulva? a. Niacin (nicotinic acid) b. Pyridoxine (vitamin B6) c. Riboflavin (vitamin B2) d. Thiamine (vitamin B1)

ANS: C Riboflavin deficiency produces the symptoms described and can be treated with riboflavin supplements. These signs do not indicate deficiencies of niacin, pyridoxine, or thiamine. PTS: 1 DIF: Cognitive Level: Application REF: p. 993

14. A patient is severely constipated and needs immediate relief. The nurse knows that which class of laxative will provide the most rapid results? a. Bulk-forming laxative, such as psyllium (Metamucil) b. Stool softener, such as docusate salts (Colace) c. Magnesium hydroxide (MOM) d. Magnesium oxide tablets

ANS: C Saline laxatives such as magnesium hydroxide (MOM) produce a watery stool, usually within 3 to 6 hours of ingestion. Bulk-forming laxatives such as psyllium do not produce a bowel movement rapidly. Stool softeners such as docusate salts do not cause patients to defecate; they simply soften the stool to ease its passage. Magnesium oxide tablets are used as magnesium supplements, not as laxatives. DIF: COGNITIVE LEVEL: Understanding (Comprehension) REF: p. 822 TOP: NURSING PROCESS: Planning

6. A patient with second-degree burns is treated with silver sulfadiazine (Silvadene). A nursing student asks the nurse about the differences between silver sulfadiazine and mafenide (Sulfamylon), because the two are similar products, and both contain sulfonamides. What does the nurse tell the student about silver sulfadiazine? a. It causes increased pain when the medication is applied. b. It has a broader spectrum of antimicrobial sensitivity. c. It has antibacterial effects related to release of free silver. d. It suppresses renal excretion of acid, causing acidosis.

ANS: C Silver sulfadiazine has antibacterial effects primarily related to the release of free silver and not to the sulfonamide portion of the molecule. Unlike mafenide, silver sulfadiazine reduces pain when applied. Silver sulfadiazine and mafenide have similar antibacterial effects. Because silver sulfadiazine does not suppress renal excretion of acid, it has fewer systemic effects.

9. A nursing student asks a nurse why a patient in hypertensive crisis is receiving both intravenous sodium nitroprusside [Nitropress] and oral hydralazine. The nurse will explain that this is done to prevent: a. cyanide poisoning. b. fluid retention. c. rebound hypertension. d. reflex tachycardia.

ANS: C Sodium nitroprusside acts rapidly and is given only intravenously. Rebound hypertension occurs immediately when the IV is stopped, so an oral antihypertensive should be given simultaneously. Cyanide poisoning can occur with sodium nitroprusside, but giving hydralazine does not counter this adverse effect. Hydralazine does not prevent fluid retention or reflex tachycardia.

2. A 16-year-old boy who is taking somatropin comes into the office because he had an asthma attack during a race at school. Because of this new development, the nurse expects which intervention to occur next? a. He will need to stop participating in school physical education classes. b. The somatropin must be discontinued immediately. c. The somatropin dosage may be adjusted. d. His growth will be documented and monitored for changes.

ANS: C Somatropin is to be used with caution in acute or chronic illnesses, such as migraine headaches, epilepsy, and asthma. It will not have to be immediately discontinued but will require close monitoring. The patient's growth will be measured and documented throughout therapy with somatropin. DIF: COGNITIVE LEVEL: Applying (Application) REF: p. 485 TOP: NURSING PROCESS: Implementation

3. A patient has 2+ pitting edema of the lower extremities bilaterally. Auscultation of the lungs reveals crackles bilaterally, and the serum potassium level is 6 mEq/L. Which diuretic agent ordered by the prescriber should the nurse question? a. Bumetanide [Bumex] b. Furosemide [Lasix] c. Spironolactone [Aldactone] d. Hydrochlorothiazide [HydroDIURIL]

ANS: C Spironolactone is a non-potassium-wasting diuretic; therefore, if the patient has a serum potassium level of 6 mEq/L, indicating hyperkalemia, an order for this drug should be questioned. Bumetanide, furosemide, and hydrochlorothiazide are potassium-wasting diuretics and would be appropriate to administer in a patient with hyperkalemia.

7. A patient has been taking chlorthalidone to treat hypertension. The patient's prescriber has just ordered the addition of spironolactone to the patient's drug regimen. Which statement by the patient indicates a need for further teaching? a. "I should continue following the DASH diet when adding this drug." b. "I should not take an ACE inhibitor when adding this drug." c. "I will need to take potassium supplements when adding this drug." d. "I will not experience a significant increase in diuresis when adding this drug."

ANS: C Spironolactone is given in addition to thiazide diuretics to balance potassium loss caused by the thiazide diuretic. Patients should be advised against taking potassium supplements with spironolactone, because hyperkalemia can result. The DASH diet may be continued. ACE inhibitors are contraindicated because they promote hyperkalemia. Spironolactone does not significantly increase diuresis.

2. A patient who is taking immunosuppressants develops a urinary tract infection. The causative organism is sensitive to sulfonamides and to another, more expensive antibiotic. The prescriber orders the more expensive antibiotic. The nursing student assigned to this patient asks the nurse why the more expensive antibiotic is being used. Which response by the nurse is correct? a. "Immunosuppressed patients are folate deficient." b. "Patients who are immunosuppressed are more likely to develop resistance." c. "Sulfonamides are bacteriostatic and depend on host immunity to work." d. "Sulfonamides intensify the effects of immunosuppression."

ANS: C Sulfonamides are usually bacteriostatic and require intact host defenses for complete elimination of infection. Immunosuppressed patients are not necessarily folate deficient. There is no increased likelihood of developing bacterial resistance in immunosuppressed patients. Sulfonamides do no affect immunosuppression.

5. A patient with type 2 diabetes mellitus takes glipizide. The patient develops a urinary tract infection, and the prescriber orders TMP/SMZ. What will the nurse tell the patient? a. Patients with diabetes have an increased risk of an allergic reaction. b. Patients taking TMP/SMZ may need increased doses of glipizide. c. The patient should check the blood glucose level more often while taking TMP/SMZ. d. The patient should stop taking the glipizide while taking the TMP/SMZ.

ANS: C Sulfonamides can intensify the effects of some drugs, including glipizide, which is a sulfonylurea-type hypoglycemic medication. These drugs may require a reduction in dose to prevent toxicity. Patients should monitor their blood glucose more closely. There is no increase in allergic reactions to TMP/SMZ in patients who are diabetic. Patients taking TMP/SMZ may need reduced doses of glipizide but should not stop taking the drug.

5. A cholinergic drug is prescribed for a patient with a new diagnosis of myasthenia gravis, and the nurse provides instructions to the patient about the medication. What is important to include in the teaching? a. Take the medication with meals to avoid gastrointestinal distress. b. Give daytime doses close together for maximal therapeutic effect. c. Take the medication 30 minutes before eating to improve swallowing and chewing. d. Take the medication only if difficulty swallowing occurs during a meal.

ANS: C Taking the medication 30 minutes before meals allows time for the onset of action and therapeutic effects during the meal. The doses should be spaced evenly apart to optimize the effects of the medication. The other options are incorrect. DIF: COGNITIVE LEVEL: Applying (Application) REF: p. 327 TOP: NURSING PROCESS: Implementation

3. A patient is being treated for chemotherapy-induced nausea and vomiting (CINV) with ondansetron [Zofran] and dexamethasone. The patient reports getting relief during and immediately after chemotherapy but has significant nausea and vomiting several days after each chemotherapy treatment. What will the nurse do? a. Contact the provider to discuss increasing the dose of ondansetron. b. Suggest giving prolonged doses of dexamethasone. c. Suggest adding aprepitant [Emend] to the medication regimen. d. Tell the patient to ask the provider about changing the ondansetron to aprepitant.

ANS: C The current regimen of choice for patients taking highly emetogenic drugs consists of three agents: aprepitant plus dexamethasone plus a 5-HT3 antagonist, such as ondansetron. Aprepitant has a prolonged duration of action and can prevent delayed CINV as well as acute CINV. Increasing the dose of ondansetron will not help treat the delayed CINV. Glucocorticoids should be given intermittently and for short periods to avoid side effects. Changing the ondansetron to aprepitant is not recommended.

4. A patient who has been hospitalized for 2 weeks has developed a pressure ulcer that contains multidrug-resistant Staphylococcus aureus (MRSA). Which drug would the nurse expect to be chosen for therapy? a. Metronidazole (Flagyl) b. Ciprofloxacin (Cipro) c. Vancomycin (Vancocin) d. Tobramycin (Nebcin)

ANS: C Vancomycin is the drug of choice for the treatment of MRSA. The other drugs are not used for MRSA. DIF: COGNITIVE LEVEL: Understanding (Comprehension) REF: p. 633 TOP: NURSING PROCESS: Assessment

9. A patient with bronchitis is taking TMP/SMZ, 106/80 mg orally, twice daily. Before administering the third dose, the nurse notes that the patient has a widespread rash, a temperature of 103° F, and a heart rate of 100 beats per minute. The patient looks ill and reports not feeling well. What will the nurse do? a. Administer the dose and request an order for an antipyretic medication. b. Withhold the dose and request an order for an antihistamine to treat the rash. c. Withhold the dose and notify the provider of the symptoms. d. Request an order for intravenous TMP/SMZ, because the patient is getting worse.

ANS: C The most severe hypersensitivity reaction with TMP/SMZ is Stevens-Johnson syndrome, which manifests with fever, malaise, and rash. The drug should be discontinued immediately if a rash occurs. Requesting an antipyretic while giving the drug is incorrect. Giving antihistamines is not indicated. Giving TMP/SMZ intravenously would make the reaction worse.

1. When the nurse is administering topical nitroglycerin ointment, which technique is correct? a. Apply the ointment on the skin on the forearm. b. Apply the ointment only in the case of a mild angina episode. c. Remove the old ointment before new ointment is applied. d. Massage the ointment gently into the skin, and then cover the area with plastic wrap.

ANS: C The old ointment should be removed before a new dose is applied. The ointment should be applied to clean, dry, hairless skin of the upper arms or body, not below the elbows or below the knees. The ointment is not massaged or spread on the skin, and it is not indicated for the treatment of acute angina. DIF: COGNITIVE LEVEL: Applying (Application) REF: p. 372 TOP: NURSING PROCESS: Implementation

12. A patient will be taking bismuth subsalicylate (Pepto-Bismol) to control diarrhea. When reviewing the patient's other ordered medications, the nurse recognizes that which medication will interact significantly with the Pepto-Bismol? a. Acetaminophen (Tylenol), an analgesic b. Levothyroxine (Synthroid), a thyroid replacement drug c. Warfarin (Coumadin), an anticoagulant d. Fluoxetine (Prozac), an antidepressant

ANS: C The oral anticoagulant warfarin is more likely to cause increased bleeding times or bruising when co-administered with adsorbents. This is thought to be because the adsorbents bind to vitamin K, which is needed to make certain clotting factors. Vitamin K is synthesized by the normal bacterial flora in the bowel. The other options are incorrect. DIF: COGNITIVE LEVEL: Applying (Application) REF: p. 815 TOP: NURSING PROCESS: Implementation

2. A patient reports having adverse effects with nicotinic acid (niacin). The nurse can suggest performing which action to minimize these undesirable effects? a. Take the drug on an empty stomach. b. Take the medication every other day until the effects subside. c. Take an aspirin tablet 30 minutes before taking the drug. d. Take the drug with large amounts of fiber.

ANS: C The undesirable effects of nicotinic acid can be minimized by starting with a low initial dose, taking the drug with meals, and taking small doses of aspirin with the drug to minimize cutaneous flushing. Fiber intake has no effect on niacin's adverse effects, and it is not within the nurse's scope of practice to suggest a change of medication dosage. DIF: COGNITIVE LEVEL: Understanding (Comprehension) REF: p. 444 TOP: NURSING PROCESS: Implementation

9. When evaluating a patient's use of a metered-dose inhaler (MDI), the nurse notes that the patient is unable to coordinate the activation of the inhaler with her breathing. What intervention is most appropriate at this time? a. Notify the doctor that the patient is unable to use the MDI. b. Obtain an order for a peak flow meter. c. Obtain an order for a spacer device. d. Ask the prescriber if the medication can be given orally.

ANS: C The use of a spacer may be indicated with metered-dose inhalers, especially if success with inhalation is limited. The other options are not appropriate interventions. DIF: COGNITIVE LEVEL: Applying (Application) REF: p. 591 TOP: NURSING PROCESS: Implementation

4. The prescriber has changed the patient's medication regimen to include the leukotriene receptor antagonist (LTRA) montelukast (Singulair) to treat asthma. The nurse will emphasize which point about this medication? a. The proper technique for inhalation must be followed. b. The patient needs to keep it close by at all times to treat acute asthma attacks. c. It needs to be taken every day on a continuous schedule, even if symptoms improve. d. When the asthma symptoms improve, the dosage schedule can be tapered and eventually discontinued.

ANS: C These drugs are indicated for chronic, not acute, asthma and are to be taken every day on a continuous schedule, even if symptoms improve. These drugs are taken orally. DIF: COGNITIVE LEVEL: Applying (Application) REF: p. 592 TOP: NURSING PROCESS: Implementation

1. A nurse is administering a vasodilator that dilates resistance vessels. The nurse understands that this drug will have which effect on the patient? a.Decreased cardiac preload b. Decreased cardiac output c. Increased tissue perfusion d. Increased ventricular contraction

ANS: C Vasodilators that dilate resistance vessels, or arterioles, cause a decrease in afterload, which allows cardiac output and tissue perfusion to increase. A decrease in preload would be the result of dilation of capacitance vessels, or veins. Dilation of arterioles increases cardiac output. Ventricular contraction results when preload is increased.

2. A patient who is taking digoxin is admitted to the hospital for treatment of congestive heart failure. The prescriber has ordered furosemide [Lasix]. The nurse notes an irregular heart rate of 86 beats per minute, a respiratory rate of 22 breaths per minute, and a blood pressure of 130/82 mm Hg. The nurse auscultates crackles in both lungs. Which laboratory value causes the nurse the most concern? a. Blood glucose level of 120 mg/dL b. Oxygen saturation of 90% c. Potassium level of 3.5 mEq/L d. Sodium level of 140 mEq/L

ANS: C This patient has an irregular, rapid heartbeat that might be caused by a dysrhythmia. This patient's serum potassium level is low, which can trigger fatal dysrhythmias, especially in patients taking digoxin. Furosemide contributes to loss of potassium through its effects on the distal nephron. Potassium-sparing diuretics often are used in conjunction with furosemide to prevent this complication. This patient's serum glucose and sodium levels are normal and of no concern at this point, although they can be affected by furosemide. The oxygen saturation is somewhat low and needs to be monitored, although it may improve with diuresis.

1. A patient has had blood pressures of 150/95 mm Hg and 148/90 mm Hg on two separate office visits. The patient reports a blood pressure of 145/92 mm Hg taken in an ambulatory setting. The patient's diagnostic tests are all normal. The nurse will expect this patient's provider to order: a. a beta blocker. b. a loop diuretic and spironolactone. c. a thiazide diuretic. d. counseling on lifestyle changes.

ANS: C This patient has primary, or essential, hypertension as evidenced by systolic pressure greater than 140 and diastolic pressure greater than 90, along with normal tests ruling out another primary cause. Thiazide diuretics are first-line drugs for hypertension. Beta blockers are effective but are most often used to counter reflex tachycardia associated with reduced blood pressure caused by therapeutic agents. Loop diuretics cause greater diuresis than is usually needed and so are not first-line drugs. This patient should be counseled on lifestyle changes as an adjunct to drug therapy but should also begin drug therapy because hypertension already exists.

3. A patient with chronic hypertension is admitted to the hospital. During the admission assessment, the nurse notes a heart rate of 96 beats per minute, a blood pressure of 150/90 mm Hg, bibasilar crackles, 2+ pitting edema of the ankles, and distension of the jugular veins. The nurse will contact the provider to request an order for which medication? a. ACE inhibitor b. Digoxin [Lanoxin] c. Furosemide [Lasix] d. Spironolactone [Aldactone]

ANS: C This patient shows signs of fluid volume overload and needs a diuretic. Furosemide is a loop diuretic, which can produce profound diuresis very quickly even when the glomerular filtration rate (GFR) is low. An ACE inhibitor will not reduce fluid volume overload. Digoxin has a positive inotropic effect on the heart, which may improve renal perfusion, but this is not its primary effect. Spironolactone is a potassium-sparing diuretic with weak diuresis effects; it is used in conjunction with other diuretics to improve electrolyte balance.

11. A nurse is admitting a patient to the hospital who reports having recurrent, crampy abdominal pain followed by diarrhea. The patient tells the nurse that the diarrhea usually relieves the pain and that these symptoms have occurred daily for the past 6 months. The patient undergoes a colonoscopy, for which the findings are normal. The nurse will plan to teach this patient to: a. use antispasmodic medications. b. avoid food containing lactose and gluten. c. keep a food, stress, and symptom diary. d. use antidiarrheal drugs to manage symptoms.

ANS: C This patient shows signs of irritable bowel syndrome (IBS), which can be managed with drug and nondrug therapies. Patients should be taught to keep a log to identify foods and stressors that trigger symptoms. Antispasmodic medications frequently are used, but there is no clear evidence of their benefit. Patients with malabsorption disorders may need to avoid lactose or gluten but only if indicated. Antidiarrheal drugs do not have clear benefits, even though they are commonly used.

1. A patient is admitted to the obstetric unit in preterm labor at 36 weeks' gestation. The prescriber orders a tocolytic agent. When teaching the patient about this medication, the nurse will tell her that tocolytics: a. are given until term to reduce fetal mortality. b. are used to help the fetal lungs mature. c. help delay delivery while glucocorticoids are given. d. help treat the infections that cause preterm labor.

ANS: C Tocolytic agents are used to postpone premature labor and, on average, for only 48 hours. During this time, glucocorticoids are given to help the fetal lungs mature. Tocolytic agents do not suppress labor long term. They do not directly help the fetal lungs to mature; they help by allowing time for glucocorticoids to be given. They do not treat infection. PTS: 1 DIF: Cognitive Level: Application REF: pp. 777-778

7. The nurse is reviewing a patient's medication history and notes that the patient is taking the cholinergic blocker tolterodine (Detrol). Which is an indication for this medication? a. Irritable bowel disease b. Induction of mydriasis c. Urge incontinence d. Reduction of secretions preoperatively

ANS: C Tolterodine (Detrol) is used for urinary frequency, urgency, and urge incontinence caused by bladder (detrusor) overactivity. The conditions in the other options are not indications. DIF: COGNITIVE LEVEL: Understanding (Comprehension) REF: p. 335 TOP: NURSING PROCESS: Planning

11. When teaching a patient who has a new prescription for transdermal nitroglycerin patches, the nurse tells the patient that these patches are most appropriately used for which situation? a. To prevent low blood pressure b. To relieve shortness of breath c. To prevent the occurrence of angina d. To keep the heart rate from rising too high during exercise

ANS: C Transdermal dosage formulations of nitroglycerin are used for the long-term prophylactic management (prevention) of angina pectoris. Transdermal nitroglycerin patches are not appropriate for the relief of shortness of breath, to prevent palpitations, or to control the heart rate during exercise. DIF: COGNITIVE LEVEL: Applying (Application) REF: p. 366 TOP: NURSING PROCESS: Implementation

7. When a patient is receiving diuretic therapy, which of these assessment measures would best reflect the patient's fluid volume status? a. Blood pressure and pulse b. Serum potassium and sodium levels c. Intake, output, and daily weight d. Measurements of abdominal girth and calf circumference

ANS: C Urinary intake and output and daily weights are the best reflections of a patient's fluid volume status. DIF: COGNITIVE LEVEL: Understanding (Comprehension) REF: p. 461 TOP: NURSING PROCESS: Assessment

A nurse is teaching an adolescent female patient about 28-day monophasic combination oral contraceptives. The provider has instructed the patient to begin taking the pills on the first Sunday after the onset of her next period. What will the nurse tell the patient? a. "If breakthrough spotting occurs, you should begin taking a new pack of pills." b. "Protection from pregnancy will begin immediately." c. "Use another form of contraception for the next month." d. "You may take the pills at different times of day."

ANS: C When beginning oral contraceptives, it is important that the patient use another form of contraception for the first month, because protection is not immediate. Breakthrough spotting is common and is not an indication for starting a new cycle. Protection from pregnancy is not immediate. It is important to take the pills at the same time each day.

1. A patient is about to receive a dose of octreotide (Sandostatin). The nurse will assess for which contraindications or cautions? (Select all that apply.) a. Carcinoid crisis b. Diarrhea c. Type 1 diabetes mellitus d. Gallbladder disease e. Chronic renal failure f. Esophageal varices

ANS: C, D, E Octreotide is used with caution in patients with diabetes (type 1 or 2), gallbladder impairment, or renal impairment. Carcinoid crisis, which may be associated with severe diarrhea and flushing, is an indication for octreotide; esophageal varices are also an indication. DIF: COGNITIVE LEVEL: Applying (Application) REF: p. 486 TOP: NURSING PROCESS: Assessment

9. A 79-year-old patient is receiving a quinolone as treatment for a complicated incision infection. The nurse will monitor for which adverse effect that is associated with these drugs? a. Neuralgia b. Double vision c. Hypotension d. Tendonitis and tendon rupture

ANS: D A black-box warning is required by the U.S. Food and Drug Administration for all quinolones because of the increased risk for tendonitis and tendon rupture with use of the drugs. This effect is more common in elderly patients, patients with renal failure, and those receiving concurrent glucocorticoid therapy (e.g., prednisone). The other options are not common adverse effects. DIF: COGNITIVE LEVEL: Understanding (Comprehension) REF: p. 629 TOP: NURSING PROCESS: Evaluation

2. A patient with severe liver disease is receiving the angiotensin-converting enzyme (ACE) inhibitor, captopril (Capoten). The nurse is aware that the advantage of this drug for this patient is which characteristic? a. Captopril rarely causes first-dose hypotensive effects. b. Captopril has little effect on electrolyte levels. c. Captopril is a prodrug and is metabolized by the liver before becoming active. d. Captopril is not a prodrug and does not need to be metabolized by the liver before becoming active.

ANS: D A prodrug relies on a functioning liver to be converted to its active form. Captopril is not a prodrug, and therefore it would be safer for the patient with liver dysfunction. DIF: COGNITIVE LEVEL: Applying (Application) REF: p. 348 TOP: NURSING PROCESS: Assessment

10. Aldesleukin [IL-2] (Proleukin) is prescribed for a patient. The nurse reviews the patient's medication list and would note a potential drug interaction if which drug class is also ordered? a. Anticoagulants b. Antiepileptic drugs c. Oral hypoglycemic drugs d. Antihypertensive drugs

ANS: D Aldesleukin, when given with antihypertensives, can produce additive hypotensive effects. The other responses are incorrect. DIF: COGNITIVE LEVEL: Applying (Application) REF: p. 760 TOP: NURSING PROCESS: Assessment

3. The nurse is preparing an infusion of amphotericin B for a patient who has a severe fungal infection. Which intervention is appropriate regarding the potential adverse effects of amphotericin B? a. Discontinuing the infusion immediately if fever, chills, or nausea occur b. Gradually increasing the infusion rate until the expected adverse effects occur c. If fever, chills, or nausea occur during the infusion, administering medications to treat the symptoms d. Before beginning the infusion, administering an antipyretic and an antiemetic drug

ANS: D Almost all patients given the drug intravenously experience fever, chills, hypotension, tachycardia, malaise, muscle and joint pain, anorexia, nausea and vomiting, and headache. For this reason, pretreatment with an antipyretic (acetaminophen), antihistamines, and antiemetics may be conducted to decrease the severity of the infusion-related reaction. The other options are incorrect. DIF: COGNITIVE LEVEL: Applying (Application) REF: p. 672 TOP: NURSING PROCESS: Implementation

5. The nurse will instruct patients about a possible systemic effect that may occur if excessive amounts of topically applied adrenergic nasal decongestants are used. Which systemic effect may occur? a. Heartburn b. Bradycardia c. Drowsiness d. Palpitations

ANS: D Although a topically applied adrenergic nasal decongestant can be absorbed into the bloodstream, the amount absorbed is usually too small to cause systemic effects at normal dosages. Excessive dosages of these medications, however, are more likely to cause systemic effects elsewhere in the body. These may include cardiovascular effects, such as hypertension and palpitations, and central nervous system effects such as headache, nervousness, and dizziness. The other options are incorrect. DIF: COGNITIVE LEVEL: Understanding (Comprehension) REF: p. 571 TOP: NURSING PROCESS: Assessment

3. A patient who has seasonal allergies in the spring and fall asks the nurse about oral antihistamines. Which response by the nurse is correct? a. "Anticholinergic effects are more common with second-generation antihistamines." b. "First-generation antihistamines, such as diphenhydramine [Benadryl], are more effective." c. "Make sure you take antihistamines only when you have symptoms to minimize side effects." d. "You should take oral antihistamines daily during each allergy season to get maximum effects."

ANS: D Antihistamines are most effective when they are taken prophylactically, and they should be administered on a regular basis throughout the allergy season, even when symptoms are not present. They are less helpful when taken after symptoms appear. Second-generation antihistamines have fewer anticholinergic effects than first-generation antihistamines. First-generation antihistamines are not more effective than second-generation antihistamines. Oral antihistamines are not as effective when given on a PRN basis.

5. While a patient is receiving antilipemic therapy, the nurse knows to monitor the patient closely for the development of which problem? a. Neutropenia b. Pulmonary problems c. Vitamin C deficiency d. Liver dysfunction

ANS: D Antilipemic drugs may adversely affect liver function; therefore, liver function studies need to be closely monitored. The other options do not reflect problems that may occur with antilipemic drugs. DIF: COGNITIVE LEVEL: Understanding (Comprehension) REF: p. 446 TOP: NURSING PROCESS: Evaluation

12. A prescriber is considering prescribing the amiodarone derivative dronedarone [Multaq] for a patient with atrial flutter. The nurse should be concerned about which of the following? a. History of asthma b. History of hypothyroidism c. PR interval of 260 msec d. QT interval of 520 msec

ANS: D Because dronedarone prolongs the QT interval by about 10 msec, it should not be used in patients with a QT interval of more than 500 msec. It does not have significant pulmonary or thyroid toxicity. It should not be used in patients with a PR interval of more than 280 msec.

7. A patient with coronary artery disease asks the nurse about the "good cholesterol" laboratory values. The nurse knows that "good cholesterol" refers to which lipids? a. Triglycerides b. Low-density lipoproteins (LDLs) c. Very-low-density lipoproteins (VLDLs) d. High-density lipoproteins (HDLs)

ANS: D HDLs are responsible for the "recycling" of cholesterol. HDLs are sometimes referred to as the "good" lipid (or good cholesterol) because they are believed to be cardioprotective. LDLs are known as the "bad" cholesterol. DIF: COGNITIVE LEVEL: Understanding (Comprehension) REF: p. 437 TOP: NURSING PROCESS: Implementation

A patient will begin using a transdermal preparation of a muscarinic antagonist for overactive bladder (OAB). The nurse teaches the patient what to do if side effects occur. Which statement by the patient indicates the need for further teaching? a."I can use sugar-free gum for dry mouth." b."I may need laxatives for constipation." c."I should keep the site covered to prevent other people from getting the medicine." d."I will take Benadryl for any itching caused by a local reaction to the patch."

ANS: D Benadryl is an antihistamine, and even though it is not classified as a muscarinic antagonist, it has anticholinergic effects. Giving it with a muscarinic antagonist greatly enhances these effects, so it should not be used. Muscarinic antagonists cause dry mouth, and patients should be taught to use sugar-free gum or candies to help with this. Muscarinic antagonists can cause constipation, and laxatives may be used. Medication applied topically can be transferred to others who come in contact with the skin, so the site should be covered.

2. A patient begins taking nifedipine [Procardia], along with metoprolol, to treat hypertension. The nurse understands that metoprolol is used to: a. reduce flushing. b. minimize gingival hyperplasia. c. prevent constipation. d. prevent reflex tachycardia.

ANS: D Beta blockers are combined with nifedipine to prevent reflex tachycardia. Beta blockers do not reduce flushing, minimize gingival hyperplasia, or prevent constipation. Beta blockers can reduce the adverse cardiac effects of nifedipine.

8. A patient has been taking a beta blocker for 4 weeks as part of his antianginal therapy. He also has type II diabetes and hyperthyroidism. When discussing possible adverse effects, the nurse will include which information? a. "Watch for unusual weight loss." b. "Monitor your pulse for increased heart rate." c. "Use the hot tub and sauna at the gym as long as time is limited to 15 minutes." d. "Monitor your blood glucose levels for possible hypoglycemia or hyperglycemia."

ANS: D Beta blockers can cause both hypoglycemia and hyperglycemia. They may also cause weight gain if heart failure is developing, and decreased pulse rate. The use of hot tubs and saunas is not recommended because of the possibility of hypotensive episodes. DIF: COGNITIVE LEVEL: Understanding (Comprehension) REF: p. 367 TOP: NURSING PROCESS: Implementation

12. A patient with type 1 diabetes who takes insulin reports taking propranolol for hypertension. Why is the nurse concerned? a. The beta blocker can cause insulin resistance. b. Using the two agents together increases the risk of ketoacidosis. c. Propranolol increases insulin requirements because of receptor blocking. d. The beta blocker can mask the symptoms of hypoglycemia.

ANS: D Beta blockers can delay awareness of and response to hypoglycemia by masking signs associated with stimulation of the sympathetic nervous system (eg, tachycardia, palpitations) that hypoglycemia normally causes. Furthermore, beta blockade impairs glycogenolysis, which is one means by which the body can counteract a fall in blood glucose; beta blockers, therefore, can worsen insulin-induced hypoglycemia. Propranolol does not cause insulin resistance. The incidence of DKA is not increased by concurrent use of propranolol and insulin. Insulin requirements are not increased because of receptor blocking by propranolol.

2. A patient is receiving thrombolytic therapy, and the nurse monitors the patient for adverse effects. What is the most common undesirable effect of thrombolytic therapy? a. Dysrhythmias b. Nausea and vomiting c. Anaphylactic reactions d. Internal and superficial bleeding

ANS: D Bleeding, both internal and superficial, as well as intracranial, is the most common undesirable effect of thrombolytic therapy. The other options list possible adverse effects of thrombolytic drugs, but they are not the most common effects. DIF: COGNITIVE LEVEL: Remembering (Knowledge) REF: p. 424 TOP: NURSING PROCESS: Evaluation

4. A patient who is taking calcium supplements receives a prescription for ciprofloxacin [Cipro] for a urinary tract infection. The nurse will teach this patient to: a. consume extra fluids while taking the ciprofloxacin to prevent hypercalciuria. b. stop taking the calcium supplements while taking the ciprofloxacin. c. take the two medications together to increase the absorption of both. d. take the calcium either 6 hours before or 2 hours after taking the ciprofloxacin.

ANS: D Cationic compounds, including calcium supplements, can reduce the absorption of ciprofloxacin, so proper interval dosing is necessary. Consuming extra fluids is not indicated. With proper interval dosing, it is not necessary to discontinue the calcium while giving the ciprofloxacin. These two medications should not be given together.

8. A patient has been diagnosed with carbapenemase-resistant Enterobacteriaceae (CRE). The nurse expects to see orders for which drug? a. Dapsone (Cubicin), a miscellaneous antibiotic b. Ciprofloxacin (Cipro), a quinolone c. Linezolid (Zyvox), an oxazolidinone d. Colistimethate sodium (Coly-Mycin), a polypeptide antibiotic

ANS: D Colistimethate (Coly-Mycin), commonly referred to as colistin, is now being used again, often as one of the only drugs available to treat CRE. The other options are incorrect. DIF: COGNITIVE LEVEL: Applying (Application) REF: p. 630 TOP: NURSING PROCESS: Evaluation

6. A calcium channel blocker (CCB) is prescribed for a patient, and the nurse provides instructions to the patient about the medication. Which instruction is correct? a. Chew the tablet for faster release of the medication. b. To increase the effect of the drug, take it with grapefruit juice. c. If the adverse effects of chest pain, fainting, or dyspnea occur, discontinue the medication immediately. d. A high-fiber diet with plenty of fluids will help prevent the constipation that may occur.

ANS: D Constipation is a common effect of CCBs, and a high-fiber diet and plenty of fluids will help to prevent it. Grapefruit juice decreases the metabolism of CCBs. Extended-release tablets must never be chewed or crushed. These medications should never be discontinued abruptly because of the risk for rebound hypertension. DIF: COGNITIVE LEVEL: Applying (Application) REF: p. 373 TOP: NURSING PROCESS: Implementation

3. The nurse is developing a plan of care for a patient who is experiencing gastrointestinal adverse effects, including anorexia and nausea, after the first course of antineoplastic therapy. What is an appropriate outcome for this patient when dealing with this problem? a. The patient will eat three balanced meals a day within 2 days. b. The patient will return to normal eating pattern within 4 weeks. c. The patient will maintain normal weight by consuming healthy snacks as tolerated. d. The patient will maintain a diet of small, frequent feedings with nutrition supplements within 2 weeks.

ANS: D Consuming small, frequent meals with nutritional supplements, and maintaining a bland diet help to improve nutrition during antineoplastic therapy. DIF: COGNITIVE LEVEL: Applying (Application) REF: p. 729 TOP: NURSING PROCESS: Planning

3. When a patient is receiving vasopressin (Pitressin), the nurse will monitor for which therapeutic response? a. Improved appetite b. Increased serum albumin levels c. Increased serum potassium levels d. Decreased urinary output

ANS: D Decreased severe thirst and decreased urinary output are the therapeutic responses expected with vasopressin. The other options are incorrect. DIF: COGNITIVE LEVEL: Understanding (Comprehension) REF: p. 487 TOP: NURSING PROCESS: Evaluation

2. A patient is taking procainamide (Pronestyl) for a cardiac dysrhythmia. The nurse will monitor the patient for which possible adverse effect? a. Bradycardia b. Shortened QT interval c. Dyspnea d. Diarrhea

ANS: D Diarrhea is a potential adverse effect of procainamide therapy. Prolonged QT interval on the ECG is also possible. The other options are incorrect. DIF: COGNITIVE LEVEL: Understanding (Comprehension) REF: p. 398 TOP: NURSING PROCESS: Evaluation

1. A patient about to receive a morning dose of digoxin has an apical pulse of 53 beats/min. What will the nurse do next? a. Administer the dose. b. Administer the dose, and notify the prescriber. c. Check the radial pulse for 1 full minute. d. Withhold the dose, and notify the prescriber.

ANS: D Digoxin doses are held and the prescriber notified if the apical pulse is 60 beats/min or lower or is higher than 100 beats/min. The other options are incorrect. DIF: COGNITIVE LEVEL: Applying (Application) REF: p. 385 TOP: NURSING PROCESS: Implementation

4. A patient has been taking digoxin at home but took an accidental overdose and has developed toxicity. The patient has been admitted to the telemetry unit, where the physician has ordered digoxin immune Fab (Digifab). The patient asks the nurse why the medication is ordered. What is the nurse's best response? a. "It will increase your heart rate." b. "This drug helps to lower your potassium levels." c. "It helps to convert the irregular heart rhythm to a more normal rhythm." d. "This drug is an antidote to digoxin and will help to lower the blood levels."

ANS: D Digoxin immune Fab (Digifab) is the antidote for a severe digoxin overdose. It is given intravenously. The other options are incorrect. DIF: COGNITIVE LEVEL: Applying (Application) REF: p. 383 TOP: NURSING PROCESS: Implementation

7. The nurse is providing teaching regarding drug therapy to the husband of a woman with Alzheimer's disease. She was diagnosed 3 months ago, has mild memory loss, and will be receiving donepezil (Aricept). What is the drug's expected action? a. Prevents memory loss in later stages b. Reverses the course of Alzheimer's disease c. Provides sedation to prevent agitation and restlessness d. May help to improve the mood and decrease confusion

ANS: D Donepezil is used to treat mild-to-moderate dementia occurring in Alzheimer's disease and may improve the symptoms of the disease. DIF: COGNITIVE LEVEL: Understanding (Comprehension) REF: p. 323 TOP: NURSING PROCESS: Implementation

5. A 72-year-old man has a new prescription for an anticholinergic drug. He is an active man and enjoys outdoor activities, such as golfing and doing his own yard work. What will the nurse emphasize to him during the teaching session about his drug therapy? a. Drowsiness may interfere with his outdoor activities. b. Increased salivation may occur during exercise and outside activities. c. Fluid volume deficits may occur as a result of an increased incidence of diarrhea. d. He will need to take measures to reduce the occurrence of heat stroke during his activities.

ANS: D Elderly patients who take an anticholinergic drug need to be reminded that they are at a greater risk for suffering heat stroke because of decreased sweating and loss of normal heat-regulating mechanisms. DIF: COGNITIVE LEVEL: Applying (Application) REF: p. 337 TOP: NURSING PROCESS: Implementation

11. The nurse is conducting a class about antilipemic drugs. The antilipemic drug ezetimibe (Zetia) works by which mechanism? a. Inhibiting HMG-CoA reductase b. Preventing resorption of bile acids from the small intestines c. Activating lipase, which breaks down cholesterol d. Inhibiting cholesterol absorption in the small intestine

ANS: D Ezetimibe selectively inhibits absorption in the small intestine of cholesterol and related sterols. The other options are incorrect. DIF: COGNITIVE LEVEL: Understanding (Comprehension) REF: p. 445 TOP: NURSING PROCESS: Assessment

5. The nurse is reviewing instructions for vaginal antifungal drugs with a patient. Which statement by the nurse is an appropriate instruction regarding these drugs? a. "The medication can be stopped when your symptoms are relieved." b. "Discontinue this medication if menstruation begins." c. "Daily douching is part of the treatment for vaginal fungal infections." d. "Abstain from sexual intercourse until the treatment has been completed and the infection has resolved."

ANS: D Female patients taking antifungal medications for the treatment of vaginal infections need to abstain from sexual intercourse until the treatment has been completed and the infection has resolved. The medication needs to be taken for as long as prescribed. Instruct patients to continue to take the medication even if they are actively menstruating. Douching is not an appropriate intervention. DIF: COGNITIVE LEVEL: Applying (Application) REF: p. 677 TOP: NURSING PROCESS: Implementation

2. A patient is in preterm labor, and the prescriber orders magnesium sulfate for its neuroprotective effect on the fetus. The order calls for 4 gm given as an intravenous bolus over 20 minutes, followed by a maintenance infusion of 2 gm/hr. What will the nurse do? a. Administer the drug as ordered. b. Question the bolus amount. c. Question the bolus rate. d. Question the maintenance amount.

ANS: D For fetal neuroprotective uses, magnesium sulfate should be given using a low-dose protocol, which is a 4-gm IV loading bolus over 20 minutes, followed by a maintenance infusion of 1 gm/hr. A high-dose protocol would have a maintenance infusion of 2 to 3 gm/hr. The nurse should question the maintenance amount. The bolus amount and rate are correct. PTS: 1 DIF: Cognitive Level: Application REF: p. 780

5. A parent asks a nurse about giving diphenhydramine [Benadryl] to a child to relieve cold symptoms. Which response by the nurse is correct? a. "Benadryl must be given in higher doses to provide relief for cold symptoms." b. "Intranasal glucocorticoids are more effective for treating cold symptoms." c. "Nasal antihistamines are more effective for treating cold symptoms." d. "Because histamine doesn't cause cold symptoms, Benadryl would not be effective."

ANS: D Histamine does not contribute to symptoms of infectious rhinitis; therefore, antihistamines are of no use in treating cold symptoms. Giving antihistamines in higher doses does not provide relief for infectious rhinitis. Intranasal glucocorticoids are not useful for infectious rhinitis. Nasal antihistamines are not effective for treating infectious rhinitis.

3. The nurse is teaching a group of patients about self-administration of insulin. What content is important to include? a. Patients need to use the injection site that is the most accessible. b. If two different insulins are ordered, they need to be given in separate injections. c. When mixing insulins, the cloudy (such as NPH) insulin is drawn up into the syringe first. d. When mixing insulins, the clear (such as regular) insulin is drawn up into the syringe first.

ANS: D If mixing insulins in one syringe, the clear (regular) insulin is always drawn up into the syringe first. Patients always need to rotate injection sites. Mixing of insulins may be ordered. DIF: COGNITIVE LEVEL: Applying (Application) REF: p. 516 TOP: NURSING PROCESS: Implementation

A patient calls the nurse to report that she forgot to take a combination OC pill during the third week of her cycle. She tells the nurse that she missed another pill earlier that week. The nurse will tell her to: a. continue the pack, skip the inert pills, and use an additional form of contraception for 7 days. b. not to worry, because up to 7 days can be missed without an increased risk of pregnancy. c. take a pill immediately, continue the pack, and use an additional form of contraception for 1 month. d. take a pill now, continue the pack, skip the placebo pills, and start a new pack on week 4.

ANS: D If one or two pills are missed during the second or third week of a 28-day cycle, the patient should be instructed to take one pill as soon as possible, continue the pack, skip the placebo pills, and go straight to a new pack. It is not necessary to use an alternative form of contraception. If three or more pills are missed, the risk of pregnancy increases.

5. Which statement is accurate about the long-term complications of diabetes? a. Long-term complications are almost always the result of hypoglycemia and ketoacidosis. b. The complication rates for patients with optimally controlled type 2 diabetes are the same as for those whose disease is not optimally controlled. c. Optimal control of type 1 diabetes produces excessive episodes of life-threatening hypoglycemia. d. Optimal control of both types of diabetes reduces the risk of eye, kidney, and nerve damage.

ANS: D In both types of diabetes, optimal control of the disease slows the development of microvascular complications. Short-term complications are more apt to result from hypoglycemia and ketoacidosis. Patients with type 2 diabetes have fewer complications if their blood sugar level is optimally controlled. Hypoglycemia does not occur more frequently in patients with optimally controlled type 1 diabetes.

2. The nurse is administering a vancomycin (Vancocin) infusion. Which measure is appropriate for the nurse to implement in order to reduce complications that may occur with this drug's administration? a. Monitoring blood pressure for hypertension during the infusion b. Discontinuing the drug immediately if red man syndrome occurs c. Restricting fluids during vancomycin therapy d. Infusing the drug over at least 1 hour

ANS: D Infuse the medication over at least 1 hour to reduce the occurrence of red man syndrome. Adequate hydration (at least 2 L of fluid in 24 hours) during vancomycin therapy is important for the prevention of nephrotoxicity. Hypotension may occur during the infusion, especially if it is given too rapidly. DIF: COGNITIVE LEVEL: Applying (Application) REF: p. 633 TOP: NURSING PROCESS: Implementation

8. The nurse is administering an interferon and will implement which intervention? a. Giving the medication with meals b. Monitoring daily weights c. Limiting fluids while the patient is taking this medication d. Rotating sites if administered subcutaneously

ANS: D Interferon is given parenterally (not orally), and injection sites need to be rotated. Fluids need to be increased during interferon therapy. The other options are incorrect. DIF: COGNITIVE LEVEL: Applying (Application) REF: p. 766 TOP: NURSING PROCESS: Implementation

A patient is about to begin treatment with isoniazid. The nurse learns that the patient also takes phenytoin [Dilantin] for seizures. The nurse will contact the provider to discuss: a. increasing the phenytoin dose. b. reducing the isoniazid dose. c. monitoring isoniazid levels. d. monitoring phenytoin levels.

ANS: D Isoniazid is a strong inhibitor of three cytochrome P450 enzymes, and inhibition of these enzymes can raise the levels of other drugs, including phenytoin. Patients taking phenytoin should have the levels of this drug monitored, and the dose should be reduced if appropriate. Reducing the dose of isoniazid is not indicated. It is not necessary to monitor isoniazid levels.

12. The nurse is reviewing new medication orders for a patient who has an epidural catheter for pain relief. One of the orders is for enoxaparin (Lovenox), a low-molecular-weight heparin (LMWH). What is the nurse's priority action? a. Give the LMWH as ordered. b. Double-check the LMWH order with another nurse, and then administer as ordered. c. Stop the epidural pain medication, and then administer the LMWH. d. Contact the prescriber because the LMWH cannot be given if the patient has an epidural catheter.

ANS: D LMWHs are contraindicated in patients with an indwelling epidural catheter; they can be given 2 hours after the epidural is removed. This is very important to remember, because giving an LMWH with an epidural has been associated with epidural hematoma. DIF: COGNITIVE LEVEL: Analyzing (Analysis) REF: p. 416 TOP: NURSING PROCESS: Planning

3. The order reads, "Give levothyroxine (Synthroid), 200 mg, PO once every morning." Which action by the nurse is correct? a. Give the medication as ordered. b. Change the dose to 200 mcg because that is what the prescriber meant. c. Hold the drug until the prescriber returns to see the patient. d. Question the order because the dose is higher than 200 mcg.

ANS: D Levothyroxine is dosed in micrograms. A common medication error is to write the intended dose in milligrams instead of micrograms. If not caught, this error would result in a thousandfold overdose. Doses higher than 200 mcg need to be questioned in case this error has occurred. The other options are incorrect. DIF: COGNITIVE LEVEL: Analyzing (Analysis) REF: p. 494 TOP: NURSING PROCESS: Planning

18. A patient who has traveler's diarrhea asks the nurse about using loperamide to stop the symptoms. What will the nurse tell the patient about this drug? a. "Loperamide is used for moderate to severe symptoms only." b. "This drug is useful as prophylaxis to prevent symptoms." c. "This drug is only effective to treat certain infectious agents." d. "Use of this drug may prolong symptoms by slowing peristalsis."

ANS: D Loperamide is a nonspecific antidiarrheal that slows peristalsis; by this action, it may delay transit of the causative organism and may prolong the infection. It is used to treat mild symptoms and is used once symptoms start. It is not an antibiotic agent and is nonspecific.

9. When starting a patient on antidysrhythmic therapy, the nurse will remember that which problem is a potential adverse effect of any antidysrhythmic drug? a. Deficiency of fat-soluble vitamins b. Hyperkalemia c. Heart failure d. Dysrhythmias

ANS: D Many antidysrhythmics are themselves capable of producing new dysrhythmias (the prodysrhythmic effect). The other options are not adverse effects of antidysrhythmic drugs. DIF: COGNITIVE LEVEL: Understanding (Comprehension) REF: pp. 397-398 TOP: NURSING PROCESS: Planning

A nurse is caring for a patient who has myasthenia gravis. The prescriber has ordered neostigmine [Prostigmin]. An important initial nursing action before administration of the medication includes assessing: a.the ability to raise the eyelids. b.level of fatigue. c.skeletal muscle strength. d.swallowing ability.

ANS: D Many patients hospitalized for myasthenia gravis do not have the muscle strength to swallow well and need a parenteral form of the medication; therefore, assessing the patient's ability to swallow is an important initial safety measure. Evaluating the patient's ability to raise the eyelids, level of fatigue, and skeletal muscle strength are important assessments before drug administration and during drug treatment, because they indicate the effectiveness of the drug and help determine subsequent doses.

A 7-year-old child who is otherwise healthy is receiving mecasermin [Increlex] replacement therapy to treat severe primary deficiency of insulin-like growth factor-1 (IGF-1). The child develops tonsillar hypertrophy. The nurse anticipates that the provider will recommend: a.antibiotics. b.reducing the dose of mecasermin. c.discontinuing the mecasermin. d.tonsillectomy.

ANS: D Mecasermin can cause hypertrophy of the tonsils, which can be managed by tonsillectomy if needed. Antibiotics are not indicated, because the tonsil enlargement is not caused by infection. Neither reducing the dose of mecasermin nor discontinuing the drug is indicated.

3. A patient is being treated for secondary amenorrhea. The nurse expects which drug to be used to treat this problem? a. Methylergonovine (Methergine) b. Estradiol transdermal (Estraderm) c. Raloxifene (Evista) d. Medroxyprogesterone (Provera)

ANS: D Medroxyprogesterone, a progestin, is one of the drugs most commonly used for secondary amenorrhea. Secondary amenorrhea is not an indication for the other drugs listed. DIF: COGNITIVE LEVEL: Understanding (Comprehension) REF: p. 538 TOP: NURSING PROCESS: Planning

7. A patient with infertility will begin taking menotropins [Repronex]. The nurse will evaluate this patient's history to determine whether the: a. patient has a history of hyperinsulinemia. b. patient has excessive prolactin secretion. c. patient's pituitary can produce LH and FSH. d. patient's ovaries are capable of responding to gonadotropins.

ANS: D Menotropins are used when gonadotropin secretion by the pituitary is insufficient to provide adequate ovarian stimulation. Candidates must have ovaries capable of responding to FSH and LH. Metformin is used for hyperinsulinemia. Women with excessive prolactin secretion are treated with cabergoline. Menotropins contain equal amounts of LH and FSH and act directly on the ovaries, so the pituitary's inability to produce these hormones is not a factor for beginning treatment with this drug. PTS: 1 DIF: Cognitive Level: Application REF: pp. 771-772 | p. 773

11. A patient takes an ACE inhibitor to treat hypertension and tells the nurse that she wants to become pregnant. She asks whether she should continue taking the medication while she is pregnant. What will the nurse tell her? a. Controlling her blood pressure will decrease her risk of preeclampsia. b. Ask the provider about changing to an ARB during pregnancy. c. Continue taking the ACE inhibitor during her pregnancy. d. Discuss using methyldopa instead while she is pregnant.

ANS: D Methyldopa has limited effects on uteroplacental and fetal hemodynamics and does not adversely affect the fetus or neonate. Controlling blood pressure does not lower the risk of preeclampsia. ACE inhibitors and ARBs are specifically contraindicated during pregnancy.

4. The nurse notes in a female patient's history that she has an order for the androgen methyltestosterone (Android). Based on this finding, the nurse interprets that the patient has which disorder? a. Fibrocystic breast disease b. Hereditary angioedema c. Hypertension d. Inoperable breast cancer

ANS: D Methyltestosterone can be used in cases of inoperable breast cancer in women. The other options are incorrect. DIF: COGNITIVE LEVEL: Understanding (Comprehension) REF: p. 557 TOP: NURSING PROCESS: Assessment

17. A patient with gastroesophageal reflux disease (GERD) is to begin taking oral metoclopramide [Reglan]. The patient asks the nurse about the medication. Which response by the nurse is correct? a. "After 3 months, if the drug is not effective, you may need to increase the dose." b. "Metoclopramide may cause hiccups, especially after meals." c. "Serious side effects may occur but will stop when the drug is discontinued." d. "You should take the drug 30 minutes before each meal and at bedtime."

ANS: D Metoclopramide should be given 30 minutes before meals and at bedtime. Metoclopramide should not be used long term, and increasing the dose after 3 months of therapy is not recommended. Metoclopramide is used to treat hiccups; it does not cause hiccups. Tardive dyskinesia is a serious and irreversible side effect.

A. Tachycardia

Discharge teaching to a patient receiving a beta-agonist bronchodilator should emphasize reporting which side effect? A. Tachycardia B. Nonproductive cough C. Hypoglycemia D. Sedation

4. A patient is in an urgent care center and is receiving treatment for mild hyponatremia after spending several hours doing gardening work in the heat of the day. The nurse expects that which drug therapy will be used to treat this condition? a. Oral supplementation of fluids b. Intravenous bolus of lactated Ringer's solution c. Normal saline infusion, administered slowly d. Oral administration of sodium chloride tablets

ANS: D Mild hyponatremia is usually treated by oral administration of sodium chloride tablets. Pronounced sodium depletion is treated by intravenous normal saline or lactated Ringer's solution. DIF: COGNITIVE LEVEL: Applying (Application) REF: p. 473 TOP: NURSING PROCESS: Implementation

9. Cyclosporine is prescribed for a patient who had an organ transplant. The nurse will monitor the patient for which common adverse effect? a. Nausea and vomiting b. Fever and tremors c. Agitation d. Hypertension

ANS: D Moderate hypertension may occur in as much as 50% of patients taking cyclosporine. The other options are potential adverse effects of other immunosuppressant drugs. DIF: COGNITIVE LEVEL: Understanding (Comprehension) REF: p. 777 TOP: NURSING PROCESS: Evaluation

5. When monitoring a patient for signs of hypokalemia, the nurse looks for what early sign? a. Seizures b. Cardiac dysrhythmias c. Diarrhea d. Muscle weakness

ANS: D Muscle weakness is an early symptom of hypokalemia, as are hypotension, lethargy, mental confusion, and nausea. Cardiac dysrhythmias are a late symptom of hypokalemia. The other options are incorrect. DIF: COGNITIVE LEVEL: Understanding (Comprehension) REF: p. 474 TOP: NURSING PROCESS: Evaluation

5. A nurse is preparing to administer oral ofloxacin to a patient. While taking the patient's medication history, the nurse learns that the patient takes warfarin and theophylline. The correct action by the nurse is to request an order to: a. reduce the dose of ofloxacin. b. increase the dose of ofloxacin. c. increase the dose of theophylline. d. monitor coagulation levels.

ANS: D Ofloxacin increases plasma levels of warfarin, so coagulation tests should be monitored. The ofloxacin dose should not be reduced or increased. Ofloxacin does not affect theophylline levels.

14. Which medication used for asthma has off-label uses to treat allergic rhinitis? a. Diphenhydramine [Benadryl] b. Fexofenadine/pseudoephedrine [Allegra-D] c. Guaifenesin [Mucinex] d. Omalizumab [Xolair]

ANS: D Omalizumab is a monoclonal antibody directed against IgE that plays a role in the release of inflammatory mediators from mast cells and basophils. It is currently only approved for allergy-mediated asthma but is being used off-label to treat other allergic symptoms. Diphenhydramine, fexofenadine/pseudoephedrine, and guaifenesin are not used to treat asthma.

2. A patient with hypothyroidism is given a prescription for levothyroxine (Synthroid). When the nurse explains that this is a synthetic form of the thyroid hormone, he states that he prefers to receive more "natural" forms of drugs. What will the nurse explain to him about the advantages of levothyroxine? a. It has a stronger effect than the natural forms. b. Levothyroxine is less expensive than the natural forms. c. The synthetic form has fewer adverse effects on the gastrointestinal tract. d. The half-life of levothyroxine is long enough to permit once-daily dosing.

ANS: D One advantage of levothyroxine over the natural forms is that it can be administered only once a day because of its long half-life. The other options are incorrect. DIF: COGNITIVE LEVEL: Applying (Application) REF: p. 493 TOP: NURSING PROCESS: Implementation

18. The nurse is teaching a review class to nurses about diabetes mellitus. Which statement by the nurse is correct? a. "Patients with type 2 diabetes will never need insulin." b. "Oral antidiabetic drugs are safe for use during pregnancy." c. "Pediatric patients cannot take insulin." d. "Insulin therapy is possible during pregnancy if managed carefully."

ANS: D Oral medications are generally not recommended for pregnant patients because of a lack of firm safety data. For this reason, insulin therapy is the only currently recommended drug therapy for pregnant women with diabetes. Insulin is given to pediatric patients, with extreme care. Patients with type 2 diabetes may require insulin in certain situations or as their disease progresses. DIF: COGNITIVE LEVEL: Applying (Application) REF: p. 506 TOP: NURSING PROCESS: Implementation

4. Furosemide (Lasix) is prescribed for a patient who is about to be discharged, and the nurse provides instructions to the patient about the medication. Which statement by the nurse is correct? a. "Take this medication in the evening." b. "Avoid foods high in potassium, such as bananas, oranges, fresh vegetables, and dates." c. "If you experience weight gain, such as 5 pounds or more per week, be sure to tell your physician during your next routine visit." d. "Be sure to change positions slowly and rise slowly after sitting or lying so as to prevent dizziness and possible fainting because of blood pressure changes."

ANS: D Orthostatic hypotension is a possible problem with diuretic therapy. Foods high in potassium should be eaten more often, and the drug needs to be taken in the morning so that the diuretic effects do not interfere with sleep. A weight gain of 5 pounds or more per week must be reported immediately. DIF: COGNITIVE LEVEL: Applying (Application) REF: p. 459 TOP: NURSING PROCESS: Implementation

16. A nurse is caring for a patient with cancer who has been undergoing chemotherapy. The patient has oral mucositis as a result of the chemotherapy, and the provider has ordered palifermin [Kepivance]. Which is an appropriate nursing action when giving this drug? a. Administering the drug as a slow IV infusion b. Flushing the IV line with heparin before infusing the drug c. Giving the drug within 6 hours of the chemotherapy d. Warning the patient about the potential for distortion of taste

ANS: D Palifermin is generally well tolerated but commonly causes reactions involving the skin and mouth, including taste distortion. The drug should be given as an IV bolus. Palifermin binds with heparin, so the IV line should not be flushed with heparin before giving palifermin. If the interval between administration of palifermin and the chemotherapeutic drugs is too short, palifermin actually may increase the severity and duration of oral mucositis; therefore, palifermin should be given 24 hours before the chemotherapeutic drugs.

4. A nurse teaches a patient about sulfonamides. Which statement by the patient indicates a need for further teaching? a. "I need to drink extra fluids while taking this medication." b. "I need to use sunscreen when taking this drug." c. "I should call my provider if I develop a rash while taking this drug." d. "I should stop taking this drug when my symptoms are gone."

ANS: D Patients should always be advised to complete the prescribed course of the antibiotic even when symptoms subside. Patients should also understand the need to drink 8 to 10 glasses of water a day, to use sunscreen, and to notify the provider if they develop a rash.

18. A nurse has provided education for a patient newly diagnosed with hypertension who is just beginning therapy with antihypertensive medications. Which statement by the patient indicates a need for further teaching? a. "I may experience serious long-term problems even if I am not having symptoms." b. "I should report side effects to the provider since other drugs may be substituted." c. "I will need to take medications on a long-term basis." d. "When my symptoms subside, I may discontinue the medications."

ANS: D Patients should be taught that hypertension treatment is lifelong and that medications must be continued even when symptoms subside. Long-term problems may still occur without symptoms. Reporting drug side effects is necessary so that other drugs may be tried if needed.

8. The nurse is teaching patients about self-injection of insulin. Which statement is true regarding injection sites? a. Avoid the abdomen because absorption there is irregular. b. Choose a different site at random for each injection. c. Give the injection in the same area each time. d. Rotate sites within the same location for about 1 week before rotating to a new location.

ANS: D Patients taking insulin injections need to be instructed to rotate sites, but to do so within the same location for about 1 week (so that all injections are rotated in one area—for example, the right arm—before rotating to a new location, such as the left arm). Also, each injection needs to be at least to 1 inch away from the previous site. DIF: COGNITIVE LEVEL: Applying (Application) REF: p. 517 TOP: NURSING PROCESS: Implementation

7. A patient with heart failure who has been taking an ACE inhibitor, a thiazide diuretic, and a beta blocker for several months comes to the clinic for evaluation. As part of the ongoing assessment of this patient, the nurse will expect the provider to evaluate: a. complete blood count. b. ejection fraction. c. maximal exercise capacity. d. serum electrolyte levels.

ANS: D Patients taking thiazide diuretics can develop hypokalemia, which can increase the risk for dysrhythmias; therefore, the serum electrolyte levels should be monitored closely. A complete blood count is not recommended. This patient is taking the drugs recommended for patients with Stage C heart failure; although the patient's quality of life and ability to participate in activities should be monitored, routine measurement of the ejection fraction and maximal exercise capacity is not recommended.

8. A nurse provides dietary counseling for a patient newly diagnosed with type 1 diabetes. Which instruction should be included? a. "You may eat any foods you want and cover the glucose increase with sliding scale, regular insulin." b. "Most of the calories you eat should be in the form of protein to promote fat breakdown and preserve muscle mass." c. "Your total caloric intake should not exceed 1800 calories in a 24-hour period." d. "You should use a carbohydrate counting approach to maintain glycemic control."

ANS: D Patients with diabetes should be given intensive insulin therapy education using either a carbohydrate counting or experience-based estimation approach in achieving glycemic control. A patient with diabetes cannot eat any foods desired and then cover the glucose increase with a sliding scale of regular insulin. Evidence suggests that there is not an ideal percentage of calories that should be ingested from carbohydrate, fat, or protein. Every patient with diabetes must be assessed individually to determine the number of total calories the person should have daily. The total caloric intake should be spread evenly throughout the day, with meals spaced 4 to 5 hours apart.

4. A patient has received an accidental overdose of intravenous atropine. Which drug will the nurse prepare to administer? a. Atenolol (Tenormin) b. Bethanechol (Urecholine) c. Dicyclomine (Bentyl) d. Physostigmine (Antilirium)

ANS: D Physostigmine salicylate is the antidote to an atropine overdose in patients who show extreme delirium or agitation and could inflict injury to themselves. Its routine use as an antidote for cholinergic-blocker overdose is controversial, however. The other options are incorrect choices. DIF: COGNITIVE LEVEL: Understanding (Comprehension) REF: p. 333 TOP: NURSING PROCESS: Planning

4. A patient is about to undergo a diagnostic bowel procedure. The nurse expects which drug to be used to induce total cleansing of the bowel? a. Docusate sodium (Colace) b. Lactulose (Enulose) c. Mineral oil d. Polyethylene glycol 3350 (GoLYTELY)

ANS: D Polyethylene glycol is a very potent laxative that induces total cleansing of the bowel and is most commonly used before diagnostic or surgical bowel procedures. The other options are incorrect. DIF: COGNITIVE LEVEL: Understanding (Comprehension) REF: p. 822 TOP: NURSING PROCESS: Planning

4. The parent of a child with cerebral palsy reports that the child has pebble-like stools most of the time and seems uncomfortable if several days have passed between stools. The nurse will suggest that the parent discuss which medication with the child's provider? a. Bisacodyl [Dulcolax] suppositories b. Magnesium citrate c. Methylcellulose [Citrucel] d. Polyethylene glycol [MiraLax]

ANS: D Polyethylene glycol is an osmotic laxative widely used for chronic constipation, which this child has, because it provides relief from abdominal discomfort, improves stool consistency, and increases frequency. Bisacodyl is not recommended for long-term use. Magnesium citrate causes increased water loss, and methylcellulose can also cause impaction.

10. A woman visits a health center requesting oral contraceptives. Which laboratory test is most important for the nurse to assess before the patient begins oral contraceptive therapy? a. Complete blood count b. Serum potassium level c. Vaginal cultures d. Pregnancy test

ANS: D Pregnancy should be ruled out before beginning oral contraceptive therapy because the medications can be harmful to the fetus; they are classified as pregnancy category X. DIF: COGNITIVE LEVEL: Applying (Application) REF: p. 541 TOP: NURSING PROCESS: Assessment

11. A patient who has been on antibiotic therapy for 2 weeks has developed persistent diarrhea. The nurse expects which medication class to be ordered to treat this diarrhea? a. Lubricants b. Adsorbents c. Anticholinergics d. Probiotics

ANS: D Probiotics work by replenishing bacteria that may have been destroyed by antibiotic therapy, thus restoring the balance of normal flora and suppressing the growth of diarrhea-causing bacteria. DIF: COGNITIVE LEVEL: Applying (Application) REF: p. 815 TOP: NURSING PROCESS: Implementation

6. A patient has been taking psyllium [Metamucil] two to three times daily for several days. The patient complains of stomach pain but has not had a stool. What will the nurse do? a. Ask the patient to drink a full glass of water. b. Give another dose of the psyllium. c. Request an order for a bisacodyl [Dulcolax] suppository. d. Palpate the patient's abdomen and auscultate for bowel sounds.

ANS: D Psyllium is a bulk-forming laxative and can cause a fecal impaction or obstruction. Abdominal pain can be a sign of impaction or obstruction, so the nurse should assess the patient for this complication. Giving fluids by mouth, administering more laxative, or giving a suppository are all contraindicated if an obstruction has developed and should not be done until this condition is ruled out.

5. A patient is taking digoxin [Lanoxin] and quinidine to treat sustained ventricular tachycardia. Before giving medications, the nurse reviews the patient's electrocardiogram (ECG) and notes a QRS complex that has widened by 50% from the baseline ECG. What will the nurse do? a. Administer the medications as ordered, because this indicates improvement. b. Contact the provider to discuss reducing the digoxin dose. c. Contact the provider to request an increase in the quinidine dose. d. Withhold the quinidine and contact the provider to report the ECG finding.

ANS: D Quinidine widens the QRS complex by slowing depolarization of the ventricles. As cardiotoxicity develops as a result of quinidine toxicity, the QRS complex widens excessively. Any widening of the QRS complex of 50% or more warrants notifying the provider, so the nurse should withhold the medication and contact the provider. Widening of the QRS complex by more than 50% of baseline indicates cardiotoxicity. Quinidine can double digoxin levels, so it is not likely that the digoxin dose would need to be increased, and an increase in the QRS complex does not indicate a need for more digoxin. The quinidine dose should not be increased, because the findings indicate cardiotoxicity from the quinidine.

2. A patient with type 1 diabetes is eating breakfast at 7:30 AM. Blood sugars are on a sliding scale and are ordered before a meal and at bedtime. The patient's blood sugar level is 317 mg/dL. Which formulation of insulin should the nurse prepare to administer? a. No insulin should be administered. b. NPH c. 70/30 mix d. Lispro [Humalog]

ANS: D Regular insulin is indicated for sliding scale coverage. Insulin is definitely indicated for this high blood sugar level. NPH is used for scheduled insulin doses and is a longer-acting insulin. A 70/30 mix is also used for scheduled insulin coverage.

7. A 51-year-old woman will be taking selective estrogen receptor modulators (SERMs) as part of treatment for postmenopausal osteoporosis. The nurse reviews potential contraindications, including which condition? a. Hypocalcemia b. Breast cancer c. Stress fractures d. Venous thromboembolism

ANS: D SERMs such as raloxifene are contraindicated in women with a venous thromboembolic disorder, including deep vein thrombosis, pulmonary embolism, or a history of such disorders. The other options are incorrect. DIF: COGNITIVE LEVEL: Understanding (Comprehension) REF: p. 542 TOP: NURSING PROCESS: Implementation

10. The nurse is providing instructions about the Advair inhaler (fluticasone propionate and salmeterol). Which statement about this inhaler is accurate? a. It is indicated for the treatment of acute bronchospasms. b. It needs to be used with a spacer for best results. c. Patients need to avoid drinking water for 1 hour after taking this drug. d. It is used for the prevention of bronchospasms.

ANS: D Salmeterol is a long-acting beta2 agonist bronchodilator, while fluticasone is a corticosteroid. In combination, they are used for the maintenance treatment of asthma and COPD. As a long-acting inhaler, Advair is not appropriate for treatment of acute bronchospasms. The other statements are incorrect. DIF: COGNITIVE LEVEL: Applying (Application) REF: p. 583 TOP: NURSING PROCESS: Implementation

8. A patient is admitted with severe hypertensive crisis. The nurse will anticipate administering which medication? a. Captopril PO b. Hydralazine [Apresoline] 25 mg PO c. Minoxidil 20 mg PO d. Sodium nitroprusside [Nitropress] IV

ANS: D Sodium nitroprusside is the drug of choice for hypertensive emergencies and is given intravenously. ACE inhibitors, such as captopril, are not used. Hydralazine may be used but should be given IV. Minoxidil is effective, but its severe side effects make it a second-line drug.

A nurse is teaching a community education class on contraceptives. The nurse tells the class that if spermicides containing nonoxynol-9 are used, the patient should take special precautions, because these spermicides have been linked to: a. human papillomavirus (HPV) infections. b. spontaneous abortions. c. endometrial cancer. d. increased transmission of the human immunodeficiency virus (HIV).

ANS: D Spermicides that contain nonoxynol-9 have been linked to the increased transmission of HIV. Spermicides containing nonoxynol-9 have not been linked to HPV infections, spontaneous abortions, or endometrial cancer.

5. A patient with polycystic ovary syndrome (PCOS) asks the nurse what she can do to improve her chances of getting pregnant. Which statement by the patient indicates a need for further teaching? a. "Clomiphene will help induce ovulation but will not treat the other symptoms of polycystic ovarian disease." b. "If I lose weight, my infertility and irregular periods could resolve without medications." c. "Metformin improves insulin sensitivity and reduces male hormone levels." d. "Spironolactone, which reduces androgens and facial hair, is helpful if I'm trying to conceive."

ANS: D Spironolactone is antiandrogenic; it can reduce hirsutism and acne, but it also can harm the fetus and should not be used in patients trying to conceive. Clomiphene helps induce ovulation but does not treat symptoms of PCOS. Patients who lose weight might experience a reversal of symptoms without medications, although weight loss may be difficult to achieve. Metformin reduces serum insulin levels by increasing insulin sensitivity and indirectly lowers androgen levels. PTS: 1 DIF: Cognitive Level: Application REF: p. 770

10. A nurse is caring for an African American patient who has been admitted to the unit for long-term antibiotic therapy with sulfonamides. The patient develops fever, pallor, and jaundice. The nurse would be correct to suspect that the patient has developed: a. Stevens-Johnson syndrome. b. kernicterus. c. hepatotoxicity. d. hemolytic anemia.

ANS: D Sulfonamides can cause hemolytic anemia in patients of African American and Mediterranean origin, usually because of a genetic deficiency. Red cell lysis can produce fever, pallor, and jaundice, and patients should be observed for these signs. The patient's signs and symptoms are not characteristic of Stevens-Johnson syndrome or hepatotoxicity. The patient's signs and symptoms are not characteristic of kernicterus, which occurs in newborns.

11. A nurse is obtaining a drug history from a patient about to receive sulfadiazine. The nurse learns that the patient takes warfarin, glipizide, and a thiazide diuretic. Based on this assessment, the nurse will expect the provider to: a. change the antibiotic to TMP/SMZ. b. increase the dose of the glipizide. c. monitor the patient's electrolytes closely. d. monitor the patient's coagulation levels.

ANS: D Sulfonamides interact with several drugs and through metabolism-related interactions can intensify the effects of warfarin. Patients taking both should be monitored closely for bleeding tendencies. Changing to the combination product will not help, because sulfonamides are still present. Sulfonamides intensify glipizide levels, so this drug may actually need to be reduced. Trimethoprim, not sulfonamides, raises potassium levels.

3. A patient who has endometriosis has been unable to conceive. She asks the nurse about medications to treat the condition. What will the nurse tell the patient? a. Combination oral contraceptives are effective for improving fertility. b. Gonadotropin-releasing hormone agonists are used for long-term treatment. c. Nonsteroidal anti-inflammatory drugs are second-line agents for treating pain. d. Pharmacologic agents used to treat endometriosis do not enhance fertility.

ANS: D Surgery reduces symptoms of endometriosis and improves fertility; drugs used to treat endometriosis only reduce discomfort. Combination oral contraceptives only treat symptoms; they do not improve fertility. Gonadotropin-releasing hormone agonists cannot be used long term because of side effects of osteoporosis and hot flashes. Nonsteroidal anti-inflammatory drugs (NSAIDs) are first-line agents for endometriosis pain. PTS: 1 DIF: Cognitive Level: Application REF: p. 770

13. A patient has been taking senna [Senokot] for several days, and the nurse notes that the urine is yellowish-brown. What does the nurse know about this symptom? a. It indicates that renal failure has occurred. b. It is caused by dehydration, which is a laxative side effect. c. It is a sign of toxicity, indicating immediate withdrawal of the drug. d. It is an expected, harmless effect of senna.

ANS: D Systemic absorption of senna, followed by renal excretion, may impart a harmless yellowish-brown or pink color to the urine. This symptom is not an indication of renal failure, dehydration, or toxicity.

20. When teaching a patient who is starting metformin (Glucophage), which instruction by the nurse is correct? a. "Take metformin if your blood glucose level is above 150 mg/dL." b. "Take this 60 minutes after breakfast." c. "Take the medication on an empty stomach 1 hour before meals." d. "Take the medication with food to reduce gastrointestinal (GI) effects."

ANS: D The GI adverse effects of metformin can be reduced by administering it with meals. The other options are incorrect. DIF: COGNITIVE LEVEL: Analyzing (Analysis) REF: p. 508 TOP: NURSING PROCESS: Implementation

A patient who has esophageal cancer is experiencing dry mouth and the provider orders oral pilocarpine to treat this symptom. What will the nurse expect to teach this patient about this medication? a."This medication may cause rapid heart rate and elevated blood pressure." b."This medication may cause constipation and gastric discomfort in large doses." c."You should experience sweating with this medication and should not have other side effects." d."You will begin taking 5 mg three times daily and may increase the dose to 10 mg."

ANS: D The dosing for pilocarpine, when used for dry mouth associated with head and neck cancers, is 5 mg three times daily, which may be titrated up to 10 mg three times daily. Tachycardia and constipation are side effects of atropine. Sweating occurs with low doses of pilocarpine. Higher doses, such as this, are associated with the full range of muscarinic effects.

9. What is the most reliable measure for assessing diabetes control over the preceding 3-month period? a. Self-monitoring blood glucose (SMBG) graph report b. Patient's report c. Fasting blood glucose level d. Glycosylated hemoglobin level

ANS: D The glycosylated hemoglobin level tells much about what the plasma glucose concentration has been, on average, over the previous 2 to 3 months. The SMBG graph report is done by the patient and indicates each blood sugar level the patient has on a daily basis. It is not as reliable as the glycosylated hemoglobin level, because the equipment used might not be accurate and the testing may not reflect actual measurements 100% of the time. The patient's report of blood sugar levels is not considered as accurate as the glycosylated hemoglobin level for the same reason that the SMBG is not. One fasting blood glucose level indicates the patient's blood sugar level for that one time when it was obtained.

8. A patient has been taking tolterodine (Detrol), but today her prescriber changed her to a newer drug, darifenacin (Enablex). What advantage does darifenacin have over the tolterodine? a. The newer cholinergic-blocker drugs are more effective. b. It helps reduce urinary retention. c. It can be used in patients with narrow-angle glaucoma. d. The incidence of dry mouth is much lower with darifenacin.

ANS: D The incidence of dry mouth is much lower with use of the newer cholinergic-blocker drugs, such as darifenacin, because the actions of these drugs are more specific for the bladder as opposed to the salivary glands. These drugs are contraindicated if narrow-angle glaucoma or urinary retention is present. The newer cholinergic-blocker drugs are not necessarily more effective. DIF: COGNITIVE LEVEL: Understanding (Comprehension) REF: p. 335 TOP: NURSING PROCESS: Implementation

14. An older adult patient will be taking a vasodilator for hypertension. Which adverse effect is of most concern for the older adult patient taking this class of drug? a. Dry mouth b. Restlessness c. Constipation d. Hypotension

ANS: D The older adult patient is more sensitive to the blood pressure-lowering effects of vasodilators, and consequently experience more problems with hypotension, dizziness, and syncope. The other options are incorrect. DIF: COGNITIVE LEVEL: Applying (Application) REF: p. 355 TOP: NURSING PROCESS: Evaluation

4. The nurse is preparing to administer an injection of monoclonal antibodies. Which additional drug will the nurse administer to minimize adverse reactions to the monoclonal antibodies? a. A nonsteroidal anti-inflammatory drug b. A benzodiazepine c. An opioid pain reliever d. A corticosteroid

ANS: D The monoclonal antibodies basiliximab and daclizimab have a tendency to cause the allergy-like reaction known as cytokine release syndrome, which can be severe and even involve anaphylaxis. In an effort to avoid or alleviate this problem, it is recommended that an injection of a corticosteroid, such as methylprednisolone, be administered before the injection of monoclonal antibodies. DIF: COGNITIVE LEVEL: Applying (Application) REF: p. 773 TOP: NURSING PROCESS: Implementation

7. Just before the second course of chemotherapy, the laboratory calls to report that the patient's neutrophil count is 450 cells/mm3. The nurse expects that the oncologist will follow which course of treatment? a. Chemotherapy will continue as scheduled. b. Chemotherapy will resume with a lowered dosage. c. Chemotherapy will resume after a transfusion of neutrophils. d. Chemotherapy will be withheld until the neutrophil count returns toward normal levels.

ANS: D The normal range for neutrophils is above 1500 cells/mm3. If neutrophils are decreased to levels of less than 500 cells/mm3 (neutropenia), there is risk for severe infection. Chemotherapy will be held until the count returns toward normal levels. DIF: COGNITIVE LEVEL: Analyzing (Analysis) REF: p. 744 TOP: NURSING PROCESS: Evaluation

3. A patient has an order for cyclosporine (Sandimmune). The nurse finds that cyclosporine-modified (Neoral) is available in the automated medication cabinet. Which action by the nurse is correct? a. Hold the dose until the prescriber makes rounds. b. Give the cyclosporine-modified drug. c. Double-check the order, and then give the cyclosporine-modified drug. d. Notify the pharmacy to obtain the Sandimmune form of the drug.

ANS: D The nurse must double-check the formulation before giving cyclosporine. Cyclosporine-modified products (such as Neoral or Gengraf) are interchangeable with each other but are not interchangeable with Sandimmune. In this case, the nurse must obtain the Sandimmune form of the drug from the pharmacy. The other options are incorrect. DIF: COGNITIVE LEVEL: Analyzing (Analysis) REF: p. 778 TOP: NURSING PROCESS: Implementation

3. During a follow-up visit, the health care provider examines the fundus of the patient's eye. Afterward, the patient asks the nurse, "Why is he looking at my eyes when I have high blood pressure? It does not make sense to me!" What is the best response by the nurse? a. "We need to monitor for drug toxicity." b. "We must watch for increased intraocular pressure." c. "The provider is assessing for visual changes that may occur with drug therapy." d. "The provider is making sure the treatment is effective over the long term."

ANS: D The physician would examine the fundus of a patient's eyes during antihypertensive therapy because it is a more reliable indicator than blood pressure readings of the long-term effectiveness of treatment. DIF: COGNITIVE LEVEL: Applying (Application) REF: p. 358 TOP: NURSING PROCESS: Evaluation

5. A patient has been placed on a milrinone (Primacor) infusion as part of the therapy for end-stage heart failure. What adverse effect of this drug will the nurse watch for when assessing this patient during the infusion? a. Hypertension b. Hyperkalemia c. Nausea and vomiting d. Cardiac dysrhythmias

ANS: D The primary adverse effects seen with milrinone are cardiac dysrhythmias, mainly ventricular. It may also cause hypotension, hypokalemia, and other effects, but not nausea and vomiting. DIF: COGNITIVE LEVEL: Applying (Application) REF: p. 381 TOP: NURSING PROCESS: Assessment

1. A nurse is discussing nutrition with a patient who expresses concern about not always consuming vitamins in amounts consistent with the recommended dietary allowances (RDAs). What will the nurse tell this patient? a. The RDA is only an estimate of the amount of vitamins required and does not represent a scientific number. b. The RDA is the amount of vitamin needed by 50% of the population regardless of age or gender. c. The RDA represents the highest amount that can be consumed safely, so amounts less than this can be sufficient. d. The RDA represents an average, so low intake one day can be compensated for by increased amounts on another day.

ANS: D The recommended dietary allowance is the average daily dietary intake sufficient to meet the nutrient requirements of nearly all healthy individuals. Because it represents an average, a low amount one day can be compensated for by an increased amount on another day. The RDA values are determined through extensive experimental data and are not estimates. The estimated average requirement (EAR) is the level that meets the nutritional requirements of 50% of healthy individuals. The tolerable upper intake level (UL) is the highest average daily intake that can be consumed without risk of adverse effects. PTS: 1 DIF: Cognitive Level: Analysis REF: p. 987

2. A patient is receiving her third course of 5-fluorouracil therapy and knows that stomatitis is a potential adverse effect of antineoplastic therapy. What will the nurse teach her about managing this problem? a. "You can take aspirin to prevent stomatitis." b. "Be sure to watch for and report black, tarry stools immediately." c. "You need to increase your intake of foods containing fiber and citric acid." d. "Be sure to examine your mouth daily for bleeding, painful areas, and ulcerations."

ANS: D The symptoms of stomatitis consist of pain or burning in the mouth, difficulty swallowing, taste changes, viscous saliva, dryness, cracking, and fissures, with or without bleeding mucosa. Teach patients to avoid consuming foods containing citric acid and foods that are hot or spicy or high in fiber. Assessing stools is important but is not related to stomatitis, and aspirin must not be used during this therapy. DIF: COGNITIVE LEVEL: Applying (Application) REF: p. 731 TOP: NURSING PROCESS: Implementation

7. A nurse is discussing microbial resistance among sulfonamides and trimethoprim with a nursing student. Which statement by the student indicates a need for further teaching? a. "Bacterial resistance to trimethoprim is relatively uncommon." b. "Resistance among gonococci, streptococci, and meningococci to sulfonamides is high." c. "Resistance to both agents can occur by spontaneous mutation of organisms." d. "Resistance to sulfonamides is less than resistance to trimethoprim."

ANS: D There is less microbial resistance to trimethoprim than there is to sulfonamides. Bacterial resistance to trimethoprim is relatively uncommon. Gonococcal, streptococcal, and meningococcal resistance to sulfonamides is especially high. For both agents, resistance can develop by spontaneous mutation.

11. A 19-year-old woman has been diagnosed with hypothyroidism and has started thyroid replacement therapy with levothyroxine (Synthroid). After 6 months, she calls the nurse to say that she feels better and wants to stop the medication. Which response by the nurse is correct? a. "You can stop the medication if your symptoms have improved." b. "You need to stay on the medication for at least 1 year before a decision about stopping it can be made." c. "You need to stay on this medication until you become pregnant." d. "Medication therapy for hypothyroidism is usually lifelong, and you should not stop taking the medication."

ANS: D These medications must never be abruptly discontinued, and lifelong therapy is usually the norm. The other options are incorrect. DIF: COGNITIVE LEVEL: Applying (Application) REF: p. 497 TOP: NURSING PROCESS: Assessment

1. A patient has been prescribed warfarin (Coumadin) in addition to a heparin infusion. The patient asks the nurse why he has to be on two medications. The nurse's response is based on which rationale? a. The oral and injection forms work synergistically. b. The combination of heparin and an oral anticoagulant results in fewer adverse effects than heparin used alone. c. Oral anticoagulants are used to reach an adequate level of anticoagulation when heparin alone is unable to do so. d. Heparin is used to start anticoagulation so as to allow time for the blood levels of warfarin to reach adequate levels.

ANS: D This overlap therapy is required in patients who have been receiving heparin for anticoagulation and are to be switched to warfarin so that prevention of clotting is continuous. This overlapping is done purposefully to allow time for the blood levels of warfarin to rise, so that when the heparin is eventually discontinued, therapeutic anticoagulation levels of warfarin will have been achieved. Recommendations are to continue overlap therapy of the heparin and warfarin for at least 5 days; the heparin is stopped after day 5 when the international normalized ratio (INR) is above 2. DIF: COGNITIVE LEVEL: Applying (Application) REF: p. 417 TOP: NURSING PROCESS: Implementation

A nurse working in a family planning clinic is preparing to administer a first dose of intramuscular DMPA [Depo-Provera] to a young adult patient. The woman tells the nurse she has just finished her period. What will the nurse do? a. Administer the injection today and counsel backup contraception for 7 days. b. Administer the injection today and tell her that protection is immediate. c. Obtain a pregnancy test to rule out pregnancy before administering the drug. d. Schedule an appointment for her to receive the injection in 3 weeks.

ANS: D To ensure that patients are not pregnant when they receive DMPA, the first injection should be given during the first 5 days of a normal menstrual period.

11. A patient is in the intensive care unit after a myocardial infarction. The nurse notes that the QT interval on this patient's electrocardiogram has been elongating. The nurse is concerned that which cardiac dysrhythmia may occur? a. AV block b. Bradycardia c. Supraventricular tachycardia d. Torsades de pointes

ANS: D Torsades de pointes is a dysrhythmia that can occur with prolongation of the QT interval and can progress to fatal ventricular fibrillation. A prolonged QT interval does not signal the development of AV block, bradycardia, or SVT.

10. A patient is preparing to travel to perform missionary work in a region with poor drinking water. The provider gives the patient a prescription for ciprofloxacin [Cipro] to take on the trip. What will the nurse instruct this patient to do? a. Combine the antibiotic with an antidiarrheal medication, such as loperamide. b. Start taking the ciprofloxacin 1 week before traveling. c. Take 1 tablet of ciprofloxacin with each meal for best results. d. Use the drug if symptoms are severe or do not improve in a few days.

ANS: D Traveler's diarrhea is generally caused by Escherichia coli; treatment is usually unnecessary, because the disease runs its course in a few days. If symptoms are severe or prolonged, an antibiotic, such as ciprofloxacin, may be helpful. Patients should be instructed to take it only if needed. Antidiarrheal medications may just slow the export of the organism and prolong the course of the disease, but they may be used when symptoms are mild for relief from discomfort. Prophylactic treatment with antibiotics is not recommended. Ciprofloxacin is given twice daily, not with meals.

2. A pregnant patient is discussing nutrition and vitamin supplements with the nurse. Which statement by the patient indicates an understanding of the use of nutrition and supplements during pregnancy? a. "I can get adequate folic acid by consuming foods fortified with synthetic folate." b. "I need reduced amounts of vitamin C while pregnant to lower my risk of hemorrhage." c. "I should take vitamin K so my baby won't be vitamin K deficient at birth." d. "Excessive amounts of vitamin A [retinol] can cause birth defects in my baby."

ANS: D Vitamin A in high doses can cause birth defects, so pregnant women should be cautioned about exceeding the UL and probably the RDA for vitamin A while pregnant. Pregnant women need to take 400 to 800 mg of supplemental folic acid in addition to that in food. Vitamin C deficiency, not excess, can lead to bleeding disorders. Maternal intake of vitamin K will not prevent infants from being born vitamin K deficient. PTS: 1 DIF: Cognitive Level: Application REF: pp. 988-989 | pp. 991-992 | p. 994

12. A patient on diuretic therapy calls the clinic because he's had the flu, with "terrible vomiting and diarrhea," and he has not kept anything down for 2 days. He feels weak and extremely tired. Which statement by the nurse is correct? a. "It's important to try to stay on your prescribed medication. Try to take it with sips of water." b. "Stop taking the diuretic for a few days, and then restart it when you feel better." c. "You will need an increased dosage of the diuretic because of your illness. Let me speak to the physician." d. "Please come into the clinic for an evaluation to make sure there are no complications."

ANS: D Vomiting and diarrhea cause fluid and electrolyte loss. The patient must not continue to take the diuretic until these problems have stopped. He needs to be checked for possible hypokalemia and dehydration. The other options are incorrect responses. DIF: COGNITIVE LEVEL: Applying (Application) REF: p. 462 TOP: NURSING PROCESS: Implementation

The nurse is caring for a patient receiving desmopressin [Stimate]. The nurse is performing a physical assessment and notes that the patient is drowsy and listless and complains of a recent onset of headache. Which finding would be most consistent with these symptoms? a.Hyperglycemia b.Hypernatremia c.Hypertension d.Water intoxication

ANS: D Water intoxication is manifested by drowsiness, listlessness, and headache. The patient's signs and symptoms are not consistent with hypernatremia or hyperglycemia, and there is no indication of hypertension.

3. The nurse is working with a graduate nurse to prepare an intravenous dose of potassium. Which statement by the graduate nurse reflects a need for further teaching? a. "We will need to monitor this infusion closely." b. "The infusion rate should not go over 10 mEq/hr." c. "The intravenous potassium will be diluted before we give it." d. "The intravenous potassium dose will be given undiluted."

ANS: D When giving intravenous potassium, the medication must always be given in a diluted form and administered slowly. Intravenous bolus or undiluted forms may cause cardiac arrest. Intravenous rates are not to exceed 10 mEq/hr unless the patient is on a cardiac monitor. Oral forms should be mixed with juice or water or taken according to instructions. DIF: COGNITIVE LEVEL: Analyzing (Analysis) REF: p. 476 TOP: NURSING PROCESS: Implementation

9. A patient accidentally took an overdose of the anticoagulant warfarin (Coumadin), and the nurse is preparing to administer vitamin K as an antidote. Which statement about vitamin K is accurate? a. The vitamin K dose will be given intramuscularly. b. The patient will take oral doses of vitamin K after the initial injection. c. The vitamin K cannot be given if the patient has renal disease. d. The patient will be unresponsive to warfarin therapy for 1 week after the vitamin K is given.

ANS: D When vitamin K is used as an antidote to warfarin therapy, the patient becomes unresponsive to warfarin for approximately 1 week after vitamin K administration. The use of vitamin K products is contraindicated in patients who are in the last few weeks of pregnancy and in patients with severe hepatic disease. Vitamin K is given subcutaneously and not intramuscularly when used to reverse warfarin effects. DIF: COGNITIVE LEVEL: Applying (Application) REF: p. 847 TOP: NURSING PROCESS: Implementation

3. During a patient's therapy with interleukins, the nurse monitors the patient for capillary leak syndrome. Which assessment finding, if present, would indicate this problem? a. Bradycardia b. A dry cough c. Bruising on the skin d. A sudden, 15-pound weight gain

ANS: D With capillary leak syndrome, the capillaries lose their ability to retain vital colloids, and these substances migrate into the surrounding tissues, resulting in massive fluid retention. As a result, heart failure, myocardial infarction, and dysrhythmias may occur. The other options do not reflect capillary leak syndrome. DIF: COGNITIVE LEVEL: Applying (Application) REF: p. 760 TOP: NURSING PROCESS: Assessment

16. The nurse is teaching a class on dysrhythmias and associated therapy. The nurse asks the class, "Which cardiac dysrhythmia would result in the lowest cardiac output, and what treatment would be effective?" The class best demonstrates understanding by responding that ____ results in the lowest cardiac output, and treatment includes ____. a. atrial flutter; lidocaine b. tachycardia; atropine c. first-degree heart block; verapamil [Calan] d. ventricular fibrillation; defibrillation

ANS: D With ventricular fibrillation there is no cardiac output, because the pumping action of the heart stops. Treatment with electrical countershock is indicated to restore cardiac function. Atrial flutter, tachycardia, and first-degree heart block do not result in the lowest cardiac output.

1. When assessing a patient who is receiving a loop diuretic, the nurse looks for the manifestations of potassium deficiency, which would include what symptoms? (Select all that apply.) a. Dyspnea b. Constipation c. Tinnitus d. Muscle weakness e. Anorexia f. Lethargy

ANS: D, E, F Symptoms of hypokalemia include anorexia, nausea, lethargy, muscle weakness, mental confusion, and hypotension. The other symptoms are not associated with hypokalemia. DIF: COGNITIVE LEVEL: Understanding (Comprehension) REF: p. 462 TOP: NURSING PROCESS: Evaluation

Carbonic anhydrase inhibitors indications

Adjunct drugs in the long-term management of open-angle glaucoma Used with miotics to lower intraocular pressure before ocular surgery in certain cases Also useful in the treatment of -Edema -High-altitude sickness -acetazolamide is used in the management of secondary HF when other diuretics aren't effective -CAIS are less potent diuretics than loop diuretics or thiazides- the metabolic acidosis they induce reduces their diuretic effect in 2-4 days

Textbook Case Studies A 50-year-old attorney has self-treated for heartburn for years by drinking large amounts of antacids. She finally made an appointment with her family practice physician, who referred her to a gastroenterologist. Her family practice physician instructed her to stop taking the antacids. 1. Why did physician ask her to stop taking the antacids?

Antacids have been found to have many side effects, such as constipation and diarrhea, and may also lead to rebound of hyperacidity, which may indeed increase the detrimental effects of acid on the refulx disease and on any ulcerated areas in the stomach. Antacids elevate gastric pH, Which is why they have been used in the past; however, antacids do not affect the gastric production of acid, which the other drugs (such as H2-blocking drugs) do so effectively. In addition, long-term self-treatment with antacids may actually delay needed therapy for certain conditions, such as Helicobacter pylori, or more serious conditions, such as gastric ulcers.

2. When discussing glucocorticoids to a patient, what statement by the nurse is accurate regarding the action of these medications? A. They decrease serum sodium and glucose levels. B. They regulate carbohydrate, fat, and protein metabolism. C. They stimulate defense mechanisms to produce immunity. D. They are produced in lower amounts during times of stress.

B

22. MOA for Glucocorticoids: A. Exerts effect by modifying enzyme activity B. Inhibit inflammatory and immune responses

B

25. Glucocorticoids are administered nasally for: A. Control of steroid-responsive bronchospastic states B. Rhinitis and to prevent the recurrence of polyps after surgical removal C. Inflammations of the eye, ear, and skin D. Shock, status asthmaticus and spinal cord injury

B

31. Adverse effects for adrenal drugs of this system include: - Convulsions, headache, vertigo, mood swings, nervousness, insomnia, "steroid psychosis" A. Cardiovascular B. CNS C. Endocrine D. GI E. Integumentary F. Musculoskeletal G. Ocular H. Other

B

5. Discharge teaching for a patient receiving glucocorticoids would include the use of which medication for pain management? A. Aspirin (Acetylsalicylic acid) B. Acetaminophen (Tylenol) C. Ibuprofen D. Naprosyn (Naproxen)

B

A provider has ordered captopril [Capoten] for a patient who has hypertension. The patient reports a history of swelling of the tongue and lips after taking enalapril [Vasotec] in the past. Which action by the nurse is correct? a. Administer the captopril and monitor for adverse effects. b. Hold the dose and notify the provider. c. Request an order to administer fosinopril instead of captopril. d. Reassure the patient that this is not a serious side effect.

B Angioedema is a potentially life-threatening reaction to ACE inhibitors. If a patient has exhibited this reaction with any ACE inhibitor, he or she should not receive ACE inhibitors again. The nurse should hold the medication and notify the provider. Fosinopril is an ACE inhibitor and should not be given.

6. A patient is started on immunosuppressant drugs after kidney transplantation and will be taking azathioprine [Imuran] as part of the drug regimen. The patient asks the nurse why it is necessary to have a specimen for a complete blood count drawn at the beginning of therapy and then periodically thereafter. The nurse explains that azathioprine can alter blood cells and tells the patient to report: a. alopecia. b. easy bruising. c. fatigue. d. gastrointestinal (GI) upset.

B Azathioprine can cause bone marrow suppression, resulting in neutropenia and thrombocytopenia; therefore, a CBC must be evaluated at baseline and periodically thereafter. Patients who have low platelet counts bruise easily, so this symptom should be reported. Alopecia occurs with azathioprine but is not a life-threatening side effect. Fatigue is not a common adverse effect. GI side effects occur but are not life threatening.

5. A patient is taking cyclosporine [Sandimmune] and prednisone to prevent organ rejection after right renal transplantation. The patient is febrile and complains of right-sided flank pain. The nurse reviews the patient's chart and finds that the patient's BUN and serum creatinine are elevated. The cyclosporine trough is 150 ng/mL. What will the nurse do? a. Be concerned that the left kidney is failing. b. Expect the provider to order intravenous methylprednisolone. c. Request an order for a urine culture. d. Suspect nephrotoxicity secondary to an elevated cyclosporine level.

B The patient is showing signs of acute organ rejection, which include pain at the graft site, fever, and elevated BUN and creatinine; therefore, intravenous methylprednisolone is indicated. Because the patient is having pain on the right side, along with fever and elevated renal function test results, there is no reason to suspect that the left kidney is failing or that an infection is present. The cyclosporine level is within normal limits.

A patient diagnosed with a pulmonary embolism is receiving a continuous heparin infusion at 1000 units/hr. Which situation would warrant a call to the physician? (Select all that apply.) A. aPTT of 65 seconds B. aPTT of 40 seconds C. Nosebleeds D. aPTT of 100 seconds E. platelet count of 300,000/mcL

B, C, D Measurement of the aPTT is essential to determine whether the heparin infusion is having the desired effect. If the normal value of the aPTT is 40 seconds, the goal is to achieve a therapeutic range of a factor of 1.5 to 2 (60 to 80 seconds). Because 40 seconds is too short (increases the risk for clotting) and 100 seconds is too long (increases the risk for bleeding), the physician requires notification for adjustment of the infusion rate. Evidence of bleeding, such as nosebleeds, hematuria, and red or tarry stools, warrant a call to the physician. An aPTT of 65 seconds indicates that a therapeutic effect has been achieved, and a platelet count of 300,000/mcL is within normal limits, indicating no evidence of thrombocytopenia.

The nurse would question an order for somatrem (Protropin) in a patient with which condition? A. Dwarfism B. Acromegaly C. Growth failure D. Hypopituitarism

B. Acromegaly Somatrem is a synthetic form of growth hormone. Acromegaly is caused by excessive growth hormone, and thus this drug would be contraindicated.

Which statement should the nurse include in the teaching for a patient who is to be started on zileuton [Zyflo]? A. "Use your zileuton [Zyflo] inhaler every 12 hours." B. "Have your blood drawn once a month for the next 3 months so that your liver function can be checked." C. "Take the zileuton [Zyflo] 2 hours before eating breakfast each day." D. "Take an extra dose of zileuton [Zyflo] if you have an asthmatic attack."

B. "Have your blood drawn once a month for the next 3 months so that your liver function can be checked." Zileuton can injure the liver, as evidenced by increased plasma levels of alanine aminotransferase (ALT). Therefore, ALT should be monitored once a month for 3 months, then every 2 to 3 months for the remainder of the first year, and periodically thereafter. Zileuton is an oral, not an inhaled, medication that is rapidly absorbed with or without food. The timing of administration with regard to meals is insignificant. Zileuton is used for asthma prophylaxis and cannot be used to abort an attack.

A nurse is teaching a patient about montelukast [Singulair]. Which statement by the patient would indicate that the nurse's teaching was effective? A. "I'll take a dose as soon as I feel short of breath and start to cough." B. "While taking this medicine, I may be able to reduce my steroid medication." C. "This is the priority medication for preventing exercise-induced asthma symptoms." D. "If I have nosebleeds or excessive bruising, I'll stop the medication immediately."

B. "While taking this medicine, I may be able to reduce my steroid medication." Montelukast is an antileukotriene agent. Combining montelukast with an inhaled glucocorticoid medication can improve asthma symptoms and thus may allow a reduction in the glucocorticoid dosage. The effects of montelukast develop slowly, so it cannot be used as a quick-relief agent. Short-acting beta2 agonists are preferred for exercise-induced asthma. Montelukast does not affect coagulation, so bleeding and bruising do not occur.

A patient with deep vein thrombosis is receiving an intravenous (IV) heparin infusion. He asks the nurse how this medication will help him. The nurse's response is accurately based on which concept? A. Heparin prevents the activation of vitamin K and thus blocks synthesis of some clotting factors. B. Heparin suppresses coagulation by helping antithrombin perform its natural functions. C. Heparin works by converting plasminogen to plasmin, which in turn dissolves the clot matrix. D. Heparin inhibits the enzyme responsible for platelet activation and aggregation within vessels.

B. Heparin suppresses coagulation by helping antithrombin perform its natural functions. Heparin is an anticoagulant that works by helping antithrombin inactivate thrombin and factor Xa, reducing the production of fibrin and thus decreasing the formation of clots.

A patient admitted with deep vein thrombosis (DVT) and subsequent pulmonary embolism (PE) requires immediate anticoagulation. What would be appropriate in this situation, given that the patient has a history of heparin-induced thrombocytopenia (HIT)? A. Warfarin (Coumadin) B. Lepirudin (Refludan) C. Bivalirudin (Angiomax) D. Eptifibatide (Integrilin)

B. Lepirudin (Refludan) Lepirudin (Refludan) and argatroban are indicated for the treatment of thrombosis in patients with a history of HIT. Because these medications are given as an initial IV bolus followed by a continuous infusion, the desired effect of direct thrombin inhibition is achieved more quickly. Because of warfarin's delayed onset of effects, it is not useful in emergency situations, such as pulmonary embolism (PE). Intravenous bivalirudin given in combination with aspirin helps prevent clot formation in patients undergoing coronary angioplasty. Eptifibatide (Integrilin), a glycoprotein IIb/IIIa receptor inhibitor, is an antiplatelet drug that is used short term to prevent ischemic events in patients who have acute coronary syndrome or who are undergoing percutaneous coronary intervention.

The nurse is caring for a patient receiving clopidogrel (Plavix) to prevent blockage of coronary artery stents. Which other drug on the patient's medication administration record may reduce the antiplatelet effects of clopidogrel? A. Aspirin (Bayer) B. Omeprazole (Prilosec) C. Acetaminophen (Tylenol) D. Warfarin (Coumadin)

B. Omeprazole (Prilosec) Omeprazole and other proton pump inhibitors may reduce the antiplatelet effects of clopidogrel. Patients sometimes take them to reduce gastric acidity and the risk of gastrointestinal (GI) bleeding.

To achieve therapeutic effectiveness, a nurse teaches a patient with chronic asthma to use an inhaled glucocorticoid medication according to which schedule? A. Only in an emergency B. On a continuing, daily basis C. To abort an asthma attack D. 2 weeks on, 2 weeks off

B. On a continuing, daily basis Glucocorticoid medications are the first-line therapy for asthma to reduce symptoms of inflammation. They should be taken for prophylaxis on a daily basis. Therapeutic effects develop slowly, so these drugs cannot be taken to abort an asthma attack or in an emergency. They are most effective when administered on a fixed schedule, not PRN.

A history of allergy to which substance is a contraindication to the use of ipratropium/albuterol [Combivent]? A. Mold B. Peanuts C. Penicillin D. Dairy products

B. Peanuts Patients with peanut allergy should avoid Combivent, which contains soya lecithin as a carrier. Soya is in the same plant family as peanuts, and about 10% of people with peanut allergy are cross-allergic to soya. Allergy to the remainder of the options is not a contraindication.

A nurse teaches a patient with chronic obstructive pulmonary disease (COPD) about the adverse effects of tiotropium [Spiriva]. Which behavior by the patient would indicate that the teaching has been effective? A. Combines tiotropium with an antacid B. Sucks on hard candy as needed C. Prevents constipation with a stool softener D. Wears long sleeves and a wide-brim hat

B. Sucks on hard candy as needed Tiotropium is an anticholinergic medication used to relieve bronchospasm associated with COPD. The most common adverse effect is dry mouth, and patients can suck on hard candy for symptomatic relief. It is not necessary to take an antacid medication, use a daily stool softener, or wear protective clothing when taking tiotropium.

Which drugs have an action similar to that of the naturally occurring hormone ADH? (Select all that apply) A. cosyntropin (Cortrosyn) B. desmopressin (DDAVP) C. somatropin (Humatrope) D. vasopressin (Pitressin) E. octreotide (Sandostatin)

B. desmopressin (DDAVP) D. vasopressin (Pitressin)

C. Cellulose

Before administering an antileukotriene medication, the nurse would assess the patient for allergies to which substance? (Select all that apply.) A. Lactose B. Povidone C. Cellulose D. Chlorhexedine

3. The nurse would question a prescription for steroids in a patient with which condition? A. Asthma B. Spinal cord injury C. Diabetes mellitus D. Rheumatoid arthritis

C

32. Adverse effects for adrenal drugs of this system include: - Growth suppression, Cushing's syndrome, menstrual irregularities, carbohydrate intolerance, hyperglycemia A. Cardiovascular B. CNS C. Endocrine D. GI E. Integumentary F. Musculoskeletal G. Ocular H. Other

C

8. The nurse has a prescription for a patient to receive prednisone (Deltasone) to treat contact dermatitis. The nurse would question this prescription for this patient with what condition? A. Asthma B. Multiple sclerosis C. Acquired immune deficiency syndrome (AIDS) D. Chronic obstructive pulmonary disease

C

A patient begins taking an ACE inhibitor and complains of a dry cough. What does the nurse correctly tell the patient about this symptom? a. It indicates that a serious side effect has occurred. b. It is a common side effect that occurs in almost all patients taking the drug. c. It may be uncomfortable enough that the drug will need to be discontinued. d. It occurs frequently in patients taking the drug but will subside over time.

C A cough occurs in about 10% of patients taking ACE inhibitors and is the most common reason for discontinuing therapy. It does not indicate a serious condition. It occurs in about 10% of all patients and is more common in women, older adults, and those of Asian ancestry. It does not subside until the medication is discontinued.

A patient who stops taking an ACE inhibitor because of its side effects will begin taking an angiotensin II receptor blocker (ARB) medication. Which side effect of ACE inhibitors will not occur with an ARB medication? a. Angioedema b. Cough c. Hyperkalemia d. Renal failure

C ARBs do not produce clinically significant hyperkalemia. Angioedema may occur with ARBs, but the incidence is lower than with ACE inhibitors. They do not promote the accumulation of bradykinin in the lungs and produce a lower incidence of cough. An increased risk of cancer may be a concern with ARBs but is not a concern with ACE inhibitors. As with ACE inhibitors, renal failure can occur in patients with bilateral renal artery stenosis or stenosis in the artery to a single remaining kidney.

The parent of a 5-year-old child who has had four urinary tract infections in the past year asks the nurse why the provider doesn't just order an antibiotic for the child's current symptoms of low-grade fever, flank pain, and dysuria since these are similar symptoms as before. Which is the most important reason given by the nurse? a. "Your child may need to be hospitalized for treatment." b. "Your child may need a urine culture before and after treatment." c. "Your child may need tests to assess for urinary tract abnormalities." d. "Your child may need additional medications, such as urinary tract antiseptics."

C Children with recurrent urinary tract infections should be assessed for underlying urinary tract abnormalities to help determine a possible cause for recurrence. This child has mild fever and therefore may not require hospitalization. Urine cultures are important when treating patients with recurrent UTI prophylactically, but this is not the most important consideration. Urinary tract antiseptics are used to treat uncomplicated lower urinary tract infections.

12. The nurse is administering medications to a patient who is receiving cyclosporine [Sandimmune]. Which medication, when administered concurrently with cyclosporine, would warrant a reduction in the dosage of cyclosporine? a. Phenytoin [Dilantin] b. Prednisone c. Ketoconazole [Nizoral] d. Trimethoprim/sulfamethoxazole [Bactrim]

C Concurrent use of ketoconazole would warrant a reduction in the dosage of cyclosporine. Ketoconazole often is given concurrently with cyclosporine so that the patient's dosage of cyclosporine, which is costly, can be reduced. Phenytoin would cause a decrease in cyclosporine levels. Prednisone often is given concurrently with cyclosporine to suppress the immune response. Trimethoprim/sulfamethoxazole reduces levels of cyclosporine in the body.

3. A nurse provides teaching to a patient who has undergone kidney transplantation and will begin taking cyclosporine [Sandimmune], a glucocorticoid, and sirolimus [Rapamune]. Which statement by the patient indicates understanding of the teaching? a. "I should take sirolimus at the same time as the cyclosporine." b. "I will need to have my blood sugar checked regularly." c. "I will need to take an antibiotic to prevent lung infections." d. "Taking this combination of drugs lowers my risk of kidney damage."

C Immunosuppressant drugs increase the risk of infections, especially the BK virus, which can cause renal damage, and other organisms that cause lung infections. Patients taking these drugs must take antibiotics for the first 12 months to help prevent infection. Sirolimus and cyclosporine should be taken 4 hours apart. Patients with diabetes may have trouble with glucose tolerance and require monitoring. Taking cyclosporine and sirolimus increases the risk of renal damage.

A pregnant female patient with bacteriuria, suprapubic pain, urinary urgency and frequency, and a low-grade fever is allergic to sulfa, ciprofloxacin, and amoxicillin. The nurse knows that the best alternative for treating this urinary tract infection is with: a. cephalexin [Keflex]. b. fosfomycin [Monurol]. c. methenamine [Hiprex]. d. nitrofurantoin [Macrodantin].

C Methenamine is an excellent second-line drug for this patient and is indicated because of the patient's multiple drug sensitivities. It is safe in pregnancy, and there is no drug resistance. Nitrofurantoin has potential harmful effects on the fetus and should not be used during pregnancy. Single-dose regimens are not recommended in pregnant women. Cephalexin can have cross-reactivity with amoxicillin.

A patient who is taking furosemide [Lasix] and digoxin will begin taking captopril [Capoten]. The nurse is providing information about the drug. Which statement by the patient indicates a need for further teaching? a. "I can use acetaminophen for analgesia if needed." b. "I should stop taking the Lasix about a week before starting the Capoten." c. "I should take this medication on a full stomach." d. "I will need to have blood tests done every 2 weeks for a few months."

C Most oral formulations of ACE inhibitors may be given without regard for meals; captopril and moexipril, however, should be given 1 hour before meals. Nonsteroidal anti-inflammatory medications should be avoided; acetaminophen is not contraindicated. Patients taking diuretics should stop taking them 1 week before starting an ACE inhibitor to minimize the risk of hypotension. A WBC with differential should be evaluated at baseline and, after treatment starts, every 2 weeks for several months to monitor for neutropenia.

11. The nurse is caring for a patient after recent renal transplantation. The patient is taking sirolimus [Rapamune] to prevent transplant rejection. What other medications would the nurse expect the patient to be taking? a. Rifampin and ketoconazole b. Carbamazepine and phenobarbital c. Cyclosporine and glucocorticoids d. Amphotericin B and erythromycin

C Sirolimus should be used in conjunction with cyclosporine and glucocorticoids to reduce the risk of transplant rejection. Rifampin can reduce and ketoconazole can increase therapeutic levels of sirolimus and cyclosporine; therefore, they are not indicated for transplant patients. Carbamazepine and phenobarbital are not indicated, because they can reduce the therapeutic levels of sirolimus and cyclosporine. Amphotericin B and erythromycin are not indicated, because they can increase the therapeutic levels of sirolimus and cyclosporine.

A patient presents to the emergency department with complaints of chills, severe flank pain, dysuria, and urinary frequency. The patient has a temperature of 102.9°F, a pulse of 92 beats per minute, respirations of 24 breaths per minute, and a blood pressure of 119/58 mm Hg. The nurse would be correct to suspect that the patient shows signs and symptoms of: a. acute cystitis. b. urinary tract infection. c. pyelonephritis. d. prostatitis.

C The nurse should suspect pyelonephritis. Pyelonephritis is characterized by fever, chills, severe flank pain, dysuria, urinary urgency and frequency, and pyuria and bacteriuria. Clinical manifestations of acute cystitis include dysuria, urinary urgency and frequency, suprapubic discomfort, pyuria, and bacteriuria. Urinary tract infections (UTIs) are very general and are classified by their location. These symptoms are specific to pyelonephritis. Prostatitis is manifested by high fever, chills, malaise, myalgia, localized pain, and various UTI symptoms but not by severe flank pain.

A 30-year-old male patient reports having two to four urinary tract infections a year. What will the nurse expect to teach this patient? a. "Make sure you void after intercourse and drink extra fluids to stay well hydrated." b. "We will treat each infection as a separate infection and treat with short-course therapy." c. "You will need to take a low dose of medication for 6 months to prevent infections." d. "You will need to take antibiotics for 4 to 6 weeks each time you have an infection."

C This patient has reinfection of his urinary tract at a rate of more than three per year, which is an indication for long-term prophylaxis. Voiding after intercourse is a good teaching point for sexually active women to prevent urinary infections, but it is not a sufficient preventive measure for recurrent infections in men. Short-course therapy may be used for each occurrence of infection if the reinfection rate is less than three per year. Long-term treatment for individual infections is recommended if relapse occurs or if infections do not clear with shorter-term therapy.

Which instruction about clopidogrel (Plavix) should be included in the discharge teaching for a patient who has received a drug-eluting coronary stent? A. "Constipation is a common side effect of clopidogrel, so take a stool softener daily." B. "If you see blood in your urine or black stools, stop the clopidogrel immediately." C. "Check with your healthcare provider before taking any over-the-counter medications for gastric acidity." D. "Keep the amounts of foods containing vitamin K, such as mayonnaise, canola and soybean oil, and green, leafy vegetables, consistent in your diet."

C. "Check with your healthcare provider before taking any over-the-counter medications for gastric acidity." Proton pump inhibitors (PPIs), such as omeprazole (Prilosec), and CYP2C1 inhibitors, such as cimetidine (Tagamet), can be purchased over the counter to treat heartburn. However, patients taking clopidogrel should consult their healthcare provider before using them. PPIs and CYP2C1 inhibitors can reduce the antiplatelet effects of clopidogrel. Diarrhea (5% incidence), not constipation, is a side effect of clopidogrel. Patients should immediately contact the healthcare provider if signs of bleeding occur, such as bloody urine, stool, or emesis. The drug should not be stopped until the prescriber advises it, because this could lead to coronary stent restenosis. Consistency of vitamin K intake is indicated while taking warfarin (Coumadin).

A 52-year-old male patient presents to the emergency department with symptoms of acute myocardial infarction. After a diagnostic workup, the healthcare provider prescribes a 15-mg IV bolus of alteplase (tPA), followed by 50 mg infused over 30 minutes. In monitoring this patient, the nurse understands that which symptom, if present, indicates the most likely adverse reaction to this drug? A. Urticaria, itching, and flushing B. Blood pressure of 90/50 mm Hg C. Decreasing level of consciousness D. Potassium level of 5.5 mEq/L

C. Decreasing level of consciousness Unlike streptokinase, alteplase does not cause an allergic reaction or hypotension. The greatest risk with this drug is bleeding, with intracranial bleeding being the greatest concern. A decreasing level of consciousness indicates intracranial bleeding. Thrombolytic agents, such as alteplase, do not typically cause an elevated potassium level.

A patient with asthma is scheduled to start taking a glucocorticoid medication with a metered-dose inhaler (MDI). The nurse should give the patient which instruction about correct use of the inhaler? A. "After you inhale the medication once, repeat until you obtain symptomatic relief." B. "Wait no longer than 30 seconds after the first puff before taking the second one." C. "Use a spacer with the inhaler and rinse your mouth after each dose administration." D. "Breathe in through the nose and hold for 2 seconds just before activating the inhaler."

C. "Use a spacer with the inhaler and rinse your mouth after each dose administration." Spacers are available for use with MDIs to prevent the patient from swallowing the dose and to allow for maximum delivery of medication to the lungs. Rinsing the mouth after administration is important for inhaled glucocorticoids to prevent candidiasis. Glucocorticoid inhalers are used for long-term prophylaxis of asthma, not for symptomatic relief. When two puffs are needed, an interval of at least 1 minute should separate the first puff from the second. Inhaling through the mouth just before activating the MDI is the proper technique.

A patient who takes cromolyn for exercise-induced bronchospasm should follow which approach for maximum therapeutic effectiveness? A. It should be used infrequently because of systemic adverse effects. B. One 10-mg tablet should be taken at least 2 hours before exercising. C. It should be administered by inhalation 15 minutes before anticipated exertion. D. It should be used as a quick-relief agent if exercise triggers asthma symptoms.

C. It should be administered by inhalation 15 minutes before anticipated exertion. Cromolyn suppresses inflammation through inhibition of histamine release. It must be administered at least 15 minutes before exertion to prevent exercise-induced bronchospasm. It is administered by inhalation only. It is not a bronchodilator and cannot abort an ongoing attack. It is one of the safest antiasthma medications and has no systemic adverse effects.

A patient admitted with atrial fibrillation is receiving a continuous heparin infusion as well as daily warfarin (Coumadin). What is an appropriate action by the nurse? A. Contact the healthcare provider for an order to stop the heparin, because the aPTT is 60 seconds. B. Contact the healthcare provider to request discontinuation of the heparin, because the patient is receiving Coumadin. C. Notify the healthcare provider that the INR is 3 and anticipate that the heparin will be discontinued. D. Prepare to administer protamine sulfate, because the patient has received two anticoagulants at the same time.

C. Notify the healthcare provider that the INR is 3 and anticipate that the heparin will be discontinued. The INR is within a therapeutic range (2 to 3), so the nurse should call the health care provider to get the heparin infusion discontinued. An aPTT of 60 indicates the heparin is therapeutic with no indication that the patient has achieved the therapeutic effect of warfarin. The patient will be on a combination of heparin and warfarin until the therapeutic effect of warfarin has been achieved, and this may take several days. The patient will have no protective effect against clotting if the heparin is reversed with protamine sulfate and the INR is not within a therapeutic range.

A patient who is about to be given octreotide is also taking a diuretic, IV heparin, ciprofloxacin (Cipro), and an opioid as need for pain. The nurse will monitor for what possible interaction? A. Hypokalemia due to an interaction with the diuretic B. Decreased anticoagulation due to an interaction with the heparin C. Prolongation of the QT internal due to an interaction with ciprofloxacin D. Increased sedation if the opioid is given

C. Prolongation of the QT internal due to an interaction with ciprofloxacin

Which outcome should a nurse establish as a priority for a patient taking an oral glucocorticoid for long-term treatment of asthma? A. Increases the daily intake of vitamin D and calcium B. Records daily peak expiratory flow rates C. Supplements additional doses at times of stress D. Uses alternate-day therapy to reduce adverse effects

C. Supplements additional doses at times of stress Adrenal suppression can be profound with oral glucocorticoid use. It is a priority that patients take supplemental oral or intravenous doses at times of stress; failure to do so can be fatal. Alternate-day dosing, using a peak flowmeter, and minimizing bone loss with vitamin D and calcium intake are important; however, they are not as important as supplemental doses of glucocorticoid at times of stress.

12. A patient who has cystic fibrosis has a Pseudomonas aeruginosa infection and the provider has ordered aztreonam [Cayston]. What will the nurse teach this patient about administration of this drug? a. Administer the drug intramuscularly twice daily. b. Give a daily dose every day for 28 days and then stop. c. Inhale the powdered drug as ordered three times each day. d. Use the nebulizer to administer the drug three times daily.

Cayston is a form of aztreonam formulated for inhalation administration for patients with cystic fibrosis who have P. aeruginosa lung infections. The reconstituted powder is given using a nebulizer system three times daily for 28 days followed by 28 days off. This form of the drug is not given IM. The dose is three times daily. The drug is reconstituted and administered via a nebulizer.

The patient is being discharged home with furosemide (Lasix). When providing discharge teaching, which instruction will the nurse include? A.Avoid prolonged exposure to the sun. B.Avoid foods high in potassium content. C.Stop taking the medication if you feel dizzy. D.Weigh yourself once a week and report a gain or loss of more than 1 pound.

Correct answer: A Rationale: Patients taking furosemide (Lasix) should avoid prolonged exposure to the sun because the drug can cause photosensitivity. Although orthostatic hypotension is a possible adverse effect of the medication, patients should not stop taking the medication without consultation with their health care provider. Patients should weigh themselves once a day and report a weight gain or loss of approximately 3 pounds. Patients taking furosemide (Lasix) should be encouraged to eat foods rich in potassium.

The patient is ordered furosemide (Lasix). Before administering furosemide, it is most important for the nurse to assess the patient for allergies to which drug class? A.Aminoglycosides B.Sulfonamides C.Macrolides D.Penicillins

Correct answer: B Rationale: The nurse should assess patients receiving furosemide (Lasix) for cross-sensitivity to sulfonamides. Although allergy to sulfonamide antibiotics is listed as a contraindication, analysis of the literature indicates that cross-reaction with the loop diuretics is unlikely to occur. Loop diuretics are commonly given to such patients in clinical practice. The nurse should closely monitor these patients.

Which location is the area where the highest percentage of sodium and water are resorbed back into the bloodstream? A.Glomerulus B.Proximal tubule C.Ascending loop of Henle D.Distal tubule

Correct answer: B Rationale: The proximal convoluted (twisted) tubule or, more simply, proximal tubule, anatomically follows the glomerulus and returns 60% to 70% of the sodium and water from the filtered fluid back into the bloodstream. Another 20% to 25% of sodium is resorbed into the bloodstream in the ascending loop of Henle. The remaining 5% to 10% of sodium resorption takes place in the distal convoluted tubule, often called simply the distal tubule, which anatomically follows the ascending loop of Henle. The glomerulus does not resorb sodium or water, but instead is the point of initial filtration of the blood.

Two days after admission, the nurse is reviewing laboratory results of the patient. Which is the most common electrolyte finding resulting from the administration of furosemide (Lasix)? A.Hypocalcemia B.Hypophosphatemia C.Hypokalemia D.Hypomagnesemia

Correct answer: C Rationale: Of all of the adverse effects of furosemide (Lasix) administration, hypokalemia is of serious clinical importance. To prevent hypokalemia, patients often receive potassium supplements along with furosemide. The other electrolyte disturbances listed do not occur as a result of furosemide (Lasix) therapy.

A patient is suspected of having adrenocortical insufficiency. The nurse expects to administer which drug to aid in the diagnosis of this condition? A. octreotide (Sandostatin) B. vasopressin (Pitressin) C. somatropin (Humatrope) D. cosyntropin (Cortrosyn)

D. cosyntropin (Cortrosyn)

1. Based on the nurse's knowledge of glucocorticoids, what instructions should be given for this drug? A. Take the medication every evening. B. Inform the patient that the drug can be taken with coffee. C. Advise that the drug does not have to be tapered before stopped. D. Advise to take drug with milk, other dairy products, or food.

D

17. Anti-adrenals, adrenal steroid inhibitors, are: A. Topical B. Inhaled C. Nasal D. Systemic E. All of the above

D

29. Use adrenal drugs with caution for patients with: A. Gastritis, reflux disease, ulcer disease B. Diabetes C. Cardiac/renal/liver dysfunction D. All of the above

D

33. Adverse effects for adrenal drugs of this system include: - Peptic ulcers with possible perforation, pancreatitis, abdominal distention A. Cardiovascular B. CNS C. Endocrine D. GI E. Integumentary F. Musculoskeletal G. Ocular H. Other

D

9. A pediatric patient has been prescribed methylprednisolone (Solu-Medrol) for an allergic response to food. The dose prescribed is 0.5 mg/kg IV every 6 hours. The patient's weight is 11.4 kg. How much will the nurse administer for one dose? A. 4.3 mg B. 3.1 mg C. 7.5 mg D. 5.7 mg

D

The nurse admitting a patient with acromegaly anticipates administering which medication? A. desmopressin (DDAVP) B. corticotropin (Acthar) C. somatropin (Nutropin) D. octreotide (Sandostatin)

D. octreotide (Sandostatin) Octreotide suppresses growth hormone, the culprit of acromegaly.

A female patient taking an ACE inhibitor learns that she is pregnant. What will the nurse tell this patient? a. The fetus most likely will have serious congenital defects. b. The fetus must be monitored closely while the patient is taking this drug. c. The patient's prescriber probably will change her medication to an ARB. d. The patient should stop taking the medication and contact her provider immediately.

D ACE inhibitors are known to cause serious fetal injury during the second and third trimesters of pregnancy. Whether injury occurs earlier in pregnancy is unknown, and the incidence probably is low. However, women should be counseled to stop taking the drug if they become pregnant, and they should not take it if they are contemplating becoming pregnant. Women who take ACE inhibitors in the first trimester should be counseled that the risk to the fetus is probably low. Women should stop taking the drug when pregnant. ARBs carry the same risk as ACE inhibitors.

A patient with hypertension is prescribed an angiotensin-converting enzyme (ACE) inhibitor. The nurse reviewing this patient's chart before administering the medication will be most concerned about which other disease process? a. Bronchial asthma b. Coronary artery disease c. Diabetes mellitus d. Renal artery stenosis

D ACE inhibitors can cause severe renal insufficiency in patients with bilateral renal artery stenosis or stenosis in the artery to a single remaining kidney. Bronchial asthma, coronary artery disease, and diabetes mellitus are not comorbidities that are contraindications to treatment with an ACE inhibitor.

A female patient who is not taking any other medications is prescribed aliskiren [Tekturna], a direct renin inhibitor (DRI). The nurse reviews medication information with the patient. Which statement by the patient indicates understanding of the teaching? a. "If I get pregnant, I should stop taking this drug by the second trimester." b. "If I take this drug with a high-fat meal, it will be more effective." c. "I should restrict my potassium intake while taking this drug." d. "I should take this medication 1 hour before sitting down to a meal."

D DRIs should be taken on an empty stomach to improve their bioavailability. Patients should not take this drug when pregnant and should stop immediately upon finding out they are pregnant. Dosing with a high-fat meal reduces the drug's bioavailability. DRIs rarely cause hyperkalemia when taken alone.

A young, nonpregnant female patient with a history of a previous urinary tract infection is experiencing dysuria, urinary urgency and frequency, and suprapubic pain of 3 days' duration. She is afebrile. A urine culture is positive for more than 100,000/mL of urine. The nurse caring for this patient knows that which treatment is most effective? a. A 14-day course of amoxicillin with clavulanic acid [Augmentin] b. A 7-day course of ciprofloxacin [Cipro] c. A single dose of fosfomycin [Monurol] d. A 3-day course of trimethoprim/sulfamethoxazole [Bactrim]

D Short-course therapy is recommended for uncomplicated, community-acquired lower urinary tract infections. The short course is more effective than a single dose, and compared with longer-course therapies, it is less costly, has fewer side effects, and is more likely to foster compliance. Amoxicillin with clavulanic acid is a second-line drug used for pyelonephritis. Fosfomycin is a second-line drug and can be useful in patients with drug allergies.

An older male patient comes to the clinic with complaints of chills, malaise, myalgia, localized pain, dysuria, nocturia, and urinary retention. The nurse would most likely suspect that the patient has: a. acute cystitis. b. urinary tract infection. c. pyelonephritis. d. prostatitis.

D The nurse should suspect prostatitis, which is manifested by high fever, chills, malaise, myalgia, and localized pain and may also be manifested by dysuria, nocturia, and urinary urgency, frequency, and retention. Clinical manifestations of acute cystitis include dysuria, urinary urgency and frequency, suprapubic discomfort, pyuria, and bacteriuria. Urinary tract infections are very general and are classified by their location. Pyelonephritis is characterized by fever, chills, severe flank pain, dysuria, and urinary frequency and urgency, as well as by pyuria and bacteriuria.

Which patient with a urinary tract infection will require hospitalization and intravenous antibiotics? a. A 5-year-old child with a fever of 100.5°F, dysuria, and bacteriuria b. A pregnant woman with bacteriuria, suprapubic pain, and fever c. A young man with dysuria, flank pain, and a previous urinary tract infection d. An older adult man with a low-grade fever, flank pain, and an indwelling catheter

D The patient with an indwelling catheter and signs of pyelonephritis shows signs of a complicated UTI, which is best treated with intravenous antibiotics. The other three patients show signs of uncomplicated urinary tract infections that are not severe and can be treated with oral antibiotics.

A patient is taking oral theophylline for maintenance therapy of stable asthma. A nurse instructs the patient to avoid using which substance to prevent a complication? A. Echinacea B. Cimetidine [Tagamet] C. Sunscreen products D. Caffeine

D. Caffeine Theophylline is a methylxanthine that provides benefits through bronchodilation. It is used to reduce the frequency and severity of asthma attacks, especially those occurring at night. Caffeine also is a methylxanthine, and its pharmacologic actions may intensify the adverse effects of theophylline on the central nervous system (CNS) and heart. Sources of caffeine should be avoided. It is not necessary to avoid taking cimetidine or echinacea or using sunscreen products while taking theophylline.

The nurse is caring for a patient who takes warfarin (Coumadin) for prevention of deep vein thrombosis. The patient has an international normalized ratio (INR) of 1.2. Which action by the nurse is most appropriate? A. Prepare to administer protamine sulfate. B. Continue with the current prescription. C. Prepare to administer vitamin K. D. Call the healthcare provider to increase the dose.

D. Call the healthcare provider to increase the dose. An INR in the range of 2 to 3 is considered the level for warfarin therapy. For a level of 1.2, the nurse should contact the healthcare provider to discuss an order for an increased dose.

The nurse is reviewing the medication list for a patient who will be starting therapy with somatropin. Which type of drug would raise a concern that needs to be addressed before the patient starts the somatropin? A. Nonsteroidal anti-inflammatory drug for arthritis B. Antidepressant drug C. Penicillin D. Glucocorticoid

D. Glucocorticoid

A patient is admitted to the emergency department with acute severe exacerbation of asthma. Which drug should the nurse anticipate will be included in the treatment plan? A. Oral theophylline [Elixophyllin] B. Subcutaneous omalizumab [Xolair] C. Inhaled mometasone furoate [Asmanex] D. High-dose albuterol [Proventil] via nebulizer treatment

D. High-dose albuterol [Proventil] via nebulizer treatment Nebulized high-dose SABAs, such as albuterol, are administered to relieve airflow obstruction. Oral theophylline is used for maintenance therapy of chronic stable asthma, not for treatment of exacerbation. Omalizumab is a second-line agent indicated for allergy-related asthma and only when preferred options have failed. Inhaled glucocorticoids, such as mometasone furoate, are not used to abort an acute attack. During an exacerbation they are administered systemically.

Fondaparinux (Arixtra) is not approved for use in which circumstance? A. Prevention of deep vein thrombosis (DVT) after knee replacement B. Treatment of acute pulmonary embolism (PE) (in conjunction with warfarin) C. Prevention of deep vein thrombosis (DVT) after abdominal surgery D. Prevention of ischemic complications in patients with unstable angina

D. Prevention of ischemic complications in patients with unstable angina Enoxaparin (Lovenox), not Arixtra, is approved for use in preventing ischemic complications in patients with unstable angina, non-Q-wave myocardial infarction (MI), and ST-segment elevation myocardial infarction (STEMI). Arixtra is approved for (1) preventing DVT after hip surgery, knee replacement, and abdominal surgery; and (2) treating acute PE and acute DVT in conjunction with warfarin.

After administering somatropin (Serostim), the nurse would assess for adverse effects by monitoring which parameters? A. Serum potassium levels B. Mental status C. Respiratory rate D. Serum glucose levels

D. Serum glucose levels Hyperglycemia and hypoglycemia are potential adverse effects of somatropin therapy.

ANS: D Long-term self-medication with antacids may mask symptoms of serious underlying diseases, such as bleeding ulcer or malignancy. Patients with ongoing symptoms need to undergo regular medical evaluations, because additional medications or other interventions may be needed.

During an admission assessment, the patient tells the nurse that he has been self-treating his heartburn for 1 year with over-the-counter Prilosec OTC (omeprazole, a proton pump inhibitor). The nurse is aware that this self-treatment may have which result? a.No serious consequences b.Prevention of more serious problems, such as an ulcer c.Chronic constipation d.Masked symptoms of serious underlying diseases

Textbook Case Studies A 50-year-old attorney has self-treated for heartburn for years by drinking large amounts of antacids. She finally made an appointment with her family practice physician, who referred her to a gastroenterologist. Her family practice physician instructed her to stop taking the antacids. In a few weeks, the attorney had an endoscopy, and it was discovered that she had gastroesophgeal reflux disease (GERD) and gastritis secondary to stress-induced hyper-acidity. The gastoenterologist has prescribed the proton pump inhibitor (PPI) omeprazole (Prilosec) 20 mg once a day. 2. What other conditions will the gastroenterologist test for during this diagnostic stage?

Evaluate baseline renal and liver function. In addition, the gastroenterologist will test for the presence of H. pylori antibodies.

37. Adverse effects for adrenal drugs of this system include: - Weight gain A. Cardiovascular B. CNS C. Endocrine D. GI E. Integumentary F. Musculoskeletal G. Ocular H. Other

H

B. less sedative effect.

Loratadine (Claritin) has an advantage over traditional antihistamines such as diphenhydramine (Benadryl) in that loratadine has A. less risk of cardiac arrhythmias. B. less sedative effect. C. increased bronchodilating effects. D. less gastrointestinal upset.

C. Allergens

Mast cell stabilizers are most effective in treating bronchoconstriction associated with which condition? A. Emphysema B. Exposure to cold C. Allergens D. Infection

C. Increase fluid intake to decrease viscosity of secretions.

Patient teaching regarding expectorants should instruct the patient to perform which action? A. Restrict fluids to decrease mucus production. B. Take the medication once a day only, usually at bedtime. C. Increase fluid intake to decrease viscosity of secretions. D. Increase fiber and fluid intake to prevent constipation.

C. "This medication works by preventing the inflammation that causes your asthma attack."

Patient teaching regarding the use of antileukotriene drugs such as zafirlukast (Accolate) would include which statement? A. "Take the medication as soon as you begin wheezing." B. "It will take about 3 weeks before you notice a therapeutic effect." C. "This medication works by preventing the inflammation that causes your asthma attack." D. "Increase fiber and fluid in your diet to prevent the common side effect of constipation."

Which measurements should a nurse obtain to evaluate the effects of androgen therapy on the epiphyses of a patient with hypogonadism? Monthly height and weight Periodic hand and wrist x-rays Body mass/fat ratio indices Blood testosterone levels

Periodic hand and wrist x-rays

Textbook Case Studies A 50-year-old attorney has self-treated for heartburn for years by drinking large amounts of antacids. She finally made an appointment with her family practice physician, who referred her to a gastroenterologist. Her family practice physician instructed her to stop taking the antacids. In a few weeks, the attorney had an endoscopy, and it was discovered that she had gastroesophgeal reflux disease (GERD) and gastritis secondary to stress-induced hyper-acidity. The gastoenterologist has prescribed the proton pump inhibitor (PPI) omeprazole (Prilosec) 20 mg once a day. 3. What is the rational for use of the PPIs to treat GERD?

Proton pump inhibitors (PPIs) are used as long-term therapy to promote and maintain the healing of gastroesophageal reflux disease (GERD) and other hypersecretory disorders. The PPI inhibits the production of hydrochloric acid in the stomach.

C. fexofenadine (Allegra) D. loratadine (Claritin) E. cetirizine (Zyrtec)

The nurse is discussing use of antihistamines for allergic rhinitis. Which medications would be included in the list of H1 antagonists used in the treatment of allergic rhinitis? (Select all that apply.) A. ranitidine (Zantac) B. nizatidine (Axid) C. fexofenadine (Allegra) D. loratadine (Claritin) E. cetirizine (Zyrtec)

ANS: B, C, E Antacids neutralize acid in the stomach. Magnesium-based antacids cause diarrhea, and aluminum-based antacids cause constipation. Calcium-based antacids often cause rebound hyperacidity.

The nurse is providing patient teaching about antacids. Which statements about antacids are accurate? (Select all that apply.) a.Antacids reduce the production of acid in the stomach. b.Antacids neutralize acid in the stomach. c.Rebound hyperacidity may occur with calcium-based antacids. d.Aluminum-based antacids cause diarrhea. e.Magnesium-based antacids cause diarrhea.

A. antihistamines are best tolerated when taken with meals B. the patient can chew gum if he or she experiences dry mouth C. drowsiness is a frequent side effect of antihistamines E.the patient should avoid drinking alcoholic beverages while on these drugs

The nurse is providing patient teaching about antihistamine use. Will include which information? select all that apply...... A. antihistamines are best tolerated when taken with meals B. the patient can chew gum if he or she experiences dry mouth C. drowsiness is a frequent side effect of antihistamines D. OTC meds are usually safte to use with antihistamines E.the patient should avoid drinking alcoholic beverages while on these drugs

A. "If I develop a puffy face, I will stop taking methylprednisolone (Medrol) immediately."

The nurse is providing teaching to a group of individuals with chronic obstructive pulmonary disease (COPD) at a community center. Which statement by one of the attendees indicates that further teaching is needed? A. "If I develop a puffy face, I will stop taking methylprednisolone (Medrol) immediately." B. "I will inform my prescriber of any weight gain of 2 pounds or more in 24 hours or 5 pounds or more in 1 week." C. "I use omalizumab (Xolair) to control my asthma but not for an acute asthma attack." D. "When taking theophylline (Theo-Dur), I will advise my prescriber if I experience epigastric pain."

B. an expectorant

The nurse is reviewing a patient's medication orders for PRN medications that can be given to a patient who has bronchitis with a productive cough. Which drug will the nurse choose? A. an antitussive B. an expectorant C. an antihistamine D. a decongestant

ANS: D All H2 receptor antagonists may inhibit the absorption of certain drugs, such as the antifungal ketoconazole, which require an acidic gastrointestinal environment for gastric absorption. The other options are incorrect.

The nurse is reviewing the medication orders for a patient who will be taking an H2 antagonist. Which drug may have an interaction if taken along with the H2 antagonist? a.ibuprofen (Motrin) b.ranitidine (Zantac) c.tetracycline (Doryx) d.ketoconazole (Nizoral)

B. increased heart rate D. nausea E. nervousness F. tremors

The nurse is teaching a group of patient about the use of bronchodilators. It is important to remind them that using bronchodilators too frequently may cause which adverse effect? select all that apply.. A. blurred vision B. increased heart rate C. decreased heart rate D. nausea E. nervousness F. tremors

A. I will rinse my mouth with water after each dose C. This medication is taken twice a day, every 12 hour E. I will call my doctor if I notice white patches inside my mouth

The nurse is teaching a patient about the inhaler Advair. Which statements by the patient indicate a correct understanding of this medication? Select all that apply.... A. I will rinse my mouth with water after each dose B. I need to use this inhaler whenever I feel SOB but not less than 4 hours between doses C. This medication is taken twice a day, every 12 hours D. I can take this inhaler if I get short of breath while exercising E. I will call my doctor if I notice white patches inside my mouth

A. glaucoma

When assessing a patient who is to receive a decongestant, the nurse will recognize that a potential contraindication to this drug would be which condition? A. glaucoma B. fever C. peptic ulcer disease D. allergic rhinitis

C. hypertension

When giving decongestants, the nurse must remember that these drugs have alpha-adrenergic stimulating effects that may result in which effect? A. fever B. bradycardia C. hypertension D. CNS depression

B. Do not use in patients younger than 2 years of age, unless prescribed.

When providing general education on use of over-the-counter medications for allergies, which is the highest priority for the nurse to include? A. Discontinue use 4 days before allergy testing. B. Do not use in patients younger than 2 years of age, unless prescribed. C. It may cause dry mouth. D. The medication treats the symptoms but is not a cure.

ANS: B Both calcium- and magnesium-based antacids are more likely to accumulate to toxic levels in patients with renal disease and are commonly avoided in this patient group. The other options are incorrect.

When reviewing the health history of a patient who will be receiving antacids, the nurse recalls that antacids containing magnesium need to be used cautiously in patients with which condition? a.Peptic ulcer disease b.Renal failure c.Hypertension d.Heart failure

ANS: D Although neuromuscular blockers can paralyze all skeletal muscles, not all muscles are affected at once. The last muscles affected by neuromuscular blockers are the muscles of respiration, including the intercostals and the diaphragm. The first to become paralyzed are the levator muscle of the eyelids and the muscles of mastication. Paralysis occurs next in the muscles of the limbs, abdomen, and glottis.

Which areas of the body show the effects of neuromuscular blockers last? a.Levator muscle of the eyelids and the muscles of mastication b.Muscles in the lower extremities c.Muscles controlling the glottis d.Muscles of respiration and the diaphragm

B. decongestants (naphazoline) E. inhaled corticosteroids (beclomethasone)

Which drugs are considered first line drugs for the treatment of nasal congestion? select all that apply... A. antihistamine (diphenhydramine) B. decongestants (naphazoline) C. antitussives (dextromethorphan) D. expectorants (guaifenesin) E. inhaled corticosteroids (beclomethasone)

B. "I will rinse my mouth with water after each use."

Which statement by a patient best demonstrates an understanding of the teaching on flunisolide (AeroBid)? A. "I will take two puffs to treat an acute asthma attack." B. "I will rinse my mouth with water after each use." C. "I will immediately stop taking my oral prednisone as soon as I start using the AeroBid." D. "I will not use my albuterol inhaler while I am taking AeroBid."

B. "This medication will help prevent the inflammatory response of my allergies."

Which statement by the patient demonstrates understanding of action or use of beclomethasone diproprionate (Beconase)? A. "I will need to taper off the medication to prevent acute adrenal crisis." B. "This medication will help prevent the inflammatory response of my allergies." C. "I will need to monitor my blood sugar more closely because it may increase secondary to Beconase." D. "I only need to take this medication when my symptoms get bad."

ANS: D Succinylcholine is used for muscle relaxation during short procedures, such as esophageal dilation, because of its short duration. The effects wear off minutes after the drug is withdrawn, because plasma pseudocholinesterase quickly degrades it. Patients who have prolonged effects may have low plasma pseudocholinesterase levels and should be evaluated if this is suspected. Mechanical ventilation may be necessary if the drug is given, but the prudent nurse would rather avoid this by taking preventive measures instead of treating the adverse reaction after the fact. Dantrolene should always be available when succinylcholine is used because of the risk of malignant hyperthermia; however, nothing indicates that this particular patient is at greater risk than others. Prolonged neuromuscular blockade is not a normal reaction

While preparing a patient for a second esophageal dilation procedure, the nurse explains that succinylcholine [Anectine] will be used for muscle relaxation. The patient is anxious and reports not being able to swallow for several hours after the previous procedure. What will the nurse do? a.Be prepared to provide mechanical ventilation after the procedure. b.Have dantrolene available, because this patient is at increased risk for side effects. c.Reassure the patient that this is expected after neuromuscular blockade. d.Request an order for a pseudocholinesterase level.

The nurse recognizes that an angiotensin-converting enzyme (ACE) inhibitor is indicated in the treatment of unstable angina if the patient also has what condition? A) Persistent hypertension B) Diabetes mellitus C) Tachydysrhythmias D) Renal insufficiency

a ACE inhibitors are only indicated in the treatment of unstable angina/anti-ischemic therapy if the patient also has signs of hypertension, left ventricular dysfunction, or congestive heart disease.

The nurse recognizes that beta blockers reduce anginal pain primarily by doing what? A) Decreasing cardiac contractility B) Increasing cardiac afterload C) Decreasing cardiac preload D) Increasing oxygen demand

a Beta blockers decrease cardiac oxygen demand by decreasing adrenergic stimulation; this decreased stimulation leads to decreased heart rate and decreased myocardial contractility. These drugs may also decrease arterial pressure, leading to a decrease in cardiac afterload.

Preparation for the NCLEX® Examination Questions 2. The nurse will monitor a client taking an aluminum-containing antacid, such as aluminum hydroxide (Amphojel), for which adverse effect? a. Constipation b. Gastrointestinal (GI) upset c. Fluid retention d. Diarrhea

a Constipation Aluminum-and calcium-containing antacids cause constipation, magnesium-containing antacids cause diarrhea, and sodium-containing antacids cause sodium and fluid retention.

The nurse is teaching a patient about risk factors for increased cardiovascular mortality. Which risk factor(s) should be included? (Select all that apply.) A) Smoking B) Sedentary lifestyle C) Increased HDL cholesterol D) Decreased LDL cholesterol

a, b, d Smoking, hypertension, hyperlipidemia, and a sedentary lifestyle increase the risk of cardiovascular disease. Patients should be strongly encouraged to quit smoking. Patients' diet should be low in saturated fats (less than 7% of total caloric intake), and total fat content should not exceed 30% of caloric intake. Patients with a sedentary lifestyle should be encouraged to establish a regular program of aerobic exercise (e.g., walking, jogging, swimming, biking).

NCLEX EXAMINATION REVIEW QUESTIONS 6. When the nurse is administering a proton pump inhibitor (PPI), which actions by the nurse are correct? (Select all that apply.) a. Giving the PPI on an empty stomach b. Giving the PPI with meals c. Making sure the patient dose not crush or chew the capsules d. Instructing the patient to open the capsule and chew the contents for best absorption e. Administering the PPI only when the patient complains of heartburn

a, c Give the PPI on an empty stomach, making sure the patient dose not crush or chew the capsules

A patient is receiving gentamicin once daily. A nursing student asks the nurse how the drug can be effective if given only once a day. The nurse explains drug dosing schedules for aminoglycosides. Which statement by the student indicates a need for further teaching? a. "Gentamicin has a longer half-life than other aminoglycosides." b. "Large doses given once daily yield higher peak levels." c. "The postantibiotic effect lasts for several hours." d. "There is less risk of ototoxicity and nephrotoxicity with large daily doses."

a. "Gentamicin has a longer half-life than other aminoglycosides." When a daily dose is given once daily instead of divided into 2 or 3 doses, a higher peak level can be achieved. The higher peak, along with the fact that aminoglycosides have a postantibiotic effect, means that the bacterial kill is just as great with one dose as with 2 or 3 doses per day. When a single daily dose is given, the risk of toxicity is reduced. Gentamicin does not have a longer half-life than other aminoglycosides.

A patient is diagnosed with a lung infection caused by P. aeruginosa. The culture and sensitivity report shows sensitivity to all aminoglycosides. The nurse knows that the rate of resistance to gentamicin is common in this hospital. The nurse will expect the provider to order which medication? a. Amikacin [Amikin] b. Gentamicin c. Paromomycin d. Tobramycin

a. Amikacin [Amikin] When resistance to gentamicin and tobramycin is common, amikacin is the drug of choice for initial treatment of aminoglycoside-sensitive infections. Gentamicin would not be indicated, because resistance is more likely to develop. Paromomycin is used only for local effects within the intestine and is given orally. Tobramycin is not indicated, because organisms can more readily develop resistance.

A patient has been taking levothyroxine for several years and reports that "for the past 2 weeks, the drug doesn't seem to work as well as before." What will the nurse do? a. Ask the patient when the prescription was last refilled. b. Expect the patient to have an elevated temperature and tachycardia. c. Suggest that the patient begin taking calcium supplements. d. Tell the patient to try taking the medication with food.

a. Ask the patient when the prescription was last refilled. Not all levothyroxine preparations have the same drug bioavailability; therefore, if a patient is experiencing differing effects, the pharmacist may have switched brands. Asking a patient about a recent refill may help to explain why the drug has different effects. An elevated temperature and tachycardia would be signs of toxicity, not of a decrease in effectiveness. Calcium supplements and food would only interfere with absorption and further reduce the drug's effectiveness.

A patient with arthritis is admitted to the hospital. The patient's serum glucose level is 350 gm/dL, and the blood pressure is 182/98 mm Hg. The nurse notes that the patient's face appears rounded and puffy. The patient complains of feeling weak. What will the nurse do? a. Ask which drugs the patient takes for arthritis. b. Contact the provider to discuss whether the patient has a pituitary carcinoma. c. Request an order for ketoconazole [Nizoral]. d. Suspect that this patient has Addison's disease.

a. Ask which drugs the patient takes for arthritis. Many patients with arthritis are treated with glucocorticoids. Because the doses necessary to suppress inflammation are larger than the physiologic doses used to treat adrenal insufficiency, patients can develop signs of cortisol excess with cushingoid symptoms. This patient has an elevated glucose level, hypertension, and the characteristic moon facies and muscle weakness of Cushing's syndrome; therefore, the nurse would be correct to ask about the medications the patient takes for arthritis. A pituitary carcinoma could be the cause but is less likely. Ketoconazole is used after chemotherapy and radiation therapy in the treatment of a pituitary adenoma. This patient does not show signs of Addison's disease.

A patient who will begin combination estrogen/progestin therapy (EPT) for menopause asks the nurse why she can't take an estrogen-only preparation. The patient has not had a hysterectomy, has a slightly increased risk of cardiovascular disease, and has mild osteopenia. The nurse will tell her that the progestin is necessary to: a. decrease her risk of endometrial cancer. b. increase bone resorption to prevent fractures. c. lower her risk of myocardial infarction (MI). d. prevent deep vein thrombosis (DVT).

a. decrease her risk of endometrial cancer. In patients who still have a uterus, progestin is necessary to reduce the risk of endometrial carcinoma. Progestins do not have effects on bone density and do not decrease risk of MI or DVT.

Which are contraindications for taking sildenafil [Viagra]? (Select all that apply.) a. Patients who are taking nitroglycerin for angina pectoris b. Patients with nonarteritic ischemic optic neuropathy (NAION) in one eye c. Patients with BPH who are taking alpha1-adrenergic antagonists d. Patients with a blood pressure of 100/60 mm Hg e. Patients with a history of myocardial infarction (MI) in the past 6 months

a. Patients who are taking nitroglycerin for angina pectoris b. Patients with nonarteritic ischemic optic neuropathy (NAION) in one eye c. Patients with BPH who are taking alpha1-adrenergic antagonists Men who take nitroglycerin should not take sildenafil, because at least 24 hours should elapse between taking nitroglycerin and taking sildenafil. Because sildenafil is associated with the development of NAION, patients with that disorder in one eye should not take sildenafil. Patients with low blood pressure should use caution in taking sildenafil, but it is not contraindicated. Patients who have had an MI in the past 6 months should use caution in taking sildenafil.

A 1-year-old child with cretinism has been receiving 8 mcg/kg/day of levothyroxine [Synthroid]. The child comes to the clinic for a well-child checkup. The nurse will expect the provider to: a. change the dose of levothyroxine to 6 mcg/kg/day. b. discontinue the drug if the child's physical and mental development are normal. c. increase the dose to accommodate the child's increased growth. d. stop the drug for 4 weeks and check the child's TSH level.

a. change the dose of levothyroxine to 6 mcg/kg/day. In the treatment of cretinism, thyroid dosing decreases with age. For infants 6 to 12 months of age, the dose is 6 mcg/kg/day. At 1 year of age, the dose is reduced to 5 to 6 mcg/kg/day. For all children, treatment should continue for 3 years. It is incorrect to increase the dose with age. After 3 years of therapy, the patient undergoes a trial of 4 weeks without the drug, followed by assessment of the TSH and T4 levels, to determine whether the drug may be discontinued.

Carbonic anhydrase inhibitors

acetazolamide: most commonly used CAI

A male patient tells the nurse he awakens once or twice each night to void and has difficulty starting his stream of urine. He describes these symptoms as "annoying." The patient's provider examines him and notes that the prostate is moderately enlarged. The patient is sexually active and tells the nurse that he does not want to take any medication that will interfere with sexual function. The nurse anticipates the provider will order: a. doxazosin [Cardura]. b. finasteride [Proscar]. c. silodosin [Rapaflo]. d. a transurethral prostatectomy.

a. doxazosin [Cardura]. Nonselective alpha1-adrenergic antagonists do not commonly affect sexual function and are useful in patients with mild to moderate symptoms, so doxazosin would be a drug of choice for this patient. Finasteride is a 5-alpha-reductase inhibitor and is used for patients with more severe enlargement of the prostate; it also reduces ejaculate volume and libido. Silodosin is a selective alpha1-adrenergic antagonist and can cause abnormal ejaculation. Transurethral prostatectomy is reserved for patients with more severe enlargement of the prostate.

When teaching a patient who has a new prescription for thyroid hormone, the nurse will instruct the patient to notify the physician if which adverse effects are noted? (select all that apply) a. palpitations b. weight gain c. angina d. fatigue e. cold intolerance

a. palpitations c. angina

Preparation for the NCLEX® Examination Questions 10. The nurse will question an order for misoprostol (Cytotec) in which patient? a. A 21-year-old man with Zollinger-Ellison syndrome b. A 32-year-old pregnant woman with a urinary tract infection c. A 45-year-old woman with GERD d. A 64-year-old man with hypertension

b A 32-year-old pregnant woman with a urinary tract infection Misoprostol (Cytotec) is a prostaglandin E analog and is believed to inhibit gastric acid secretion and protect the gastric mucosa from injury by enhancing the local production of mucus. However, it is also an abortifacient and therefore is contraindicated in pregnancy. The drug may be useful in treating patients with Zollinger-Ellison syndrome (a hypersecretory syndrome) and GERD. Hypertension is not a contraindication for its use.

A patient who is taking ketoconazole tells the nurse that her periods have become irregular. What will the nurse tell her? a. This indicates that she should begin taking oral contraceptives. b. This is caused by a reversible effect on estradiol synthesis. c. This is a serious side effect that warrants discontinuation of the drug. d. This is a sign of hepatic toxicity, and the drug dose should be lowered.

b. This is caused by a reversible effect on estradiol synthesis.

A nurse is explaining congenital adrenal hyperplasia (CAH) to a group of nursing students. Which statement by a student indicates understanding of the teaching? a. "CAH is caused by a deficiency of ACTH production." b. "CAH is the result of an inability to synthesize glucocorticoids." c. "Newborn screening provides a definitive diagnosis for CAH" d. "The enzyme 21-alpha-hydroxylase increases the production of androgens."

b. "CAH is the result of an inability to synthesize glucocorticoids." CAH results from an inborn deficiency of the enzymes needed for glucocorticoid synthesis. The condition is marked by increased production of ACTH, because the pituitary attempts to enhance glucocorticoid synthesis. Newborn screening indicates a deficiency of 21-alpha-hydroxylase; if this is noted, follow-up testing must be done to confirm a diagnosis of CAH. The enzyme 21-alpha-hydroxylase increases the production of glucocorticoids. When this enzyme is deficient, androgens are produced in excess because of stimulation by ACTH.

A nurse is teaching a patient who will begin taking methimazole [Tapazole] for Graves' disease about the medication. Which statement by the patient indicates understanding of the teaching? a. "Because of the risk for liver toxicity, I will need frequent liver function tests." b. "I should report a sore throat or fever to my provider if either occurs." c. "I will need a complete blood count every few months." d. "It is safe to get pregnant while taking this medication."

b. "I should report a sore throat or fever to my provider if either occurs." Agranulocytosis is rare but can occur with methimazole, so patients should report signs of infection, such as a sore throat or fever. Liver toxicity is not a side effect, so liver function tests are not indicated. Because agranulocytosis often develops rapidly, periodic blood counts do not guarantee early detection. Methimazole is contraindicated in the first trimester of pregnancy.

The nurse is providing patient education to a patient who will begin taking fludrocortisone [Florinef] as adjunctive therapy to hydrocortisone. Which statement by the patient indicates understanding of the teaching? a. "I should move from sitting to standing slowly." b. "I should report any swelling of my hands and feet." c. "I should report weight loss to my provider." d. "I should report excessive urine output."

b. "I should report any swelling of my hands and feet." Fludrocortisone is a mineralocorticoid that regulates sodium, potassium, and water balance. Water and sodium retention is a particular concern, so the patient should be taught to report any signs of fluid retention, such as swelling of the hands and feet. Fludrocortisone elevates the blood pressure, so hypotension is not a concern. Because fluid retention causes weight gain, patients should be taught to report any increase in weight. Patients should report decreased urine output.

The nurse prepares a patient with Graves' disease for radioactive iodine (131I) therapy. Which statement made by the patient best demonstrates understanding of 131I therapy? a. "I will have to isolate myself from my family for 1 week so that I don't expose them to radiation." b. "This drug will be taken up by the thyroid gland and will destroy the cells to reduce my hyperthyroidism." c. "This drug will help reduce my cold intolerance and weight gain." d. "I will need to take this drug on a daily basis for at least 1 year."

b. "This drug will be taken up by the thyroid gland and will destroy the cells to reduce my hyperthyroidism." Iodine-131 can be used to destroy thyroid tissue in patients with hyperthyroidism; no further teaching is necessary. The patient does not need to isolate himself from others. The treatment will not reduce intolerance to cold, nor will it affect weight gain. The patient will not need the treatment daily.

The nurse is caring for a pregnant patient recently diagnosed with hypothyroidism. The patient tells the nurse she does not want to take medications while she is pregnant. What will the nurse explain to this patient? a. Hypothyroidism is a normal effect of pregnancy and usually is of no consequence. b. Neuropsychologic deficits in the fetus can occur if the condition is not treated. c. No danger to the fetus exists until the third trimester. d. Treatment is required only if the patient is experiencing symptoms.

b. Neuropsychologic deficits in the fetus can occur if the condition is not treated. Maternal hypothyroidism can result in permanent neuropsychologic deficits in the child. Hypothyroidism is not a normal effect of pregnancy and is a serious condition that can affect both mother and fetus. The greatest danger to the fetus occurs in the first trimester, because the thyroid does not fully develop until the second trimester. Early identification is essential. Symptoms often are vague. Treatment should begin as soon as possible, or mental retardation and other developmental problems may occur.

Preparation for the NCLEX® Examination Questions 11. The nurse would teach a client prescribed simethicone (Mylicon) to avoid which substance? a. Over-the-counter antacids b. Histamin 2- receptor antagonists c. Carbonated beverages d. Milk and dairy products

c Carbonated beverage Simethicone is used to decrease gas and belching. both of which can be aggravated or caused by ingesting carbonated beverages. It may be given in combination with other medications used to decrease acidity.

Preparation for the NCLEX® Examination Questions 12. How dose sucralfate (Carafate) achieve a therapeutic effect? a. By inhibiting the production of gastric acid secretion b. by enhancing gastric absorption c. By forming a protective barrier over the gastric mucosa d. By neutralizing gastric acid

c By forming a protective barrier over the gastric mucosa Sucralfate has a local effect only on the gastric mucosa. If forms a protective barrier that can be thought of as a liquid protective bandage in the stomach. This liquid bandage adheres to the gastric lining, protecting against adverse effects related to gastric acid. It also stimulates healing of any ulcerated areas of the gastric mucosa.

The nurse is teaching a patient who has a new prescription for the antithyroid drug propylthiouracil (PTU). Which statement by the nurse is correct? a. There is no food restrictions b. You need to avoid foods high in iodine, such as iodized salt, seafood, and soy products c. This drug is given to raise the thyroid hormone levels in your blood d. Take this drug in the morning on an empty stomach

b. You need to avoid foods high in iodine, such as iodized salt, seafood, and soy products

A patient who takes the loop diuretic ethacrynic acid is given intravenous gentamicin for an infection. After several days of treatment with gentamicin, the nurse reviews the patient's most recent laboratory results and notes a gentamicin trough of 2.1 mcg/mL and normal blood urea nitrogen (BUN) and serum creatinine levels. The nurse will question the patient about: a. gastrointestinal (GI) symptoms. b. headache, dizziness, or vertigo. c. presence of rash. d. urine output.

b. headache, dizziness, or vertigo. Ethacrynic acid has ototoxic properties, and patients who take this drug with an aminoglycoside have an increased risk of ototoxicity, especially when trough levels of the aminoglycoside are elevated. A trough level of 2.1 mcg/mL is above normal limits for gentamicin, so this patient should be asked about early signs of ototoxicity. There is no indication to evaluate for GI symptoms, rash, or urine output

A nurse is preparing to administer a dose of gentamicin to a patient who is receiving the drug 3 times daily. The nurse will monitor ____ levels. a. peak b. peak and trough c. serum drug d. trough

b. peak and trough When divided doses of aminoglycosides are given, it is important to measure both peak and trough levels of the drug, because it is more difficult to achieve therapeutic peaks in lower doses without causing toxicity. Trough levels are drawn when single-dosing regimens are used, because high peak levels are guaranteed.

A patient is admitted to the hospital and will begin taking levothyroxine [Synthroid]. The nurse learns that the patient also takes warfarin [Coumadin]. The nurse will notify the provider to discuss ____ the ____ dose. a. reducing; levothyroxine b. reducing; warfarin c. increasing; levothyroxine d. increasing; warfarin

b. reducing; warfarin Levothyroxine accelerates the degradation of vitamin K-dependent clotting factors, which enhances the effects of warfarin. Patients taking warfarin who start taking levothyroxine may need to have their warfarin dose reduced. It is not correct to increase or decrease the levothyroxine dose or to increase the warfarin dose.

A 68-year-old male patient receives a prescription for 25-mg tablets of sildenafil [Viagra] for erectile dysfunction. When he asks the nurse how to take the medication, the nurse will tell him to: a. begin taking one tablet twice daily, 12 hours apart. b. start with one tablet about 1 hour before anticipated sexual activity. c. take 25 to 100 mg per dose 30 minutes to 4 hours before sexual activity. d. take two tablets 1 hour before sexual activity with a high-fat meal.

b. start with one tablet about 1 hour before anticipated sexual activity. Patients older than 65 years should start with a low dose of 25 mg and may take the drug 1 hour before anticipated sexual activity. Sildenafil is taken when needed and not on a routine basis. Dosing at 25 to 100 mg per dose 30 minutes to 4 hours before sexual activity is the standard recommendation for men younger than 65 years. Two tablets is a high dose; consumption of a high-fat meal would interfere with absorption of sildenafil.

A patient will begin treatment with posaconazole [Noxafil] to treat oropharyngeal candidiasis that has not responded to fluconazole. The provider has ordered 200 mg three times daily. Which action by the nurse is correct? a. Administer the drug as ordered. b. Contact the provider to discuss decreasing the dose to twice daily. c. Discuss a 400-mg twice daily drug regimen with the provider. d. Suggest to the provider that this dose may be too high for this indication.

c. Discuss a 400-mg twice daily drug regimen with the provider.

A patient is being treated with amphotericin B [Abelcet] for a systemic fungal infection. After several weeks of therapy, the provider orders flucytosine [Ancobon] in addition to the amphotericin. The nurse understands that the rationale for this combination is that it: a. broadens the antifungal spectrum. b. improves the effectiveness of the amphotericin B. c. lowers the dose of amphotericin B and reduces toxicity. d. treats fungal central nervous system (CNS) infection.

c. lowers the dose of amphotericin B and reduces toxicity.

A nurse provides teaching to a patient who has had a hysterectomy and is about to begin hormone therapy to manage menopausal symptoms. Which statement by the patient indicates understanding of the teaching? a. "Because I am not at risk for uterine cancer, I can take hormones indefinitely." b. "I can take estrogen to reduce my risk of cardiovascular disease." c. "I should take the lowest effective dose for the shortest time needed." d. "I will need a progestin/estrogen combination since I have had a hysterectomy."

c. "I should take the lowest effective dose for the shortest time needed." For patients who have undergone hysterectomy, progestin is unnecessary; estrogen-only preparations still carry increased risk of breast cancer and should be taken in the lowest effective dose for the shortest time possible. Even though uterine cancer is no longer a possibility, breast cancer is still a risk. Studies have shown no protection against coronary heart disease but increased risk of stroke and breast cancer with estrogens.

A healthy male patient who does not have erectile dysfunction asks about medications to improve sexual stamina. What will the nurse tell this patient? a. "Medications for ED can improve the duration of erection in healthy men." b. "Medications for ED can improve the quality of erection in healthy men." c. "Medications for ED have no effect on erections in healthy men." d. "Medications for ED will cause priapism in healthy men."

c. "Medications for ED have no effect on erections in healthy men." Medications for ED will have little or no effect on erection quality or duration in otherwise healthy men who do not have ED. These medications will not cause priapism in otherwise healthy men.

The nurse is teaching a nursing student about the mechanism by which antimicrobial agents achieve selective toxicity. Which statement by the student indicates a need for further teaching? a. "Some agents disrupt the bacterial cell wall." b. "Some agents act to block the conversion of para-aminobenzoic acid (PABA) to folic acid." c. "Some agents cause phagocytosis of bacterial cells." d. "Some agents weaken the cell wall, causing cell wall lysis."

c. "Some agents cause phagocytosis of bacterial cells." Antimicrobial agents do not cause phagocytosis of bacterial cells. They do disrupt and weaken the bacterial cell wall; because human cells do not have cell walls, antimicrobial agents are not toxic to human cells. Some agents block the conversion of PABA to folic acid; humans do not synthesize folic acid and are not harmed by this process.

A patient is given 1 mg of dexamethasone at 11:00 PM; a plasma cortisol level recorded at 8:00 PM the next day is normal. The nurse knows that this is an indication that the patient has what condition? a. Addison's disease b. Congenital adrenal hyperplasia c. Cushing's syndrome d. Secondary adrenal insufficiency

c. Cushing's syndrome The overnight dexamethasone suppression test, which is performed by administering dexamethasone as described, is used to diagnose Cushing's syndrome. In normal individuals, dexamethasone suppresses the release of adrenocorticotropic hormone (ACTH), thereby suppressing the synthesis and release of cortisol, which results in a low cortisol level. In patients with Cushing's syndrome, the cortisol level is normal or only mildly low. This is not an indication of Addison's disease, which is diagnosed by administering cosyntropin. Neither test is useful for diagnosing congenital adrenal hyperplasia or secondary adrenal insufficiency.

A child has received amoxicillin [Amoxil] for three previous ear infections, but a current otitis media episode is not responding to treatment. The nurse caring for this child suspects that resistance to the bacterial agent has occurred by which microbial mechanism? a. Alteration of drug target molecules b. Antagonist production c. Drug inactivation d. Reduction of drug concentration at the site of action

c. Drug inactivation Drug inactivation can occur when microbes produce drug-metabolizing enzymes. Penicillin-resistant organisms, including many that cause otitis media, produce penicillinase. Alteration of drug target molecules, drug inactivation, and reduction of the drug concentration occur with other antimicrobials.

A nurse obtaining an admission history on an adult patient notes that the patient has a heart rate of 62 beats per minute, a blood pressure of 105/62 mm Hg, and a temperature of 96.2°F. The patient appears pale and complains of always feeling cold and tired. The nurse will contact the provider to discuss tests for which condition? a. Cretinism b. Graves' disease c. Hypothyroidism d. Plummer's disease

c. Hypothyroidism This patient is showing signs of hypothyroidism: a low heart rate, low temperature, pale skin, and feeling cold and tired. In adults, thyroid deficiency is called hypothyroidism. In children, thyroid deficiency is called cretinism. Graves' disease and Plummer's disease are conditions caused by thyroid excess.

A recent campaign, initiated by the Centers for Disease Control (CDC), to delay the emergence of antibiotic resistance in hospitals, has what as one of its objectives? a. Allowing patients to stop antibiotics when symptoms subside b. Allowing prescribers to develop their own prescribing guidelines c. Increased adherence to prescribed antibiotics d. Increased use of antibiotics among parents of young children

c. Increased adherence to prescribed antibiotics The CDC initiative has identified three objectives to help delay the emergence of resistance to antibiotics in hospitals. One of the objectives is to increase adherence to prescribed antibiotics among users. Allowing patients to stop using antibiotics before the prescription ends is not one of the objectives and may increase the risk of resistance. Other objectives include ensuring that prescribers adhere to appropriate prescribing guidelines instead of developing their own and decreasing antibiotic use among parents of small children.

A patient with hypothyroidism begins taking PO levothyroxine [Synthroid]. The nurse assesses the patient at the beginning of the shift and notes a heart rate of 62 beats per minute and a temperature of 97.2°F. The patient is lethargic and difficult to arouse. The nurse will contact the provider to request an order for which drug? a. Beta blocker b. Increased dose of PO levothyroxine c. Intravenous levothyroxine d. Methimazole [Tapazole]

c. Intravenous levothyroxine Intravenous administration of levothyroxine is used for myxedema coma. This patient is showing signs of severe hypothyroidism, or myxedema. A beta blocker is useful in patients who show signs of hyperthyroidism to minimize cardiac effects. Because the half-life of oral levothyroxine is so long, increasing the PO dose will not provide immediate relief of this patient's symptoms. Methimazole is used to treat hyperthyroidism.

Which patients should be given antibiotics prophylactically? (Select all that apply.) a. Children who attend day care with other children who have strep throat b. Children undergoing chemotherapy who have mild neutropenia c. Patients with certain congenital heart defects at risk for bacterial endocarditis d. Patients with compound fractures undergoing surgical repair e. Postoperative patients who have undergone emergency cesarean sections

c. Patients with certain congenital heart defects at risk for bacterial endocarditis d. Patients with compound fractures undergoing surgical repair Antimicrobial prophylaxis is recommended for patients with congenital or other heart disease who have an increased risk for bacterial endocarditis and for patients undergoing certain surgeries in which the risk for infection is very high, including those who have compound fractures. Children who are exposed to strep throat will need to have confirmed strep infection prior to receiving antibiotics. Patients with milk neutropenia do not need antimicrobial prophylaxis. Patients undergoing emergency C-sections need antimicrobial prophylaxis prior to surgery, not after.

Which are benefits of using a combination of two or more antibiotics? (Select all that apply.) a. Reduced cost b. Reduced risk of superinfection c. Reduced toxicity d. Reduced resistance e. Reduced risk in severe infection

c. Reduced toxicity d. Reduced resistance e. Reduced risk in severe infection In some situations, an antibiotic combination can reduce toxicity, because the dosage of a more toxic agent can be reduced. Drug combinations can be used to suppress the emergence of resistant bacteria in the treatment of tuberculosis. With a severe infection, a combination of antibiotics is useful until culture results are known to ensure that all possible sources of infection are treated. Using multiple antibiotics is more costly and poses a greater risk of superinfection.

A patient in her twenties with Graves' disease who takes methimazole [Tapazole] tells a nurse that she is trying to conceive and asks about disease management during pregnancy. What will the nurse tell her? a. Methimazole is safe to take throughout pregnancy. b. Propylthiouracil should be taken throughout her pregnancy. c. The patient should discuss changing to propylthiouracil from now until her second trimester with her provider. d. The patient should discuss therapy with iodine-131 instead of medications with her provider.

c. The patient should discuss changing to propylthiouracil from now until her second trimester with her provider. Methimazole is not safe during the first trimester of pregnancy, because it is associated with neonatal hypothyroidism, goiter, and cretinism; however, it is safe in the second and third trimesters. Propylthiouracil is recommended for pregnant patients only in the first trimester and during lactation only if a thionamide is absolutely necessary. Iodine-131 is used in women older than 30 years who have not responded to medication therapy and is contraindicated during pregnancy

A patient who has chronic adrenal insufficiency is admitted to the hospital for an open cholecystectomy. The nurse obtaining the admission history learns that the patient takes hydrocortisone 25 mg PO daily in the morning. The patient's surgery is scheduled for the next morning. The nurse will expect an order to: a. administer the usual morning dose of hydrocortisone 25 mg PO. b. administer hydrocortisone 50 mg PO in the morning. c. administer hydrocortisone 50 mg IV before surgery. d. withhold the morning dose of hydrocortisone and give it after surgery.

c. administer hydrocortisone 50 mg IV before surgery. Patients who take steroids need extra steroid before situations that cause stress, such as surgery. Failure to administer the increased dose can prove fatal. For surgeries that cause moderate stress, such as a cholecystectomy, patients should be given 50 mg of hydrocortisone intravenously the day of the procedure, followed by a taper over 1 to 2 days to the usual replacement dose. Giving the usual dose or giving an increased oral dose is not indicated

Preparation for the NCLEX® Examination Questions 3. For a client with chronic renal failure, the nurse MOST likely will question a prescription for which type of antacid? a. Aluminum-containing antacids b. Calcium-containing antacids c. Sodium-containing antacids d. Magnesium-containing antacids

d Magnesium-containing antacids Magnesium-containing antacids can cause hypermagnesemia in clients with chronic renal failure. Aluminum-containing antacids may be used as a phosphate binder in clients with chronic renal failure. Sodium-and aluminum-containing antacids are chemically more easily excreted in clients with renal compromise. although calcium-containing antacids may accumulate in the bloodstream of clients with renal failure, they may also be appropriate because these patients may be hypocalcemic.

Preparation for the NCLEX® Examination Questions 4. What condition will the nurse monitor for with a client using sodium bicarbonate to treat gastric hyperacidity? a. Hypercalcemia b. Hyperkalemia c. Metabolic acidosis d. Metabolic alkalosis

d Metabolic alkalosis Solutions containing sodium bicarbonate (a base) can cause metabolic alkalosis. Serum potassium and serum calcium would decrease, not increase, with alkalosis.

Preparation for the NCLEX® Examination Questions 5. Which nursing diagnosis is appropriate for a patient receiving famotidine (Pepcid)? a. Ineffective peripheral tissue perfusion related to hypertension b. Risk for infection related to immunosuppression c. Impaired urinary elimination related to retention d. Risk for injury related to thrombocytopenia

d Risk for injury related to thrombocytopenia. A serious side effect of famotidine is thrombocytopenia, which is manifested by a decrease in platelet count and an increased risk of bleeding. The patient receiving famotidine may experience hypotension as an adverse effect, not hypertension. Famotidine dose not cause immunosuppression or urinary retention.

A nurse is providing education to a patient who is beginning therapy with AndroGel testosterone gel. What statement made by the patient demonstrates a need for further teaching? a. "I should not shower or swim for at least 5 to 6 hours after application." b. "I should avoid direct skin-to-skin contact with my spouse where the medication was applied." c. "I should have my blood drawn for laboratory tests in 14 days." d. "I should apply the medication to my genitals for best results."

d. "I should apply the medication to my genitals for best results."

A nurse provides teaching for a female patient with anemia who has had cancer chemotherapy and who will begin treatment with testosterone. Which statement by the patient indicates understanding of the teaching? a. "Facial hair may develop with this drug but will go away over time." b. "I may experience an increase in breast size while taking this drug." c. "Testosterone may increase my high-density lipoprotein (HDL) cholesterol and reduce my low-density lipoprotein (LDL) cholesterol." d. "Testosterone treats anemia by stimulating the synthesis of a renal hormone."

d. "Testosterone treats anemia by stimulating the synthesis of a renal hormone."

A patient has an invasive aspergillosis infection. Which antifungal agent is the drug of choice for this infection? a. Amphotericin B b. Fluconazole [Diflucan] c. Posaconazole [Noxafil] d. Voriconazole [Vfend]

d. Voriconazole [Vfend]

An adolescent male patient is beginning androgen therapy for delayed puberty. His parents ask the nurse when this treatment may be stopped. The nurse will offer which response? a. After 3 to 4 years of therapy b. If acne and facial hair develop c. When complete sexual maturation has occurred d. When testicular enlargement occurs

d. When testicular enlargement occurs

Tadalafil [Cialis] was prescribed 4 weeks ago for a patient with erectile dysfunction. The patient also takes prazosin [Minipress] for hypertension. Which statement by the patient best demonstrates understanding of the use of tadalafil [Cialis]? a. "I can take this up to 2 times a day, just before intercourse." b. "I have a little renal trouble, so I should take a higher dose." c. "I should take this medication with food." d. "I should take this medication no more than once a day."

d. "I should take this medication no more than once a day." Tadalafil should be taken no more than once a day and without regard to meals. Tadalafil has a half-life of 17.5 hours and should not be taken twice a day. Men with moderate renal or hepatic insufficiency should take a reduced dosage. This patient may be confusing the actions of tadalafil and prazosin.

A nurse is teaching a patient who has been diagnosed with hypothyroidism about levothyroxine [Synthroid]. Which statement by the patient indicates a need for further teaching? a. "I should not take heartburn medication without consulting my provider." b. "I should report insomnia, tremors, and an increased heart rate to my provider." c. "If I take a multivitamin with iron, I should take it 4 hours after the Synthroid." d. "If I take calcium supplements, I may need to decrease my dose of Synthroid."

d. "If I take calcium supplements, I may need to decrease my dose of Synthroid." Patients taking calcium supplements should take these either 4 hours before or after taking levothyroxine, because they interfere with levothyroxine absorption. Many heartburn medications contain calcium, so patients should consult their provider before taking them. Insomnia, tremors, and tachycardia are signs of levothyroxine toxicity and should be reported. Iron also interferes with levothyroxine absorption, so dosing should be 4 hours apart.

A nurse provides teaching to a group of nursing students about the risks and benefits of hormone therapy (HT), including estrogen therapy (ET) and combination estrogen/progestin therapy (EPT). Which statement by a student indicates understanding of the teaching? a. "ET can provide protection against coronary heart disease and reverse osteoporosis." b. "EPT is generally safer than ET, especially in women who have undergone hysterectomies." c. "In women with established coronary heart disease, EPT can protect against myocardial infarction." d. "Principle benefits of ET are suppression of menopausal symptoms and prevention of bone loss."

d. "Principle benefits of ET are suppression of menopausal symptoms and prevention of bone loss." ET can be used to suppress menopausal symptoms and to prevent osteoporosis, but it carries risks of breast cancer and stroke, while conferring no preventive benefit for coronary heart disease. ET does not reverse osteoporosis but may help prevent it. EPT is not safer than ET; progestins appear to increase the risk of breast cancer. EPT does not prevent myocardial infarction in patients with coronary heart disease.

A patient has a free T4 level of 0.6 ng/dL and a free T3 of 220 pg/dL. The patient asks the nurse what these laboratory values mean. How will the nurse respond? a. "These laboratory values indicate that you may have Graves' disease." b. "These results suggest you may have hyperthyroidism." c. "We will need to obtain a total T4 and a total T3 to tell for sure." d. "We will need to obtain a TSH level to better evaluate your diagnosis."

d. "We will need to obtain a TSH level to better evaluate your diagnosis." A free T4 level of less than 0.9 ng/dL and a free T3 of less than 230 pg/dL are consistent with hypothyroidism, but measurement of the thyroid-stimulating hormone (TSH) level is necessary to distinguish primary hypothyroidism from secondary hypothyroidism. Total T3 and T4 levels are not as helpful as free T3 and T4 levels. These laboratory values indicate hypothyroidism, not hyperthyroid conditions such as Graves' disease.

A patient who has been newly diagnosed with adrenal hormone deficiency will begin taking hydrocortisone. The nurse provides teaching for this patient. Which statement by the patient indicates understanding of the teaching? a. "I may take all of my daily dose in the morning or divide it in half and take it twice daily." b. "I will need to take this medication until my symptoms completely clear, and then I may stop." c. "Side effects are common with hydrocortisone, even with therapeutic doses." d. "When I am sick, I should take three times the normal dose for 3 days in a row."

d. "When I am sick, I should take three times the normal dose for 3 days in a row." Patients who take hydrocortisone as replacement need to be taught to increase their daily dose during times of stress, because the drug causes adrenal suppression, and the adrenals will not be able to release cortisone as usual during stress. A general rule of thumb is the "3 by 3 rule": take 3 times the usual dose for 3 days when sick. The daily dose is usually given once in the morning; if late-day fatigue occurs, patients may split the dose and take two-thirds in the morning and one-third in the late afternoon or evening. Treatment is lifetime. Side effects are uncommon when hydrocortisone is given in therapeutic doses.

A parent asks a nurse if the provider will prescribe an antibiotic for a child who attends school with several children who have strep throat. The child is complaining of a sore throat and has a fever. What will be the nurse's response? a. "Because strep throat is likely, your child should be treated empirically." b. "With good hand washing, your child should not get strep throat." c. "Your child probably has strep throat, so your provider will order an antibiotic." d. "Your child should come to the clinic to have a throat culture done today."

d. "Your child should come to the clinic to have a throat culture done today." Whenever possible, the infecting organism should be identified before antibiotics are started, even if there is a strong suspicion that a particular organism is present. The nurse is correct in telling the parent to bring the child to the clinic for a throat culture. Fever and sore throat have other causes, so it is not correct to treat this child empirically, especially because these symptoms are not severe or life threatening. Although good hand washing is always indicated, this child already has symptoms and needs to be evaluated. Antibiotics should not be started until indicated by cultures.

5. A postmenopausal patient who has had a hysterectomy and who has a family history of coronary heart disease reports experiencing vaginal dryness and pain with intercourse but tells the nurse that she doesn't want to take hormones because she is afraid of adverse effects. The nurse will suggest that the woman ask her provider about: a. Depo Provera. b. Estraderm. c. low-dose estrogens. d. Premarin vaginal.

d. Premarin vaginal. Estrogens for intravaginal administration are used for local effects, primarily to treat vulval and vaginal atrophy. Because these preparations bypass the liver, the total dose is reduced and there is a lower risk of systemic effects. Depo Provera is a progesterone and is not indicated. Transdermal estrogen is used to treat hot flushes. Low-dose estrogens still have systemic effects.

A patient with a history of benign prostatic hyperplasia is admitted to the unit. The patient is taking multiple medications, including terazosin [Hytrin]. During the evening rounds, the patient begins to complain of dizziness and nasal congestion. Upon assessment, the patient is somnolent and has a blood pressure of 101/42 mm Hg. The nurse is correct to suspect: a. priapism. b. sinus infection. c. allergic reaction. d. adverse effects.

d. adverse effects. The principal adverse effects of terazosin and doxazosin are hypotension, fainting, dizziness, somnolence, and nasal congestion. No evidence indicates priapism, sinus infection, or an allergic reaction.

When assessing the elderly patient, the nurse keeps in mind that certain nonspecific symptoms may represent hypothyroidism in these patients, such as: a. leukopenia, anemia b. loss of appetite, polyuria c. weight loss, dry cough d. cold intolerance, depression

d. cold intolerance, depression

A patient has a localized skin infection, which is most likely caused by a gram-positive cocci. Until the culture and sensitivity results are available, the nurse will expect the provider to order a ____-spectrum ____ agent. a. broad; systemic b. broad; topical c. narrow; systemic d. narrow; topical

d. narrow; topical When infections are treated before the causative agent has been identified, and after cultures have been obtained, antibiotics may be used based on the knowledge of which microbes are most likely to cause infection at that particular site. Because this is a localized infection, a topical agent is recommended. Unless the infection is very serious, a narrow-spectrum antibiotic is best.

A patient arrives in the emergency department with a heart rate of 128 beats per minute and a temperature of 105°F. The patient's skin feels hot and moist. The free T4 level is 4 ng/dL, the free T3 level is 685 pg/dL, and the TSH level is 0.1 microunits/mL. The nurse caring for this patient will expect to administer: a. intravenous levothyroxine. b. iodine-131 (131I). c. methimazole [Tapazole]. d. propylthiouracil (PTU).

d. propylthiouracil (PTU). Propylthiouracil is used for patients experiencing thyroid storm, and this patient is showing signs of that condition. Levothyroxine is given IV for hypothyroidism. 131I is used in patients over age 30 who have not responded to other therapies. Methimazole is used long term to treat hyperthyroidism, but PTU is more useful for emergency treatment.

A nurse preparing to administer intravenous gentamicin to a patient notes that the dose is half the usual dose for an adult. The nurse suspects that this is because this patient has a history of: a. antibiotic resistance. b. interpatient variation. c. liver disease. d. renal disease.

d. renal disease. The aminoglycosides are eliminated primarily by the kidneys, so in patients with renal disease, doses should be reduced or the dosing interval should be increased to prevent toxicity. Patients with antibiotic resistance would be given amikacin. Interpatient variation may occur but cannot be known without knowing current drug levels. Aminoglycosides are not metabolized by the liver, so liver disease would not affect drug levels.

The nurse is providing patient education about the application of transdermal estrogen spray. Which statement made by the patient best demonstrates understanding of the application of this medication? "I should apply this medication to my: a. waistline and shoulders." b. abdomen and arms." c. breasts and abdomen." d. thighs and calves."

d. thighs and calves." The top of the thighs and the back of the calves are the preferred sites for application of the transdermal spray. The waistline and abdomen are used for the patches. The gel is applied to arms. Breasts are never used for application of transdermal estrogen.

A 50-year-old postmenopausal patient who has had a hysterectomy has moderate to severe vasomotor symptoms and is discussing estrogen therapy (ET) with the nurse. The patient is concerned about adverse effects of ET. The nurse will tell her that: a. an estrogen-progesterone product will reduce side effects. b. an intravaginal preparation may be best for her. c. side effects of ET are uncommon among women her age. d. transdermal preparations have fewer side effects.

d. transdermal preparations have fewer side effects Transdermal preparations of estrogen have fewer adverse effects, use lower doses of estrogen, and have less fluctuation of estrogen levels than do oral preparations. Progesterone is contraindicated in women who have undergone hysterectomy. Intravaginal preparations are most useful for treating local estrogen deficiency such as vaginal and vulvar atrophy. Side effects of ET are the same at the patient's age as for other women using ET.

Adverse Effects: Antithyroid Drugs

most serious are liver and bone marrow toxicity, drowsiness, vertigo, paresthesia, hepatitis, loss of taste, smoky urine, decreased urine output, agranulocytosis, leukopenia, thrombocytopenia, hypothrominemia, lymphadenopathy,bleeding, rash, pruritus, myalgia, arthralgia, increased BUN and serum creatinine levels, enlarged thyroid gland nephritis

Hyperthyroidism during Pregnancy

treated with dosage adjustments every 4 weeks to maintain TSH level at the lower end of normal range- fetal growth may be retarded if maternal hypothyroidism remains untreated during pregnancy

1. A patient with Pneumocystis jirovecii pneumonia will be receiving pentamidine (Pentam 300) as an intravenous piggyback (IVPB) dose. The medication has been added to a 100-mL bag of D5W for the infusion, and it needs to infuse over 120 minutes. The nurse will set the infusion pump to infuse at what rate for this IVPB medication? _______

ANS: DIF: COGNITIVE LEVEL: Applying (Application) REF: N/A TOP: NURSING PROCESS: Implementation

1. A patient with a viral infection is to receive ganciclovir (Cytovene) 5 mg/kg/day IVPB every morning. The patient weighs 110 pounds. Identify how many milligrams will this patient receive for this dose. _______

ANS: 250 mg DIF: COGNITIVE LEVEL: Applying (Application) REF: N/A TOP: NURSING PROCESS: Implementation

A patient who is taking nelfinavir [Viracept] calls the nurse to report moderate to severe diarrhea. What will the nurse expect the provider to recommend? a.An over-the-counter antidiarrheal drug b.Immediate discontinuation of the nelfinavir c.Reducing the dose of nelfinavir by half d.Taking the nelfinavir with food to avoid side effects

ANS: A A dose-limiting effect of nelfinavir is moderate to severe diarrhea, which can be managed with OTC antidiarrheal medications. Unless the symptoms become severe, withdrawing the nelfinavir is not indicated. Reducing the dose by half or taking it with food is not indicated.

A patient with HIV and mucocutaneous HSV is being treated with foscarnet after failing treatment with acyclovir. After 2 weeks, the patient's dose is increased to 90 mg/kg over 2 hours from 40 mg/kg over 1 hour. The patient reports numbness in the extremities and perioral tingling. What will the nurse do? a.Notify the provider and request an order for a serum calcium level. b.Notify the provider of potential foscarnet overdose. c.Request an order for a creatinine clearance level. d.Request an order of IV saline to be given before the next dose.

ANS: A Foscarnet frequently causes hypocalcemia and other electrolyte and mineral imbalances. Paresthesias, numbness in the extremities, and perioral tingling can indicate hypocalcemia, so a calcium level should be drawn. These are not signs of foscarnet overdose. Nephrotoxicity may occur, but these are not signs of renal complications, so a creatinine clearance is not indicated. If nephrotoxicity occurs, prehydration with IV saline is indicated to reduce the risk of renal injury.

A patient has lamivudine-resistant hepatitis B and has been taking entecavir [Baraclude] for 2 years. The patient asks the nurse why the provider has recommended taking the drug for another year. What will the nurse tell the patient? a."Entecavir can reverse fibrosis and cirrhosis of the liver when taken long term." b."It is necessary to continue taking entecavir to avoid withdrawal symptoms." c."The drug will be given until the infection is completely eradicated." d."You will need to continue taking entecavir to prevent lactic acidosis and hepatotoxicity."

ANS: A Recent evidence indicates that, with long-term use (3 years), entecavir can reverse fibrosis and cirrhosis. The drug is not continued to avoid withdrawal symptoms. Patients who stop taking entecavir may experience acute exacerbations of hepatitis B; the disease is not eradicated. Entecavir can cause lactic acidosis and hepatotoxicity; it does not prevent these adverse effects.

The nurse is caring for a patient who is HIV positive and has a previous history of drug and alcohol abuse. The patient is being treated with combination therapies, including didanosine [Videx]. Which laboratory findings would most concern the nurse? a.Increased serum amylase and triglycerides and decreased serum calcium b.Decreased serum amylase and serum triglycerides and increased serum calcium c.Decreased hemoglobin and hematocrit d.Increased serum amylase, decreased triglycerides, and increased platelets

ANS: A The nurse should be concerned about increased serum amylase triglycerides and a decreased serum calcium, which are symptoms of pancreatitis, the major adverse effect of didanosine. The other laboratory test results and assessment findings are not consistent with pancreatitis and are not a concern for the nurse.

The nurse is caring for a patient receiving intravenous acyclovir [Zovirax]. To prevent nephrotoxicity associated with intravenous acyclovir, the nurse will: a.hydrate the patient during the infusion and for 2 hours after the infusion. b.increase the patient's intake of foods rich in vitamin C. c.monitor urinary output every 30 minutes. d.provide a low-protein diet for 1 day before and 2 days after the acyclovir infusion.

ANS: A The nurse should ensure that the patient is hydrated during the acyclovir infusion and for 2 hours after the infusion to prevent nephrotoxicity. Increasing vitamin C would not help prevent nephrotoxicity. Monitoring urine output is important but would not help prevent nephrotoxicity. A low-protein diet is not indicated after an acyclovir infusion.

A male patient with hepatitis C will begin triple drug therapy with pegylated interferon alfa 2a [Pegasys], ribavirin [Ribasphere], and boceprevir [Victrelis]. The patient tells the nurse that his wife is pregnant. What will the nurse tell him? a.Boceprevir is contraindicated in males whose partners are pregnant. b.He should use a barrier contraceptive when having sex. c.He should use dual drug therapy with pegylated interferon alfa and ribavirin only. d.This combination drug therapy is safe for him to use.

ANS: A The triple combination is dangerous for pregnant women whose partners are using it, so it is contraindicated for any man whose partner is pregnant. Barrier contraceptives should be used by couples to prevent pregnancy when either partner is taking the triple combination therapy. Ribavirin is teratogenic and is not safe when a partner is pregnant. This combination is not safe for pregnant women whose partners are taking these drugs.

A young female patient is seen in a rural clinic after complaining of abdominal pain. The patient is wearing dirty clothing and is barefoot. The provider orders a complete blood count, which shows that the patient is anemic. The nurse may suspect that this patient has which of the following infestations? a. Ancylostomiasis (hookworm) b. Ascariasis (giant roundworm) c. Enterobiasis (pinworm) d. Trichuriasis (whipworm)

ANS: A Ancylostomiasis is most common when hygiene is poor and the patient habitually goes barefoot. Symptomatic anemia may occur in menstruating women or undernourished individuals. Ascariasis is usually asymptomatic. Enterobiasis is characterized by perianal itching. Trichuriasis is usually asymptomatic but may cause rectal prolapse if the worm burden is very large.

2. A patient is in the HIV clinic for a follow-up appointment. He has been on antiretroviral therapy for HIV for more than 3 years. The nurse will assess for which potential adverse effects of long-term antiretroviral therapy? (Select all that apply.) a. Lipodystrophy b. Liver damage c. Kaposi's sarcoma d. Osteoporosis e. Type 2 diabetes

ANS: A, B, D, E Anti-HIV drugs produce strain on the liver and may result in liver disease. A major adverse effect of protease inhibitors is lipid abnormalities, including lipodystrophy, or redistribution of fat stores under the skin. In addition, dyslipidemias such as hypertriglyceridemia can occur, and insulin resistance and type 2 diabetes symptoms can result. The increase in long-term antiretroviral drug therapy due to prolonged disease survival has led to the emergence of another long-term adverse effect associated with these medications—bone demineralization and possible osteoporosis. Kaposi's sarcoma is an opportunistic disease associated with HIV, not a result of long-term drug therapy. DIF: COGNITIVE LEVEL: Applying (Application) REF: p. 652 TOP: NURSING PROCESS: Assessment

The nurse is providing education to a group of patients who are HIV positive. The nurse is discussing the various medications used to treat HIV infection. A patient asks about nevirapine [Viramune]. Which statements by the nurse most accurately reflect the facts about nevirapine? (Select all that apply.) a."Some herbal preparations can reduce the levels of this drug." b."This agent can damage the liver; therefore, liver function tests are needed periodically." c."Usually no adverse effects occur when this medication is used alone." d."The drug must be dosed five times per day at evenly spaced intervals." e."You should call your healthcare provider immediately if you develop a rash."

ANS: A, B, E Some herbal preparations can reduce the levels of nevirapine, which can damage the liver. The patient should call the healthcare provider immediately if a rash develops. Many adverse effects are associated with the use of nevirapine. In adults, nevirapine is dosed once daily initially, not five times a day.

1. A patient who is diagnosed with genital herpes is taking topical acyclovir. The nurse will provide which teaching for this patient? (Select all that apply.) a. "Be sure to wash your hands thoroughly before and after applying this medicine." b. "Apply this ointment until the lesion stops hurting." c. "Use a clean glove when applying this ointment." d. "If your partner develops these lesions, then he can also use the medication." e. "You will need to avoid touching the area around your eyes." f. "You will have to practice abstinence when these lesions are active."

ANS: A, C, E, F This medication needs to be applied as long as prescribed, and the medication needs to be applied with clean gloves. Prescriptions should not be shared; if the partner develops these lesions, the partner will have to be evaluated before medication is prescribed, if needed. Eye contact should be avoided. The presence of active genital herpes lesions requires sexual abstinence. DIF: COGNITIVE LEVEL: Analyzing (Analysis) REF: p. 656 | p. 658 TOP: NURSING PROCESS: Implementation

1. Hydroxychloroquine (Plaquenil) is prescribed as part of malaria prophylaxis for a patient who will be traveling. The nurse will discuss which potential adverse effects with the patient? (Select all that apply.) a. Diarrhea b. Constipation c. Insomnia d. Dizziness e. Rash f. Headache

ANS: A, D, E, F Diarrhea, anorexia, nausea, vomiting, dizziness, rash, and headache are potential adverse effects of hydroxychloroquine. See Table 43-1 for other common adverse effects. The other options are not adverse effects of this drug. DIF: COGNITIVE LEVEL: Applying (Application) REF: p. 682 TOP: NURSING PROCESS: Implementation

A patient who is being treated for onchocerciasis with ivermectin [Stromectol] has a Mazotti reaction. Which assessment findings would the nurse expect? (Select all that apply.) a. Fever b. Nausea and vomiting c. Rash with pruritus d. Blurred vision e. Bone and joint pain

ANS: A,C,E Signs and symptoms associated with a Mazotti reaction are fever, rash with pruritus, and bone and joint pain. Nausea and vomiting and blurred vision are not signs and symptoms associated with a Mazotti reaction.

A patient has a positive test for hepatitis C and is admitted to the hospital. The admission laboratory tests reveal a normal ALT, and a liver biopsy is negative for hepatic fibrosis and inflammation. The nurse will prepare this patient for: a.dual therapy with pegylated interferon alfa and ribavirin. b.no medication therapy at this time. c.pegylated interferon alfa only until ALT levels are elevated. d.triple drug therapy with pegylated interferon alfa, ribavirin, and boceprevir.

ANS: B Current recommendations are that treatment is used only for patients with HCV viremia, persistent elevation of ALT, and evidence of hepatic fibrosis and inflammation upon liver biopsy. Dual therapy has been the regimen of choice for patients with the above symptoms, but the addition of a protease inhibitor has been shown to improve outcomes. It is not correct to give pegylated interferon alfa until ALT levels are elevated. Triple drug therapy is used for patients with the above symptoms.

A patient with HIV contracts herpes simplex virus (HSV), and the prescriber orders acyclovir [Zovirax] 400 mg PO twice daily for 10 days. After 7 days of therapy, the patient reports having an increased number of lesions. The nurse will expect the provider to: a.extend this patient's drug therapy to twice daily for 12 months. b.give intravenous foscarnet every 8 hours for 2 to 3 weeks. c.increase the acyclovir dose to 800 mg PO 5 times daily. d.order intravenous valacyclovir [Valtrex] 1 gm PO twice daily for 10 days

ANS: B Foscarnet is active against all known herpesviruses and is used in immunocompromised patients with acyclovir-resistant HSV or VZV. This patient is demonstrating resistance to acyclovir, so extending acyclovir therapy or increasing the acyclovir dose will not be effective. Valacyclovir is not approved for use in immunocompromised patients because of the risk for thrombotic thrombocytopenic purpura/hemolytic uremic syndrome

A patient taking stavudine [Zerit] telephones the clinic and reports numbness and tingling in the hands and feet. What should the nurse tell the patient? a.The numbness is an expected side effect of the medication and will diminish once the drug is withdrawn. b.The medication will probably be stopped, and the patient should come into the clinic for further evaluation. c.The dose may be too high, and the patient should cut the tablet in half. d.The patient should take the medication on a full stomach to reduce absorption of the drug.

ANS: B The patient has early signs and symptoms of neuropathy, which may resolve if the drug is stopped. The patient should be taught early in treatment to report these symptoms immediately. Numbness is not an expected side effect and these symptoms may diminish once the drug is withdrawn. The patient should never be advised to cut the dose in half unless instructed to do so by a prescriber. Taking the medication on a full stomach will not affect the amount of medication absorbed.

The nurse is performing a physical assessment on a patient who is receiving treatment with abacavir, zidovudine, and lamivudine [Trizivir]. The patient complains of fatigue. Upon further assessment, the nurse finds a rash and notes that the patient has a temperature of 101.1°F. What is the nurse's best course of action? a.Tell the patient that this is an expected response to these medications and to continue the agents as prescribed. b.Have the patient hold the medications and arrange for an immediate evaluation by the prescriber. c.Have the patient continue the abacavir but discontinue the other two agents for 3 weeks. d.Instruct the patient to continue all three medications and administer an antihistamine for the symptoms.

ANS: B The patient should discontinue all the medications. Immediate assessment by the provider is required, because the patient is showing early symptoms of a fatal hypersensitivity reaction. This is not an expected response; it indicates a serious reaction, which the patient should report to the prescriber immediately. The patient should not continue the medications for any additional dosages.

2. A patient who has undergone a lung transplant has contracted cytomegalovirus (CMV) retinitis. The nurse expects which drug to be ordered for this patient? a. Acyclovir (Zovirax) b. Ganciclovir (Cytovene) c. Ribavirin (Virazole) d. Amantadine (Symmetrel)

ANS: B Ganciclovir is indicated for the treatment of cytomegalovirus retinitis. Acyclovir is used for herpes simplex types 1 and 2, herpes zoster, and chickenpox; amantadine is used for influenza type A; and zanamivir is used for influenza types A and B. DIF: COGNITIVE LEVEL: Understanding (Comprehension) REF: p. 647 TOP: NURSING PROCESS: Planning

A patient who frequently travels to Africa has been treated for an acute attack of P. vivax malaria. The patient asks the nurse about medications to prevent relapse. Which response by the nurse is correct? a. "The form of malaria that you have does not relapse after a clinical cure has been achieved." b. "Until you are no longer traveling to Africa, relapse drugs will be postponed." c. "Using mosquito netting and DEET when you travel will prevent relapse." d. "You will be given prophylactic medications when you travel to prevent relapse."

ANS: B Relapse prevention, or a clinical cure, is postponed in patients with P. vivax malaria as long as the risk of reinfection continues. P. vivax malaria does relapse, so this response is incorrect. Using nondrug measures can help prevent reinfection but are not guaranteed to do so. Prophylactic medications prevent acute symptoms but not invasion of the liver.

The nurse is caring for a patient with severe symptomatic intestinal amebiasis who has just completed a 10-day course of metronidazole [Flagyl]. Which medication would the nurse expect to be ordered for the patient? a. Atovaquone [Mepron] b. Iodoquinol [Yodoxin] c. Nitazoxanide [Alinia] d. Pentamidine [Pentam]

ANS: B The metronidazole concentration in the stomach remains low; therefore, some amebas survive. Using iodoquinol as the subsequent drug achieves high concentrations, killing the remaining amebas. Pentamidine, atovaquone, and nitazoxanide are indicated for the treatment of diseases other than amebiasis.

9. The nurse is preparing to administer morning medications to a patient who has been newly diagnosed with tuberculosis. The patient asks, "Why do I have to take so many different drugs?" Which response by the nurse is correct? a. "Your prescriber hopes that at least one of these drugs will work to fight the tuberculosis." b. "Taking multiple drugs reduces the chance that the tuberculosis will become drug resistant." c. "Using more than one drug can help to reduce side effects." d. "Using multiple drugs enhances the effect of each drug."

ANS: B The use of multiple medications reduces the possibility that the organism will become drug resistant. The other options are incorrect. DIF: COGNITIVE LEVEL: Applying (Application) REF: p. 662 TOP: NURSING PROCESS: Assessment

A patient is being treated for trichinosis. The patient asks the nurse why the provider has ordered prednisone in addition to the anthelmintic medication. The nurse will tell this patient that this is ordered to: a. prevent swelling of the legs caused by larval infestation of lymphatics. b. reduce the inflammation that occurs during larval migration. c. suppress the patient's allergic response to the anthelmintic agent. d. suppress dermatologic symptoms that occur with heavy infestation.

ANS: B Trichinosis is acquired by eating undercooked pork containing encysted larvae, which migrate from the intestine to the skeletal muscle. Prednisone is given to reduce the inflammation that results from larval migration. This parasite does not affect lymphatics or cause elephantiasis. Prednisone is not given to counter allergic reactions to the drug. Dermatologic symptoms are not part of trichinosis infestations.

9. The nurse is providing counseling to a woman who is HIV positive and has just discovered that she is pregnant. Which anti-HIV drug is given to HIV-infected pregnant women to prevent transmission of the virus to the infant? a. Acyclovir (Zovirax) b. Zidovudine (Retrovir) c. Ribavirin (Virazole) d. Foscarnet (Foscavir)

ANS: B Zidovudine, along with various other antiretroviral drugs, is given to HIV-infected pregnant women and even to newborn babies to prevent maternal transmission of the virus to the infant. The other drugs are non-HIV antiviral drugs. DIF: COGNITIVE LEVEL: Applying (Application) REF: p. 654 TOP: NURSING PROCESS: Planning

The nurse asks a graduate nurse, "When a patient in the initial phase of HIV infection is assessed, which findings would you expect to see?" The graduate nurse would be correct to respond with which conditions? (Select all that apply.) a.Respiratory distress b.Fever c.Myalgia d.Lymphadenopathy e.Insomnia

ANS: B, C, D Fever, myalgia, and lymphadenopathy are early signs associated with HIV infection. Respiratory distress and insomnia are not consistent findings in the initial phase of HIV infection.

The nurse is caring for a patient who is human immunodeficiency virus (HIV) positive and is taking high doses of zidovudine [Retrovir]. The nurse is providing patient education about the adverse effects of the medication. Which statement by the patient demonstrates a need for further teaching? a."I may experience fatigue from anemia." b."I may be more susceptible to infection from neutropenia." c."I may have a deficiency of vitamin B6." d."I may have a deficiency of folic acid."

ANS: C A deficiency of vitamin B12, not vitamin B6, would be expected; this statement indicates that further teaching is required. With high-dose zidovudine, the patient can expect anemia, neutropenia, and folic acid deficiency.

The nurse is caring for a patient who is taking a protease inhibitor (PI). Upon review of the laboratory test results, the nurse notes that the patient has newly elevated plasma triglycerides and cholesterol. The nurse expects that the prescriber will manage these levels with: a.lovastatin [Mevacor]. b.simvastatin [Zocor]. c.modified diet and exercise. d.Pancrease.

ANS: C All PIs can elevate plasma levels of cholesterol and triglycerides. Potential interventions for hyperlipidemia include modified diet, exercise, and lipid-lowering agents. Lovastatin and simvastatin should be avoided, because they can accumulate to dangerous levels. Pancrease is not indicated to lower triglycerides and cholesterol.

A nurse provides teaching for a patient with cytomegalovirus (CMV) retinitis who will receive the ganciclovir ocular implant [Vitrasert]. Which statement by the patient indicates a need for further teaching? a."My vision may be blurred for 2 to 4 weeks after receiving the implant." b."Surgical placement of the implant is an outpatient procedure." c."The implant will remain in place permanently." d."The implant will slow progression of CMV retinitis."

ANS: C Ganciclovir ocular implants must be replaced every 5 to 8 months and do not remain in place permanently. It is correct that vision may be blurred for 2 to 4 weeks after placement of the implant, that placement is an outpatient procedure, and that the implant will slow progression of CMV retinitis.

A patient who is pregnant has a history of recurrent genital herpesvirus (HSV). The patient asks the nurse what will be done to suppress an outbreak when she is near term. The nurse will tell the patient that: a.antiviral medications are not safe during pregnancy. b.intravenous antiviral agents will be used if an outbreak occurs. c.oral acyclovir [Zovirax] may be used during pregnancy. d.topical acyclovir [Zovirax] must be used to control outbreaks.

ANS: C Oral acyclovir is devoid of serious adverse effects and may be used safely during pregnancy. It is incorrect to tell this patient that antiviral medications are not safe during pregnancy. Oral acyclovir is used to suppress recurrent genital herpes near term; intravenous antiviral medications are not indicated. It is not necessary to rely on topical medications because oral acyclovir is safe.

A female patient who has hepatitis C is being treated with pegylated interferon alfa and ribavirin [Ribasphere]. It will be important for the nurse to teach this patient that: a.if she gets pregnant, she should use the inhaled form of ribavirin [Virazole]. b.if she is taking oral contraceptives, she should also take a protease inhibitor. c.she should use a hormonal contraceptive to avoid pregnancy. d.she will need a monthly pregnancy test during her treatment.

ANS: D Ribavirin causes severe fetal injury and is contraindicated during pregnancy. Women taking ribavirin must rule out pregnancy before starting the drug, monthly during treatment, and monthly for 6 months after stopping treatment. Inhaled ribavirin is also embryo lethal and teratogenic. Adding a protease inhibitor will reduce the efficacy of oral contraceptives. Women using ribavirin should use two reliable forms of birth control.

A patient starting therapy with efavirenz [Sustiva] asks about the timing of the medication with regard to meals. What patient education about the administration of this medication should the nurse provide? a.The drug must be taken within 30 minutes after a meal. b.The drug is best taken with a high-fat meal. c.The drug can be taken anytime without regard to meals. d.The drug should be taken once daily on an empty stomach.

ANS: D The nurse should advise the patient that the medication should be taken once daily on an empty stomach. Thirty minutes after a meal is too soon to take the medication. The medication is taken on an empty stomach, because high-fat meals increase plasma levels by 39% with capsules and by 79% with tablets. The medication must not be taken with high-fat meals.

The nurse is caring for a patient who is HIV positive and is taking zidovudine [Retrovir]. Before administering the medication, the nurse should monitor which laboratory values? a.Ketones in the urine and blood b.Serum immunoglobulin levels c.Serum lactate dehydrogenase d.Complete blood count (CBC)

ANS: D The nurse should monitor the patient's CBC to determine whether the patient has anemia and neutropenia. Ketones are not an adverse effect of zidovudine. Nothing indicates a need to monitor the immunoglobulin levels or serum lactate dehydrogenase.

7. A patient is taking a combination of antiviral drugs as treatment for early stages of a viral infection. While discussing the drug therapy, the patient asks the nurse if the drugs will kill the virus. When answering, the nurse keeps in mind which fact about antiviral drugs? a. They are given for palliative reasons only. b. They will be effective as long as the patient is not exposed to the virus again. c. They can be given in large enough doses to eradicate the virus without harming the body's healthy cells. d. They may also kill healthy cells while killing viruses.

ANS: D Because viruses reproduce in human cells, selective killing is difficult; consequently, many healthy human cells, in addition to virally infected cells, may be killed in the process, and this results in the serious toxicities that are involved with these drugs. The other options are incorrect. DIF: COGNITIVE LEVEL: Understanding (Comprehension) REF: p. 645 TOP: NURSING PROCESS: Implementation

3. An infant has been hospitalized with a severe lung infection caused by the respiratory syncytial virus (RSV) and will be receiving medication via the inhalation route. The nurse expects which drug to be used? a. Acyclovir (Zovirax) b. Ganciclovir (Cytovene) c. Amantadine (Symmetrel) d. Ribavirin (Virazole)

ANS: D The inhalational form of ribavirin (Virazole) is used primarily in the treatment of hospitalized infants with severe lower respiratory tract infections caused by RSV. The other drugs listed are not used for the treatment of RSV. DIF: COGNITIVE LEVEL: Understanding (Comprehension) REF: p. 648 TOP: NURSING PROCESS: Planning

A 20-kg pediatric patient is being treated for toxoplasmosis, and the provider orders pyrimethamine [Daraprim] 10 mg twice daily. The nurse caring for this patient will anticipate which additional order from the provider? a. Administer 10 to 15 mg of folinic acid with each dose of pyrimethamine. b. Give the medication intravenously over 30 to 45 minutes. c. Increase the dose to 20 mg twice daily after 2 to 4 days. d. Teach the family to monitor for muscle pain and joint stiffness.

ANS: A Folinic acid is given with pyrimethamine to minimize adverse effects, and the dose is 10 to 15 mg twice daily, given with the pyrimethamine dose. The medication is given orally. The dose should be cut in half, not increased, after 2 to 4 days. Muscle pain and joint stiffness are adverse effects of sodium stibogluconate.

A patient has severe P. falciparum malaria and is unable to tolerate oral medications. Which regimen is the U.S. Food and Drug Administration (FDA)-approved treatment of choice for this patient? a. Intravenous quinidine gluconate plus intravenous doxycycline [Vibramycin] b. Intravenous artesunate followed by atovaquone/proguanil [Malarone] c. Intravenous quinine [Qualaquin] plus intravenous clindamycin d. Intravenous mefloquine plus intravenous primaquine

ANS: A For patients with severe malaria, IV quinidine gluconate is the only drug approved by the FDA for parenteral therapy of malaria; it should be accompanied by clindamycin, doxycycline, or tetracycline. Artesunate is considered the drug of choice for severe malaria, but it has not yet been approved by the FDA and must be used under the provisions of an Investigational New Drug protocol from the Centers for Disease Control and Prevention (CDC). Quinine is effective but has serious cardiotoxic side effects, so it is not a drug of first choice. Mefloquine and primaquine are not available as an IV medication.

A patient is taking diethylcarbamazine [Hetrazan] to treat a filarial infestation. The patient reports intense itching and a rash after several days of therapy. What will the nurse tell this patient? a. These symptoms are caused by death of the parasite. b. This is a minor, direct adverse effect of the drug. c. This is a sign the infestation is worsening. d. This warrants discontinuation of the drug.

ANS: A Indirect effects of therapy with diethylcarbamazine, such as itching and a rash, are the result of death of the parasite. These effects are transient. They are not direct effects of the drug. They do not indicate worsening of the infestation. They do not warrant discontinuation of the drug.

5. The nurse is administering intravenous acyclovir (Zovirax) to a patient with a viral infection. Which administration technique is correct? a. Infuse intravenous acyclovir slowly, over at least 1 hour. b. Infuse intravenous acyclovir by rapid bolus. c. Refrigerate intravenous acyclovir. d. Restrict oral fluids during intravenous acyclovir therapy.

ANS: A Intravenous acyclovir is stable for 12 hours at room temperature and often precipitates when refrigerated. Intravenous infusions must be diluted as recommended (e.g., with 5% dextrose in water or normal saline) and infused with caution. Infusion over longer than 1 hour is suggested to avoid the renal tubular damage seen with more rapid infusions. Adequate hydration should be encouraged (unless contraindicated) during the infusion and for several hours afterward to prevent drug-related crystalluria. DIF: COGNITIVE LEVEL: Applying (Application) REF: p. 656 TOP: NURSING PROCESS: Implementation

A child is diagnosed with cryptosporidiosis, and the prescriber orders nitazoxanide [Alinia]. While preparing the child's parents to administer this drug, the nurse will: a. determine whether the child is taking immunosuppressant medications. b. instruct the family to have the child take the drug on an empty stomach. c. teach the parents that yellow discoloration of the sclerae represents a serious side effect. d. tell the parents that headache, dry mouth, and a metallic taste are common.

ANS: A Nitazoxanide is approved for diarrhea caused by Cryptosporidium parvum in children only and is only effective in children who are immunocompetent. The nurse should assess whether the child is taking immunosuppressant medications. This drug should be given with food. Yellow discoloration of the sclerae may occur but resolves when the drug is discontinued. Headache, dry mouth, and a metallic taste are common side effects of metronidazole.

A patient with HIV who takes protease inhibitors develops tuberculosis and will begin treatment. Which drug regimen will be used for this patient? a. Isoniazid, pyrazinamide, ethambutol + rifabutin b. Isoniazid, pyrazinamide, ethambutol c. Isoniazid, rifampin, pyrazinamide, ethambutol d. Isoniazid + rifabutin

ANS: A Patients with HIV who take protease inhibitors are susceptible to drug interactions with rifampin, which accelerates the metabolism of protease inhibitors. Rifabutin can be substituted for rifampin in patients with HIV, because the degree of acceleration of this metabolism is less. A three-drug regimen would increase drug resistance, as would a two-drug regimen.

2. A patient has an infestation with flukes. The nurse anticipates the use of which drug to treat this infestation? a. Praziquantel (Biltricide) b. Pyrantel (Pin-X) c. Metronidazole (Flagyl) d. Ivermectin (Stromectol)

ANS: A Praziquantel is an anthelmintic that is used to kill flukes. Metronidazole is used to treat protozoal infections. The other drugs listed are used for other helminthic infestations. DIF: COGNITIVE LEVEL: Understanding (Comprehension) REF: p. 687 TOP: NURSING PROCESS: Planning

The nurse is providing patient education to a 26-year-old college student who is being treated for Trichomonas vaginalis with metronidazole [Flagyl]. Which statement by the patient indicates understanding of the teaching? a. "I should not consume alcohol while taking this medication." b. "If I have dark urine, I should stop taking the drug." c. "The drug is safe throughout pregnancy." d. "Vertigo and headaches may occur and are signs of toxicity."

ANS: A The patient should be advised to avoid drinking alcohol while taking this medication, because it may cause a disulfiram-like reaction. Darkening of the urine may occur but does not warrant discontinuing the drug. The drug is mutagenic for fetuses and should be avoided before and/or during pregnancy. Vertigo and headaches are side effects but are not signs of toxicity.

A nurse is teaching a group of nursing students about the two most common forms of malaria, Plasmodium vivax malaria and Plasmodium falciparum malaria. Which statement by a student indicates a need for further teaching? a. "Once erythrocyte forms of P. falciparum are eliminated, relapse will not occur." b. "Patients with P. vivax malaria experience relapse with increased frequency over time." c. "Patients with P. falciparum malaria experience symptoms at irregular intervals." d. "Patients with P. vivax malaria rarely encounter drug resistance."

ANS: B Relapse is likely with P. vivax malaria, because dormant parasites remain in the liver. Over time, episodes of relapse become less frequent and eventually stop entirely. Relapse does not occur with P. falciparum malaria, because this type does not form hypnozoites that can become dormant. Symptoms of P. falciparum malaria occur at irregular intervals, unlike those of P. vivax malaria, which peak every 48 hours. Drug resistance by P. vivax is relatively uncommon.

8. A patient is being evaluated for a possible helminthic infection. The nurse knows that which statement about anthelmintic therapy is true? a. The drugs may cause severe drowsiness. b. Anthelmintics are very specific in their actions. c. Anthelmintics are effective against broad classes of infestations. d. The drugs are used to treat protozoal infections such as intestinal amebiasis.

ANS: B Anthelmintics are very specific in their actions, and it is important to identify the cause of the infestation before beginning treatment. They are not used to treat protozoal infections, and they do not cause severe drowsiness. DIF: COGNITIVE LEVEL: Understanding (Comprehension) REF: p. 687 TOP: NURSING PROCESS: Assessment

A child has been diagnosed with enterobiasis, and the prescriber orders mebendazole [Vermox] to treat the infestation. When teaching this child's parents about the treatment, the nurse will include which statement? a. "A single dose of mebendazole will eradicate the infestation." b. "Everyone in the household should be treated with mebendazole." c. "Serious complications of this type of infestation are common." d. "Treatment should be limited to family members with symptoms."

ANS: B Everyone in the family should be treated simultaneously, because pinworms are easily spread. To treat pinworms, an initial dose of mebendazole 100 mg is given, and the dose is repeated in 2 weeks. The infestation rarely causes serious complications. Treatment is not limited to symptomatic infestations.

A patient comes to a clinic for tuberculosis medications 2 weeks after beginning treatment with a four-drug induction phase. The patient's sputum culture remains positive, and no drug resistance is noted. At this point, the nurse will expect the provider to: a. change the regimen to a two-drug continuation phase. b. continue the four-drug regimen and recheck the sputum in 2 weeks. c. obtain a chest radiograph and consider adding another drug to the regimen. d. question the patient about adherence to the drug regimen.

ANS: B In patients with positive pretreatment sputum test results, sputum should be evaluated every 2 to 4 weeks until cultures are negative and then monthly thereafter. In the absence of drug resistance, treatment with the same regimen should continue. Sputum cultures should become negative in over 90% of patients in 3 or more months. The induction phase should last 2 months, so this patient should remain on a four-drug regimen. It is not necessary to order a chest radiograph or to add another drug at this stage of treatment. The patient is stable and has not developed symptoms that cause concern, so the patient does not need to be questioned about adherence.

6. A patient with an intestinal infection that is positive for the Giardia lamblia organism will be taking an antiprotozoal drug. The nurse will include which information in the teaching plan for this patient? a. The urine may become dilute and pale during therapy. b. Taking the medications with food reduces gastrointestinal upset. c. The medications should be taken on an empty stomach. d. The drugs may be discontinued once the diarrhea subsides.

ANS: B Taking these drugs with food reduces gastrointestinal upset. Antiprotozoal drugs may cause the urine to turn dark. These drugs should be administered for the prescribed length of time to ensure complete eradication of the infection. DIF: COGNITIVE LEVEL: Applying (Application) REF: p. 690 TOP: NURSING PROCESS: Implementation

A family has just returned from a camping trip, and all family members have diarrhea and colicky abdominal pain after eating. The parents report that they drank water from a well while on the camping trip. While awaiting stool cultures, the nurse will expect the provider to order which agent empirically? a. Iodoquinol [Yodoxin] b. Metronidazole [Flagyl] c. Miltefosine [Impavido] d. Sodium stibogluconate [Pentostam]

ANS: B This family most likely has giardiasis, which can be acquired by drinking contaminated water. Metronidazole is the drug of choice for this infection. Iodoquinol is used for asymptomatic amebiasis. Miltefosine and sodium stibogluconate are used to treat leishmaniasis.

8. A patient who has started drug therapy for tuberculosis wants to know how long he will be on the medications. Which response by the nurse is correct? a. "Drug therapy will last until the symptoms have stopped." b. "Drug therapy will continue until the tuberculosis develops resistance." c. "You should expect to take these drugs for as long as 24 months." d. "You will be on this drug therapy for the rest of your life."

ANS: C Drug therapy commonly lasts for 24 months if consistent drug therapy has been maintained. The other options are incorrect. DIF: COGNITIVE LEVEL: Analyzing (Analysis) REF: pp. 666-667 TOP: NURSING PROCESS: Implementation

7. The nurse is reviewing antimalarial drug therapy with a patient and instructs the patient to watch for and report which potential adverse reactions? a. Drowsiness b. Insomnia c. Visual disturbances d. Constipation

ANS: C Encourage the patient to contact the prescriber if there is unresolved nausea, vomiting, profuse diarrhea, or abdominal pain and to report immediately any visual disturbances, dizziness, or respiratory difficulties. DIF: COGNITIVE LEVEL: Applying (Application) REF: p. 690 TOP: NURSING PROCESS: Implementation

A child is seen in the clinic after complaining of intense perianal itching. The provider diagnoses the child with pinworms and orders mebendazole [Vermox]. The nurse will expect to teach the child's parents to: a. administer the drug with a high-fat meal to improve absorption. b. avoid driving or other hazardous activities until the drug's effects wear off. c. give each family member one dose of the drug now and another dose in 2 weeks. d. report serious adverse effects that can occur with death of the causative parasite.

ANS: C Mebendazole is administered as a single dose once and then again in 2 weeks when given to treat pinworms. It should be given to all household members when one person is infested. Albendazole is given with a high-fat meal. Praziquantel can cause drowsiness. Ivermectin causes a Mazotti reaction, which occurs in patients treated for onchocerciasis with symptoms resulting from death of the parasite.

6. A patient is receiving cidofovir (Vistide) as part of treatment for a viral infection, and the nurse is preparing to administer probenecid, which is also ordered. Which is the rationale for administering probenecid along with the cidofovir treatment? a. Probenecid has a synergistic effect when given with cidofovir, thus making the antiviral medication more effective. b. The probenecid also prevents replication of the virus. c. Concurrent drug therapy with probenecid reduces the nephrotoxicity of the cidofovir. d. The probenecid reduces the adverse gastrointestinal effects of the cidofovir.

ANS: C Probenecid is recommended as concurrent drug therapy with cidofovir to help alleviate the nephrotoxic effects of probenecid. The other options are incorrect. DIF: COGNITIVE LEVEL: Applying (Application) REF: p. 646 TOP: NURSING PROCESS: Planning

2. A patient who has been taking isoniazid (INH) has a new prescription for pyridoxine. She is wondering why she needs this medication. The nurse explains that pyridoxine is often given concurrently with the isoniazid to prevent which condition? a. Hair loss b. Renal failure c. Peripheral neuropathy d. Heart failure

ANS: C Pyridoxine (vitamin B6) may be beneficial for isoniazid-induced peripheral neuropathy. The other options are incorrect. DIF: COGNITIVE LEVEL: Understanding (Comprehension) REF: p. 687 TOP: NURSING PROCESS: Implementation

8. A young adult calls the clinic to ask for a prescription for "that new flu drug." He says he has had the flu for almost 4 days and just heard about a drug that can reduce the symptoms. What is the nurse's best response to his request? a. "Now that you've had the flu, you will need a booster vaccination, not the antiviral drug." b. "We will need to do a blood test to verify that you actually have the flu." c. "Drug therapy should be started within 2 days of symptom onset, not 4 days." d. "We'll get you a prescription. As long as you start treatment within the next 24 hours, the drug should be effective."

ANS: C These drugs need to be started within 2 days of influenza symptom onset; they can be used for prophylaxis and treatment of influenza. The other options are incorrect. DIF: COGNITIVE LEVEL: Applying (Application) REF: p. 648 TOP: NURSING PROCESS: Planning

6. The nurse is counseling a woman who will be starting rifampin (Rifadin) as part of antitubercular therapy. The patient is currently taking oral contraceptives. Which statement is true regarding rifampin therapy for this patient? a. Women have a high risk for thrombophlebitis while on this drug. b. A higher dose of rifampin will be necessary because of the contraceptive. c. Oral contraceptives are less effective while the patient is taking rifampin. d. The incidence of adverse effects is greater if the two drugs are taken together.

ANS: C Women taking oral contraceptives and rifampin need to be counseled about other forms of birth control because of the impaired effectiveness of the oral contraceptives during concurrent use of rifampin. DIF: COGNITIVE LEVEL: Understanding (Comprehension) REF: p. 668 TOP: NURSING PROCESS: Implementation

5. A patient has been taking antitubercular therapy for 3 months. The nurse will assess for what findings that indicate a therapeutic response to the drug therapy? a. The chronic cough is gone. b. There are two consecutive negative purified protein derivative (PPD) results over 2 months. c. There is increased tolerance to the medication therapy, and there are fewer reports of adverse effects. d. There is a decrease in symptoms of tuberculosis along with improved chest x-rays and sputum cultures.

ANS: D A therapeutic response to antitubercular therapy is manifested by a decrease in the symptoms of tuberculosis, such as cough and fever, and by weight gain. The results of laboratory studies (culture and sensitivity tests) and the chest radiographic findings will be used to confirm the clinical findings of resolution of the infection. DIF: COGNITIVE LEVEL: Analyzing (Analysis) REF: p. 667 TOP: NURSING PROCESS: Evaluation

A patient is being treated with albendazole [Albenza] for neurocysticercosis caused by larval forms of the pork tapeworm. Which statement by the patient indicates a need for further teaching about the drug regimen? a. "I may need to take this medication for a month before the infestation is cleared." b. "I should take this drug with a fatty meal to improve absorption." c. "I will need to have liver function tests before and during treatment." d. "I will take the drug in 3 consecutive cycles of 28 days, followed by 14 drug-free days."

ANS: D Albendazole is given in consecutive cycles when dosing is done for cystic hydatid disease, not neurocysticercosis. For treatment of neurocysticercosis, patients generally take albendazole for 8 to 30 days. Albendazole can cause mild to moderate liver impairment, so patients should have liver function testing before and during treatment. The drug should be taken with a high- fat meal to improve absorption.

A patient who has contracted falciparum malaria has received a prescription for artemether/lumefantrine [Coartem] and tells the nurse she thinks she may be pregnant. What will the nurse tell this patient? a. "Ask your provider about taking only the artemether component." b. "The medication should be taken with food to prevent nausea." c. "This drug is safe during the first trimester of pregnancy." d. "You should talk to your provider about a different medication."

ANS: D Artemether/lumefantrine has undergone premarketing studies in pregnant women with no adverse effects, but all of the women were in their third trimester, but, since animal studies have demonstrated teratogenesis, is Pregnancy Risk Category C. The nurse should advise her to discuss another medication with her provider. The artemether component only is not recommended as a single drug. Taking the medication with food does not prevent nausea. It is not known whether the drug is safe during the first trimester.

A patient tells the nurse that a close friend has ascariasis. The patient expresses worry about becoming infested. What will the nurse tell this patient? a. "Ascariasis usually resolves without treatment, so you should not worry." b. "Avoid going barefoot and practice good hygiene and you won't become infested." c. "Treatment is always indicated if you are exposed, because ascariasis is contagious." d. "You should be tested, because serious complications can occur without treatment."

ANS: D Because this patient is worried about possible infestation, testing should be done so that treatment can be initiated if necessary. Because ascariasis can have serious complications, treatment is always indicated. Ascariasis infestations do not result from poor hygiene or from going barefoot. Ascariasis is not contagious, so treatment should begin only if the diagnosis is certain.

3. A patient who is being treated for malaria has started therapy with quinine and tetracycline. He asks the nurse why he is on an antibiotic when malaria is caused by a parasite. Which response by the nurse is correct? a. "The tetracycline prevents reinfection by the malarial parasite." b. "The antibiotic is combined with quinine to reduce the side effects of the quinine." c. "An antibacterial drug prevents the occurrence of superinfection during antimalarial therapy." d. "The two drugs are more effective against malaria when given together."

ANS: D The combination of quinine and tetracycline takes advantage of their synergistic protozoacidal effects. The other responses are incorrect. DIF: COGNITIVE LEVEL: Applying (Application) REF: p. 681 TOP: NURSING PROCESS: Implementation

A recent immigrant from the Middle East is being treated for P. falciparum malaria and has completed a course of chloroquine [Aralen Phosphate]. What will the nurse do? a. Ask the provider about genetic screening for this patient before starting the next medication. b. Contact the provider to discuss nondrug methods to prevent reinfection, such as DEET insect repellents. c. Teach the patient to notify the provider if symptoms recur so that relapse medication may be started. d. Tell the patient that no further treatment is necessary, because relapse in unlikely.

ANS: D P. falciparum malaria does not relapse once a clinical cure has been achieved, so no further treatment is necessary. Although this patient is from the Middle East and is at increased risk for glucose-6-phosphate dehydrogenase (G6PD) deficiency, genetic testing is unnecessary unless primaquine is given. Because no further treatment is indicated, genetic testing is not necessary. There is no need to use nondrug methods, because the patient is no longer in an area where infection is likely. This form of malaria will not relapse, so there is no need to teach the patient to report new symptoms.

1. The nurse is discussing adverse effects of antitubercular drugs with a patient who has active tuberculosis. Which potential adverse effect of antitubercular drug therapy should the patient report to the prescriber? a. Gastrointestinal upset b. Headache and nervousness c. Reddish-orange urine and stool d. Numbness and tingling of extremities

ANS: D Patients on antitubercular therapy should report experiencing numbness and tingling of extremities, which may indicate peripheral neuropathy. Some drugs may color the urine, stool, and other body secretions reddish-orange, but this is not an effect that needs to be reported. Patients need to be informed of this expected effect. The other options are incorrect. DIF: COGNITIVE LEVEL: Applying (Application) REF: p. 668 TOP: NURSING PROCESS: Implementation

4. A woman is traveling to a country where she will be at high risk for malarial infection. What will the nurse teach her regarding prophylactic therapy with hydroxychloroquine (Plaquenil)? a. Hydroxychloroquine is better absorbed and has fewer adverse effects if taken on an empty stomach. b. The drug is started 3 weeks before exposure but can be discontinued once she leaves the area. c. The medication is taken only when she observes mosquito bites because it can have toxic effects if taken unnecessarily. d. The drug is usually started 1 to 2 weeks before traveling to endemic areas and is continued for 4 weeks after leaving the area.

ANS: D Prophylaxis of malaria with hydroxychloroquine is usually started 1 to 2 weeks before exposure and continued for 4 weeks after the person has left the area. The medication should be taken with food to decrease gastrointestinal upset. DIF: COGNITIVE LEVEL: Applying (Application) REF: p. 683 TOP: NURSING PROCESS: Implementation

10. A patient newly diagnosed with tuberculosis (TB) has been taking antitubercular drugs for 1 week calls the clinic and is very upset. He says, "My urine is dark orange! What's wrong with me?" Which response by the nurse is correct? a. "You will need to stop the medication, and it will go away." b. "It's possible that the TB is worse. Please come in to the clinic to be checked." c. "This is not what we usually see with these drugs. Please come in to the clinic to be checked." d. "This is an expected side effect of the medicine. Let's review what to expect."

ANS: D Rifampin, one of the first-line drugs for TB, causes a red-orange-brown discoloration of urine, tears, sweat, and sputum. Patients need to be warned about this side effect. The other options are incorrect. DIF: COGNITIVE LEVEL: Applying (Application) REF: p. 668 TOP: NURSING PROCESS: Assessment

The nurse is caring for a newly admitted African American patient who has just returned from a mission trip to Africa. The patient contracted malaria and is being treated with primaquine. What will the nurse monitor? a. The patient's blood for evidence of hyperlipidemia b. The patient's eyes for evidence of icterus c. The patient's skin for photosensitivity d. The patient's urinary output for evidence of darkening urine

ANS: D The nurse should frequently monitor the patient's urine for evidence of hemoglobin, indicating hemolysis, which is marked by darkening urine. The patient need not be monitored for photosensitivity, because primaquine does not cause that condition. The patient need not be monitored for icterus, because primaquine does not cause icterus. The patient's blood need not be monitored for evidence of hyperlipidemia.

A patient taking metronidazole [Flagyl] for asymptomatic intestinal amebiasis complains of tingling and numbness of the hands and feet. What action by the nurse is most appropriate? a. Encourage the patient to exercise to improve circulation. b. Inform the patient that numbness is a common and reversible side effect. c. Stress the need to avoid constrictive clothing and crossing the legs. d. Withhold the drug and notify the prescriber.

ANS: D The nurse should withhold the drug and notify the prescriber, because these symptoms may indicate neurologic involvement. Numbness is not a common side effect of metronidazole. Encouraging the patient to move the hands and feet frequently to improve circulation would not address the neurologic symptoms. Advising the patient to avoid constrictive clothing and crossing the legs would not address the neurologic symptoms.


Related study sets

Pharmacology - Cardiovascular Medications

View Set

Ch 15 - Physiological and Behavioral Responses of the Neonate

View Set

Chapter 51: Care of Patients with Musculoskeletal Trauma

View Set

Chapter 11- Measuring the Cost of Living

View Set

Chapter 37: Nursing Care of the Child With an Infectious or Communicable Disorder

View Set

Art History 225B Post-Impressionism & SYmbolism

View Set